Está en la página 1de 749

case

()

()

()
miss miss
( miss
)

1 4

James Stewart: Calculus, Early Transcendentals, Seventh
Edition, International Metric Version, c2012.

Chapter 1
1.1

Functions and Models

Four Ways to Represent a Function, page 10

Definition 1 (page 10). A function () f is a rule that assigns to each element


x in a set D exactly one element, called f (x) in a set E.
y
x

f (x)

x
a
D

f (x)

f (x)
f (a)
f

x
E

(b)

(a)

(c)

Figure 1: (a) machine diagram; (b) arrow diagram; (c) graph () of a function.
We usually consider functions for which the sets D and E are sets of real numbers.





domain (), codomain (), range ().


value of f at x (or f of x).
independent variable, dependent variable.

Example 2. Find the domain of the following function:


(1) f (x) =

x2
.
1+x

(2) f (x) = (x 2)

D = {x R |
q

1+x
.
1x

D = {x R |

(3) f (x) = log(x + 2) + log(x 2).

D = {x R |

(4) f (x) = tan x.

D = {x R |

Example 3. Sketch the graph f (x) = x and g(x) = x2 .

Figure 2: f (x) = x and g(x) = x2 .

1.1-1

}.
}.
}.
}.

Vertical Line Test (page 15). A curve in the xy-plane is the graph of a function
of x if and only if no vertical line intersects the curve more than once.
Example 4. Give examples that one curve is the graph of a function and one curve
is not the graph of a function.

Figure 3: Left curve is a graph of a function; Right curve is not a graph of a function.
There are four possible ways to represent a function:






verbally (by a description in words).

numerically (by a table of values).

visually (by a graph).


algebraically (by an explicit formula).

Example 5. Recall that the absolute value () of a number a, denoted by |a|,


is the distance from a to 0 on the real number line. Plot the graph of the absolute
value function
(
x if x 0
f (x) = |x| =
.
x if x < 0

Figure 4: f (x) = |x|.


Example 6. Sketch the graph of the Heaviside function H(x), which is defined by
(
0 if x < 0
H(x) =
.
1 if x 0

1.1-2

Figure 5: The Heaviside function.


Example 7. Sketch the graph of the sign function sgn(x), which is defined by

1 if x > 0
sgn(x) =
0 if x = 0 .

1 if x < 0

Figure 6: The sign function.


Example 8. Sketch the graph of the greatest integer function f (x) = [[x]]: the
largest integer not greater than x. () Sometimes it is called the floor
function.

Figure 7: The floor function.

Useful inequality: x 1 [[x]] x < [[x]] + 1.


1.1-3

Definition 9 (Odd Function and Even Function, page 1718).


(a) If a function f satisfies f (x) = f (x) for every number x in the domain,
then f is called an odd function. ()
(b) If a function f satisfies f (x) = f (x) for every number x in the domain, then
f is called an even function. ()




All odd functions are symmetric about


All even functions are symmetric about

.
.

Figure 8: Odd function and even function.


Example 10. Any function defined on R can be decomposed as the sum of an odd
function and an even function.
Proof. Define two functions
g(x) =

f (x) f (x)
2

and h(x) =

We will show that


g(x) is an odd function:

h(x) is an even function:

f (x) = g(x) + h(x):

1.1-4

f (x) + f (x)
.
2




The decomposition is unique.


Why we use the name odd and even functions?

Definition 11 (Increasing and Decreasing Functions, page 19).


(a) A function f is called increasing () on an interval I if f (x1 ) < f (x2 )
whenever x1 < x2 in I.
(b) A function f is called decreasing () on an interval I if f (x1 ) > f (x2 )
whenever x1 < x2 in I.

Figure 9: Increasing function and decreasing function.


Remark 12. Some textbooks or papers use strictly increasing (decreasing), or
monotone increasing (decreasing).

1.1-5

1.2

Mathematical Models: A Catalog of Essential


Functions, page 23

Mathematical models
Why do we learn mathematics? One reason is that mathematics can help us solve
problems.
Real-world problem
Real-world predictions

Mathematical model

Mathematical conclusions

Essential functions
Definition 1 (page 27). A function P is called polynomial if
P (x) = an xn + an1 xn1 + + a2 x2 + a1 x + a0 ,
where n is a nonnegative integer and the numbers a0 , a1 , . . . , an are constants called
the coefficients of the polynomial. If the leading coefficient an 6= 0, then the degree
of the polynomial is n.

A polynomial of degree 1 is of the form P (x) = mx + b and so it is a linear

function. ()

A polynomial of degree 2 is of the form P (x) = ax2 + bx + c and so it is a


quadratic function. ()

A polynomial of degree 3 is of the form P (x) = ax3 + bx2 + cx + d and so it is a


cubic function. ()

Figure 1: Some cubic functions: f1 (x) = x3 + x, f2 (x) = x3 , f3 (x) = x3 x.

1.2-1

Definition 2 (page 28). A function of the form f (x) = xa , where a is a constant,


is called a power function.

Figure 2: Families of power functions


We usually consider several cases.
(a) a = n, where n is a positive integer.
(b) a = n1 , where n is a positive integer. root function.
(c) a = 1. reciprocal function.

Figure 3: Graphs of root function and reciprocal function.


Definition 3 (page 30). A rational function f is a ratio of two polynomials:
f (x) =

P (x)
,
Q(x)

where P and Q are polynomials. The domain consists of all values of x such that
Q(x) 6= 0.
Definition 4 (page 30). A function f is called an algebraic function if it can be
construct using algebraic operations (addition, subtraction, multiplication, division,
and roots) starting with polynomials.
1.2-2

Example 5. The mass of a particle with velocity v is


m0
,
m = f (v) = q
2
1 vc2

where m0 is the rest mass of the particle and c = 3 105 km/s is the speed of light
in a vacuum.
Definition 6 (page A26). Trigonometric functions: Let P (x, y) by any
the terminal side of and let r be the distance |OP |. Then we define
y
x
y
x
r
sin =
cos =
tan =
cot =
sec =
csc =
r
r
x
y
x

point on
r
.
y

Figure 4: y = sin x, y = cos x, y = tan x.

Figure 5: y = cot x, y = sec x, y = csc x.


Angles can be measured in degrees or in radians (abbreviated as rad). The angle
given by a complete revolution contain 360 , which is the same as 2 rad. Therefore,
rad = 180 .
Example 7. Complete the table.

sin
cos

1.2-3

2
3

3
4

5
6

3
2

There are a lot of trigonometric Identities:




sin csc = 1, cos sec = 1, tan cot = 1.


cos tan = sin ,

sin cot = cos ,

sin sec = tan ,

cos csc = cot ,

tan csc = sec ,

cot sec = csc .

sin2 + cos2 = 1, tan2 + 1 = sec2 ,

sin() = sin ,
cot() = cot ,

sin( + 2) = sin ,
cot( + ) = cot ,

cos() = cos ,
sec() = sec(),

1 + cot2 = csc2 .
tan() = tan ,
csc() = csc .

cos( + 2) = cos , tan( + ) = tan ,


sec( + 2) = sec , csc( + 2) = csc .

sin(x y) = sin x cos y cos x sin y,





cos(x y) = cos x cos y sin x sin y,


tan xtan y
.
tan(x y) = 1tan
x tan y
sin 2x = 2 sin x cos x,
cos 2x = cos2 x sin2 x = 2 cos2 x 1 = 1 2 sin2 x.
cos2 x =
sin2 x =

1+cos 2x
,
2
1cos 2x
.
2

sin x cos y = 21 (sin(x + y) + sin(x y))

cos x cos y = 21 (cos(x + y) + cos(x y))


sin x sin y = 12 (cos(x + y) cos(x y))

Definition 8 (Bounded function). A function f (x) is called bounded on an interval


I if there exists a constant M such that |f (x)| < M for x I.
Example 9. sin x, cos x are bounded functions on R. sin
|x| is not bounded function on R.

1
x

is bounded on x 6= 0.

Definition 10 (Periodic function). A function f (x) is called periodic with period


T (nonzero constant) if f (x + T ) = f (x) for x is defined.
Example 11. sin x, cos x, sec x and csc x are periodic functions with period 2; tan x
and cot x are periodic functions with period .
Definition 12 (Exponential functions, page 32). The exponential function are the
functions of the form f (x) = ax , where the basis a is a positive constant.

1.2-4

Figure 6: Exponential function f (x) = ax . (a) a > 1. (b) 0 < a < 1.

Definition 13 (Logarithmic functions, page 32). The logarithmic function f (x) =


loga x, where the base a ia a positive constant, are the inverse functions of the
exponential functions.

Figure 7: Logarithmic functions f (x) = loga x.

1.2-5

1.3

New Functions from Old Functions, page 36

Given a function f (x), we will discuss the new function


g(x) = af (bx + c) + d
for constant a, b, c, d and the effects of these constants.






The effect of a is vertical dilation.


The effect of b is horizontal dilation.
The effect of c is horizontal shift.
The effect of d is vertical shift.
y

Figure 1: The effect of d = 2 and a = 2.


y

Figure 2: The effect of c = 1 and b = 2.

 0 < b < 1: b > 1: c < 0: c > 0:


 f (x) af (bx + c) + d, b c a d
1.3-1

Example 1. Starting from f (x) = x2 , plot the function g(x) = 13 (2x + 5)2 1.
Solution.
y

Figure 3: x2 , (2x)2 , (2x + 5)2 , 13 (2x + 5)2 , and 31 (2x + 5)2 1.


Definition 2 (page 40). Given two functions f and g, the composite function f g
(also called the composition of f and g) is defined by
(f g)(x) = f (g(x)).

The notation f g means that the function g is applied first and then f is applied
second. In general f g 6= g f .

Example 3 (Composition of a function and the absolute value function). Discuss


the relations between f (x), |f (x)|, and f (|x|).
Solution.
y

Figure 4: Left: Given f (x), plot |f (x)|. Right: Given f (x), plot f (|x|).

1.3-2

1.5

Exponential Functions, page 51

Definition 1 (page 51). An exponential function is a function of the form


f (x) = ax ,
where a is a positive constant.

Figure 1: Three kinds of expoinential functions y = ax : 0 < a < 1, a = 1, a > 1.

Figure 2: Compare y = ax and y = bx , where a < b. (case 0 < a < b < 1 and
1 < a < b.)
Laws of Exponents (page 53). If a and b are positive numbers and x and y are
any real numbers, then
(1) ax+y = ax ay .
(2) axy =

ax
.
ay

(3) (ax )y = axy .


(4) (ab)x = ax bx .
2

Question 2. What is the meaning of 23 ?


1.5-1

Applications of exponential functions






: , 20

The number e
Example 3. 1 ,
: 100%, ,


(1) ?
(2) , ?
Solution. (1) :



= 1 +
,

, n ,

12

365

8760

525600
31536000

2
2.44140625
2.61303529
2.71456748
2.71812669
2.71827924
2.71828178

(2)

n
 2
 
 3
 n
1
1
1
1
1
n
n
n
n
1+
+ C2
= 1 + C1
+ C3
+ + Cn
n
n
n
n
n
1
1
1
1+1+ + ++
2! 3!
n!
1
1
1
+
++
1+1+
12 23
(n 1) n

 



1 1
1 1
1
1
1+1+
+
++

1 2
2 3
n1 n
1
1 + 1 + 1 < 3,
n
1.5-2


n
Leonhard Euler 1727 n , 1 + n1
n
, , e 1 + n1 n

1.5-3

1.6

Inverse Functions and Logarithms, page 58

Definition 1 (page 59). A function f is called a one-to-one function if it never


takes on the same value twice; that is,
f (x1 ) 6= f (x2 ) whenever x1 6= x2 .

Equivalent statement of one-to-one function is: If f (x1 ) = f (x2 ), then x1 = x2 .

x
y
z
w

x
y
z
w

X
Y
Z
W
A

X
Y
Z
W
A

Figure 1: f is one-to-one; g is not.


Horizontal Line Test (page 59). A function is one-to-one if and only if no horizontal line intersects its graph more than once.
Example 2. Give examples that one function is one-to-one and one function is not
one-to-one.

Figure 2: Left curve is an one-to-one function; Right curve is not an one-to-one


function.
Definition 3 (page 60). Let f be a one-to-one function with domain A and range
B. Then its inverse function f 1 has domain B and range A and is defined by
f 1 (y) = x f (x) = y.
for any y in B.
1.6-1





domain of f 1 = range of f .
range of f 1 = domain of f .
Do not mistake that 1 in f 1 for an exponent. Thus
f 1 (x) does NOT mean




1
= (f (x))1 .
f (x)

f 1 (x) = y f (y) = x.
We get the following cancellation equations:
f 1 (f (x)) = x for every x in A
f (f 1 (x)) = x for every x in B.

Question 4. How do we find the inverse function of a one-to-one function f ?


(1) Write y = f (x).
(2) Solve this equation for x in terms of y (if possible).
(3) To express f 1 as a function of x, interchange x and y. The resulting equation
is y = f 1 (x).
Example 5. Find a formula for the inverse of the function f (x) =

4x1
.
2x+3

Solution.

The graph of f (x) and the graph of f 1 (x) are symmetric about

Figure 3: Symmetry of a function and its inverse function.

Any increasing (decreasing) function has inverse function.


(By Horizontal Line Test).
1.6-2

Example 6. Plot the graph f (x) = xa and g(x) = x a .

Figure 4: f (x) = xa and g(x) = x a .

Logarithmic Functions
Definition 7 (page 62). If a > 0 and a 6= 1, the exponential function f (x) = ax is
either increasing or decreasing. It therefore has an inverse function f 1 (x), which
is called the logarithmic function with base a and is denoted by loga .
Since f 1 (x) = y f (y) = x, we have loga x = y ay = x.




loga (ax ) = x for every x R.


aloga x = x for every x > 0.

Figure 5: Logarithmic functions.


Laws of Logarithms (page 63). If x and y are positive numbers, then
(1) loga (xy) = loga x + loga y.
 
(2) loga xy = loga x loga y.

(3) loga (xr ) = r loga x (where r is any real number).


1.6-3

Definition 8 (page 64). The logarithm with base e 2.718281828 . . . is called the
natural logarithm and has a special notation:
loga x = ln x.
Hence we have






ln x = y ey = x.
ln(ex ) = x for all x R.
eln x = x for all x > 0.
ln e = 1.

Property 9 (Change of Base Formula, page 65). For any positive number a (a 6= 1),
we have
loga x =

ln x
.
ln a

Inverse Trigonometric Functions


The sine function f (x) = sin x is not one-to-one, but the function f (x) = sin x, 2
x 2 is one-to-one.
Definition 10 (page 67). The inverse function of this restricted sine function exists
and is denoted by sin1 x or arcsin x. It is called the inverse sine function or the
arcsine function.

Figure 6: sin x and sin1 x.





sin1 x 6=

1
.
sin x

sin1 (sin x) = x for 2 x 2 .


sin(sin1 x) = x for 1 x 1.
1.6-4

Example 11. Evaluate (a) sin1 (

3
)
2

and (b) tan(arcsin 23 ).

Solution.

Definition 12 (page 68).


(1) We restrict the cosine function y = cos x on 0 x and define the inverse
cosine function cos1 x or arccos x.
(2) We restrict the tangent function y = tan x on 2 < x <
inverse tangent function tan1 x or arctan x.

and define the

(3) We restrict the cotangent function y = cot x on 0 < x < and define the
inverse cotangent function cot1 x or arccot x.
(4) We restrict the secant function y = sec x on 0 x <
define the inverse secant function sec1 x or arcsec x.

(5) We restrict the cosecant function y = csc x on 0 < x


define the inverse cosecant function csc

x or arccsc x.

For arcsec x and arccsc x, domains are not agreed.

Figure 7: cos1 x and tan1 x.

Example 13. Simplify the expression sin(tan1 x).


Solution.

1.6-5

or x <

3
,
2

and

or < x

3
,
2

and

Chapter 2
2.1

Limits and Derivatives

The Tangent and Velocity Problem, page 82

The tangent problem, page 82


Example 1. Plot the parabola f (x) = x2 . Observe all secant lines passing through
the point P (1, f (1)) and Qx (1 + x, f (1 + x)), where x 6= 0 is a number close
to 0.
Solution.

We can compute the slope of secant line LP Qx to get


(1 + x)2 12
f (1 + x) f (1)
=
(1 + x) 1
x
(1 + x + 1)(1 + x 1)
= 2 + x.
=
x

mP Qx =

So the equation of secant line LP Qx is


y 1 = (2 + x)(x 1).
When x is close to 0, the slope mP Qx is close to 2. That means the family
of secant lines LP Qx is close to the line y 1 = 2(x 1), which passes through
P (1, f (1)) and the slope is 2.
We call y 1 = 2(x 1) the tangent line of f (x) = x2 at x = 1.

The velocity problem, page 84

 ,
 ; ()
Example 2. Suppose that a ball is dropped from the upper observation deck of
Taipei 101. Find the velocity of the ball after 5 seconds.
2.1-1

Solution. If the distance falled after t seconds is denoted by s(t) and measured in
meters, then Galileos law is expressed by the equation
s(t) =

1
9.8 t2 = 4.9t2 .
2

We can approximate the velocity at instant time t = 5 by computing the average


velocity over the brief time interval
change in position
s(5 + 10n ) s(5)
=
time elapsed
(5 + 10n ) 5
n 2
2
4.9 (5 + 10n + 5)(5 + 10n 5)
4.9 ((5 + 10 ) 5 )
=
=
10n
10n
n
n
= 4.9 (10 + 10 ) = 49 + 4.9 10 .

average velocity =

That is,
Time interval

Average velocity (m/s)

5 t 5.1
5 t 5.01

49.49
49.049

5 t 5.001
5 t 5.0001

49.0049
49.00049

5 t 5.00001

49.000049

It appears that as we shorten the time period, the average velocity is becoming
closer to 49m/s. The instantaneous velocity when t = 5 is defined to be the limiting
value of these average velocities over shorter an shorter time periods that start at
t = 5. Thus the instantaneous velocity after 5 second is v = 49m/s.
Remark 3. Time periods 10n we choose in Example 2 are just some samples. In
general, we can use t to represent any time interval and do the same calculation
to get the average velocity form 5 to 5 + t is 4.9 (10 + t). The average velocity
is becoming closer to 49m/s as well when we shorten the time period.

2.1-2

2.2

The Limit of a Function, page 87

(One-Sided) Limit
Definition 1 (page 92). We write
lim f (x) = L
xa

and say the left-hand limit of f (x) as x approaches a (or the limit of f (x) as x
approaches a from the left) is equal to L if we can make the values of f (x) arbitrarily
close to L by taking x to be sufficiently close to a and x less than a.
Similarly, if we require that x be greater than a, we get the right-hand limit of
f (x) as x approaches a is equal to L and we write
lim f (x) = L.

xa+




The symbol x a means that we consider only x < a.


The symbol x a+ means that we consider only x > a.

Figure 1: Left-hand limit and Right-hand limit.


Definition 2 (The Limit of a Function, page 87). Suppose f (x) is defined when x
is near the number a. Then we write
lim f (x) = L
xa

if we can make the value of f (x) arbitrarily close to L by taking x to be sufficiently


close to a but not equal to a.





An alternative notation for lim f (x) = L is f (x) L as x a.


xa

The value at x = a is not important.


Observe the behavior of f (x) near x = a.
2.2-1

Figure 2: Limit of a function.

lim f (x) = L if and only if lim f (x) = L and lim+ f (x) = L.

xa

xa

xa

Example 3. Find the limit of the Heaviside function at x = 0.

y
1
x
Figure 3: The Heaviside function H(x).
Solution. lim H(x) =
x0

, lim+ H(x) =
x0

, lim H(x)

x0

Example 4. The graph of a function f (x) is shown in Figure 4. Use it to state the
values (if they exist) of the following:
y
3
2
1
1

Figure 4: The graph of f (x).

(a1) lim f (x)

(b1) lim+ f (x)

(c1) lim f (x)

x1

x1

x1

(a2) lim f (x)

(b2) lim+ f (x)

(c2) lim f (x)

x2

x2

x2

(a3) lim f (x)

(b3) lim+ f (x)

(c3) lim f (x)

x3

x3

2.2-2

x3

(d1) f (1)
(d2) f (2)
(d3) f (3)

Example 5. Guess the value of lim


x0

sin x
.
x

y
1

Figure 5: The graph of

sin x
.
x

Example 6. Investigate the limit lim sin x .


x0

y
1

1
Figure 6: The graph of

sin
.
x

Infinite Limits
Definition 7 (page 93). Let f be a function defined on both sides of a, except
possibly at a itself. Then
lim f (x) =
xa

means that the values of f (x) can be made arbitrarily large (as large as we please)
by taking x sufficiently close to a, but not equal to a.

Another notation for lim f (x) = is f (x) as x a.


xa

2.2-3

Figure 7: Infinite limit lim f (x) = .


xa

Definition 8 (page 94). Let f be a function defined on both sides of a, except


possibly at a itself. Then
lim f (x) =
xa

means that the values of f (x) can be made arbitrarily negative by taking x sufficiently close to a, but not equal to a.

Figure 8: Infinite limit lim f (x) = .


xa

Similar definition can be given for the one-sided infinite limits:


lim f (x) =
xa

lim f (x) = .

lim f (x) =

lim f (x) =

xa+

xa+

xa

Definition 9 (page 94). The line x = a is called a vertical asymptote of the curve
y = f (x) if at least one of the following statement is true:
lim f (x) =
xa

lim f (x) =
xa

lim f (x) =
xa

lim f (x) =
xa

lim f (x) =

xa+

lim f (x) = .

xa+

Example 10.
(a) f (x) = tan x has vertical asymptotes

(b) f (x) = sec x has vertical asymptotes

(c) f (x) =

1
x

has a vertical asymptote

(d) f (x) = log x has a vertical asymptote


2.2-4

2.3

Calculating Limits Using the Limit Laws,


page 99

Theorem 1. If lim f (x) exists, then it is unique.


xa

Theorem 2 (Limit Laws, page 99). Suppose that c is a constant and the limits
lim f (x) and lim g(x) exist. Then
xa

xa

(1) lim (f (x) + g(x)) = lim f (x) + lim g(x).


xa

xa

(Sum Law)

xa

(2) lim (f (x) g(x)) = lim f (x) lim g(x).


xa

xa

(3) lim (cf (x)) = c lim f (x).


xa

(Difference Law)

xa

(Constant Multiple Law)

xa

(4) lim (f (x)g(x)) = lim f (x) lim g(x).


xa

xa

f (x)
xa g(x)

(5) lim

lim f (x)
xa

lim g(x)

if lim g(x) 6= 0.

(Quotient Law)

xa

xa

The followings are some special limits:



n
(6) lim (f (x))n = lim f (x) where n is a positive integer.
xa

(Product Law)

xa

(Power Law)

xa

(7) lim c = c.
xa

(8) lim x = a.
xa

(9) lim xn = an where n is a positive integer.


xa

(10) lim

x =

a where n is a positive integer. (If n is even, we assume that

xa

a > 0.)
q
p
(11) lim n f (x) = n lim f (x) where n is a positive integer. (If n is even, we
xa

xa

assume that lim f (x) > 0.)


xa

 ,

x2 1
.
x1 x1

Example 3. Find the limit lim


Solution.

2.3-1

Solution 2. Let x = 1 + x, then x 1 is equivalent to x 0, and

 : Section 2.1, Example 1.


Example 4. Find the limit lim
t0

t2 +93
.
t2

Solution.

Example 5. Show that lim |x| = 0.


x0

Solution.
Example 6. Prove that lim

does not exist.

|x|
x0 x

Solution.

Exercise 7. Find the limit lim


x0

|2x1||2x+1|
.
x

(94 )

Example 8. Compute lim [[x]] and lim+ [[x]].


x0

x0

Solution. Left-hand limit:

Right-hand limit:

Exercise 9. Evaluate the limit lim


x2

x 2x
.
2
|x 4|

(101 )

Theorem 10 (page 105). If f (x) g(x) when x is near a (except possibly at a)


and the limits of f and g both exist as x approaches a, then
lim f (x) lim g(x).
xa

xa

 f (x) < g(x), xa


lim f (x) lim g(x)
xa
2.3-2

Theorem 11 (The Squeeze Theorem, page 105). If f (x) g(x) h(x) when x is
near a (except possibly at a) and
lim f (x) lim h(x) = L
xa

xa

then
lim g(x) = L.
xa

Figure 1: The Squeeze Theorem.



Example 12. Show that lim x sin x1 = 0.
x0

Solution.

Example 13. Show that lim


x0

sin x
x

(93 )

= 1.

Solution (page 192). Assume first that x lies between 0 and 2 . Figure 2 shows a
sector of a circle with center O, central angle x, and radius 1.

Figure 2: A sector of a circle with center O, central angle x, and radius 1.


2.3-3

Since area of triangle OAB < area of sector OAB < area of triangle OAC,
we have

For 2 < x < 0, since sin x, x, and tan x are odd functions, we get tan x < x <
sin x, then

Hence lim
x0

lim
x0

sin 2x
2x

sin x
x

= 1.
.

Exercise 14. Find the limit lim


x0

sin(|x|)
x

and lim+
x0

sin(|x|)
.
x

(101 )

sin 3x
.
x0 |3x2||x+2|

(91 )

Exercise 15. Find the limit lim

Example 16. Find the limit lim


x0

1cos x
x2

and lim

Solution. By the half-angle formula: sin2

Exercise 17. Find the limit lim


x0

x0
x
2

tan xsin x
.
x3

cos x1
.
x

1cos x
,
2

(92,93 )

we get

(Hint: tan x =

sin x
.)
cos x

(92,93 )

cos x1
.
x0 sin(x sin x)

(101 )

cos(sin x)1
.
tan2 x
x0

(101 )

Exercise 18. Find the limit lim


Solution.

Exercise 19. Find the limit lim

2.3-4

Example 20. Find the following limits:



x2 sin x1
;
(a) lim
x0
sin x


x3 sin x1
(b) lim
.
x0 sin(x2 )
(91 ; 95 )

Solution.

1sin2 (ax)cos(ax)
.
2
x0 1+sin (bx)cos(bx)

Exercise 21. Find the limit lim


Example 22. Given lim
x2

x2
px+q3

= 9, find p and q.

Solution. We compute

Exercise 23. Is there a number a such that


3x2 + ax + a + 3
x2
x2 + x 2
lim

exists? If so, find the value of a and the value of the limit.

2.3-5

(101 )
(95 )

2.4

The Precise Definition of a Limit (page 108)

Definition 1 (- language, page 110). Let f be a function defined on some open


interval that contains the number a, except possibly at a itself. Then we say that
the limit of f (x) as x approaches a is L, and we write
lim f (x) = L
xa

if for every number > 0 there is a number > 0 such that


if 0 < |x a| <

then |f (x) L| < .

Figure 1: Limit of f (x) as x approaches a is L.






: for all; exist


, :

Example 2. Prove that lim (2x + 3) = 5.


x1

Solution.
Observation: We calculate |(2x + 3) 5| = |2x 2| = 2|x 1|. We want to
find > 0 such that
if 0 < |x 1| < ,

then 2|x 1| < .

That is, if 0 < |x 1| < , then |x 1| < .


2

This suggests that we can choose = 2 . (or smaller)


Proof:

2.4-1

Example 3. Prove that lim x2 = 9.


x3

Solution.
Observation: We calculate |x2 9| = |x + 3||x 3| < . We want to find > 0
such that
if 0 < |x| < ,

then |x + 3||x 3| < .

Notice that if we can find a positive constant M such that |x + 3| < M,


then |x + 3||x 3| < M|x 3| and we can make M|x 3| < by taking
|x 3| <

= .

Since we are interested only in values of x that close to 3, it is reasonable to


assume |x 3| < 1, then |x + 3| < 7, so M = 7 is a choice.
Proof:

 Proof
Example 4. Prove the Limit Sum Law: Suppose that the limits lim f (x) and
xa

lim g(x) exist. Then lim (f (x) + g(x)) = lim f (x) + lim g(x).
xa

xa

xa

xa

Proof.

Remark 5. ? , lim f (x) =


xa
L, L ?
, there exists > 0, for all > 0, there exists 0 < |x a| < such
that |f (x) L| .
2.4-2

Example 6. The Dirichlet function is defined by


(
0 if x is rational
f (x) =
.
1 if x is irrational
Prove that lim f (x) does not exist.
x0

Solution. Suppose lim f (x) = L. Notice that both rational numbers and irrational
x0
numbers are dense in real numbers.
If L 21 ,

If L < 12 ,

You cant plot the graph of the Dirichlet function.

Example 7. The Riemann function is defined by


(
1
if x = pq , (p, q) = 1 is rational
q
f (x) =
.
0 if x is irrational
Then the limit of f (x) exists as x approaches to any irrational number.
Exercise 8.
(a) Give an - proof to show that lim f (x) = L implies lim |f (x)| = |L|.
xa

xa

(b) Show that the reversed direction of the above statement is not true.
(96 )
Definition 9 (Definition of Left-Hand Limit, page 113).
lim f (x) = L
xa

if for every number > 0 there is a number > 0 such that


if a < x < a then |f (x) L| < .
2.4-3

Definition 10 (Definition of Right-Hand Limit, page 113).


lim f (x) = L

xa+

if for every number > 0 there is a number > 0 such that


if a < x < a +
Exercise 11. Prove that lim+

then |f (x) L| < .

x = 0.

x0

Definition 12 (page 115). Let f be a function defined on some open interval that
contains the number a, except possibly at a itself. Then
lim f (x) =
xa

means that for every number M there is a number > 0 such that
if 0 < |x a| <
1
2
x0 x

Example 13. Prove that lim

then f (x) > M.

= .

Solution.
Observation: Let M be a given positive number. We want to find a number
> 0 such that if 0 < |x| < , then x12 > M. Notice that
1
1
1
> M x2 <
|x| < .
2
x
M
M
This suggests us to choose =

1
M

(or smaller).

Proof:

Definition 14 (page 116). Let f be a function defined on some open interval that
contains the number a, except possibly at a itself. Then
lim f (x) =
xa

means that for every number N there is a number > 0 such that
if 0 < |x a| <

then f (x) < N.

2.4-4

2.5

Continuity (page 118)

Definition 1 (continuous at a point, page 118). A function f (x) is continuous at


x = a if
lim f (x) = f (a).

xa

We say that f (x) is discontinuous at x = a (or f (x) has a discontinuity at x = a) if


f (x) is not continuous at a.

Figure 1: f (x) is continuous at x = a.

The definition of continuous at x = a requires three things:

(1) f (x) is defined at x = a.


(2) The limit lim f (x) exists. (Both left-hand limit lim f (x) and right-hand limit
xa

xa

lim+ f (x) exist and are equal.)

xa

(3) lim f (x) = f (a).


xa

Another viewpoint of continuous at x = a:


Limit and function are exchangeable. That is, lim f (x) = f (lim x) = f (a).
xa

xa

Continuous between mathematics and everyday language.


There are three types of discontinuity:

(1) removable discontinuity: We can redefine the value of the function f (x) at
x = a such that f (x) is continuous at x = a.
(2) infinite discontinuity
(3) jump discontinuity

2.5-1

Figure 2: Three types of discontinuity.


Definition 2 (continuous from the right (or left), page 120.). A function f (x) is
continuous from the right at x = a if
lim f (x) = f (a),

xa+

and f (x) is continuous from the left at x = a if


lim f (x) = f (a),

xa

Figure 3: (a) f (x) is continuous from the right. (b) f (x) is continuous from the left.
Example 3. Discuss the continuity of the following functions:
( 2
( 2
x x2
x x2
2
if
x
=
6
2
if x 6= 2
x x2
x2
x2
, g(x) =
f (x) =
, h(x) =
.
x2
1
if x = 2,
3
if x = 2
Solution.

2.5-2

Example 4. Discuss the continuity of floor function f (x) = [[x]] at integers.


Solution.

Example 5. The Riemann function is defined by


(
1
if x = pq is rational number
q
.
f (x) =
0 if x is irrational number
Then the Riemann function is continuous at irrational numbers.
Definition 6 (continuous on an interval, page 120.). A function f (x) is continuous
on an interval if it is continuous at every point in the interval. If f (x) is defined
only on one side of an endpoint of the interval, we understand continuous at the
endpoint to mean continuous from the right or continuous from the left.
Theorem 7 (properties of continuous functions, page 121.). If f (x) and g(x) are
continuous at x = a, and c is a constant, then the following functions are also
continuous x = a:
(1) (f + g)(x) = f (x) + g(x)
(2) (f g)(x) = f (x) g(x)
(3) cf (x), cg(x)
(4) f (x)g(x)
(5)

f (x)
g(x)

if g(a) 6= 0.

Proof of (1). Since f (x) and g(x) are continuous at x = a, we have

Therefore

2.5-3

Theorem 8 (page 123). The following type of functions are continuous at every
number in their domains:
polynomials: P (x) = an xn + an1 xn1 + + a2 x2 + a1 x + a0 .
rational functions: f (x) =

P (x)
.
Q(x)

root functions: f (x) = x n , n is a positive integer.


trigonometric functions: sin x, cos x, tan x, cot x, sec x, csc x.
inverse trigonometric functions: sin1 x, cos1 x, tan1 x, cot1 x, sec1 x,
csc1 x.
exponential functions f (x) = ax , ex .
logarithmic functions f (x) = loga x, ln x.
Theorem 9. If f is continuous at b an lim g(x) = b, then lim f (g(x)) = f (b). In
xa
xa
other words,
lim f (g(x)) = f (lim g(x)).
xa
 
x
Example 10. Evaluate lim sin1 1
1x
xa

x1

Solution.



Exercise 11. Find the limit lim sin1 x sin x1 .
x0

(99 )

Theorem 12. If g is continuous at a and f is continuous at g(a), then the composition function f g given by (f g)(x) = f (g(x)) is continuous at a.
Proof.

2.5-4

Theorem 13 (The Intermediate Value Theorem, page 125). Suppose that f (x) is
continuous on the closed interval [a, b] and let N be any number between f (a) and
f (b), where f (a) 6= f (b). Then there exists a number c in (a, b) such that f (c) = N.

Figure 4: The Intermediate Value Theorem





f (x) is continuous is important.


Closed interval [a, b] is important.
The number is not necessarily unique.

Applications of Intermediate Value Theorem






Example 14. Suppose f is a continuous function on [a, b] and a f (x) b for all
x [a, b]. Show that there exists c [a, b] such that f (c) = c.
Solution.

2.5-5

(100 )

2.6

Limits at Infinity: Horizontal Asymptotes (page


130)

Definition 1 (page 130). Let f be a function defined on some interval (a, ). Then
lim f (x) = L
x

means that the values of f (x) can be made arbitrarily close to L by taking x sufficiently large.

Another notation for lim f (x) = L is f (x) L as x .


x

Figure 1: Example illustrating lim f (x) = L.


x

Definition 2 (page 131). Let f be a function defined on some interval (, a).


Then
lim f (x) = L
x

means that the value of f (x) can be made arbitrarily close to L by taking x sufficiently large negative.

 x x
Definition 3 (page 131). The line y = L is called a horizontal asymptote of the
curve y = f (x) if either
lim f (x) = L or
x

lim f (x) = L.
x

Figure 2: Example illustrating horizontal asymptotes.

2.6-1

Remark 4. If an one-to-one function has a vertical asymptote, then its inverse function has a horizontal asymptote. For example, tan x and tan1 x. Another example
is ln x and ex .
Theorem 5 (page 133). If r > 0 is a rational number, then
1
= 0.
x xr
lim

If r > 0 is a rational number such that xr is defined for all x, then


1
= 0.
x xr
lim

Example 6. Evaluate
3x2 x 2
.
x 5x2 + 4x + 1
lim

Solution.

Suppose P (x) and Q(x) are two polynomials with leading terms an and bm . Then

if

P (x)
lim
=
if
x Q(x)

if
1

Example 7. Evaluate lim (2x + 3x + 5x ) x .


x

Solution.

2.6-2

(91, 102 )


Example 8. Evaluate the limit lim x( x2 x x) sin x1 .

(94 )

Solution.

3
Example 9. Find the limit lim x 2 ( x + 2 2 x + 1 + x).

(102 )

Solution.

Example 10. We know lim 1 +


x

Solution.

Example 11. Let f (x) =


(a) lim
x


1 x
x

= e. Find lim
x

1+


1 x
.
x

(97 )

x2 + x. Compute the following limits:

f (x)
.
x

(89 )

(b) lim (f (x) Ax).


x

2.6-3

Solution.

Example 12. Find lim

x2 + 4x + 5

x2 + x + 1.

(98 )

Solution.

Exercise 13. Find the limit lim

x2 2x + x.

(93 )

Example 14. Suppose , are two constants and lim ( x2 + 3x + 2x) = 0.


x

(91 )

Find and .
Solution.

Exercise 15.
(a) Use the Squeeze Theorem to evaluate lim
x

sin x
.
x

(b) How many times does the graph cross the asymptote?
2.6-4

Infinite Limits at Infinity


Precise Definition
Definition 16 (page 137). Let f be a function defined on some interval (a, ).
Then
lim f (x) = L
x

means that for every > 0 there is a corresponding number N such that
if x > N

then |f (x) L| < .

Definition 17 (page 138). Let f be a function defined on some interval (, a).


Then
lim f (x) = L
x

means that for every > 0 there is a corresponding number N such that
if x < N

then |f (x) L| < .

Definition 18 (page 140). Let f be a function defined on some interval (a, ).


Then
lim f (x) =
x

means that for every positive number M there is a corresponding positive number
N such that
if x > N

then f (x) > M.

2.6-5

2.7

Derivatives and Rates of Change (page 143)

Definition 1 (page 143). The tangent line () to the curve y = f (x) at the point
P (a, f (a)) is the line through P with slope
f (x) f (a)
f (a + h) f (a)
= lim
xa
h0
xa
h

m = lim

provided that this limit exists.

Figure 1: tangent line is the limiting position of the secant line ().

Example 2. Find an equation of the tangent line to the hyperbola y =


point (1, 1).

1
x

at the

Solution. Let f (x) = x1 . Then the slope of the tangent at (1, 1) is

Therefore an equation of the tangent at the point (1, 1) is

Definition 3 (page 145). If f (x) is the position function, then the average velocity
is
average velocity =

displacement
f (a + h) f (a)
=
,
time
h

and the velocity (or instantaneous velocity) v(a) at time t = a be the limit of these
average velocities:
f (a + h) f (a)
.
h0
h

v(a) = lim

The speed of the particle is the absolute value of the velocity |v(a)|
2.7-1

Example 4. Suppose that a ball is dropped from the upper observation deck of
Taipei 101, 508m above the ground.
(a) What is the velocity of the ball after 5 seconds?
(b) How fast is the ball traveling when it hits the ground?
Solution. Using the equation of motion s = f (t) = 4.9t2 , we have

.
(a) The velocity after 5 is
q
2
(b) First we solve 4.9t1 = 508. This gives t1 = 508
. The velocity of the ball as it
4.9
hits the ground is

Definition 5 (page 146). The derivative of a function f at a number a, denoted by


f (a), is
f (a + h) f (a)
f (x) f (a)
= lim
h0
h0
xa
h

f (a) = lim
if this limit exists.

If we use the point-slope form () of the equation of a line, we can write an


equation of the tangent line to the curve y = f (x) at the point P (a, f (a)):
y f (a) = f (a)(x a)


Example 6. Consider
f (x) =

x sin x1 if x 6= 0
,
0
if x = 0

where is a natural number. Determine whether f (0) exists.


Solution. By the definition of derivative, we have

2.7-2

Rates of Change
Suppose y is a quantity that depends on another quantity x. Thus y is a function
of x and we write y = f (x). If x changes from x1 to x2 , then the change in x (also
called the increment () of x) is x = x2 x1 , and the corresponding change
in y is y = f (x2 ) f (x1 ). The difference quotient
f (x2 ) f (x1 )
y
=
x
x2 x1
is called the average of the change of y with respect to x over the interval [x1 , x2 ].
We say
f (x2 ) f (x1 )
y
= lim
.
x
x
x0 x
x2 x1
2
1

instantaneous rate of change = lim

The derivative f (a) is the instantaneous rate of change of y = f (x) with respect to
x when x = a.




Velocity of an object: the rate of change of displacement with respect to time.


Marginal cost (): the rate of change of production cost with respect to
the number of items produced.






Interest (in economics): the rate of change of the debt with respect to time.
Power (in physics, ): the rate of change of work with respect to time.
Rate of reaction (in chemistry): the rate of change in the concentration ()
of a reactant with respect to time.
Rate of change of the population of a colony of bacteria with respect to time.
(biology)

2.7-3

2.8

The Derivative as a Function (page 154)

Definition 1 (page 154). The derivative of f is


f (x + h) f (x)
.
h0
h

f (x) = lim

The domain of f is the set {x|f (x) exists}.

Example 2. Let f (x) = x3 . Find f (x).


Solution.

Other Notations
If we use the traditional notation y = f (x) to indicate that the independent variable
is x and the dependent variable is y, then some common alternative notations for
the derivative are as follows:
f (x) = y =

df
d
dy
=
=
f (x) = Df (x) = Dx f (x).
dx
dx
dx

d
are called differentiation operators because thy indicate the
The symbols D and dx
operation of differentiation.

We use the notation



dy
dx x=a

to indicate the value of a variable

dy
dx

or

dy
dx

x=a

at a specific number a, which is a synonym for

f (a).
Definition 3 (page 157). A function f is differentiable at a if f (a) exists. It is
differentiable on an open interval (a, b) [or (a, ) or (, a) or (, )] if it is
differentiable at every number in the interval.
Theorem 4 (page 158). If f is differentiable at a, then f is continuous at a.

 The converse of Theorem 4 is false. For instance, f (x) = |x|.


 ,

2.8-1

How Can a Function Fail to Be Differentiable?


(1) corner or kink: the graph of f has no tangent at this point and f is not
differentiable there.
(2) discontinuity: f is not continuous at a, then f is not differentiable at a.
(3) vertical tangent line: f is continuous at a and lim |f (x)| = .
xa

Figure 1: Three ways for f not to be differentiable at a.

Higher Derivatives
If f is differentiable function, then its derivative f is also a function, so f may have
a derivative of its own, denoted by (f ) = f . This new function f is called the
second derivativeof f . We write the second derivative of y = f (x) as
 
d2 y
d dy
= 2.
dx dx
dx

acceleration: the instantaneous rate of change of velocity with respect to time.

The third derivative f is the derivative of the second derivative: f = (f ) . If


y = f (x), then alternative notations for the third derivative are


d3 y
d d2 y

=
.
y = f (x) =
dx dx2
dx3
In general, the n-th derivative (n 4) of f is denoted by f (n) . If y = f (x), we write
y n = f (n) (x) =

jerk: the rate of change of acceleration.

2.8-2

dn y
.
dxn

Example 5. Let f (x) = x|x|. Find f (x) and f (x).


Solution.


 C k (R) k
Example 6. Suppose
f (x) =

1cos x
sin x

x>0
.
ax + b x 0

Find a and b such that f is continuous and differentiable at x = 0.

(89 )

Solution.

Homework. Suppose
f (x) =

x2

x1

ax + b x > 1

Find a and b such that f is continuous and differentiable at x = 1.


Homework.
2

(a) If f (x) = x 3 , show that f (0) does not exist.


(b) If a 6= 0, find f (a).
2

(c) Show that y = x 3 has a vertical tangent line at (0, 0).


2.8-3

(92 )

Chapter 3
3.1

Differentiation Rules

Derivatives of Polynomials and Exponential


Functions (page 174)

Property 1 (Derivative of a Constant Function, page 174).


d
(c) = 0.
dx
Proof. Let f (x) = c the constant function. Then from the definition of a derivative,
we have
f (x) =

Property 2 (The Power Rule, page 175176). If n is any real number, then
d n
(x ) = nxn1 .
dx
Proof. Let f (x) = xn . Here we check the case n Z and show the general case in
, we compute
Section 3.6. First, for n N, by the
f (x) =
=
Next, we check the case negative integer n:
f (x) =
=




an bn = (a b)(an1 + an2 b + + abn2 + bn1 ).


1

Application: a b = (a n b n )(a

n1
n

+a

n2
n

Homework. Show that the derivative of f (x) =

3.1-1

bn + + an b

n2
n

+b

x = x 2 is f (x) =

n1
n

).

1 1
2 x

= 12 x 2 .

Property 3 (The Constant Multiple Rule, page 177). If c is a constant and f is a


differential function, then
d
d
(c(f (x))) = c f (x).
dx
dx
Proof. Let g(x) = cf (x). Then
g (x) =
=

Property 4 (The Sum and Difference Rule, page 177178). If f and g are both
differentiable, then
d
d
d
(f (x) g(x)) =
f (x)
g(x).
dx
dx
dx
Proof. Let F (x) = f (x) + g(x). Then
F (x) =
=
=

Homework. Find the derivative of f (x) = x4 +

4
x

x + 2.

Example 5. Compute the derivative of the exponential function f (x) = ax .


Solution.
f (x) =
=
Definition 6 (the Number e, page 180). e is the number such that
eh 1
= 1.
h0
h
lim

The number e 2.71828.

Property 7 (Derivative of the Natural Exponential Function, page 180).


d x
(e ) = ex .
dx
3.1-2

3.2

The Product and Quotient Rules (page 184)

Property 1 (The Product Rule, page 185). If f an g are both differentiable, then
d
d
d
(f (x)g(x)) = g(x) f (x) + f (x) g(x).
dx
dx
dx
Proof. Let F (x) = f (x)g(x), then
F (x + h) F (x)
=
h0
h

F (x) = lim
=
=
=
=





(f (x)g(x)) 6= f (x)g (x).


.

(f (x)g(x)h(x)) =
Leibniz Rule: (f g)(n) (x) =

n
P

Ckn f (nk) (x)g (k) (x).

k=0

Property 2 (The Quotient Rule). If f an g are both differentiable, then


d
dx
Proof. Let F (x) =

f (x)
.
g(x)

f (x)
g(x)

d
d
f (x) f (x) dx
g(x)
g(x) dx
.
(g(x))2

Then

3.2-1

Example 3. Compute
Solution.

d2
dx2

Example 4. The curve y =

f (x)
g(x)

1
1+x2

is called a witch of Maria Agnesi. Find an equation

of the tangent line to this curve at the point (1, 21 ).


Solution.

x
Homework. The curve y = 1+x
2 is called a witch of Maria Agnesi. Find an equation
3
).
of the tangent line to this curve at the point (3, 10

Homework. If f is a differentiable function, find an expression for


(a) y =

x2
f (x)

(b) y =

3.2-2

1 + xf (x)

.
x

dy
.
dx

3.3

Derivatives of Trigonometric Functions (page


191)

Recall some useful limits:


sin
= 1,
0
lim

cos 1
1
cos 1
=

lim
= 0.
0
0
2
2

lim

Example 1. Calculate the derivative of f () = sin and g() = cos .


Solution. We compute
f () =
=
g () =
=
Example 2. Calculate the derivative of tan , cos , sec , csc .
Solution.
d
tan =
d
=
d
cot =
d
d
sec =
d
d
csc =
d

Example 3.
(a) (sin2 ) = (sin sin ) =
(b) (cos2 ) = (cos cos ) =

sin2 + cos2 = 1 (sin2 + cos2 ) = 0.

Homework.
(a) (tan2 ) = (tan tan ) =
3.3-1

(b) (sec2 ) = (sec sec ) =


Example 4. Compute

dn
d n

sin and

dn
d n

cos .

Homework. If f (x) = ex cos x, find f (x) and f (x).

3.3-2

Chapter 4
4.1

Applications of Differentiation

Maximum and Minimum Values (page 274)

Definition 1 (page 274). Let c be a number in the domain D of a function f . Then


f (c) is the
absolute maximum value of f on D if f (c) f (x) for all x in D.
absolute minimum value of f on D if f (c) f (x) for all x in D.

An absolute maximum (or minimum) is sometimes called a global maximum


(or minimum).

The maximum an minimum values of f are called extreme values of f .

Figure 1: Absolute maximum value and absolute minimum value of f .

The importance of for all.

Definition 2 (page 274). The number f (c) is a


local maximum value of f on D if f (c) f (x) when x is near c.
local minimum value of f on D if f (c) f (x) when x is near c.
Remark 3. We say that something is true near c, we mean that it is true on some
open interval containing c.

Figure 2: Local maximum value and local minimum value of f .

The importance of near.


4.1-1

Example 4. State the absolute (and local) maximum (and minimum) values of the
function y = f (x).
y

y = f (x)

1
x

Figure 3: Find absolute (and local) maximum (and minimum) values of the function.
Solution.
Absolute maximum:
Local maximum:
Absolute minimum:
Local minimum:
Theorem 5 (The Extreme Value Theorem, page 275). If f is continuous on a closed
interval [a, b], then f attains an absolute maximum value f (c) and an absolute value
f (d) at some numbers c and d in [a, b].





The importance of continuous.


The importance of closed interval.
The importance of an and some.

Example 6. Give examples that if f is not continuous, or f is continuous on (a, b),


the Extreme Value Theorem does not hold. Give a continuous function that it
attains maximum values and minimum values at more than one numbers.
Solution.

4.1-2

Theorem 7 (Fermats Theorem, page 276). If f has a local maximum or minimum


at c, and if f (c) exists, then f (c) = 0.




The condition f (c) exists is important. For example,


The converse of Fermats Theorem is false in general. For example,

Proof. Here we prove the local maximum case. Since f (c) f (x) if x is sufficiently
close to c, this implies that if h is sufficiently close to 0, with h being positive or
negative, then

If h > 0, we have

f (c+h)f (c)
h

0. Since f (c) exists, we get

f (c + h) f (c)
=
h0
h

f (c) = lim
If h < 0, we have

f (c+h)f (c)
h

0. Since f (c) exists, we get

f (c + h) f (c)
=
h0
h

f (c) = lim
Hence f (c) = 0.

Definition 8 (page 277). A critical number of a function f is a number c in the


domain of f such that f (c) = 0 or f (c) does not exist.

All critical numbers are in the domain of f .


3

Example 9 (page 278). Find the critical numbers of f (x) = x 5 (4 x).


Solution. We compute
f (x) =
Therefore the critical numbers are
Homework. Prove that the function f (x) = x101 + x51 + x + 1 has neither a local
maximum nor a local minimum.
Theorem 10 (page 278). If f has a local maximum or minimum at c, then c is a
critical number of f .

4.1-3

The Closed Interval Method (page 278). To find the absolute maximum and
minimum values of a piecewise continuous function on a closed interval [a, b]:
(1) Find the values of f at the critical numbers of f in (a, b).
(2) Find the values of f at the endpoints of the interval, that is, f (a) and f (b).
(3) The largest and smallest of the values from (1) and (2) are absolute maximum
value and absolute minimum value, respectively.
Homework. Find the absolute maximum and absolute minimum values of f (x) =
2x3 3x2 12x + 1 on [2, 3].

4.1-4

4.2

The Mean Value Theorem (MVT) (page 284)

Question 1. A highway from Taipei to Kaohsiung is 330 km and the speed limit
is 110 km/h. Man A drove the car on the high way from Taipei at 9 : 00 AM to
Kaohsiung at 11 : 59 AM. Did he exceed the speed limit?
Theorem 2 (Rolles Theorem, page 284). Let f be a function that satisfies the
following three hypotheses:
(1) f is continuous on the closed interval [a, b].
(2) f is differentiable on the open interval (a, b).
(3) f (a) = f (b).
Then there is a number c in (a, b) such that f (c) = 0.

Figure 1: Rolles Theorem.


Proof. There are three cases.
(I) f (x) = k, a constant. We have f (x) = 0, so the number c can be taken to be
any number in (a, b).
(II) f (x) > f (a) for some x in (a, b). By the

, f has a max-

imum somewhere in [a, b]. Since f (a) = f (b), it must attain this maximum
value at a number c in the open interval (a, b). Then f has a
at c, and f is differentiable at c. By
(III) f (x) < f (a) for some x in (a, b). By the

, we know f (c) = 0.
, f has a min-

imum value in [a, b], and since f (a) = f (b), it attains this local minimum value
, f (c) = 0.
at a number c (a, b). By

4.2-1





The importance of continuous on [a, b].


The importance of differentiable.
The Rolles Theorem only says the existence.

Example 3. Give examples that each condition in Rolles Theorem is required.


Solution.

Example 4 (page 284). Prove that the equation x3 + x 1 = 0 has exactly one
real root.
Solution.

Homework. Let f (x) = a(x3 + x 2) + b(x3 + 3x + 4), ab > 0. Show that f (x) = 0
has exactly one real solution.
(98 )
Theorem 5 (The Mean Value Theorem (MVT), page 285). Let f be a function that
satisfies the following hypotheses:
(1) f is continuous on the closed interval [a, b].
(2) f is differentiable on the open interval (a, b).
Then there is a number c in (a, b) such that
f (c) =

f (b) f (a)
ba

or, equivalently,
f (b) f (a) = f (c)(b a).
4.2-2

Figure 2: The Mean Value Theorem.


Proof. Define a new function
h(x) =
We will verify that h(x) satisfies the three hypotheses of Rolles Theorem.
(1) The function h is continuous on [a, b]: It is the sum of f and a first-degree
polynomial, both of which are continuous.
(2) The function h is differentiable on (a, b): Both f and the first-degree polynomial are differentiable. In fact, we have
h (x) =
(3) h(a) = h(b) = 0:
h(a) =
h(b) =
By




, there is a number c (a, b) such that h (c) = 0. Therefore,

The importance of continuity, differentiable, and existence.


Why this theorem is called the Mean Value Theorem?

4.2-3

Example 6. Suppose that f (0) = 3 and f (x) 5 for all values of x. How large
can f (2) possibly be?
Solution.

Theorem 7. If f (x) = 0 for all x in an interval (a, b), then f is constant on (a, b).
Proof. Let x1 and x2 be any two numbers in (a, b) with x1 < x2 . Since f is differentiable on (a, b), it must be differentiable on (x1 , x2 ) and continuous on [x1 , x2 ]. By
to f on the interval [x1 , x2 ], we get a number c
applying the
such that x1 < c < x2 and

Therefore f has the same value at any two numbers x1 and x2 in (a, b). So f (x) is
constant on (a, b).
Corollary 8. If f (x) = g (x) for all x in an interval (a, b), then f g is constant
on (a, b); that is, f (x) = g(x) + c where c is a constant.
Proof. Let F (x) =

. Then

for all x (a, b). Thus, F (x) is constant; that is, f g is constant.

f (x) =

x
.
|x|

Example 9. Show that

for any x, y (, ).


y |x y|
x

tan tan
2
2
2

(89,101 )

Solution. If x = y, the inequality holds. If x 6= y, without loss of generality, we


assume < x < y < . Consider the function f (t) = tan 2t , then
f (t) is
f (t) is
4.2-4

By the
, there is a number c (x, y) such that f (x) f (y) =

, we
f (c)(x y), which implies |f (x) f (y)| = |f (c)||x y|. Since f (t) =
. So |f (x) f (y)| 12 |x y|, which means
have |f (c)| =

x
y |x y|

.
tan tan
2
2
2

Homework. Use the Mean Value Theorem to prove the inequality


| sin a sin b| |a b| for all a and b.
Homework. Show that
ln(1 + x)
1
<
<1
1+x
x
(99 )

for x > 0.

Example 10. A number a is called a fixed point of a function f (x) if f (a) = a.


Prove that if f (x) is differentiable and f (x) 6= 1 for all real number x, then f has
at most one fixed point.
(97 )
Solution.

Solution 2.

Theorem 11 (Cauchys Mean Value Theorem, Appendix F, A45). Suppose that the
functions f and g are continuous on [a, b] and differentiable on (a, b), and g (x) 6= 0
for all x in (a, b). Then there is a number c (a, b) such that
f (c)
f (b) f (a)
=
.

g (c)
g(b) g(a)
4.2-5

Proof. The key point is to find a new function F (x) and apply the Mean Value
Theorem.
F (x) =

Example 12. Suppose f (x) is continuous on [a, b], differentiable on (a, b), and
f (a) = f (b) = 0. Show that for any real number , there exists (a, b) such that
f () + f () = 0.
Solution.

Homework. Suppose both f (x), g(x) are continuous on [a, b], and differentiable on
(a, b). Suppose g (x) 6= 0 for any x (a, b). Show that there exists (a, b) such
that
f () f (a)
f ()
=
.
g ()
g(b) g()

Appendix
Example 13. Evaluate


3
3
lim sin((x + 2) 4 ) sin(x 4 ) .

Solution.

4.2-6

(95 )

4.3

How Derivatives Affect the Shape of a Graph


(page 290)

Increasing/Decreasing Test (page 290).


(a) If f (x) > 0 on an interval, then f is increasing on that interval.
(b) If f (x) < 0 on an interval, then f is decreasing on that interval.
Proof.
, there is c (x1 , x2 ) such that

(a) Let x1 < x2 . By the

.
, there is c (x1 , x2 ) such that

(b) Let x1 < x2 . By the

Example 1. Find where the function f (x) = 3x4 4x3 12x2 + 5 is increasing and
where it is decreasing.
Solution. We compute
f (x) =
Solutions of f (x) = 0 are

.
. Hence

f (x) is increasing on

.
.

f (x) is decreasing on

The First Derivative Test (page 291). Suppose that c is a critical number of a
continuous function f .
(a) If f changes from positive to negative at c, then f has a local maximum at c.
(b) If f changes from negative to positive at c, then f has a local minimum at c.
(c) If f does not change sign at c (for example, if f is positive on both side of c
or negative on both sides), then f has no local maximum or minimum at c.

4.3-1

Figure 1: The First Derivative Test.


Example 2. Find the local minimum and maximum values of the function f in
Example 1.
Solution.
x

27

f
Hence
f has local maximum

f has local minimum

Definition 3 (page 293). If the graph f lies above all of it tangents on an interval
I, then it is called concave upward on I. If the graph f lies below all of it tangents
on an interval I, then it is called concave downward on I.

Some textbooks or papers use convex instead of concave upward.

Figure 2: Concave upward and concave downward.


Concavity Test (page 293).
(a) If f (x) > 0 for all x in I, then the graph of f is concave upward ()
on I.
(b) If f (x) < 0 for all x in I, then the graph of f is concave downward (
) on I.
4.3-2

Definition 4 (page 294). A point P on a curve y = f (x) is called an inflection


point () if f is continuous there and the curve changes from concave upward
to concave downward or from concave downward to concave upward at P .

Figure 3: Inflection points.

Example 5. Find the concave upward, concave downward intervals, and inflection
points of the function f in Example 1. Sketch the graph of f .
Solution. We compute
f (x) =

So
x
f
f
f

31

32
9

27

The points of inflections are


f is concave upward on
f is concave downward on

.
.
.

Figure 4: The graph of f (x) = 3x4 4x3 12x2 + 5.

4.3-3

Homework. Suppose a cubic function f (x) has the coefficient of leading order 1,
and the graph of f (x) passes through (0, 4). The graph of f (x) is decreasing on
(1, 3) and decreasing on (, 1) and (3, ). Find f (x) and its inflection point. (96

)
The Second Derivative Test (page 295). Suppose f is continuous near c.
(a) If f (c) = 0 and f (c) > 0, then f has a local minimum at c.
(b) If f (c) = 0 and f (c) < 0, then f has a local maximum at c.
Example 6. Show that f (x) =

sin x
x

is a decreasing function on (0, 2 ).

(89

Section 2.3 | sin x| |x|

Solution.

Homework. Let f (x) =

tan x
.
x

Show that when 0 < x1 < x2 < 2 , f (x1 ) < f (x2 ).

(93 )
Example (TA) 7.
(a) Show that the function f (x) = xx is strictly increasing on (e1 , ).
(b) If f is the inverse function of f of part (a), find lim
y

4.3-4

g(y) ln ln(y)
.
ln y

(100 )

Appendix
Example 8. Classify all cubic functions f (x) = ax3 + bx2 + cx + d.
Solution.

4.3-5

4.4

Indeterminate Forms and lHospitals Rule


(page 301)

In this section, we want to introduce new method to deal with the limit such as
ln x
,
x x 1
lim

or

ex
.
x x2
lim

Definition 1 (page 301302).


(a) If we have a limit of the form
f (x)
,
xa g(x)
lim

where both f (x) 0 and g(x) 0 as x a, it is called an indeterminate


form of type 00 .
(b) If we have a limit of the form
f (x)
,
xa g(x)
lim

where both f (x) (or ) and g(x) (or ) as x a, it is called

an indeterminate form () of type


.
L Hospitals Rule (page 302). Suppose f and g are differentiable and g (x) 6= 0
on an open interval I that contains a (except possibly at a). Suppose that
lim f (x) = 0

and

lim f (x) =

and

xa

lim g(x) = 0

xa

or that
xa

lim g(x) =

xa

0
0

(In other words, we have an indeterminate form of type

or

.)

Then

f (x)
f (x)
= lim
lim
xa g (x)
xa g(x)
if the limit on the right side exists (or is or ).
(x)

 : (1) ; (2) xa
lim fg (x)

L
(x) L
(x) L
(x)
 xa
lim fg(x)
= M.
= lim fg (x)
= = lim fg (x)
xa
xa

4.4-1

(k)

(k)

Example 2.
(a) Find lim+
t0

tln(1+t)
.
t2

(b) Use (a) to find lim+


t0

tln(1+t)
.
t

(102 )

Solution.

x(ex +1)2(ex 1)
.
x3
x0

(96 )

Homework. Find the limit lim

Indeterminate Products, page 305


Definition 3 (page 305). If we have a limit of the form
lim f (x)g(x),

xa

where lim f (x) = 0 and lim g(x) = (or ) as x a, it is called an indetermixa


xa
nate form of type 0 .
We can deal with it by writing the product f g as a quotient:
fg =

f
1/g

or f g =

g
,
1/f

and this converts the given limit into an indeterminate form of type

0
0

or

Example 4 (page 305). Evaluate lim+ x ln x.


x0

Solution.

Homework. Let f (x) =

cot 2 x
,0
ln x

< x < 1. Find lim f (x) and lim f (x). (100


x1

4.4-2

xa

Indeterminate Differences, page 305


Definition 5 (page 305). If lim f (x) = and lim g(x) = , then the limit
xa

xa

lim (f (x) g(x))

xa

is called an indeterminate form of type .


We can try to convert the difference into a quotient (for instance, by using a
common denominator (), or rationalization (), or factoring out a common
factor ()) so that we have an indeterminate form of type
Example 6. Find the limit lim

x0

1
x2

Solution.

Homework. Compute

1
sin2 x


.

0
0

or

(95 )

lim (sec x tan x).

x( 2 )

Indeterminate Powers, page 306


Definition 7 (page 306). Several indeterminate forms arise from the limit
lim (f (x))g(x)

xa

(1) lim f (x) = 0 and lim g(x) = 0: type 00 .


xa

xa

(2) lim f (x) = and lim g(x) = 0: type 0 .


xa

xa

(3) lim f (x) = 1 and lim g(x) = : type 1 .


xa

xa

Each of these three cases can be treated either by taking the natural logarithm:
let y = (f (x))g(x) , then ln y = g(x) ln f (x) or by writing the function as an exponential: (f (x))g(x) = eg(x) ln f (x) . In either method we are led to the indeterminate
product g(x) ln f (x), which is of type 0 .

4.4-3

Example 8 (page 307). Find lim+ xx .


x0

Solution.

Homework. Find the limit lim

x0

1+tan x
1+sin x

 13
x

Homework. Evaluate the limit lim 1 +


x

Homework. Find the limit lim

x0

tan x
x

. (, lHospital Rule )
(96 )
3
x

 12
x

4.4-4


5 x
.
2
x

(97 )
(100 )

lHospital Rule
f (x)

xa g (x)

: (1) ; (2) lim

lim x
x0 1+sin x
lim xsin x
x x+sin x
; sin x, cos x (as x ) sin x1 , cos x1 , x1 , ln1x (as x 0)
x2 sin x1
x0 sin x

lim

lim x ln x

x0+

, ,
lim 1 1
x0 x2 e x
lim tan xx
x0 xsin x
LHospital Rule ,
(sin x)ex
(x+sin
x)e2x
x

lim

, Squeeze Theorem, definition of derivative,


()
lim x sin x1

x0

lim

x0

sin x
x

lim

sin x
x

lim

ln(1+x)
x

x0

4.4-5

Proof of lHospitals Rule (Appendix A46)


f (x)

xa g (x)

We are assuming that lim f (x) = 0 and lim g(x) = 0. Let lim
xa

F (x) =

xa

f (x) if x 6= a
,
0
if x = 1

G(x) =

= L. Define

g(x) if x 6= a
.
0
if x = 1

Then both F and G are continuous on I since f and g are continuous on {x I|x 6=
a} and
lim F (x) = lim f (x) = 0 = F (a),

xa

xa

lim G(x) = lim g(x) = 0 = G(a).

xa

xa

Furthermore, F and G are differentiable on (a, x) (or (x, a)) since F = f and G =
g . Since G 6= 0, by the Cauchys Mean Value Theorem, there is a number y such
that a < y < x (or x < y < a) and
F (y)
F (x) F (a)
F (x)
=
=
.

G (y)
G(x) G(a)
G(x)
Hence
F (x)
F (y)
f (y)
f (x)
= lim+
= lim+
= lim+
= L,
xa G(x)
ya G (y)
ya g (y)
xa g(x)

f (x)
F (x)
F (y)
f (y)
lim
= lim
= lim
= lim
= L.
xa g(x)
xa G(x)
ya G (y)
ya g (y)
lim+

and

Therefore,
f (x)
= L.
xa g(x)
lim

This proves lHospitals Rule for the case where a is finite.


If a is infinite, we let t = x1 . Then t 0+ as x , so we have

f ( 1t ) t12
f ( 1t ) L
f ( 1t )
f (x)
f (x)

=
lim
lim
= lim+ 1 = lim+ 1
.
=
lim
1
x g (x)
x g(x)
t0 g ( ) 12
t0 g( )
t0+ g ( )
t
t
t
t

4.4-6

4.5

Summary of Curve Sketching (page 311)

Guidelines for sketching a curve


1. Domain: the set of x for which f (x) is defined.
2. Intercepts: y-intercept f (0), x-intercepts: let y = 0 and solve for x.
3. Symmetry: even function, odd function, periodic function.
4. Asymptotes: horizontal asymptotes, vertical asymptotes, slant asymptotes
().
5. Intervals of increase or decrease: use the Increasing/Decreasing test.
6. Local maximum and minimum values: find the critical numbers of f
(f (c) = 0 or f (c) does not exist.)
7. Concavity and points of inflection: compute f (x) and use the Concavity
Test.
8. Sketch the Curve: use the information in items 17, draw the graph.

DISAFMSG

Definition 1 (page 315). If


lim (f (x) (mx + b)) = 0,
x

where m 6= 0, then the line y = mx + b is called a slant asymptote.


Example 1 (page 312). Sketch the curve y =

2x2
.
x2 1

Example 2 (page 313). Sketch the curve y =

2
x
.
x+1

Example 3 (page 314). Sketch the curve y = xex .


Example 4 (page 314). Sketch the curve y =

cos x
.
2+sin x

Example 5 (page 315). Sketch the curve y = ln(4 x2 ).


Example 6 (page 316). Sketch the curve y =

x3
.
x+1

Homework. Sketch the curve y = x 3 (2x + 5).

(89 )

Homework. Sketch the curve f (x) = x4 + 23 x3 8x2 8x + 13.

(91 )

4.5-1

Homework. Sketch the curve y =

x3 2x2 +x1
.
x2

(92 )

Homework. Sketch the curve y =

x
.
x2 +9

(93 )

(94 )

(95 )

Homework. Sketch the curve y = x 3 (x 4).


Homework. Sketch the curve y = x 3 ( 52 x).
1

Homework. Sketch the curve y = x 3 (x 3) 3 .

(96 )

Homework. Sketch the curve y = 20x3 3x5 .

(97 )

Homework. Sketch the curve y =

2
x +1
x2 4

for x < 2 and x > 2..

Homework. Sketch the curve y = (2x2 + 3x)ex .

(99 )
2

Homework. Sketch the curve y = (x 1) 3 (x + 1) 3 .


Homework. Sketch the curve y =

(x+1)2
.
x2 +1

Homework. Sketch the curve y = x

x2
6

(98 )

(100 )
(101 )

2 ln x
,
3

x > 0..

(102 )

Homework. Sketch the curve y =

x2 2x+4
.
x2

(89 )

Homework. Sketch the curve y =

(x+1)3
.
(x1)2

(93 )

Homework. Sketch the curve y = x 3 (x 2)2 .

(94 )

Homework. Sketch the curve y =

(x2)2
.
x+1

(95 )

Homework. Sketch the curve y =

x3 x1
.
x2

(96 )

Homework. Sketch the curve y = sin 2x + 4 sin x x, where x [ 2 , 3


].
2
(97 )
Homework. Sketch the curve y =

x2 (x2)
.
(x+1)2

(98 )

Homework. Sketch the curve


y = f (x) =

|x|x if x 6= 0
1

if x = 0.
(99 )
1

(100 )

Homework. Sketch the curve y = (x + 2)e x .


Homework. Sketch the curve y =

3x 3
.
x1

(101 )
5

Homework. Sketch the curve y = (x 1) 3 (x2 1) 3 .


4.5-2

(102 )

Example 1 (page 312). Sketch the curve y =

2x2
.
x2 1

Solution.
A. The domain is

B. The x- and y-intercept are both .


C. Since

, the function f is

D. Since
2x2
=
x x2 1
lim

the line

is a

. The denominator is 0 when

we compute the following limits:


2x2
=
x1 x2 1
2x2
=
lim + 2
x1 x 1

2x2
=
x1 x2 1
2x2
lim 2
=
x1 x 1

lim+

Therefore the lines

and

lim

are vertical asymptotes.

E. Direct computation gives


y =

Since f (x) > 0 when

and f (x) < 0 when

f is increasing on
F. The only critical number is
at 0, f (0) = 0 is a

and decreasing on

. Since f changes from positive to negative


by the First Derivative Test.

G. Direct computation gives


f (x) =

We know f (x) > 0 on


and f (x) < 0 on
cave upward on the interval
. It has no point of inflection since

. Thus the curve is conand concave downward on


.

H. Using this information to sketch the curve. ()

4.5-3

Example 2 (page 313). Sketch the curve y =

2
x
.
x+1

Solution.
A. The domain is

B. The x- and y-intercept are both .


C. Symmetry: None.
D. Since
lim
x

x2
=
x+1

there is no horizontal asymptote. Since


lim +

x1

the line

x2
=
x+1

is a vertical asymptotes.

E. Direct computation gives


y =

, so the only critical number is . Since


We see that f (x) = 0 when
and f (x) < 0 when
, f is increasing on
f (x) > 0 when
and decreasing on

F. Since f (0) = 0 and f changes from negative to positive at 0, f (0) = 0 is a


by the First Derivative Test.
G. Direct computation gives
f (x) =

Since the numerator is always


, We know f (x) > 0 for all x in the
and there is no
domain of f , which means f is concave upward on
point of inflection.
H. Using this information to sketch the curve. ()

4.5-4

Example 3 (page 314). Sketch the curve y = xex .


Solution.
A. The domain is

B. The x- and y-intercept are both .


C. Symmetry: None.
D. Since
lim xex =

there is no horizontal asymptote. By the lHospital Rule, we have


lim xex = lim
x

x
ex

is a horizontal asymptote.

so the

E. Direct computation gives


y =
Since f (x) > 0 when
and decreasing on

.
and f (x) < 0 when
.

, f is increasing on

F. Since f (1) = 0 and f changes from negative to positive at x = 1, f (1) =


e1 is a
by the First Derivative Test.
G. Direct computation gives
f (x) =

Since f (x) > 0 if


and f (x) < 0 if
and concave downward on
on

, f is concave upward
. The inflection point is

.
H. Using this information to sketch the curve. ()

4.5-5

Example 4 (page 314). Sketch the curve y =

cos x
.
2+sin x

Solution.
A. The domain is

.
and y-intercept is

B. The x-intercepts are

C. Symmetry: f is neither even nor odd. Since f (x + 2) = f (x) for all x, f


is
and has period
. Thus, the following steps we only consider
0 x 2 and then extend the curve by translation.
D. Asymptotes: None.
E. Direct computation gives
y =

Thus f (x) > 0 when


increasing on

. So f is
and decreasing on

F. From part E and First Derivative Test, we see that the local minimum value
is

and local maximum value is

G. Direct computation gives


f (x) =
Since f (x) > 0 if
downward on

, f is concave upward on
. The inflection point is

H. Using this information to sketch the curve.

4.5-6

and concave
.

Example 5 (page 315). Sketch the curve y = ln(4 x2 ).


Solution.
A. The domain is

B. The y-intercept is f (0) = ln 4. To find the x-intercept, we set ln(4 x2 ) = 0,


. Therefore the x-intercepts are
.
so we have
C. Since f (x) = f (x), f is

and the curve is symmetric about the

D. Since
lim ln(4 x2 ) =

x2+

lim ln(4 x2 ) =

x2

are vertical asymptotes.

the lines
E. Direct computation gives

y =

and f (x) < 0 when


Since f (x) > 0 when
and decreasing on
.
ing on
F. The only critical number is
at 0, f (0) = ln 4 is a

, f is increas-

. Since f changes from positive to negative


by the First Derivative

Test.
G. Direct computation gives
f (x) =

Since f (x) < 0 for all x, the curve is


no inflection point.
H. Using this information to sketch the curve.

4.5-7

on

and has

Example 6 (page 316). Sketch the curve y =

x3
.
x+1

Solution.
A. The domain is

B. The x- and y-intercept are both .


C. Since

, the function f is

D. Since x2 + 1 is never 0, there is no vertical asymptote. Since f (x) as


x and f (x) as x , there is no horizontal asymptote. Long
division gives
x3
=
x2 + 1
x
=
f (x) x = 2
x +1
f (x) =

is a

So the line

,
.
.

E. Direct computation gives


y =

Since f (x) > 0 when

, f is increasing on

F. Although f (0) = 0, f does not change sign at 0, so there is


or

G. Direct computation gives


f (x) =
Since f (x) = 0 when

.
, we set up the following chart.

The points of inflection are

H. Using this information to sketch the curve.

4.5-8

4.7

Optimization Problems (page 325)

Steps in solving optimization problems


1. Understand the problem: What is the unknown? What are the given
quantities? What are the given conditions?
2. Draw a diagram: In most problems it is useful to draw a diagram and
identify the given and required quantities on the diagram.
3. Introduce notation: Assign a symbol to the quantity that is to be maximized
or minimized (call it Q for now). Also select symbols a, b, c, . . . , x, y for other
known quantities and label the diagram with these symbols.
4. Express Q in terms of some of the other symbols.
5. If Q has been expressed as a function of more than one variable, use the
given information to find relationships among these variables. Then use these
equations to eliminate all but one of the variables. Thus we get Q = f (x).
6. Use the methods of Section 4.1 and 4.3 to find the absolute maximum or
minimum value of f .
Example 1 (Snells Law, , page 335). Let v1 be the velocity of light in
air and v2 the velocity of light in water. According to Fermats Principle, a ray of
light will travel from a point A in the air to a point B in the water by a path ACB
that minimizes the time taken. Show that
v1
sin 1
= ,
sin 2
v2
where 1 (the angle of incidence) and 2 (the angle of refraction) are known. This
equation is known as Snells Law.
Solution.

4.7-1

Example 2 (, page 335). A steel pipe is being carried down a hallway 3m


wide. At the end of the hall there is a right-angled turn into a narrower hallway 2m
wide. What is the length of the longest pipe that can be carried horizontally around
the corner?
(92 )
Solution.

Example 3 (; , page 336). A painting in an art gallery has height h


and is hung so that its lower edge is a distance d above the eye of an observer. How
far from the wall should the observer stand to get the best view? (In other words,
where should the observer stand so as to maximize the angle subtended at his eye
by the painting?)
(96 )
Solution.

4.7-2

Example 4. A right circular cone is inscribed in a sphere of radius r. Find the


largest possible volume of such a cone. In this case, what is the height and radius
of the cone?
(89, 95 )
Solution.

Example 5. The upper right-hand corner of a piece of paper, 30 cm by 20 cm, is


folded over to the bottom edge. How would you fold it so as to minimize the length
of the fold? In other words, how would you choose x to minimize y? (91 )
Solution.

4.7-3

Example 6.
(a) Find the point (denote P ) on the line y =
Q(3, 0).

x that is closest to the point

(b) Show that the line P Q is orthogonal to the tangent line of y =

x at P .

Solution.

Homework. See the figure below.


x
x

x
x
Figure 1: A square piece of cardboard with 30 cm wide.
A box with cover is to be constructed from a square piece of cardboard, 30 cm wide,
by cutting out a square or a rectangle (shaded region) from each of the four corners
an bending up the remaining cardboard (unshaded region) along the dotted lines.
What is the largest volume that such a box can have? Justify that the volume you
obtain actually is the maximum volume.
4.7-4

(97 )

Homework. A cylindrical can is to be made to hold 1 L of oil. Find the dimensions


that will minimize the cost of the metal to manufacture the can. (The can contains
top, bottom, and sides.)
Homework. A cone-shaped drinking cup is made from a circular piece of paper
of radius R by cutting out a sector and joining the edges CA and CB. Fin the
maximum capacity of such a cup.
B

A
R
C

Figure 2: Making a cone-shaped drinking cup.

Homework. What is the smallest possible area of the triangle that is cut off by
the first quadrant and whose hypotenuse is tangent to the parabola y = 1 x2 at
some point?
(94 )

4.7-5

4.9

Antiderivative (page 344)

Definition 1 (page 344). A function F is called an antiderivative () of f


on an interval I if F (x) = f (x) for all x in I.
Theorem 2 (page 344). If F is an antiderivative of f on an interval I, then the
most general antiderivative of f on I is
F (x) + C
where C is an arbitrary constant.
Proof. If F and G are any two antiderivative of f , then F (x) = f (x) = G (x).
Form the corollary of the Mean Value Theorem (Section 4.2 Corollary 8), we know
G(x) F (x) = C, where C is a constant. So G(x) = F (x) + C
This is a table of antidifferentiation formulas. We use the notation F (x) = f (x)
and G (x) = g(x).
Function

Particular antiderivative

Function
2

Particular antiderivative

cf (x)

cF (x)

sec x

tan x

f (x) + g(x)
xn (n 6= 1)

F (x) + G(x)

sec x tan x

xn+1
n+1

sec x
sin1 x

1
x
x

ln |x|

1
1x2
1
1+x2

tan1 x

e
cos x

ex
sin x

cosh x
sinh x

sinh x
cosh x

sin x

cos x

Example 3. Find the most general antiderivative of the function. (Let F (x) is the
antiderivative of the function f (x).)
(1) f (x) = e2 .

F (x) =

(2) f (x) = x(2 x)2 .

F (x) =

(3) f (x) = x x3.14 .

F (x) =

(4) f (x) =

2+x2
.
1+x2

F (x) =

Homework. Find the most general antiderivative of the function


f (x) = sin x

4.9-1

1
1
+ 4.
x x

Example 4. Find f .
(1) f (x) = 2 cos x + sec2 x, 2 < x < 2 , f ( 3 ) = 4.
(2) f (x) = 2et + 3 sin x, f (1) = 0, f (2) = 0.

Differential equations.

Solution.

Homework. Find f , where f (x) = x2 , x > 0, f (1) = 0, f (2) = 0.

4.9-2

5 10

James Stewart: Calculus, Early Transcendentals, Seventh
Edition, International Metric Version, c2012.

Chapter 5
5.1

Integrals

Areas and Distances (page 360)

The Area Problem, page 360


Example 1. Use rectangles to estimate the area under the parabola y = x2 from 0
to 1.
Solution.

Definition 2 (page 365). The area A of the region S that lies under the graph
of the continuous function f is the limit of the sum of the areas of approximating
rectangles:
A = lim Rn = lim (f (x1 )x + f (x2 )x + + f (xn )x).
n

It can also be shown that we get the same value if we use left endpoints:
A = lim Ln = lim (f (x0 )x + f (x1 )x + + f (xn1 )x).
n

5.1-1

In fact, instead of using left endpoints or right endpoints, we could take the height
of the i-th rectangle to be the value of f at anynumber xi in the i-th subinterval
[xi1 , xi ]. We call numbers x1 , x2 , . . . , xn the sample points.
In general, we form lower sums (and upper sums) by choosing the sample points
xi so that f (xi ) is the minimum (and maximum) value of f on the i-th subinterval.
Homework.
(a) Let An be the area of a polygon with n equal sides inscribed in a circle with
radius r. By dividing the polygon into n congruent triangles with central angle
2
,
n

show that
1
An = nr 2 sin
2

2
n

(b) Show that lim An = r 2 .


n

The Distance Problem, page 367


We can find the distance traveled by an object during a certain time period if the
velocity of the object is known at all times.

5.1-2

5.2

The Definite Integral, page 371

Definition of a Definite Integral (page 372). If f is a function defined for a


.
x b, we divide the interval [a, b] into n subintervals of equal with x = ba
n
We let x0 = a, x1 , x2 , . . . , xn = b be the endpoints of these subintervals and we
let x1 , x2 , . . . , xn be any sample points in these subintervals, so xi lies in the i-th
subinterval [xi1 , xi ]. Then the definite integral of f from a to b is
Z

f (x) dx = lim
n
a

n
X

f (xi )x

i=1

provided that this limit exists and gives the same value for all possible choices of
sample points. If it does exist, we say that f is integrable on [a, b].

Figure 1: Definition of a definite integral.


The precise meaning of the limit that defines the integral is as follows:
For every number > 0, there is an integer N such that
Z

n
b

X

f (x) dx
f (xi )x <

a

i=1

for every integer n > N and for every choice of xi in [xi1 , xi ].

There are some notations we should know:

integral sign:
integrand:

f (x) dx

limits of integration:
lower limit:
upper limit:

5.2-1








The procedure of calculating an integral is called integration.


The dx simply indicates that the independent variable is x. (dummy variable)
Rb
The definite integral a f (x) dx is a number; it does not depend on x.
n
P
The sum
f (xi )x is called a Riemann sum.
i=1
Rb
The geometric meaning of a f (x) dx is the net area of y = f (x) from a to b.
In fact, the subinterval widths are not necessary equal width.
Z

f (x) dx =
a

lim

maxxi 0

n
X

f (xi )xi .

i=1

Not all functions are integrable. For example, the Dirichlet function, or f (x) =
on 0 < x 1.

Example 1. Show that f (x) =

1
x

1
x

on 0 < x 1 is not integrable.

Solution.

Homework. Show that the Dirichlet function


(
0 if x is rational
f (x) =
1 if x is irrational
on [0, 1] is not integrable.
Hint: Both rational numbers and irrational numbers are dense in [0, 1].
Theorem 2 (page 373). If f is continuous on [a, b], or if f has only a finite number
of jump discontinuous, then f is integrable on [a, b]; that is, the definite integral
Rb
f (x) dx exists.
a

Theorem 2 is not the general result. An amazing thing is that Riemann function
on [0, 1] is integrable.
5.2-2

If f is integrable on [a, b], then the limit of Riemann sum exists and gives the same
value no matter how we choose the sample points xi . To simplify the calculation of
the integral, we often taken the sample points to be right endpoints. Then xi = xi
and the definition of an integral simplifies as follows.
Theorem 3 (page 374). If f is integrable on [a, b], then
Z
where x =

ba
n

f (x) dx = lim
n
a

n
X

f (xi )x,

i=1

and xi = a + ix.

Example 4 (page 377). Set up an expression for


Solution.

Rx
1

ex dx as a limit of sums.

Example 5. Change the following limits of sums as integrals:




1
1
1
2
lim n
+
++
n
n3 (n + 1)3
(n + (2n 1))3
(89 )
Solution.

5.2-3

Homework. Change the following limits of sums as integrals:


 
n
X
i
.
(a) lim
sin
n
2n
2n
i=1
(b) lim
n

n
X
i=1

(c) lim
n

 2
i
1
1
.
n n

1
+
n3

1
(e) lim
n n

(93 )

2
++
n3

n
(d) lim ln

(93 )

!
n
.
n3

(94 )

n!
.
n
ln

n+1
n
n+1
n

(96, 100 )


ln

n+2
n
n+2
n

++

ln

n+n
n
n+n
n

!

(98 )

Evaluating Integrals
Example 6 (page 377). Evaluate the expression in Example 4.
Solution.

Example 1 in section 5.1 is also an evaluating integral of

The Midpoint Rule

R1
0

x2 dx.

We often choose the sample point xi to be the right endpoint of the i-th subinterval
because it is convenient for computing the limit. But if the purpose is to find an
approximation to an integral, it is usually better to choose xi to be the midpoint of
the interval, which we denote by xi . Any Riemann sum is an approximation to an
integral, but if we use midpoints we get the following approximation.

5.2-4

Midpoint Rule (page 378).


Z b
n
X
f (x) dx
f (
xi )x = (f (
x1 ) + + f (
xn ))x,
a

where x =

ba
n

i=1

and xi = 12 (xi1 + xi ) = midpoint of [xi1 , xi ].

Properties of the Definite Integral


Properties of the Integral (page 379381).
Z b
(1)
c dx = c(b a), where c is any constant.
a

(2)

(f (x) + g(x)) dx =

(3)

cf (x) dx = c

f (x) dx +

g(x) dx.

f (x) dx, where c is any constant.

(f (x) g(x)) dx =

(5)

(4)

f (x) dx

f (x) dx +

f (x) dx =

g(x) dx.
a

f (x) dx.
a

(6) If f (x) 0 for a x b, then

f (x) dx 0.

(7) If f (x) g(x) for a x b, then

f (x) dx

(8) If m f (x) M for a x b, then m(b a)

g(x) dx.
a

f (x) dx M(b a).


a

Example 7 (page 385). If f is continuous on [a, b], show that


Z b
Z b




f
(x)
dx
|f (x)| dx.


a

Solution.

5.2-5

5.3

The Fundamental Theorem of Calculus


(page 386)

The Fundamental Theorem of Calculus, Part 1 (page 388). If f is continuous


on [a, b], then the function g defined by
Z x
g(x) =
f (t) dt a x b
a

is continuous on [a, b] and differentiable on (a, b), and g (x) = f (x).


Proof. For x and x + h in (a, b), we have
g(x + h) g(x) =
=
so for h 6= 0,
g(x + h) g(x)
=
h
Assume that h > 0. Since f is continuous on [x, x + h], the
says that there are u, v [x, x + h] such that f (u) = m and f (v) = M, where m
and M are the absolute minimum and maximum values of f on [x, x + h]. So
Z x+h
mh
f (t) dt Mh
x

Now we let h 0, then u x and v x, so


lim f (u) = f (lim u) = f (x) and
h0

ux

lim f (v) = f (lim v) = f (x)


vx

h0

because f is continuous at x. By the

, we have

g(x + h) g(x)
= f (x).
h0
h

g (x) = lim

If x = a or b, the above discussion can be modified by considering one-sided limit.


Since g is differentiable on [a, b], g is continuous on [a, b].
Remark 1. The Fundamental Theorem of Calculus, Part 1, can be written as
Z x
d
f (t) dt = f (x)
dx a
when f is continuous.
5.3-1

Example 2 (page 389). Find the derivative of the function g(x) =


Solution.

Example 3. Find the derivative of h(x) =

sin x

1 + r 3 dr.

1 + t2 dt.

(98 )

Solution.

d
Example 4. Compute
dx
Solution.

x3
x2

Homework. Let f (x) = e

g(x)

t3

1
dt.
+2

, where g(x) =

)
d
Homework. Compute
dx

x3

(91 )

x
2

t
dt. Find f (2).
1 + t4

(90

(92 )

t sin t dt.

x
tan x

Z
1
d
du.
(93 )
Homework. Compute
dx x
u4 + 1
Z cos
Homework. Let f () =
sin(t2 ) dt. Find f () and f ().
(94 )
0


Z
Z x
x2
x2
t2
Example 5. Let A =
A
e
dx. Compute the limit lim xe
e dt .
x

(101 )

Solution.

5.3-2

Homework. Find the limit lim+


x0

Homework. Find the limit lim


x0

Rx

sin(t2 ) dt
.
x sin(x2 )

R x2
0

t
1+t3
x4

dt

(89 )

(95 )

R1

2
cos x t

dt x2
.
(97 )
x0
x4
The Fundamental Theorem of Calculus, Part 2 (page 391). If f is continuous
on [a, b], then
Z b
f (x) dx = F (b) F (a),
Homework. Find the limit lim

where F (x) is any antiderivative of f (x), that is, a function such that F (x) = f (x).
Rx
Proof. Let g(x) = a f (t) dt. From the Fundamental Theorem of Calculus, Part
1, we know g (x) = f (x), so g(x) is an antiderivative of f . If F is any other
antiderivative of f on [a, b], then F and g differ by a constant:

for a < x < b. Remark that it also holds when x = a and x = b.


We put x = a in the formula of g(x) to get

So
F (b) F (a) =

We often use notation F (x)|ba = F (b) F (a).

The Fundamental Theorem of Calculus (page 393). Suppose f is continuous


on [a, b].
Z x
(1) If g(x) =
f (t) dt, then g (x) = f (x).
a

(2)

f (x) dx = F (b) F (a), where F is any antiderivative of f , that is, F = f .

Homework (page 396). Find a function f and a number a such that


Z x

f (t)
x for all x > 0.
6+
dt
=
2
t2
a
5.3-3

5.4

Indefinite Integrals and the Net Change Theorem (page 397)

Table of Indefinite Integrals (page 398).


Z
Z
Z
Z
Z
cf (x) dx = c f (x) dx
(f (x) + g(x)) dx = f (x) dx + g(x) dx
Z
k dx = kx + C
Z
Z
1
xn+1
n
+ C (n 6= 1)
dx = ln |x| + C.
x dx =
n+1
x
Z
Z
ax
x
x
+ C.
e dx = e + C
ax dx =
ln a
Z
Z
sin x dx = cos x + C
cos x dx = sin x + C.
Z
Z
2
sec x dx = tan x + C
csc2 x dx = cot x + C.
Z
Z
sec x tan x dx = sec x + C
csc x cot x dx = csc x + C.
Z
Z
1
1
1

dx
=
tan
x
+
C
dx = sin1 x + C.
2
x2 + 1
Z
Z 1x
sinh x dx = cosh x + C
cosh x dx = sinh x + C
We adopt the convention that when a formula for a general indefinite integral is
given, it is valid only on an interval. For example, the general antiderivative of the
function f (x) = x1 , x 6= 0 is
(
ln |x| + C1 if x > 0
F (x) =
.
ln |x| + C2 if x < 0

Rb
R
A definite integral a f (x) dx is a number; an indefinite integral f (x) dx is a
function (family of functions).

Applications
Recall that the Fundamental Theorem of Calculus, part 2:
The Fundamental Theorem of Calculus, Part 2 (page 391). If f is continuous
on [a, b], then
Z b
f (x) dx = F (b) F (a),
a

where F (x) is any antiderivative of f (x), that is, a function such that F (x) = f (x).
5.4-1

We put f (x) = F (x) into the Theorem and get


Net Change Theorem (page 401). The integral of a rate of change is the net
change:
Z b
F (x) dx = F (b) F (a).
a

This principle can be applied to all of the rates of change in the natural and
social sciences. For example,
If an object moves along a straight line with position function s(t), then its
velocity is v(t) = s (t), so
Z t2
v(t) dt = s(t2 ) s(t1 )
t1

is the net change of position, or displacement, of the particle during the time
period from t1 to t2 .
If we want to calculate the distance the object travels during the time interval,
we have to consider the intervals when v(t) 0 and also the intervals when
v(t) 0. In both cases the distance is computed by integrating |v(t)|, the
speed. Therefore,
Z t2
|v(t)| dt = total distance traveled.
t1

Figure 1: Displacement and distance

The acceleration of the object is a(t) = v (t), so


Z t2
a(t) dt = v(t2 ) v(t1 )
t1

is the change in velocity from time t1 to time t2 .


5.4-2

5.5

The Substitution Rule (page 407)

The Substitution Rule (page 408). If u = g(x) is a differentiable function whose


range is an interval I and f is continuous on I, then
Z
Z
Z

f (g(x))g (x) dx = f (g(x)) dg(x) = f (u) du.


Proof. Suppose F is an antiderivative of f , then we have
d
F (g(x)) = F (g(x))g (x) = f (g(x))g (x).
dx
So F (g(x)) is an antiderivative of f (g(x))g (x). Let u = g(x), then
Z
Z
Z

f (g(x))g (x) dx = F (g(x)) + C = F (u) + C = F (u) du = f (u) du.


The middle formula comes from the definition of differential: dg(x) = g (x) dx.
The Substitution Rule for Definite Integrals (page 411). If g is continuous
on [a, b] and f is continuous on the range of u = g(x), then
Z b
Z g(b)

f (g(x))g (x) dx =
f (u) du.
a

g(a)

Proof. Let F be an antiderivative of f . Then F (g(x)) is an antiderivative of


f (g(x))g (x), by Part 2 of the Fundamental Theorem, we have
Z b
b

f (g(x))g (x) dx = F (g(x)) = F (g(b)) F (g(a)).


a

On the other hand, for the right hand side of the equation, we have
Z g(b)
g(b)

= F (g(b)) F ((a)).
f (u) du = F (u)
g(a)

g(a)

Example 1. Compute the integral

x7
dx.
x4 + 1

Solution.

5.5-1

(93 )

Homework. Compute the integral

64

Homework. Compute the integral

Example 2 (page 410). Calculate


Solution.

Example 3. Find

x
dx.
x+1

4 x+73x

dx.
6
x

(94 )
(97 )

tan x dx.

sec2
d.
1 + tan

(91 )

Solution.

Example (TA) 4. Compute the integral

sec2 tan2 d.

(94 )

Solution.

Homework. Find

sec x tan x

dx.
sec x

(92 )
5.5-2

Example 5. Find

cos(ax + b) dx.

(ln x)k
dx.
x

Solution.

Example 6. Find
Solution.

Example 7. Find the integral

esin x
dx.
sec x

(97 )

Solution.

Homework. Compute the integral

ln 7

(ex + 1) 3

Example 8. Let F (x) =


Solution.

d
Homework. Find
dx

Rx

e2x

dx.

(x t)t sin(t2 ) dt. Find F (x).

(99 )
(89 )

e t (x t)
dt.
t2

Be careful that there is x in the integrand.


Z
2t
d
dt.
Homework (page 414). Find
dx
2t + 3
5.5-3

(100 )

Example 9. Compute the integral

1
2
1
4

Solution.

Homework. Compute the integral

sin1 x
p
dx.
x(1 x)

(96 )

tan1 x
dx.
1 + x2

(96 )

Integrals of Symmetric Functions (page 412). Suppose f is continuous on


[a, a].
Z a
Z a
(a) If f is even, then
f (x) dx = 2
f (x) dx.
0

(b) If f is odd, then


Proof. We compute
Z a
Z
f (x) dx =
a

f (x) dx = 0.
a

f (x) dx +
a

f (x) dx =
0

f (x) dx +

Let u = x, then du = dx and when x = a, u = a. Therefore


Z a
Z u
Z u

f (x) dx =
f (u)(du) =
f (u) du.
0

(a) If f is even, then


Z a
f (x) dx =

, so we get

=
, so we get

(b) If f is odd, then


Z a
f (x) dx =
a

Homework. Find

x2 tan x dx.

5.5-4

f (x) dx,

Chapter 6
6.1

Applications of Integration

Areas Between Curves (page 422)

Theorem 1 (page 422). The area A of the region bounded by the curves y =
f (x), y = g(x), and the lines x = a, x = b, where f and g are continuous and
f (x) g(x) for all x [a, b], is
A = lim
n

n
X

(f (xi )

g(xi ))x

(f (x) g(x)) dx.

i=1

Proof. This is because


A = (area under y = f (x)) (area under y = g(x))
Z b
Z b
Z b
=
f (x) dx
g(x) dx =
(f (x) g(x)) dx.
a

Theorem 2 (page 425). The area between the curves y = f (x) and y = g(x) and
between x = a and x = b is
Z b
A=
|f (x) g(x)| dx.
a

Proof. This is because


|f (x) g(x)| =

f (x) g(x) if f (x) g(x)


.
g(x) f (x) if g(x) f (x)

Example 3 (page 427). Sketch the region enclosed by y = tan x, y = 2 sin x, 3


x

and find its area.

6.1-1

Example 4 (page 426). Find the area enclosed by the line y = x 1 and the
parabola y 2 = 2x + 6.
Solution.

Homework (page 428). The curve with equation y 2 = x2 (x + 3) is called Tschirnhausens cubic. If you graph this curve you will see that part of the curve forms a
loop. Find the area enclosed by the loop.
Example (TA) 5 (page 428). For what values of m do the line y = mx and the
curve y =

x
x2 +1

enclose a region? Find the area of the region.

Solution.

6.1-2

6.2

Volumes (page 430)

Definition 1 (page 431). Let S be a solid that lies between x = a and x = b. If


the cross-sectional area of S in the plane Px , through x and perpendicular to the
x-axis, is A(x), where A is a continuous function, then the volume of S is
V = lim
n

n
X

f (xi )x

A(x) dx.

i=1

Definition 2 (page 435). The solids are obtained by revolving a region about a line
is called solids of revolution.
In general, we calculate the volume of a solid of revolution by the formula
Z b
Z d
V =
A(x) dx or V =
A(y) dy,
a

where
If the cross-section is a disk, then A = (radius)2 .
If the cross-section is a washer, then A = (outer radius)2 (inner radius)2 .
Example 3 (page 431). Show that the volume of a sphere of radius r is V = 34 r 3 .
Solution.

Homework. Show that the volume of a right circular cone with height h and radius
r is V = 31 r 2 h.
Example 4 (page 440). Compute the volume of the solid torus.
Solution.

6.2-1

Example 5 (page 434). Consider the region R enclosed by the curves y = x and
y = x2 .
(a) Find the volume of the solid obtained by rotating the region about the line
y = 2.
(b) Find the volume of the solid obtained by rotating the region about the line
x = 1.
Solution.

2e
, y = 1, x = 1, and x = 4.
Homework. Let be the region enclosed by y = 2x1
(99 )
Find the volume generated by revolving about the line x = 12 .

We now find the volumes of two solids that are not solids of revolution.
Example 6 (page 437). Find the volume of a pyramid whose base is a square with
side L and whose height is h.
Solution.

6.2-2

Homework (page 436). A solid has a circular base of radius 1. Parallel crosssections perpendicular to the base are equilateral triangles. Find the volume of the
solid.
Example 7 (page 437). A wedge is cut out of a circular cylinder of radius 4 by two
planes. One plane is perpendicular to the axis of the cylinder. The other intersects
the first at an angle of 30 along a diameter of the cylinder. Find the volume of the
wedge.
Solution.

Example 8 (page 440). Find the volume common to two circular cylinders, each
with radius r, if the axis of the cylinder intersect at right angles.
Solution.

6.2-3

Example (TA) 9 (page 440). A bowl is shaped like a hemisphere with diameter
30 cm. A heavy ball with diameter 10 cm is placed in the bowl and water is poured
into the bowl to a depth of h centimeters. Find the volume of water in the bowl.
Solution.

6.2-4

6.3

Volumes by Cylindrical Shells (page 441)

Definition 1 (page 431). The volume of the solid obtained by rotating about the
y-axis the region under the curve y = f (x) from a to b is
V = lim
n

n
X
i=1

2
xi f (
xi )x =

2xf (x) dx,

where 0 a < b.

Example 2 (page 444). Find the volume of the solid obtained by rotating about

the y-axis the region bounded by y = sin(x2 ) and y = 0 for 0 x .


Solution.

Example 3. Find the volume of the solid obtained by rotating about x = 1 the
region bounded by y = 6x2 , x = 1, and y = 0.

(94 )

Solution.

 , ,
6.3-1

Homework. The region in the first quadrant enclosed by x = y 2 , x-axis and x = 1


is rotated about the line y = 3. Use the disk method (Section 6.2) and cylindrical
shell method to find the volume of the solid. The answers should be the same.
(98 )
Homework (page 445). Use the method of cylindrical shells to find the volume
generated by rotating the region by x = y 2 + 1, x = 2 about y = 2.

6.3-2

6.5

Average Value of a Function (page 451)

Definition 1 (page 452). We define the average value of f on the interval [a, b] as
Z b
n
1
1 X

fave = lim
f (xi )x =
f (x) dx.
n b a
b

a
a
i=1

Example 2. Find the average of f (x) = sin x on [0, ].


Solution.

The Mean Value Theorem for Integrals (pgae 452). If f is continuous on [a, b],
then there exists a number c in [a, b] such that
 Z b

Z b
1
f (x) dx.
or
f (x) dx = f (c)(b a).
f (c) = fave =
ba a
a
Rx
Proof. Consider F (x) = a f (x) dx. Since f (x) is continuous on [a, b], F (x) is continuous on [a, b] and differentiable on (a, b). By the Mean Value Theorem, there
exists c (a, b) such that
. By the Fundamental Theo. Hence
rem, we have
f (c) =

Example 3. Given an increasing continuous function f (x) on [a, b]. Find the line
Rb
y = L such that a |f (x) L| dx is minimum.
Solution.

6.5-1

Example 4. Given a continuous function f (x) on [a, b]. Find the line y = L such
Rb
that a (f (x) L)2 dx is minimum.
Solution.

Median and average.

Homework (page 460). The figure shows a curve with the property that, for
every point P on the middle curve y = 2x2 , the areas A and B are equal. Find an
equation for .
(93 )
y
y = 2x2

y = x2

B
A

x
Figure 1: Find an equation for such that the areas A and B are equal.

6.5-2

Chapter 7
7.1

Techniques of Integration

Integration by Parts (page 464)

The rule that corresponds to the Product Rule for differentiation is called the rule
for integration by parts.
The Product Rule states that if f and g are differentiable functions, then
d
(f (x)g(x)) = f (x)g (x) + g(x)f (x).
dx
In the notation for indefinite integrals this equation becomes
Z
(f (x)g (x) + g(x)f (x)) dx = f (x)g(x), or
Z
Z

f (x)g (x) dx + g(x)f (x) dx = f (x)g(x).


So we can rearrange this equation as
Z
Z

f (x)g (x) dx = f (x)g(x) g(x)f (x) dx.

(1)

Formula (1) is called the formula for integration by parts.


Let u = f (x) and v = g(x), then the differentials are du = f (x)dx and dv =
g (x)dx. By the Substitution Rule, the formula for integration by parts becomes
Z
Z
u dv = uv v du.
(2)
Example 1 (page 464). Evaluate

x sin x dx.

x sin x dx = x cos x + n

Solution.

Reduction formula: For n N,


Homework. Find

x2 sin(3x) dx.

xn1 cos x dx.

(89 )
7.1-1

ln x dx.

Homework (page 465). Evaluate

(ln x)2 dx.

(ln x) dx = x(ln x) n

Example 2 (page 465). Evaluate


Solution.

Reduction formula: For n N,

Example 3. Find the integral

x(ln x)2 dx.

(ln x)n1 dx.


(95 )

Solution.

Homework. Evaluate the integral

x ex dx.

Homework (page 465). Evaluate

x2 ex dx.

x e dx = x e n

Example 4 (page 465). Evaluate

(97 )

x ln x dx, R.

Solution.

Reduction formula: For n N,

n x

7.1-2

n x

xn1 ex dx.

Example 5 (page 466). Evaluate

ex sin x dx.

tan1 x dx.

Solution.

Example 6 (page 467). Evaluate


Solution.

Homework. Evaluate the integral


Homework. Evaluate

esin

1
2

x tan (x ) dx and
1

(94 )

dx.
Z

x(tan1 x)2 dx

(101 )

Example 7. Consider the region R enclosed by the curves y = cos x and y = sin x,
and 0 x 4 . Find the volume of the solid obtained by rotating the region about
the y-axis.
(93 )
Solution.

7.1-3

Homework. Find the volume of the solid obtained by rotating the region bounded
by the given curves about the specified line.
(a) y = 1 + sin x, y = 0, and 0 x 2; about the y-axis.
(b) y = cos x, y = 0, and 0 x 2 ; about the y-axis.

(95 )
(96, 97, 102 )

Example 8. Find the average of the horizontal chords in y = sin x, 0 x .


(91 )
Solution.

7.1-4

7.2

Trigonometric Integrals (page 471)

In this section we use trigonometric identities to integrate certain combinations of


trigonometric functions.
Z
Example 1 (page 473). Evaluate sinm x cosn x dx, where m, n 0 are integers.
Solution.
(a) If m = 2k + 1, then
Z
Z
Z
m
n
2k+1
n
sin x cos x dx = sin
x cos x dx = sin2k x cosn x d cos x
=
=

k
X

(1 cos x) cos x d cos x =

Z X
k

Cik 1ki (1)i cos2i x cosn x d cos x

i=0

(1)i+1 Cik

i=0

cosn+2i x d(cos x) =

k
X
(1)i+1 C k
i

i=0

n + 2i + 1

cosn+2i+1 x + C.

(b) If n = 2k + 1, then
Z
Z
Z
m
n
m
2k+1
sin x cos x dx = sin x cos
x dx = sinm x cos2k x d(sin x)
=
=

sin x(1 sin x) d sin x =

k
X
i=0

(1)

Cik

sin

m+2i

x d sin x =

sinm x

k
X

Cik 1ki (1)i sin2i x d sin x

i=0

k
X
i=0

(1)i Cik
m + 2i + 1

sinm+2i+1 x + C.

(c) If m = 2k, n = 2l, then using the half-angle identities


sin2 x =

1 cos 2x
2

and

cos2 x =

1 + cos 2x
,
2

we have
Z
Z
m
n
sin x cos x dx = sin2k x cos2l x dx
=

Z 

1 cos 2x
2

k 

1 + cos 2x
2

l

dx =

Z
k X
l
X
(1)i Cik Cjl
i=0 j=0

If i + j is odd, we reduce the integral to case (b).


If i + j is even, we use half-angle identities again.
Z
Homework (page 471). Evaluate sin5 x cos2 x dx.
7.2-1

2k+l

cosi+j 2x dx.

Example 2 (page 472). Evaluate

sin2 x dx.

Solution.

Example (TA) 3 (page 467, 469). Show that the following reduction formulae:
Z
Z
1
n1
n
n1
(a)
sin x dx = cos x sin
sinn2 x dx, n 2.
x+
n
n
Z
Z
1
n1
n
n1
(b)
cos x dx = sin x cos
cosn2 x dx, n 2.
x+
n
n
Solution.

Example (TA) 4. From the reduction formulae, find

Solution.

Homework (page 472). Evaluate

sin4 x dx.
7.2-2

sin

2n+1

dx and

sin2n dx.

Example 5 (page 475). Compute the integrals

tan x dx and

sec x dx.

Solution.

 (), ()
Example (TA) 6 (page 469). Show that the following reduction formulae:
Z
Z
tann1 x
n
(a)
tan x dx =
tann2 dx, n 6= 1.
n1
Z
Z
tan x secn2 x n 2
n
(b)
sec x dx =
secn2 x dx, n 6= 1.
+
n1
n1
Solution.

Homework (page 475). Evaluate

tan3 x dx.

Homework (page 475). Evaluate

sec3 x dx.
7.2-3

(97 )

Example 7 (page 474). Evaluate

tanm x secn x dx, where m, n N.

Solution.
(a) If n = 2k, k N, then
Z
Z
Z
m
n
m
2k
tan x sec x dx = tan x sec x dx = tanm x sec2k2 x d tan x
=
=

tan x(tan x + 1) d tan x =

k
X

Cik

i=0

tanm+2i x d tan x =

k
X
i=0

tan x

k
X

Cik tan2i x d tan x

i=0

Cik
tanm+2i+1 x + C.
m + 2i + 1

(b) If m = 2k + 1, k N, then
Z
Z
Z
m
n
2k+1
n
tan x sec x dx = tan
x sec x dx = tan2k x secn1 x d sec x
=
=

(sec x 1) sec

k
X
i=0

n1

x d sec x =

Z X
k

Cik sec2(ki) (1)i secn1 x d sec x

i=0

(1)i Cik

sec2(ki)+n1 x d sec x =

k
X
i=0

(1)i Cik
sec2(ki)+n x + C.
2(k i) + n

(c) If m = 2k, n = 2l + 1, then


Z
Z
Z
m
n
2k
2l+1
Im = tan x sec x dx = tan x sec
x dx = tan2k1 x sec2l x d sec x
Z
2k1
2l+1
= tan
x sec
x sec x d(tan2k1 x sec2l x)
Z
2k1
2l+1
= tan
x sec
x sec x(2k 1) tan2k2 sec2 x sec2l x dx
Z
sec x tan2k1 x(2l) sec2l1 x sec x tan x dx
Z
2k1
2l+1
= tan
x sec
x (2k 1) tan2k2 sec2l+3 x dx
Z
2l tan2k x sec2l+1 x dx
Z
2k1
2l+1
= tan
x sec
x (2k 1) tan2k2 (tan2 x + 1) sec2l+1 x dx
Z
2l tan2k x sec2l+1 x dx
= tanm1 x secn x (m 1)Im (m 1)Im2 (n 1)Im

7.2-4

Hence we get

1
tanm1 x secn x (m 1)Im2 ,
m+n1
R
R
and the reduction formula will reduce the integral to secn dx = sec2l+1 dx,
l N.
Z
Homework (page 473). Evaluate tan6 x sec4 x dx.
Im =

Homework (page 474). Evaluate

tan5 x sec7 x dx.

Example 8 (page 476). Evaluate the following integrals:


Z
Z
Z
sin mx cos nx dx;
cos mx cos nx dx;
sin mx sin nx dx.
Solution. Recall the following identities:
1
sin x cos y = (sin(x + y) + sin(x y))
2
1
cos x cos y = (cos(x + y) + cos(x y))
2
1
sin x sin y = (cos(x + y) cos(x y))
2
Then we have
Z
Z
1
sin mx cos nx dx =
sin((m + n)x) + sin((m n)x) dx
2
=
Z

1
cos mx cos nx dx =
2

cos((m + n)x) + cos((m n)x) dx

=
Z

1
sin mx sin nx dx =
2

cos((m + n)x) cos((m n)x) dx

7.2-5

7.3

Trigonometric Substitution (page 478)

Trigonometric identities are also useful to make substitutions for some radical functions.
Table of Trigonometric Substitutions.
Expression

a2 x2

a2 + x2

x2 a2

Substitution

Identity

x = a sin , 2

1 sin2 = cos2

x = a tan , 2 < <

1 + tan2 = sec2

x = a sec , 0 <

or

3
2

sec2 1 = tan2

Example 1 (page 479). Find the area enclosed by the ellipse


Solution.

Example 2 (page 482). Find

x
dx.
3 2x x2

Solution.

Homework (page 479). Evaluate

9 x2
dx.
x2

7.3-1

x2 y 2
+ 2 = 1.
a2
b

Example 3 (page 480). Find

x2

1
dx.
x2 + 4

Solution.

Homework. Evaluate the integral

x2

Example 4 (page 481). Find

1
dx.
1 + x2

(94 )

1
dx, where a > 0.
a2

x2

Solution.

Homework. Find the integral

x3

Homework. Evaluate the integral

2
dx,
x2 1

x > 1.

x
dx.
x2 + 2x + 2

7.3-2

(89 )
(98 )

Example 5 (page 482). Find

3 3
2

x3
3

(4x2 + 9) 2

dx.

Solution.

Homework (page 484). Find the area of the crescent-shaped region (called a lune)
bounded by arcs of circles with radii r and R.

r
R

Figure 1: The crescent-shaped region.

7.3-3

7.4

Integration of Rational Functions by Partial


Fractions (page 484)

In this section we show how to integrate any rational function (a ratio of polynomials) by expressing it as a sum of simpler fractions, called partial fractions (
).
Z
1
dx, where k N.
Example 1. Discuss the integral
(ax + b)k
Solution.

Example 2. Discuss the integral


Solution.

Ax + B
dx, where b2 4ac < 0, k N.
+ bx + c)k

(ax2

7.4-1

Integrate rational functions


Step 1: Perform the long division ()
Definition 3 (page 485). Consider a rational function f (x) =
Q(x) are polynomials.

P (x)
,
Q(x)

where P (x) and

(a) If the degree of P (x) is less than the degree of Q(x), such a rational function
f (x) is called proper.
(b) If the degree of P (x) is greater or equal to the degree of Q(x), such a rational
function f (x) is called improper.
If f (x) is improper, then we use the long division to get
f (x) =
and

R(x)
Q(x)

R(x)
P (x)
= S(x) +
,
Q(x)
Q(x)

is proper.

Step 2: Factor the denominator Q(x) as a product of linear factors (ax + b)


and irreducible quadratic factors ax2 + bx + c, b2 4ac < 0. ()
Step 3: Express the proper rational function
fractions. (, )

R(x)
Q(x)

as a sum of partial

Definition 4 (page 485). A rational function is called a partial fraction if it is of


the form
A
(ax + b)n

or

Ax + B
.
+ bx + c)n

(ax2

(1) Q(x) is a product of distinct linear factors. That is,


Q(x) = (a1 x + b1 )(a2 x + b2 ) (ak x + bk ),
where no factor is repeated, then there exist constants A1 , A2 , . . . , AK such
that
A1
A2
Ak
R(x)
=
+
++
.
Q(x)
a1 x + b1 a2 x + b2
ak x + bk
(2) Q(x) is a product of linear factors, some of which are repeated. Suppose the
first linear factor (a1 x + b1 ) is repeated r times, then instead of the single term
A1
, we would use
a1 x+b1
A2
Ar
A1
+
++
.
2
a1 x + b1 (a1 x + b1 )
(a1 x + b1 )r
7.4-2

(3) Q(x) contains irreducible quadratic factors, none of which is repeated. That
is, Q(x) has the factor ax2 + bx + c, where b2 4ac < 0, then the expression
R(x)
will have a term of the form
for Q(x)
Ax + B
,
ax2 + bx + c
and then we will use the formula
Z
 
1
1
1 x
+ C.
dx = tan
x2 + a2
a
a
(4) Q(x) contains a repeated irreducible quadratic factor. If Q(x) has the factor
(ax2 + bx + c)r , where b2 4ac < 0, then instead of the single partial fraction,
the sum
A2 x + B2
Ar x + Br
A1 x + B1
+
+

+
ax2 + bx + c (ax2 + bx + c)2
(ax2 + bx + c)r
occurs in the

R(x)
.
Q(x)

Example 5 (Case (1)). Show that

sec d = ln | sec + tan | + C.

Solution.

Example 6 (Case (2)). Find the integral

Solution.

7.4-3

x2 + 3x + 2
dx.
x3 3x + 2

(90 )

Homework. Evaluate

x2 + 4
dx.
x4 + 3x3 + 2x2

Example 7 (Case (3)). Evaluate the integral

(99 )
Z

2x2 + 5x + 3
dx.
(x2 + 2x + 2)(x 1)
(94 )

Solution.

Homework. Evaluate the integral

1
dx.
x3 + 1

1
dx.
x3 + x2 + x
Z
1
dx.
Homework. Evaluate the integral
3
x 1
Z 3
x x2 + x + 1
Homework. Evaluate
dx.
x3 + x2 + x + 1
Homework. Find the integral

7.4-4

(93 )
(100 )
(98 )
(102 )

Example 8. Find the integral

2x + 1
dx.
(x2 + 1)2

(95 )

Solution.

Rationalizing substitutions, page 492


Some nonrational functions can be changed into rational functions by means of
appropriate substitutions. In particular, when an integrand contains an expression
p
p
of the form n g(x), then the substitution u = n g(x) may be effective.
Z
1

dx.
(96 )
Example 9. Find the integral
3
x x2 + 3
Solution.

Homework. Evaluate the integral

Z p

1+
x

7.4-5

dx.

(97 )

Convert rational functions of sin x and cos x, page 493


The German mathematician Karl Weierstrass (1815-1897) noticed that the substitution t = tan( x2 ) will convert any rational function of sin x and cos x into an ordinary
rational function of t.
If t = tan( x2 ), < x < , then we have
cos

x
2

1
=
1 + t2

and

sin

and

sin x =

By double-angle formula, we get


cos x =

1 t2
1 + t2

x
2

t
.
1 + t2

2t
.
1 + t2

Furthermore, we can compute


dx =
Example 10. Find the integral

2
dt.
1 + t2

1
dx.
2 + cos x

(89 )

Solution.

Homework. Find the integral


Homework. Evaluate

1
dx.
2 + sin x

1
dx.
3 5 sin x
7.4-6

(96 )
(99 )

7.5

Strategy for Integration (page 494)

We have learned the following techniques to integrate a function:


Substitution rule, section 5.5.
Integration parts, section 7.1.
Trigonometric substitution, section 7.3.
Partial fractions, section 7.4.
In this section, we present a collection of miscellaneous integrals in random and the
main challenge is to recognize which technique or formula to use.
Table of Indefinite Integrals (page 495).
Z
Z
 
 
1
1
1
1 x
1 x

+
C
+ C, a > 0
dx
=
tan
dx
=
sin
2 x2
x2 + a2
a
a
a
a


Z
Z

x a
1
1
1

2 a2 + C
+C

ln
x
dx =
dx
=
ln
x
+


2a
x + a
x2 a2
x2 a2
Once you are armed with these basic integration formulae, if you dont immediately see how to attack a given integral, you might try the following four-steps
strategy.
(1) Simplify the integrand if possible. Use algebraic manipulation or trigonometric
identities to simplify the integrand.
Z

x(1 + x) dx =
Z
tan
d =
sec2
Z
(sin + cos )2 d =
(2) Look for an obvious substitution.
Z
x
dx =
x2 1
(3) Classify the integrand according to its form.
(a) Trigonometric function: product of powers of sin x and cos x, of tan x and
sec x, or cot x and csc x.
(b) Rational functions:

P (x)
,
Q(x)

where P (x) and Q(x) are polynomials.


7.5-1

(c) Integration by parts: product of a power of a polynomial and a transcendental function (trigonometric, exponential, or logarithmic).
p

(d) Radicals: x2 a2 , n ax + b, or n g(x).

(4) Try again: remember that there are basically only two methods of integration:
substitution and parts.
(a) Try substitution: inspiration, ingenuity, desperation.
(b) Try parts: it is sometimes effective on single function, such as sin1 x,
tan1 x, ln x (inverse functions).
(c) Manipulate the integrand:
Z
1
dx =
1 cos x
=
(d) Relate the problem to previous problems.
Z
tan2 x sec x dx =
(e) Use several methods: substitution, integration by parts, etc.
Z
tan3 x
dx.
Example 1 (page 497). Compute
cos3 x

Solution.

Solution 2.

Example 2 (page 497). Compute


Solution.

dx.

7.5-2

Example 3 (page 497). Compute


Solution.

Z r

1x
dx.
1+x

Can we integrate all continuous functions?


Definition 4 (page 498). Elementary functions are all polynomials, rational functions, power functions, exponential functions, logarithmic functions, trigonometric
and inverse trigonometric functions, hyperbolic and inverse hyperbolic functions,
and all functions that can be obtained from these by the five operations of addition,
substraction, multiplication, division, and composition.
R

If f (x) is an elementary function, then f (x) is an elementary function, but


f (x) dx need not be an elementary function. For example,
Z p
(1)
1 2 sin2 x dx: elliptic integral (),

(8.1 )
Z x
2
2
(2)
et dt: error function (), ()
0
Z
Z
2
(3)
sin(x ) dx, cos(x2 ) dx: Fresnel integral (),
(4)

sin x
dx,
x

(5)

ex
dx: exponential integral ()
x

cos(ex ) dx: sine integral function (cosine integral function)

1
dx: logarithmic integral ()
ln x
Z
x3 + 1 dx:
(7)
(6)

,
( ) :
r
Z
Z
Z

sin x
2
x
2
e
dx = ,
,
dx = .
sin(x ) dx =
2

7.5-3

7.5 Exercises and 7 Review


Evaluate the integral. (page 499-500; 530.)
Z
Z 1

2
1. cos x(1 + sin x) dx
2.
(3x + 1) 2 dx

3.

sin x + sec x
dx
tan x

6.

9.

12.

15.

4.

7.

10.

13.

16.

tan d
1

1
4

x2

sin t cos t dt
2
2

19.
22.

25.

28.

31.

Z r

34.

37.

x2

dx
1 x2

x+ex

dx
ln x

p
dx
x 1 + (ln x)2
3x2 2
dx
x2 2x 8
sin

at dt

1+x
dx
1x

1 + 4 cot x
dx
4 cot x
tan3 sec2 d

11.

x2

14.

x3

dx
1 + x2

17.

t sin t cos t dt

x
dx
3 x4

r 4 ln r dr

x1
dx
4x + 5

t cos t dt

20.

23.

26.

29.

e2 dx
1

(1 +

x) dx

x4

1
3

(1 x2 ) 2

ln(x +

32.

35.

38.

x2 + 1) dx

24.

27.

30.

33.

36.

39.

sec tan
d
sec2 sec

42.

tan1 x
dx
x2

45.

x5 ex dx

48.

e t
dt
18.
t
1
Z

21. tan1 x dx
6z + 5
dz
2z + 1

1
dx
1 + ex
2

|ex 1| dx

2x 1
dx
2x + 3

cos 2x cos 6x dx

dx

3x2 2
dx
x3 2x 8

x
dx
+ x2 + 1

p
dy
4y 2 4y 3
Z
x
43.
dx
1 + x3
Z
(x 1)ex
46.
dx
x2
40.

8.

x1
dx
4x 5

2t
dt
(t 3)2

5.

tan1

y
e
dy
1 + y2

sin cot
d
sec

41.

44.

47.

x3
dx
(x 1)4

tan d
1+

ex

7.5-4

dx

3 2x x2 dx

x2 tan x
dx
1 + cos4 x

x 2 1 x2 dx

49.

52.

55.

58.

dx
x 4x + 1
1
dx
4
x(x + 1)
1
dx
x+x x
x ln x

dx
x2 1

50.

53.

x sinh mx dx
1

dx
x+x x

56.
59.

dx
2
x 4x + 1

cos x cos (sin x) dx

51.

dx
x 4x2 + 1

54.

(x + sin x)2 dx

57.

60.

x2

1
d
1 + cos2

Z
65.

sin 2x
dx
1 + cos4 x

66.

dx
x+1+ x

Z
68.

x2
dx
x6 + 3x3 + 2

69.

70.

1
dx
1 + 2ex ex

e2x
dx
1 + ex

72.

73.

x + sin1 x

dx
1 x2

61.

1
d
1 + cos

64.

67.

1
p
dx
x+1

dx
x(2 + x)4

76.

79.

x sin2 x cos x dx

82.

sin x cos x
dx
sin4 x + cos4 x

85.

88.

91.

sin(ln t)
dt
t

94.

e2x
dx
1 + e4x

97.

x1
dx
x2 + 2x

sin ecos d

62.

71.

Z x
4 + 10x
74.
dx
2x
Z
xex
dx
77.
1 + ex
Z
sec x cos 2x
dx
80.
sin x + sec x
Z 2
(x + 1)2
dx
83.
x
1
Z
6
86.
t sin 2t dt

63.

75.
78.
81.

x ln x dx

89.

92.

95.

98.

sec6
d
tan2

sin cos d

dx

7.5-5

1
dx
4x2 1

xe

dx

ln(tan x)
dx

sin
x
cos
x
4
Z 3
1 + x2
dx
x2
1
Z
ln(x + 1)
dx
x2
Z
1
dx
(x 2)(x2 + 4)
Z
1 + sin x
dx
1 sin x
Z

1 sin x dx
Z

84.
87.

2t2

90.

93.

93.

x2 + 2
dx
x+2

99.

x sec x tan x dx

tan1 x
dx
1 + x2

x 3 x + c dx

x
dx
(x + 1)2
1
dt
+ 3t + 1
1
dx
1

ex

x2 1
dx
x

100.

x2 + 8x 3
dx
x3 + 3x2

103.

dx
2
x 4x

106.

e cos x dx

109.

112.

115.

(4 x2 )

118.

1 tan
d
1 + tan

121.

101.
104.
107.

cos x sin 2x dx 110.

dx
ex 1 e2x

113.

1
2

3
2

dx

xe2x
dx
(1 + 2x)2

116.

tan5 sec3 d

105.

dx
x x2 + 1

108.

x sin x cos x dx

111.

114.

117.

(cos x + sin x)2 cos 2x dx 120.

te

dt

3x3 x2 + 6x 4
dx
(x2 + 1)(x2 + 2)
Z
3
x+1

dx
3
x1
Z ln 10 x x
e e 1
dx
ex + 8
0
Z
(sin1 x)2 dx

119.
122.

Z
Z

e
1

Homework. Find the limit lim

R x2
0

1
p
dx.
x 1 + (ln x)2

102.

x
dx
1 + |x|
x sin x
dx
cos3 x

1
p
dx
3
x + x2

2 x
dx
x

tan
d
sin 2

Homework. Find the integral

Homework. Find the integral

x+1
dx
2
9x + 6x + 5

x
dx.
8 2x2 x4

(90 )
(95 )

etx (2t2 + 1) dt
.
x4

(96 )

Hint: ,
Homework. Consider the region bounded by the curves y = sin
6
(x2 +3x+2)

x
2

and y =

for 0 x 1. Note that the two curves meet at x = 1. Find the volume
of revolving the region (a) about the y-axis; (b) about x-axis.
(101 )

7.5-6

7.8

Improper Integrals (page 519)

In this section we extend the concept of a definite integral to the case where the
interval is infinite and also to the case where f has an infinite discontinuity in [a, b].
In either case the integral is called an improper integral ().

Type 1: Infinite Intervals


Definition of an Improper Integral of Type 1 (page 520).
Rt
(a) If a f (x) dx exists for every number t a, then
Z
Z t
f (x) dx = lim
f (x) dx
t

provided this limits exists (as a finite number).


Rb
(b) If t f (x) dx exists for every number t b, then
Z b
Z b
f (x) dx = lim
f (x) dx
t

provided this limits exists (as a finite number).


R
Rb
The improper integrals a f (x) dx and f (x) dx are called convergent () if
the corresponding limits exists and divergent () if the limit does not exist.
R
Ra
(c) If both a f (x) dx and f (x) dx convergent, then we define
Z
Z a
Z
f (x) dx =
f (x) dx +
f (x) dx

In part (c) any real number a can be used.


Example 1 (page 519). Discuss the areas of the infinite region R under the curve
y = x1p , p > 0 and to the right x = 1.
Solution.

7.8-1

Type 2: Discontinuous Integrands


Definition of an Improper Integral of Type 2 (page 523).
(a) If f is continuous on [a, b) and is discontinuous at b, then
Z b
Z t
f (x) dx = lim
f (x) dx
tb

if this limits exists (as a finite number).

(b) If f is continuous on (a, b] and is discontinuous at a, then


Z b
Z b
f (x) dx
f (x) dx = lim+
ta

if this limits exists (as a finite number).


Rb
The improper integrals a f (x) dx is called convergent () if the corresponding
limits exists and divergent () if the limit does not exist.
Rc
(c) If f has a discontinuity at c, where a < c < b, and both a f (x) dx and
Rb
f (x) dx convergent, then we define
c
Z b
Z c
Z b
f (x) dx =
f (x) dx +
f (x) dx
a

Example 2 (page 528). Discuss the areas of the region R under the curve y =
1
, p > 0, and between x = 0 and x = 1.
xp
Solution.

Example 3. Compare Example 1 with Example 2.


Solution.

7.8-2

A Comparison Test for Improper Integrals


Comparison Theorem. Suppose that f and g are continuous functions with f (x)
g(x) 0 for x a.
(a) If
(b) If




R
a

R
a

f (x) dx is convergent, then


g(x) dx is divergent, then

R
a

R
a

g(x) dx is convergent.

f (x) dx is divergent.

f (x) g(x) 0,
for x a for some x b, b a.

Figure 1: Comparison Theorem.

Example 4.
(a) Find the values of a for which the improper integral
verges.
(b) Evaluate the integral

x (1

1
dx con+ x)

sec2 x dx.

Solution.

7.8-3

(97 )

Example 5. Suppose the improper integral

Find the value of a and the value of integral.

x
a

2
x2 1 x + 1

dx converges.
(96 )

Solution.

Homework. Determine the real number a so that the integral



Z 2
1
1

dx

x x2 + 4 ax
0
(100 )

converges.
Example 6 (page 528). Let In =

xn ex dx. Find the reduction formula.

Solution.

Homework. Let In =

xn ex dx.

(a) Compute I3 .

(95 )

(b) Find the general formula of In .

(98 )
7.8-4

Example 7.
(a) Determine the values of > 0 such that

(b) Find the integral

Solution.

ln x
dx is convergent.
x
(102 )

ln x
dx.
x3

(89 )

n!
.
(96 )
n n
Remark: We have derived the integral expression of this limit in Homework 8,
section 5.2.
Z
4

Example (TA) 8. Evaluate


dx.
(99 )
x(x + 4)
0
Solution.
Homework. Find the limit lim

Example 9. Evaluate the improper integral


Solution.

7.8-5

2
0

x(2 x)
dx.
x

(101 )

Homework. Is the proper integral

1
dx convergent or divergent?
(1 x)2
(90 )

Homework. Find the volume of solid generated by revolving the curve y = ex sin x,
(100 )

x 0, about the x-axis.

Example
(TA) 10 (page 529). Show that if a > 1 and b > a+1, then the integral
Z
xa
dx is convergent.
1 + xb
0
Solution.

7.8-6

Chapter 8
gration
8.1

Further Applications of Inte-

Arc Length (page 538)

The Arc Length Formula (page 539). If f is continuous on [a, b], then the length
of the curve y = f (x), a x b, is
s
 2
Z b
Z bp
dy

2
dx.
(1)
1 + (f (x)) dx =
1+
L=
dx
a
a

If a curve has the equation x = g(y), c y d, and g (y) is continuous, then by


interchanging the roles of x and y, we obtain the following formula for its length:
s
 2
Z dp
Z d
dx

2
L=
1 + (g (y)) dy =
1+
dy.
(2)
dy
c
c
Proof. Suppose that a curve C is defined by the equation y = f (x), where f (x) is
continuous and a x b. We obtain a polygonal approximation to C by dividing
the interval [a, b] into n subintervals with endpoints x0 , x1 , . . . , xn and equal width
x. If yi = f (xi ), then the point Pi (xi , yi ) lies on C and the polygon with vertices
P0 , P1 , . . . , Pn is an approximation to C.

Figure 1: We use the length of inscribed polygons to approximate the length of C.


We define the length L of the curve C with equation y = f (x), a x b, as the
limit of the lengths of these inscribed polygons (if the limit exists):
L = lim
n

n
X

|Pi1 Pi |

i=1

When f is continuous on [a, b] (we say f C 1 ([a, b])), then by the Mean Value
Theorem, there is a number xi (xi1 , xi ) such that
p
p
|Pi1Pi | = (xi xi1 )2 + (yi yi1 )2 = (x)2 + (f (xi ) f (xi1 ))2
p
p
= (x)2 + (f (xi )x)2 = 1 + (f (xi ))2 x.

8.1-1

Therefore,
L = lim
n

n
X

|Pi1 Pi | =

i=1

To get the formula (1), by similar discussion, we divide c y d into n subinterval


with endpoints y0 , y1 , . . . , yn and equal width y, and rewrite |Pi1 Pi | as
p
(xi xi1 )2 + (yi yi1)2 = (g(yi ) g(yi1))2 + (y)2
p
p
= (g(yi)x)2 + (y)2 = 1 + (g(yi))2 y,

|Pi1 Pi | =

so
L = lim
n

n
X

|Pi1 Pi | =

i=1

Example 1. Show that the circumference of a circle with radius r is 2r.


Solution.

Example 2 (page 543). Find the length of the curve x =


Solution.

8.1-2

1
y(y
3

3), 1 y 9.

Example 3 (page 543). Find the length of the curve x 3 + y 3 = 1.


Solution.

Homework. Let f (x) = ln x 18 x2 , 1 x 2. Find the length of the graph of f .


(90 )
Z 12
x
t3 + 1 dt, 12 x 1.
Homework. Find the length of the curve y =
1

(94 )

Homework. Given 0 < a < b, find the arc length of y = ln

ex +1
ex 1

for a x b.
(98 )

The Arc Length Function


We will find it useful to have a function that measures the arc length of a curve
from a particular starting point to any other point on the curve. If a smooth curve
C has the equation y = f (x), a x b, let s(x) be the distance along C from the
initial point P0 (a, f (x)) to the point Q(x, f (x)). Then
Z xp
s(x) =
1 + (f (t))2 dt
a

is a function, called the arc length function.


By the Fundamental Theorem of Calculus, we get
s
 2
dy
ds p
= 1 + (f (x))2 = 1 +
dx
dx

In differential sense, we have


s
 2
dy
ds = 1 +
dx (ds)2 = (dx)2 + (dy)2 .
dx
Similarly, we have

ds =

1+

dx
dy

8.1-3

2

dy.

8.2

Area of a Surface of Revolution (page 545)

In this section, we will derive the formula of the area of a surface of revolution.
Example 1 (page 545). Find the lateral surface area of a circular cylinder with
radius r and height h.
Solution.

Example 2 (page 546). Find the lateral surface area of a circular cone with base
radius r and slant height l.
Solution.

Example 3 (page 546). Find the lateral surface area of a band, which is a portion
of a circular cone with upper radius r1 , lower radius r2 , and slant height l.
Solution.

8.2-1

Example 4 (page 546). Find the surface area of the surface obtained by rotating
the curve y = f (x), a x b, about the x-axis.
Solution.

Definition 5. Surface area of the surface obtained by rotating the curve y =


f (x), a x b, about x-axis is
s
 2
Z b
Z b
p
dy

2
dx.
2y 1 +
S=
2f (x) 1 + (f (x)) dx =
dx
a
a
Recall that the differential of the arc length function is
s
 2
dx
2
2
2
(ds) = (dx) + (dy) ds = 1 +
dy.
dy

(1)

If the curve is described as x = g(y), c y d, then the formula for surface area
(rotating about x-axis) becomes
s
 2
Z d
dx
S=
2y 1 +
dy.
(2)
dy
c
Formula (1) and (2) can be formally written as
Z
2y ds.
For rotation about the y-axis, the surface area formula (formally) becomes
s
s
 2
 2
Z
dy
dx
dx = 1 +
dy
S = 2x ds, where ds = 1 +
dx
dy
8.2-2

Example 6. Find the area of surface obtained by rotating y = sin x, 0 x ,


(93, 97, 100 )

about the x-axis.


Solution.

Example 7 (page 550). Consider the region R = {(x, y)|x 1, 0 y x1 } rotating


about the x-axis.
(a) Show that the volume of the resulting solid is finite.
(b) Show that the surface area is infinite. (The surface is called Gabriels horn.)
Solution.

Homework.
Z
(a) Find sec3 x dx.
(b) The curve y = ln x, 0 < x < 1, is rotated about y-axis. Find the area of the
resulting surface.
(99 )
8.2-3

Homework. Find the exact area of the surface obtained by rotating the curve
3

x = 31 (y 2 + 2) 2 , 1 y 2 about the x-axis.


Homework.
(a) The ellipse
x2 y 2
+ 2 = 1, a > b
a2
b
is rotated about the x-axis to form a surface called an ellipsoid, or prolate
spheroid. Find the surface area of this ellipsoid.
(b) If the ellipse in part (a) is rotated about its minor axis (the y-axis), the resulting ellipsoid is called an oblate spheroid. Find the surface area of this
ellipsoid.
Example (TA) 8. Let h(x) =

x x2 + sin1 ( x), 0 x 1.

(a) Find the length of the curve y = h(x).


(b) Find the area of the surface generated by rotating the curve y = h(x) about
the x-axis.
(102 )
Solution.

8.2-4

8.3

Applications to physics and engineering


(page 554)

Among the many applications of integral calculus to physics and engineering, we


consider two here: force due to water pressure and centers of mass.
Strategy: Partition Approximation Summation Limit Evaluate

Hydrostatic Pressure () and Force, page 552


Suppose that a thin horizontal plate with area A m2 is submerged in a fluid of density
(or mass density) kg/m3 at a depth d m below the surface of the fluid.

Figure 1: A thin horizontal plate is submerged in a fluid.


The fluid volume directly above the plate is V =

.
.

The mass of the fluid above the plate is m =


The force exerted by the fluid on the plate is F =
acceleration due to gravity.

, where g is the

The pressure P on the plate is defined to be the force per unit area:
P =

The SI unit for measuring pressure is N/m2 , which is called a pascal Pa, and
the kilopascal (kPa) is often used.
When using US Customary units, we write P = gd = d, where = d is the
weight density. For instance, the weight density of water is = 62.5 lb/ft3.
Example. The density of water is = 1000 kg/m3 , the pressure at the bottom
of a swimming pool 2 m(= 6.56 ft) deep is
P = gd = 1000 (kg/m3 ) 9.8 (m/s2 ) 2 (m) = 19, 600 (Pa) = 19.6 (kPa)
= d = 62.5 (lb/ft3) 6.56 (ft) = 410 (lb/ft2).
8.3-1

An important principle of fluid pressure is the experimentally verified fact that


at any point in a liquid the pressure is the same in all directions. Thus the pressure
in any direction at a depth d in a fluid with mass density is given by
P = gd = d.
This helps us determine the hydrostatic force against a vertical plate of wall or dam
in a fluid. This is not a straightforward problem because the pressure is not constant
but increases as the depth increases.
Example 1 (page 553). A dam has the shape of the trapezoid () shown in
Figure 2. The height is 20 m and the width is 50 m at the top and 30 m at the
bottom. Find the force on the dam due to hydrostatic pressure if the water level is
4 m from the top to the dam.
50 m

15
0
20 m

10

xi
a

30 m
x
Figure 2: Find the force on the dam due to hydrostatic pressure.

Solution.
Partition: We choose a vertical x-axis with origin at the surface of the water and
directed downward as in Figure 2. The depth of the water is 16 m, so we divide
the interval [0, 16] into subintervals of equal length with endpoints xi and we choose
xi [xi1 , xi ]. The i-th horizontal strip of the dam is approximated by a rectangle
with height x and width wi .
Approximation: From similarity, we have
10
a
=
a=

16 xi
20
and so
wi =

8.3-2

If Ai is the area of the i-th strip, then


Ai wi x =
If x is small, then the pressure Pi on the i-th strip is almost constant and we can
get Pi 1000gxi . The hydrostatic force Fi acting on the i-th strip is the product
of the pressure and the area: Fi = Pi Ai 1000gxi (46 xi )x.
Summation Limit Evaluate: Adding these forces and taking the limit as n ,
we obtain the total hydrostatic force on the dam:
F = lim
n

n
X

1000gxi (46 xi )x =

i=1

=
Example 2 (page 554). Find the hydrostatic force on one end of a cylindrical drum
with radius 3 ft if the drum is submerged in water 10 ft deep.
y

di

10

9 (yi )2

y
yi

x
x2 + y 2 = 9

Figure 3: Find the hydrostatic force.

Solution.
Partition: In this example it is convenient to choose the axes as in Figure 3 so that
the origin is placed at the center of the drum. Then the circle has a simple equation,
x2 +y 2 = 9. We divide the circular region into horizontal strips of equal width. From
p
the equation of the circle, we see that the length of the i-th strip is 2 9 (yi)2 and
so its area is
Ai =
8.3-3

Approximation: The pressure on this strip is approximately (in US Customary units)


di = 62.5(7 yi ),
and so the force on the strip is approximately
di Ai =
Summation, Limit, Evaluate: The total force is obtained by adding the forces on all
the strips and taking the limit:
F = lim
n

n
X
i=1

p
62.5(7 yi )2 9 (yi)2 y =

=
Homework (page 560). A vertical plate is submerged in water and has the indicated
shaped. Explain how to approximate the hydrostatic force against one side of the
plate by a Riemann sum. Then express the force as an integral and evaluate it.
a

Figure 4: Find the hydrostatic force.


Homework. A vertical dam has semicircular gate as shown in the Figure 5. Find
the hydrostatic force against the gate.

}2 m
12 m

4m
Figure 5: Find the hydrostatic force.

8.3-4

water level

Moments and Center of Mass


Our main objective is to find the point P on which a thin plate of any given shape
balances horizontally as in Figure 6. The point is called the center of mass ()
(or center of gravity) of the plate.
P

Figure 6: Find the center of mass of a plate.


Example 3 (page 555). Two masses m1 and m2 are attached on a rod of negligible
mass on opposite sides of a fulcrum () and at distances d1 and d2 from the
fulcrum. Find the condition that the rod will balance.
Solution. From the Law of the Lever, the rod will balance if m1 d1 = m2 d2 .
o

m1

m2

Figure 7: Find the condition that the rod will balance.


If we set up a coordinate, x-axis on the rod for example, and set the position of
m1 , m2 at x1 and x2 , respectively, and denote the position of the center of mass x.
We can write down the condition m1 d1 = m2 d2 as

The numbers m1 x1 and m2 x2 are called moments of masses m1 and m2 (with respect
to the origin).
Example 4 (page 555). Generalize the result in Example3 to n particles with masses
m1 , m2 , . . . , mn located at the points x1 , x2 , . . . , xn on the x-axis.
Solution. The center of mass of the system is located at

x =

8.3-5

where m =

n
P

mi is the total mass of the system, and the sum of the individual

i=1

moments M =

n
P

mi xi is called the moment of the system about the origin.

i=1

Example 5 (page 555). Find the center of mass of the system of n particles with
masses m1 , m2 , . . . , mn located at the points (x1 , y1 ), (x2 , y2 ), . . . , (xn , yn ) in the xyplane.
y
m1
(x1 , y1 )

m3
(x3 , y3 )

x
m2
(x2 , y2 )

Figure 8: A system of 3-particles with m1 , m2 , m3 in the xy-plane.

Solution. We define the moment of the system about the y-axis to be


My =

n
X

mi xi .

i=1

Remark that the index y of My means y-axis, so we compute the horizontal


moment. Similarly, we define the moment of the system about the x-axis as
Mx =

n
X

mi yi .

i=1

As in 1-dimensional case, the coordinates (


x, y) of the center of mass are given in
terms of the moments by
x =
where m =

n
P

My
,
m

y =

Mx
,
m

mi is the total mass.

i=1

Since m
x = My and m
y = Mx , the center of mass (
x, y) is the point where a
single particle of mass m would have the same moments as the system.

8.3-6

Example 6 (page 556). We consider a flat plate (called a lamina ()) with
uniform density that occupies a region R of the plane. Locate the center of mass
of the plate, which is called the centroid () of R.
Solution. The symmetry principle says that if R is symmetric about a line l, then
the centroid of R lies on l. Thus the centroid of a rectangle is its center.
y

1 f (
Ci (
xi , 2
xi ))

x
xi xi+1

Figure 9: Find the centroid of the region R.


Suppose that the region R is of the type shown in Figure 9. We divide the interval [a, b] into n subintervals with endpoints x0 , x1 , . . . , xn and equal width x. We
choose the sample point xi to be the midpoint xi of the i-th subinterval. This determines the polygonal approximation to R. The centroid of the i-th approximating
xi )). Its area is f (
xi )x, so its mass is
rectangle Ri is its center Ci (
xi , 21 f (
f (
xi )x.
The moment of Ri about the y-axis is the product of its mass and the distance from
Ci to y-axis, which is xi . Thus
My (Ri ) = (f (
xi )x)
xi = xi f (
xi )
xi x.
So
My = lim
n

n
X

xi f (
xi )x =

i=1

Similarly, we can get the moment of Ri about the x-axis:


1
1
Mx (Ri ) = (f (
xi )x) f (
xi ) = (f (
xi ))2 x.
2
2
So
n
X
1
xi ))2 x =
(f (
Mx = lim
n
2
i=1

8.3-7

Just for systems of particles, the center of mass of the plate is defined so that
m
x = My and m
y = Mx . But the mass of the plate is the product of its density
and its area:
Z b
m = A =
f (x) dx,
a

and so the center of mass of the plate on the centroid of R is located at the point
(
x, y), where
Z
Z
1 b1
1 b
xf (x) dx, y =
(f (x))2 dx
x =
A a
A a 2

Example 7 (page 557). Find the center of mass of a semicircular plate of radius r.
Solution.

Homework (page 558). Find the centroid of the region bounded by the curves
y = cos x, y = 0, x = 0, and x = 2 .
If the region R lies between two curves y = f (x) and y = g(x), where f (x)
g(x), then the similar arguments can show that the centroid of R is (
x, y), where
Z
Z
1 b
1 b1
x =
x(f (x) g(x)) dx, y =
[(f (x))2 (g(x))2 ] dx
A a
A a 2
Example 8 (page 559). Find the centroid of the region bounded by the line y = x
and the parabola y = x2 .
Solution.

8.3-8

Theorem of Pappus (page 559). Let R be a plane region that lies entirely on one
side of a line l in the plane. If R is rotated about l, then the volume of the resulting
solid is the product of the area A of R and the distance d traveled by the centroid of
R.
Example 9 (page 560). A torus is formed by rotating a circle of radius r about a
line in the plane of the circle that is a distance R(> r) from the center of the circle.
Find the volume of the torus.
Solution.

Homework. Let R be the region bounded by y = sin x, y = 0, x = 0, and x = 2 .


(a) Find the centroid of the region R.
(b) Find the volume of the solid obtained by revolving R about 8x + 6y = 25.
(96 )
Homework. Let R be the region bounded by y = cos x, y = 0, x = 0, and x = 2 .
(a) Find the volume of the solid obtained by revolving R about x-axis.
(102 )

(b) Find the centroid of R.

Homework (page 561). Find the centroid of the region bounded by y = sin x, y =
cos x, x = 0, and x = 4 .

8.3-9

Chapter 9
9.1

Differential Equations

Modeling with Differential Equations


(page 580)

A ordinary differential equation is an equation that contains an unknown function


and some of its derivatives. In a real world problem, we use the mathematical model
in form of a differential equation because we often notice that changes occur and we
want to predict future behavior on the basis of how current values change.

Models of Population Growth, page 580


Example 1 (page 580). One model for the growth of a population is based on
the assumption that the population grows at a rate proportional to the size of
the population. That is a reasonable assumption for a population of bacteria or
animals under ideal condition (unlimited environment, adequate nutrition, absence
of predators, immunity from disease.)
Lets identify and name the variables in this model:
t = time.
P = P (t) = the number of individuals in the population.
So we can get the differential equation:
dP
= kP,
dt

(1)

where k is the proportionality constant.


We know from Chapter 3 that exponential functions satisfy the equation (1).
Here we can solve the equation by the method of integration:
dP
= kP
dt

In this formula, we allow C to vary through all the real numbers, and we get the
family of solution. But in real world problem, populations have only positive values
and so we are interested only in the solutions with C0 > 0. (We probably concerned
only with values of t greater than the initial time t = 0.) We can put t = 0 and
get P (0) = C0 ek 0 = C0 , so the constant C0 turns out to be the initial population
P (0).
9.1-1

Example 2 (Logistic differential equation, page 581). Example 1 shows a model


for population growth under ideal conditions, but we have to recognize that a more
realistic model must reflect that the fact that a given environment has limited resources. Many populations starts by increasing in an exponential manner, but the
population levels off when it approaches its carrying capacity M (or decreases toward M if it ever exceed M). For a model to take into account both trends we make
to assumptions:

dP
dt

kP if P is small. (Initially, the growth rate is proportional to P ).

dP
dt

< 0 if P > M. (P decreases if it ever exceeds M.)

A simple expression that incorporates both assumptions is given by the equation




P
dP
.
(2)
= kP 1
dt
M
Why this differential equation is reasonable?
If P is small compared with M, then
If P > M, then 1

P
M

P
M

is closed to 0 and so

is negative and so

dP
dt

dP
dt

kP .

< 0.

Equation (2) is called logistic differential equation and we can solve the equation by
the method of integration:


dP
k
P
=
= kP 1
P (M P )
dt
M
M

If the initial population P (0) lies between 0 and M, then


.
population

dP
dt

If the population exceeds the carrying capacity (P > M), then


.
the population

, and the

dP
dt

, and

In either case, if the population approached the carry capacity (p M), then
dP
, which means the population levels off.
dt
The graphs move away from the equilibrium solution P = 0 and move toward
the equilibrium solution P = M.
9.1-2

A Model for the Motion of a Spring, page 582


We consider the motion of an object with mass m at the end of a vertical spring.
Hooks Law says that if the spring is stretched (or compressed) x units from its
natural length, then it exerts a force that is proportional to x:
restoring force = kx,
where k is a positive constant (called the spring constant). If we ignore any external
resisting force (due to air resistance or friction), by Newtons Second Law, we have
m

d2 x
= kx.
dt2

(3)

This is an example of a second-order differential equation. All solutions of (3) can


be written as
x(t) = A sin kt + B cos kt,

General Differential Equations, page 582


In general, a differential equation is an equation that contains an unknown function
and one or more of its derivatives. The order of a differential equation is the order
of the highest derivative that occurs in the equation.
A function f is called a solution of a differential equation if the equation is
satisfied when y = f (x) and its derivatives are substituted into the equation.
When apply differential equations, we are usually not as interested in finding
a family of solution (the general solution). In many problems we need to find the
particular solution that satisfies a condition of the form y(t0 ) = y0 . This is called an
initial condition, and the problem of finding a solution of the differential equation
that satisfies the initial condition is called an initial-value problem.
Homework (page 585). Psychologists interested in learning theory study learning
curves. A learning curve is the graph of a function P (t), the performance of someone
learning a skill as a function of the training time t. The derivative dP
represents the
dt
rate at which performance improves.
If M is the maximum level of performance of which the learner is capable, then
dP
= k(M P (t))
dt
is a model for learning, where k is a positive constant. Solve it as a linear differential
equation and use you solution to graph the learning curve.
9.1-3

9.2

Direction Fields and Eulers Method


(page 585)

Unfortunately, it is impossible to solve most differential equations in the sense of


obtaining an explicit formula for the solution. In this section we show that, despite
the absence of an explicit solution, we can still learn a lot about the solution through
a graphical approach (direction fields) or a numerical approach (Eulers method).

Direction Field, page 585


Suppose we have a first-order differential equation of the form
dy
= F (x, y),
dx
where F (x, y) is some expression in x and y. The differential equation says that
the slope of a solution curve at a point (x, y) on the curve is F (x, y). If we draw
short line segments with slope F (x, y) at several points (x, y), the result is called a
direction field (or slope field).
This line segments indicate the direction in which a solution curve is heading, so
the direction field helps us visualize the general shape of these curves.
Example 1 (page 592). Match the differential equation with its direction field.
(a) y = 2 y. (b) y = x(2 y). (c) y = x + y 1. (d) y = sin x sin y.
3

-3

-2

-1

-2

-1

-1

-1

-2

-2

-3

( )

-3

( )

-3

3
3

-3

-2

-1

-3

-2

-2

-3

-1
-1

-1

( )

-2

( )

-3

Figure 1: Direction field method.

9.2-1

Eulers Method, page 589


Eulers methods says to start at the point given by the initial value and proceed
in the direction indicated by the direction field. Stop after a short time, look at
the slope at the new location, and proceed in that direction. Keep stopping and
changing direction according to the direction field.
Euler Method (page 590). Approximate values for the solution of the initial-value
problem y = F (x, y), y(x0) = y0 , with step size h, at xn = xn1 + h, are
yn = yn1 + hF (xn1 , yn1 ),

n = 1, 2, 3, . . . .

Eulers method does not produce the exact solution to an initial-value problem
it gives approximations. But by decreasing the step size, we obtain successively
better approximations to the exact solution.
Example 2 (page 590). Use Eulers method with step sizes 0.5, 0.1, 0.01, 0.001 to
construct a table of approximate values for the solution of the initial-value problem
y = x + y,

y(0) = 1.

Compare the graph of the Euler approximates with the exact solution 2ex x 1.
Solution. We can use Mathematica to get
Step size Euler estimate of y(0.5) Euler estimate of y(1)
0.5 1.5000
0.1 1.7210
0.01 1.78926
Exact solution

2.50000
3.18749
3.40963

1.79744

3.43656

2.5

1.5

0.2

0.4

0.6

Figure 2: Euler method.

9.2-2

0.8

Autonomous, page 589


A differential equation of the form
y = f (y)
in which the independent variable is missing from the right side, is called autonomous. For such an equation, the slopes corresponding to two different points
with the same y-coordinate must be equal. This means that if we know one solution
to an autonomous differential equation, then we can obtain infinitely many others
just by shifting the graph of the known solution to the right or left.

9.2-3

9.3

Separable Equations (page 594)

In this section, we examine a certain type of differential equation that can be solved
explicitly.
Definition 1 (page 594). A separable equation is a first-order differential equation
dy
in which the expression for dx
can be factored as a function of x times a function of
y. In other words, it can be written in the form

dy
= g(x)f (y).
dx
Equivalently, if f (y) 6= 0, we could write
1 dy
= g(x).
f (y) dx
To solve this equation, we integrate both sides of the equations:
Z
Z
Z
Z
1
1 dy
dx = g(x) dx
dy = g(x) dx.
f (y) dx
f (y)
Example 2. Solve the initial value problem:
(
(xy 2 + y 2 + x + 1) dx + (y 1) dy = 0,
.
y(2) = 0
(101 )
Solution.

Homework. Solve the differential equation:

1 x2
dy
=
0<x1
.
dx
x

y(1) = 0
9.3-1

(89 )

Homework. Solve the differential equation:


(
dy
= ey+sin x x ( 2 , 6 )
sec x dx
y(0) = 0

.
(95 )

Homework. Solve

dy
y 2 sin x
=
with the initial condition y(0) = 1. (99 )
dx
1 + y3

Homework. Solve the initial value problem y =

(y4)(2x+1)
,
x2 +1

y(0) = 1.
(97 )
y

Example 3 (page 600). Solve the differential equation xy = y + x e x by making


the change of variable v = xy .
Solution.

Orthogonal Trajectories, page 597


Definition 4 (page 597). An orthogonal trajectory () of a family of curves
is a curve that intersects each curve of the family orthogonally, that is, at right
angles.
In section 3.5, Example 6, we have list many examples:
x2 + y 2 = r 2 , ax + by = 0.
x2 + y 2 = ax, x2 + y 2 = by.
y = cx2 , x2 + 2y 2 = k.
y = ax3 , x2 + 3y 2 = b.
Given a family of curves, we can get an orthogonal trajectory of the family by solving
the differential equation.
9.3-2

Example 5 (page 597). Show that the orthogonal trajectories of the family of curves
x2 + y 2 = r 2 is ax + by = 0.
Solution.

Homework.
(a) Show that the orthogonal trajectories of the family of curves x2 + y 2 = ax is
x2 + y 2 = by.
(b) Show that the orthogonal trajectories of the family of curves y = cx2 is x2 +
2y 2 = k.
(c) Show that the orthogonal trajectories of the family of curves y = ax3 is x2 +
3y 2 = b.

(98 )

Mixing Problems, page 598


A typical mixing problem involves a tank of fixed capacity filled with a thoroughly
mixed solution of some substance, such as salt. A solution of a given concentration
enters the tank at a fixed rate and mixture, thoroughly stirred, leaves at a fixed
rate, which may differ from the entering rate.
If y(t) denotes the amount of substance in the tank at time t, then y (t) is the rate
at which the substance is being added minus the rate at which it is being removed.
The mathematical description of this situation often leads to a first-order separable
differential equation.
We can use the same type of reasoning to model a variety of phenomena: Chemical reactions, discharge of pollutants into a lake, injection of a drug into the bloodstream.
9.3-3

Example 6 (page 599). A tank contains 20 kg of salt dissolved in 5000 L of water.


Brine that contains 0.03 kg of salt per liter of water enters the tank at a rate of
25 L/min. The solution is kept thoroughly mixed and drains from the tank at the
same rate. How much salt remains in the tank after half an hour?
Solution. Let y(t) be the amount of salt (in kilograms) after t minutes. We are
given that y(0) = 20 and we want to find y(30). Notice that
dy
= (rate in) (rate out),
dt
where (rate in) is the rate at which salt enters the tank and (rate out) is the rate at
which salt leaves the tank. We have
(rate in) =
(rate out) =
dy
=
dt
Solving this separable differential equation, we obtain

Homework (page 601). Experiments show that the reaction H2 + Br2 2HBr
satisfies the rate law
1
d[HBr]
= k[H2 ][Br2 ] 2
dt
and so this reaction the differential equation becomes
1
dx
= k(a x)(b x) 2 ,
dt
where x = [HBr] and a and b are the initial concentrations of hydrogen and bromine.

(a) Find x as a function of t in the case where a = b. Use the fact that x(0) = 0.
(b) If a > b, find t as a function of x.
9.3-4

9.4

Models for Population Growth (page 605)

Other Models for Population Growth, page 612


The Law of Natural Growth and the logistic differential equation are not the only
equations that have been proposed to model population growth.
Example 1 (page 615). Another model for a growth function for a limited population is given by the Gompertz function, which is a solution of the differential
equation
dP
= c ln
dt

M
P

P,

where c is a constant and M is the carrying capacity.


Example 2 (page 615). In a seasonal-growth model, a periodic function of time is
introduced to account for seasonal variations in the rate of growth. Such variations
could, for example, be caused by seasonal changes in the availability of food. For
example, we can consider the seasonal-growth model
dP
= kP cos(rt ),
dt

P (0) = P0 ,

where k, r, and are positive constants. We can alter the differential equation as
dP
= kP cos2 (rt ),
dt

P (0) = P0 ,

Example 3 (page 612, 615). The differential equation




P
dP
c
= kP 1
dt
M
has been used to model populations that are subject to harvesting of one sort or
another. (Think of a population of fish being caught at a constant rate.)
Example 4 (page 612, 615). For some species there is a minimum population level
m below which the species tends to become extinct. Such populations have been
modeled by the differential equation


dP
m
P 
1
,
= kP 1
dt
M
P
where the extra factor 1

m
,
P

take into account the consequences of a sparse popu-

lation.

9.4-1

9.5

Linear Equations (page 616)

Definition 1 (page 616). A first-order linear differential equation (


) is one that can be put into the form
dy
+ P (x)y = Q(x),
dx

(1)

where P (x) and Q(x) are continuous functions on a given interval.


Every first-order linear differential equation can be solved by the method of integrating factor (). The idea comes from the Product Rule of the derivative.
Suppose I(x) is a differentiable function. We multiply I(x) on both sides of (1):
I(x)

dy
+ I(x)P (x)y = I(x)Q(x),
dx

(2)

We hope left hand side of equation (2) is the derivative of the product I(x)y(x):
dy dI
d
(I(x)y(x)) = I(x)
+
y.
dx
dx dx

(3)

Suppose that such I(x) exists, then we can solve the equation (1) as follows:
d
(I(x)y(x)) = I(x)Q(x)
dx

Now we prove the existence of I(x). Equation (3) gives


d
dy dI
dy
(I(x)y(x)) = I(x)
+
y = I(x)
+ I(x)P (x)y,
dx
dx dx
dx
so we get a condition:
dI
= I(x)P (x) for y 6= 0.
dx
This is a separable differential equation for I, so we can solve it:
1 dI
= P (x)
I(x) dx

Hence one solution is I(x) = e

P (x) dx

Conclusion (page 617). To solve the linear differential equation y +P (x)y = Q(x),
R
multiply both sides by the integrating factor I(x) = e P (x) dx and integrate both
sides.
9.5-1

Remark 2. 1,
Example 3.
(a) Find the particular solution of 3xy y = ln x + 1, x > 0, satisfying y(1) = 2.
(98 )

(b) Find the value y(e).


Solution.

Example 4. Solve the initial value problem:


(
y + y sec2 x = xe tan x
y(0) =

.
(100 )

Solution.

Homework. Solve the differential equation xy 2y = x3 sec x tan x, x > 0, y( 4 ) =


0.
(96 )
9.5-2

Homework. Solve the differential equation x(x+1)y +y +(x+1)2 cos x = 0, x > 0,


with y() = + 1.
(97 )
dy
+ 3x(x2 + 1)y = 2x with the initial condition y(0) =
Homework. Solve (x2 + 1)2
dx
3.
(99 )
Homework. Solve the initial value problem:
(
(sec x)y + y = (tan x)ecos xsin x 0 x < 2 ,
.
y(0) = 0
(101 )
Homework. Solve xy 3y = 5x3
(a) with the initial condition y(1) = 2.
(102 )

(b) with the initial condition y(1) = 2.

Homework. Let y = h(x) be decreasing on [0, 2 ) and is continuously differentiable


on (0, 2 ) with h(0) = 0. Let s(x) denote the arc length of y = h(x) from (0, 0) to
(x, h(x)).
(a) Write down the formula for s(x).
(b) Suppose that s(x) is also given by s(x) =

eh(t) dt. Find the function h(x)

explicitly.
(c) Find the function s(x) explicitly.

(102 )

Application to Electric Circuits, page 587, 619.


The simple electric circuit shown in Figure 1 contains an electromotive force (
) (usually a battery or generator) that produces a voltage of E(t) volts (V) (
; : ) and a current of I(t) amperes (A) (; : ) at time t. The
circuit also contains a resistor with a resistance of R ohms () (; : )
and an inductor with an inductance of L henries (H) (; : ). Ohms
Law gives the drop in voltage due to the resistor as RI. The voltage drop due to
the inductor is L dI
. One of Kirchhoffs laws says that the sum of the voltage drops
dt
is equal to the supplied voltage E(t). Thus we have
L

dI
+ RI = E(t),
dt

which is a first-order differential equation that models the current I at time t.


9.5-3

switch
Figure 1: Electric circuits.
Example 5 (page 619). Suppose that the resistance is 12 and the inductance is
4 H. If a battery gives a constant voltage of E(t) = 60 sin 30t V and the switch is
closed when t = 0. Find I(t).
Solution.

Homework (page 619). Suppose that the resistance is 12 and the inductance is
4 H. If a battery gives a constant voltage of E(t) = 60 V and the switch is closed
when t = 0.
(a) Find I(t).
(b) Find the current after 1 s.
(c) Find the limit value of the current.

9.5-4

Chapter 10 Parametric Equations and


Polar Coordinates
10.1

Arc Length (page 636)

Definition 1 (page 636). Suppose that x and y are both given as functions of a
third variable t (called a parameter) by the equations
x = f (t),

y = g(t),

(called parameter equations). Each value of t determines a point (x, y), which we
can plot in a coordinate plane. As t varies, the point (x, y) = (f (t), g(t)) varies and
traces out a curve C, which we call a parametric curve.
Sometimes t can be realized as time and we can interpret (x, y) = (f (t), g(t))
as the position of a particle at time t, but in many cases, t does not necessarily
represent time, it is just a variable.
Example 2. How do we express the following curves by parametric equations?
Curve
Straight line passing through (x0 , y0)
Circle with center (x0 , y0) and radius r
Ellipse

(x x0 )2 (y y0 )2
+
=1
a2
b2

Hyperbola

x=
( y=
x=
(

Parabola (x x0 )2 = 4p(y y0 )
2

Parametric Equation
(
x=
( y=

(x x0 )
(y y0 )

=1
2
a
b2

y=
x=
y=

x=
y=

Example 3. Compare the following parametric equations:


(a) x = (cos t, sin t), 0 t 2.
(b) x = (cos t, sin t), 0 t 4.
(c) x = (cos 2t, sin 2t), 0 t .
(d) x = (sin t, cos t), 0 t 2.
10.1-1

Example 4 (page 638). Sketch the curve with parametric equations x = sin t, y =
sin2 t.
Solution.

The Cycloid, page 639


Example 5 (page 639). The curve traced out by a point P on the circumference of
a circle as the circle rolls along a straight line is called a cycloid (). See Figure 1.
y

r
x

O
Figure 1: The cycloid.

If the circle has radius r and rolls along the x-axis and if one position of P is the
origin, find parametric equations for the cycloid.
Solution.

10.1-2

There are many interesting problems related to cycloids.


Brachistochrone Problem (), page 640
Find the curve along which a particle will slide in the shortest time (under the
influence of gravity) from a point A to a lower point B not directly beneath A.
A

B
Figure 2: Brachistochrone Problem.
The Swiss mathematician John Bernoulli, who posed this problem in 1696,
showed that among all possible curves that join A to B, the particle will take the
least time sliding from A to B if the curve is part of an inverted arch of a cycloid.
Tautochrone Problem (), page 640
The Dutch physicist Huygens had already shown that the cycloid is also the solution
to the tautochrone problem: no matter where a particle P is placed on an inverted
cycloid, it takes the same time to slide to the bottom.

P
P

Figure 3: Brachistochrone Problem.


Huygens proposed that pendulum clocks should swing in cycloidal arcs because
then the pendulum would take the same time to make a complete oscillation whether
it swings through a wide or a small arc.

10.1-3

Graphing Devices, page 638


We can use graphing devices to sketch complicated curves. The curves shown in
Figure 4 are almost impossible to produce by hand.
1

1.5
1.5

0.5

-1.5

-1

-0.5

0.5

0.5

0.5

-1.5

1.5

-1

-0.5

0.5

1.5

-2

-1

-0.5

-0.5

-0.5

-1

-1

-1.5

-1

-1.5

Figure 4: (a) x = sin t + 21 cos 5t + 14 sin 13t, y = cos t + 12 sin 5t + 14 cos 13t, t [0, 2].
(b) x = sin t+ 21 sin 5t+ 41 cos 2.3t, y = cos t+ 21 cos 5t+ 41 sin 2.3t, t [0, 20].
(c) x = sin t sin 2.3t, y = cos t, t [0, 20].


Homework (page 644). Find parametric equations for the set of all points P as
shown in the Figure 5 such that |OP | = |AB|. This curve is called the cissoid of
Diocles after the Greek scholar Diocles, who introduced the cissoid as a graphical
method for constructing the edge of a cube whose volume is twice that of a given
cube.
y
B
A

P
O

x = 2a
x

Figure 5: The cissoid of Diocles.

10.1-4

Homework (page 653). A string is wound around a circle and then unwound while
being held taut. The curve traced by the point P at the end of the string is called
the involute of the circle. If the circle has radius r and center O and the initial
position of P is (r, 0), an if the parameter is chosen as the Figure 6, show that
parametric equations of the involute are
x = r(cos + sin ),

y = r(sin cos ).

y
T
r

P
x

Figure 6: The involute of the circle.

10.1-5

10.2

Calculus with Parametric Curves (page 645)

We now apply the methods of calculus to parametric curves. In particular, we solve


problem involving tangents, area, arc length, surface area, and volume.

Tangents, page 645


Suppose f and g are differentiable functions and we want to find the tangent line at
a point on the curve x = f (t), y = g(t), where y is also a differentiable function of
x. Then the Chain Rule gives
dy
dy dx
=
.
dt
dx dt
if

dx
dt

6= 0, we can solve for

dy
:
dx

dy
=
dx

dy
dt
dx
dt

dx
6= 0.
dt

if

We can compute the second derivative

d2 y
dx2

as follows:

 
d dy
d dy
d2 y
dt dx
= dx .
=
dx2
dx dx
dt

d2 y
6
=
:
dx2

d2 y
dt2
d2 x
dt2

Example 1 (page 646).


(a) Find the tangent to the cycloid x = r( sin ), y = r(1 cos ) at the point
where = 3 .
(b) At what points its tangents horizontal? When is it vertical?
Solution.

10.2-1

Homework (page 651). Find

dy
dx

and

d2 y
.
dx2

For which values of t is the curve concave

upward?
(a) x = t2 + 1, y = t2 + t.
(b) x = cos 2t, y = cos t, 0 < t < .

Areas, page 647


We know that the area under a curve y = F (x) from a to b is A =

Rb
a

F (x) dx,where

F (x) 0. If the curve is traced out once by the parameter equations x = f (t) and
y = g(t), t , then we can calculate an area formula by using the Substitution
Rule for Definite Integrals as follows:
Z b
Z
A=
y dx =
g(t)f (t) dt or
a

g(t)f (t) dt.

Example 2 (page 647). Find the area under one arch of the cycloid x = r(
sin ), y = r(1 cos ).
(98 )
Solution.

Homework (page 651). Find the area of the region enclosed by the astroid x 3 +y 3 =
2
a3 .
Homework (page 653). A cow is tied to a silo with radius r by a rope just long
enough to reach the opposite side of the silo. Find the area available for grazing by
the cow.
Hint: Part of the curve is the involute of the circle. (See Homework in section
10.1.)
10.2-2

Arc Length, page 648


We already know how to find the length L of a curve C given in the form y =
F (x), a x b. If F (x) is continuous, then
s
 2
Z b
dy
dx.
1+
L=
dx
a
Suppose that C can also be described by the parametric equations x = f (t) and
y = g(t), t , where dx
= f (t) > 0. This means that C is traversed once,
dt
from left to right, as t increases from to and f () = a, f () = b. Then we
obtain
s
s
s
 2

2
Z  2  2
Z
Z b
dy
dy/dt
dx
dy
dx
dt =
dx =
+
dt.
L=
1+
1+
dx
dx/dt
dt
dt
dt

a
The above formula is generally true even if C cant expressed in the form y = F (x).
Theorem 3 (page 649). If a curve C is described by the parametric equations x =
f (t), y = g(t), t , where f and g are continuous on [, ] and C is traversed
exactly once as t increases from to , then the length of C is
s
Z  2  2
dy
dx
+
dt.
L=
dt
dt

Example 4 (page 649). Find the length of one arch of the cycloid x = r(
sin ), y = r(1 cos ).
(100 )
Solution.

Homework. Find the length of the astroid x 3 + y 3 = 1.


(95 )

Homework. Consider the parametric curve with x (t) = 3t, y (t) = 2t t t2 , 0


t 1. Find the arc length of this curve.
(99 )
Homework. Find the arc length of the curve x = t sin 2t, y = t cos 2t, 0 t 1.
(102 )
10.2-3

Surface Area, page 650


In the same way as for arc length, we can obtain a formula for surface area. If the
curve given by the parametric equations x = f (t), y = g(t), t , is rotated
about the x-axis, where f , g are continuous and g(t) 0, then the area of the
resulting surface is given by
s 
 2
Z
2
dy
dx
+
dt.
S=
2y
dt
dt

R
R
The general symbolic formulas S = 2y ds and S = 2x ds are still valid, but
for parametric curves we use
s 
 2
2
dy
dx
+
dt.
ds =
dt
dt
Example 5 (page 650). Show that the surface area of a sphere of radius r is 4r 2.
Solution.

Homework (page 652). Find the exact area of the surface obtained by rotating the
curve x = a cos3 , y = a sin3 , 0

about the x-axis.

Volume
See section 6.2 and 6.3.
Homework. Find the volume of the solid generated by revolving the region bounded
by one arch of the cycloid: x = r( sin ), y = r(1 cos ), and y = 0 about the
(101 )

x-axis.

10.2-4

10.3

Polar Coordinates (page 654)

A coordinate system represents a point in the plane by an ordered pair of numbers.


We usually use Cartesian coordinates (, ), which are directed
distances from two perpendicular axes. Here we describe another coordinate system
introduced by Newton, called the polar coordinate system ().
P (r, )

P (r, )

r
O

polar axis

r
+
x
O

polar axis

Q(r, ) = (r, + )
Figure 1: Polar coordinate system. We choose a point in the plane that is called the
pole and is labeled O. Then we draw a ray starting at O called polar axis, which is
usually corresponds to the positive x-axis in Cartesian coordinates.
Here are some remarks about the polar coordinate system.
If P is any other point in the plane, let r be the distance from O to P and let
be the angle between the polar axis and the line OP . Then the point P is
represented by the ordered pair (r, ) are called polar coordinates of P .
We use the convention that an angle is positive if measured in the counterclockwise direction from the polar axis and negative in the clockwise direction.
If P = O, then r = 0 and we agree that (0, ) represents the pole for any value
of .
The points (r, ) an (r, ) lie on the same line through O and at the same
direction |r| from O, but on opposite sides of O. If r > 0, the point (r, ) lies
in the same quadrant as ; if r < 0, it lies in the quadrant on the opposite side
of the pole.
Notice that (r, ) represents the same point as (r, + ).
The connection between polar and Cartesian coordinates:
y
x
, sin = .
r
r
(b) x = r cos , y = r sin .
y
(c) r 2 = x2 + y 2 , tan = .
x

()

(a) cos =

()
()
10.3-1

Polar Curves, page 656


The graph of a polar equation r = f (), or more generally F (r, ) = 0, consists of all
points P that have at least one representation (r, ) whose coordinates satisfy the
equation.
Example 1 (page 656-657). Plot the following curves represented by the polar
equation and find a Cartesian equation for this curve.
(a) r = 2.
(b) = 4 .
(c) r = 2 cos .
(d) r = 1 + sin .
(e) r = cos 2.
Solution.

Symmetry, page 659


When we sketch polar curves it is sometimes helpful to take advantage of symmetry.
(a) If a polar equation is unchanged when is replaced by , the curve is symmetric about
.
(b) If the equation is unchanged when r is replaced by r, or when is replaced
by + , the curve is symmetric about
.
(c) If the equation is unchanged when is replaced by , the curve is symmetric
about
.
10.3-2

Tangents to Polar Curves, page 659


To find a tangent line to a polar curve r = f (), we regard as a parameter and
write its parametric equations as
(
x = r cos = f () cos
y = r sin = f () sin .

Using the method for finding slopes of parametric curves and the Product Rule, we
have
dy
=
dx

(1)

Horizontal tangents:

(provided that

Vertical tangents:

(provided that

dy
d

dx
d

6= 0).

6= 0).

Tangent lines at the pole: we put r = 0 into formula (1) and get
dy
=
dx
Example 2 (page 660).
(a) For the cardioid r = 1 + sin , find the slope of the tangent line when = 3 .
(b) Find the points on the cardioid where the tangent line is horizontal or vertical.
Solution.

10.3-3

Homework. r = 1 + cos x

(96 )

Homework (page 663). Find a formula for the distance between the points with
polar coordinates (r1 , 1 ) and (r2 , 2 ).
Homework (page 664). Find the slope of the tangent line to the given polar curve
at the point specified by the value of . (a) r = 1 , = . (b) r = cos 2, = 4 .
Homework (page 664). Find the points on the given curve the tangent line is
horizontal or vertical. (a) r = 3 cos . (b) r = 1 sin .

Graphing Polar Curves with graphing Devices


We can use graphing devices to sketch complicated curves. The curves shown in
Figure 2 are almost impossible to produce by hand.
1

1.5
1

0.5

0.5

0.5
-1

-0.5

0.5

-2

-1

-1

-0.5

0.5

-0.5
-0.5

-1

-0.5

-1.5

-1

-1

4
, 22 ]. (b) r = sin2 (1.2) +
Figure 2: (a) r = sin2 (2.4) + cos
(2.4), [ 2
2.4 2.4

cos3 (6), [0, 6] (c) r = sin 85 , [0, 10].

Some interesting curves and their polar equations.


(a) r = a sin(b): rose or rhodonea curve ().
(b) r = a + b: Archimedean spiral (; ).
(c) r = aeb : logarithmic spiral ().
(d) r 2 = sin 2: lemniscate ().
(e) r = esin 2 cos(4): butterfly curve ().
(f) r = 1 + c sin : limacons de Pascal. ().
(g) r = 1 + 2 sin( 2 ): nephroid of Freeth.
p
(h) r = 1 0.8 sin2 : hippopede.
(i) r = | tan || cot | : MacMan curve.

10.3-4

10.4

Areas and Length in Polar Coordinates


(page 665)

In this section, we develop the formula for the area of the region whose boundary is
given by a polar equation.
Example 1 (page 665). The area of a sector of a circle with the radius r and the
.
radian is A =
Example 2 (page 665). Find the area of a region R bounded by the polar curve
r = f () and by rays = a and = b, where f () is a positive continuous function
and 0 < b a 2.
Solution.

Example 3 (page 666). Find the area enclosed by one loop of the four-leaved rose
r = cos 2.
Solution.

10.4-1

Example 4 (page 667). Find the area of the region R bounded by curves with
polar equations r = f (), r = g(), = a, and = b, where f () g() 0 and
0 < b a 2.
Solution.

To find all points of intersection of two polar curves, it is recommended that you
draw the graphs of both curves.
Example 5 (page 667). Find all points of intersection of the curves r = cos 2 and
r = 21 .
Solution.

Example (TA) 6. Find the area of the region that lies inside the curve r = 2+cos 2
but outside the curve r = 2 + sin .
(102 )
Solution.

10.4-2

Arc Length, page 667


To find the length of a polar curve r = f (), a b, we regard as a parameter
and write the parameter equations of the curves as
(
x = r cos = f () cos
y = r sin = f () sin .

Then using the Product Rule and differentiating with respect to , we obtain
dx
=
d
dy
=
d
so


dx
d

2

dy
d

2

=
Assuming that f () is continuous, we can write the arc length as
s
Z b  2  2
dx
dy
+
d =
L=
d
d
a
Example 7 (page 668). Find the length of the cardioid r = 1 + sin .
Solution.

10.4-3

Example (TA) 8 (page 669). Find the exact length of the polar curve r = 5 , 0
2.
Solution.

Homework (page 659). Find the area of the region that lies inside both curves
r = sin 2 and r = cos 2.
Homework.
(a) Find all points of intersection of the curves r = cos and r = 1 cos .
(b) Find the area of the region that lies inside the circle r = cos and outside the
cardioid r = 1 cos .
(95 )
Homework. Consider the cardioid by r = 1 cos , 0 2.
(a) Find the area enclosed by this curve.
(97 )

(b) Find the length of this curve.


Homework.

(a) Plot the region A which is inside the circle r = 6 cos and outside the cardioid
r = 2(1 + cos ).
(b) Find the area of A.
(c) Find the length of the boundary of the region A.

(98 )

Homework. Consider the cardioid r = 1 2 cos .


(a) Find the slope of the tangents to the cardioid at the region.
(b) Find the area of the region between the inner and outer loops of the cardioid.
(100 )
Homework. Find the area both inside r 2 = 2 cos 2 and inside r = 1.
(101 )
10.4-4

Surface Area, page 670


The area of the surface generated by rotating the polar curve r = f (), a b
(where f () is continuous and 0 a < b ) about the polar axis is
Surface area =
=
The area of the surface generated by rotating the polar curve r = f (), a b
(where f () is continuous and 0 a < b ) about the line = 2 is
Surface Area =
=
Homework. Find the surface area of the solid generated by revolving the cardioid
r = 1 + sin about y-axis.
(99 )
Homework. Find the area of the surface generated by revolving the curve r =
(101 )
sin , 0 2 about the x-axis.

Volume
See section 6.2 and 6.3.

10.4-5

10.6

Conic Sections in Polar Coordinates


(page 678)

We have learned from high school about conic sections, such as ellipse (),
parabola (), and hyperbola (). At that time, we defined the parabola
in terms of a focus () and directrix (), but defined the ellipse and hyperbola
in terms of two foci.
In this section we give another unified treatment of all three types of conic
sections in terms of a focus and directrix. Furthermore, if we place the focus at the
origin, then a conic section has a simple polar equation.
Theorem 1 (page 678). Let F be a fixed point (called the focus) and L be a fixed
line (called the directrix) in a plane. Let e be a fixed positive number (called the
eccentricity ). The set of all points P in the plane such that
|P F |
=e
|P L|

(1)

(that is the ratio of the distance from F to the distance from L is the constant e) is
a conic section. The conic is an ellipse if e < 1, a parabola if e = 1, or a hyperbola
if e > 1.
y
P
r

L:x=d
Figure 1: Conic sections in polar coordinates.
Proof. If e = 1, then |P F | = |P L|, so it is a parabola.
We place the focus F at the origin and the directrix parallel to the y-axis an d
units to the right. (See Figure 1.) If the point has polar coordinates (r, ), then we
know
|P F | = r,

and |P L| = d r cos .

10.6-1

So the condition (1) becomes


r = e(d r cos ).

(2)

If we square both sides of this polar equation and convert to rectangular coordinates,
we get
x2 + y 2 =
After completing the square, we have

2
e2 d
y2
e2 d2
x+
+
=
.
1 e2
1 e2
(1 e2 )2
If e < 1, this is an ellipse of the form
(x h)2 y 2
+ 2 = 1,
a2
b

where h =

a2 =

b2 =

We have to show the focus of an ellipse defined in Theorem 1 is the same as the
focus defined in Section 10.5. It suffices to check
c2 = a2 b2 =

e4 d2
e2 d

c
=
= h.
(1 e2 )2
1 e2

If e > 1, this is a hyperbola of the form


(x h)2 y 2
2 = 1,
a2
b

where h =

a2 =

b2 =

We have to show the focus of a hyperbola defined in Theorem 1 is the same as the
focus defined in Section 10.5. It suffices to check
c2 = a2 + b2 =

e4 d2
e2 d

c
=
= h.
(e2 1)2
e2 1

Remark 2. The eccentricity is given by e = ac .


Theorem 3 (page 680). A polar equation of the form
ed
ed
or r =
1 e cos
1 e sin
represents a conic section with eccentricity e. The conic is an ellipse if e < 1, a
parabola if e = 1, or a hyperbola if e > 1.
r=

Proof. From (2), we get the polar equation for the conic.
r = e(d r cos )

10.6-2

Homework. Show that each conic in Figure 2 has the corresponding polar equation.
y
y

y
r

L:y=d

x
r

P
x

L : x = d
(a)

L : y = d
(c)

(b)

Figure 2: Conic sections. (a) r =

ed
1e cos

(b) r =

ed
1+e sin

(c) r =

ed
.
1e sin

Keplers Law, page 682


In 1609 the German mathematician and astronomer Johannes Kepler, on the basis
of huge amount of astronomical data, published the following three laws of planetary
motion.
Keplers Law (page 682).
(1) A planet revolves around the sun in an elliptical orbit with the sum at one
focus.
(2) The line joining the sun to a planet sweeps out equal areas in equal times.
(3) The square of the period of revolution of a planet is proportional to the cube
of the length of the major axis of its orbit.
Theorem 4 (page 683). The polar equation of an ellipse with focus at the origin,
semimajor axis a, eccentricity e, and directrix x = d can be written in the form
r=

a(1 e2 )
.
1 + e cos

Proof. This is because


a2 =

e2 d2

(1 e2 )2

10.6-3

planet
r

sun

aphelion

perihelion

Figure 3: Perihelion and aphelion.


The position of a planet that are closest to and farthest from the sun are called
its perihelion () and aphelion (), respectively, and correspond to the
vertices of the ellipse. The distance from the sun to the perihelion and aphelion are
called the perihelion distance and aphelion distance, respectively.
Theorem 5 (page 683). The perihelion distance from a planet to the sun is a(1 e)
and the aphelion distance is a(1 + e).
Proof.
At perihelion, we have = 0, so r =
At aphelion, we have = , so r =

a(1e2 )
1+e cos 0

a(1e2 )
1+e cos

a(1e)(1+e)
1+e

a(1e)(1+e)
1e

= a(1 e).

= a(1 + e).

Example 6 (page 683).


(a) Find a polar equation for the elliptical orbit of the earth around the sun (at
one focus) given that the eccentricity is about 0.017 and the length of the
major axis is about 2.99 108 km.
(b) Find the distance from the earth to the sun at perihelion and at aphelion.
Solution.
(a) The length of the major axis is 2a = 2.99 108 , so a = 1.495 108 . We are
given that e = 0.017, so and equation of the earths orbit around the sun is
r=

a(1 e2 )
1.495 108 (1 (0.017)2)
=
.
1 + e cos
1 + 0.017 cos

(b) The perihelion distance from the earth to the sun is


a(1 e) (1.495 108 )(1 0.017) 1.47 108 (km).
and the aphelion distance is
a(1 + e) (1.495 108 )(1 + 0.017) 1.52 108 (km).
10.6-4

Homework (page 684). The orbit of Halleys comet, last seen in 1986 and due to
return in 2062, is an ellipse with eccentricity 0.97 and one focus at the sun. The
length of its major axis is 36.18 AU. (An astronomical unit (AU) is the mean distance
between the earth and the sun, about 93 million miles.) Find a polar equation for
the orbit of Halleys comet. What is the maximum distance from the comet to the
sun?

10.6-5

: () Catenary

(catenary), ,
y
T

(x, y)

(0, a)

1:
1 , y-, (0, a),
H y(x) , (x, y)
T (), T x- () ,
(0, a) (x, y)
Z xp
1 + (y (t))2 dt
= = s =
0

, :

T sin = s
T cos = H,

dy

tan =
= y (x) =
dx
H

x , ,
tan = y (x) =

x
0

1 + (y (t))2 dt,

p
1 + (y (x))2
H

, : v(x) =
p
y (x), v (x) = H 1 + (v(x))2 , v(x)
(separable equation),
Z
Z

p
dv(x) =
dx =
x + C,
H
H
1 + (v(x))2
1

, , tan = v(x), 2 < < 2 , sec2 d = dv(x),

Z
Z
Z
1
1
2
p
p
dv(x) =
sec d = sec d
1 + (v(x))2
1 + tan2
p

2
= ln | sec + tan | + C = ln
1 + (v(x)) + v(x) + C,
p
( 1 + (v(x))2 + v(x) , )
p


ln
x + C
1 + (v(x))2 + v(x) =
H
v(0) = y (0) = 0 () C = 0
,
p
p

1 + (v(x))2 + v(x) = e H x 1 + (v(x))2 = e H x v(x),


,

1 + (v(x)) = e

2
x
H

2v(x)e

x
H

e H x e H x

+ (v(x)) v(x) = y (x) =


2
2

,
H
y(x) =

e H x + e H x
2

+ C ,

C , C = 0 y(0) = a a =
 x
x 
x
e a + e a
y(x) = a
= a cosh

2
a
. , , :
(8.1)
(2.7)
(5.5)
(9.3)
(9.3)
() (7.3)
sec x (7.2)
ex (Chain Rule) (3.4)
cosh x (3.11)
2

H
,

: Brachistochrone Problem
A B () , A B
A

1:
, (Snells Law)
:1 A 1 ( v1 ) 2 ( v2 ) B ,
1 2
sin 1
v1
sin 1
sin 2
=

sin 2
v2
v1
v2
A

y0
v1 y
1
v2
y2

1, v1

..
.

2, v2

vn

yn1
yn

B
y

2:
A B n , i = 1, . . . , n, yi = yi yi1
vi , i [yi1 , yi ] ,
, ,
sin 1
sin 2
sin n
=
= =
,
v1
v2
vn

n ,
sin
= k = ,
v
1

(1)

x-y A y- y(x)
,


1
1
1
=q
=p
=
sin = cos


2
sec 2
1 + (y (x))2
1 + tan2
2

, , y =
(1) :
k

v2
2g

v=

2gy, g

dy
1
=c
=
2gy = p
y(x)(1 + (y (x)) ) =
2

2
2gk
dx
1 + (y (x))
1

cy

(separable equation), :
r
y
dy
dx =
cy

, x y ,
(), :
y = c sin2 , 0 2 , dy = 2c sin cos d,
s
c sin2
2c sin cos d = 2c sin2 d = c(1 cos 2) d,
dx =
c cos2



sin 2
c
x=c
+ x( = 0) = (2 sin 2) = r( sin )
2
2


c
1 cos 2
= (1 cos 2) = r(1 cos ),
y=c
2
2

r =

c
2

= 2

. , :
(4.7)
(5.1)
(2.7)
(9.3)
() (7.2, 7.3)
(9.3)
(cycloid) (10.1)
2

:
A, B, C, D (1, 1), (1, 1), (1, 1), (1, 1),
1 , : A (B, C, D) B
(C, D, A)
(1) ? ?
(2) A ?
(3) A ?
(4) A ?
B

1:
(1) , A B
, , ,

, 1 , , ,
, , A B , B
,A 1 B , A, B 2
, 2
(2) A r = r(),

:
A : (r cos , r sin ),





, r sin +
= (r sin , r cos ),
B : r cos +
2
2
C : (r cos( + ), r sin( + )) = (r cos , r sin ),





3
3
D : r cos + , r sin +
= (r sin , r cos ),
2
2
1

r = r(t), = (t), A
r cos r sin r sin + r cos
p
, p
(r )2 + ( )2
(r )2 + ( )2

AB = (r(sin + cos ), r(cos sin )) , r 6= 0,


r cos r sin
r sin + r cos
=
,
(sin + cos )
cos sin

r (t) + r(t) (t) = 0,


dr
=
d

dr
dt
d
dt

= r

1
dr = d,
r

, ln |r| = + c r = c0 e ,


r = 2 > 0, ; r = 2, = 4 c0 = 2 e 4 ,

A r = 2 e 4 , 4 1
(3) , (improper integral):
Z
Z
Z p

2
2
2
(r) + (r ()) d =
d = 2 e lim
=
2e

= 2 e 2 lim

e d

i =



e = 2 e 2 lim e + e 2 = 2

= 2

(4) t 2 , , ,

. :
(10.3)
(2.7)
(10.4)
(9.3)
(9.3)
ex (Chain Rule) (3.4)
(7.8)
(2.1)
1

, r = 0 = A t = 0 t = 2

: (Four-leaved rose)
r = cos 2
1

0.5

-1

-0.5

0.5

-0.5

= 4
-1

1: (Four-leaved rose)
2 : = 4 , r = 0 ,
= 4 , r = 0 ,

(a)

(b)

2: (a) ; (b)
(a) (polar equation) , ,

Z
Z 
4 1
1 4 1 + cos 4
2
=
d
(cos 2) d =
2 4
2
4 2

 =



sin 4 4
1 1
1 1
=
+

,
=
=

2 2
8
2 2 2
8
=
4

(b) , , x() = cos 2 cos , y() = cos 2 sin ,


, :
Z
Z
4
4
=
y() dx() =
cos 2 sin d(cos 2 cos )

=
=

Z
Z

cos 2 sin (2 sin 2 cos cos 2 sin ) d

(sin2 2 cos 2 + cos2 2 sin2 ) d,



 = 4
Z
Z

4
1 4
1 1 3
1
2
2
sin 2 cos 2 d =
sin 2 d sin 2 =
sin 2
= ,
2 4
2 3
3
4
=
4

Z
4

cos 2 sin d =

=
=
=
=


1 cos 2
cos 2
d
2
4
Z
Z
1 4
1 4
2
cos 2 d
cos3 2 d

2 4
2 4

Z 
Z
1 4 1 + cos 4
1 4
d
(1 sin2 2) d sin 2
2 4
2
4 4

 =

 = 4

sin 4 4
1 3
1
1
+
sin 2 sin 2


4
4
4
3
= 4
= 4


2
1
1 1
2
= ,

4 2 4
3
8 3

8 ,
: ,
, , :
(r = f (), ), ,
x() = r() cos x () = r () cos r() sin
y() = r() sin y () = r () sin + r() cos ,

ds =

Z
p

2
((x ()) + (y ()) d =

p
r 2 + (r ())2 d,

, ,
: , ,
. , :
(10.3)
(5.2, 10.4)
(5.2, 10.4)
() (7.2)
(10.2, 10.4)
(7.5)
2

( 398, 495 )
Z

x dx =
n

e dx =
x

sin x dx =
2

sec x dx =
sec x tan x dx =
sec x dx =
tan x dx =

1
dx =
2
x +1
Z
1
dx =
x2 + a2
Z
1
dx =

2
x a2
Z

1
dx =
x
ax dx =
cos x dx =
csc2 x dx =
csc x cot x dx =
csc x dx =
cot x dx =

1
dx =
1 x2
Z
1

dx =
a2 x2
Z
1
dx =

2
x a2
Z

sin2 + cos2 =
1 + tan2 =
1 + cot2 =
sin(x y) =
cos(x y) =
sin 2 =
cos 2 =
sin2 =
cos2 =
sin x cos y =
cos x cos y =
sin x sin y =

( 398, 495 )
Z

xn+1
+C
x dx =
n+1
n

(n 6= 1)

ex dx = ex + C
sin x dx = cos x + C
sec2 x dx = tan x + C
sec x tan x dx = sec x + C
sec x dx = ln | sec x + tan x| + C
tan x dx = ln | sec x| + C

1
dx = tan1 x + C
+1
Z
 
1
1
1 x
+C
dx
=
tan
x2 + a2
a
a


Z
x a
1
1
+C

dx =
ln
x2 a2
2a x + a
Z

x2

1
dx = ln |x| + C
x
Z
ax
ax dx =
+C
ln a
Z
cos x dx = sin x + C
Z
csc2 x dx = cot x + C
Z
csc x cot x dx = csc x + C
Z
csc x dx = ln | csc x cot x| + C
Z
cot x dx = ln | sin x| + C
Z
1

dx = sin1 x + C
2
1x
Z
x
1

+ C, a > 0
dx = sin1
a
a2 x2
Z

1
dx = ln |x + x2 a2 | + C

x2 a2
Z

sin2 + cos2 = 1
1 + tan2 = sec2
1 + cot2 = csc2
sin(x y) = sin x cos y cos x sin y,
cos(x y) = cos x cos y sin x sin y
sin 2 = 2 sin cos
cos 2 = cos2 sin2 = 2 cos2 1 = 1 2 sin2
2
sin2 = 1cos
2
2
cos2 = 1+cos
2

sin x cos y = 12 (sin(x + y) + sin(x y))


cos x cos y = 21 (cos(x + y) + cos(x y))
sin x sin y = 21 (cos(x + y) cos(x y))

11 14

James Stewart: Calculus, Early Transcendentals, Seventh
Edition, International Metric Version, c2012.

Chapter 11
11.1

Infinite Sequences and Series

Sequences (page 690)

Definition 1 (page 690).


(1) A sequence () is a list of numbers written in a definite order:
a1 , a2 , a3 , a4 , . . . , an , . . . .
The number a1 is called the first term, a2 is the second term, and in general
an is the n-th term.
(2) An infinite sequence () is a sequence that each term an has a successor
an+1 .
(3) The sequence {a1 , a2 , a3 , . . .} is also denoted by {an } or {an }
n=1 .
Example 2 (page 690). Some sequences can be defined by giving a formula for the
n-th term. There are three methods to describe a sequence. Notice that n doesnt
have to start at 1.
an =

n
,
n+1

n
{ 21 , 32 , 34 , . . . , n+1
, . . .}.

(b) { (1) 3n(n+1) }


n=1 ,

an =

(1)n (n+1)
,
3n

(c) { n 3}
n=3 ,

an =

4
{ 23 , 93 , 27
, . . . , (1) 3n(n+1) , . . .}.

{0, 1, 2, 3, . . . , n 3, . . .}.

(d) {cos n
} ,
6 n=0

, n 0,
an = cos n
6

n
(a) { n+1
}
n=1 ,
n

n 3, n 3,

{1,

3 1
, , 0, . . . , cos n
, . . .}.
2 2
6

Example 3 (page 691). Here are some sequences that dont have a simple defining
equation.
(a) The Fibonacci sequence () {fn } is defined recursively by the conditions
f1 = f2 = 1,

fn = fn1 + fn2 ,

n 3.

The first few terms are {1, 1, 2, 3, 5, 8, 13, 21, 34, 55, . . .}. This sequence arose
when the 13th-century Italian mathematician known as Fibonacci solved a
problem concerning the breeding of rabbits.

(b) If we let an be the digit in the n-th decimal place of the number 2, then {an }
is a well-defined sequence whose first few terms are {4, 1, 4, 2, 1, 3, 5, 6, 2, . . .}.
11.1-1

Definition 4 (page 692). ()


(1) A sequence {an } has the limit L and we write
lim an = L

or

an L as n

if we can make the terms an as close to L as we like by taking n sufficiently


large.
(2) If lim an exists, we say the sequence converges (or is convergent, ). Othn
erwise, we say the sequence diverges (or is divergent, ).
(3) If an becomes large as n becomes large, we use the notation lim an = .
n

Theorem 5. If lim an exists, then it is unique.


n

Property 6 (Limit Laws for Sequences, page 693). If {an } and {bn } are convergent
sequences and c is a constant, then
(1) lim (an + bn ) = lim an + lim bn .
n

(2) lim (an bn ) = lim an lim bn .


n

(3) lim c an = c lim an . In particular, lim c = c.


n

(4) lim (an bn ) = lim an lim bn .


n

(5) lim

an
n bn

(6) lim apn =


n

lim an
n

lim bn

if lim bn 6= 0.

lim an
n

p

if p > 0 and an > 0.

The Squeeze Theorem (, page 695). If an bn cn for n n0 and


lim an = lim cn = L, then lim bn = L.
n

Theorem 7. If lim an = L, then the limit of any subsequences lim ank = L.


n






, !

, ,

Theorem 8 (page 694). If lim |an | = 0, then lim an = 0.


n

Proof. Since

, by the

, we have lim an = 0.
n

11.1-2

Theorem 9 (page 693). If lim f (x) = L and f (n) = an when n is an integer, then
x
lim an = L.
n

Theorem 10 (page 695). If lim an = L and the function f is continuous at L,


n
then
lim f (an ) = f (L).
n

Figure 1: Relations between functions and sequences.

 9 10, , !
 10 :





 n
lim an = 0, lim |an | = lim an = 0 ()
n
n

Example 11. Discuss the convergence or divergence of the following sequences:


n
n2 +1
(a) an = 2n
(b) bn = nn!n (c) cn = (1)
(d) dn = lnnn (e) en = sin( n ).
2 +3n
n
Solution.

Homework (page 700). Determine whether the sequence converges or diverges. If


2
2
it converges, find the limit. (a) an = nn3 +4n (b) bn = (2n1)!
(c) cn = cos2n n (d)
(2n+1)!
n

(e) en = n n + 1 n + 3.
dn = 1 + n2
11.1-3

Theorem 12 (page 696). The sequence {r n }


n=1 is convergent if 1 < r 1 and
divergent for all other values of r. Furthermore, we have
(
0 if 1 < r < 1
lim r n =
n
1 if r = 1.
Proof. Consider f (x) = ax . We know lim ax = if a > 1; lim ax = 0 if 0 < a < 1.
x

(1) Let a = r, we get

(2) If r = 1,
(3) If r = 0,
(4) If 1 < r < 0,
(5) If r = 1,
(6) If r < 1,

Homework. Show that lim nr n = 0 if |r| < 1.


n

Definition 13 (page 696). A sequence {an } is called increasing () if an < an+1


for all n 1, that is, a1 < a2 < a3 < . It is called decreasing () if an > an+1
for all n 1. A sequence is monotonic () if it is either increasing or decreasing.
Definition 14 (page 697). A sequence {an } is bounded above () if there is a
number M such that an M for all n 1. It is bounded below () if there is
a number m such that m an for all n 1. If it is bounded above and below, then
{an } is a bounded sequence ().
Monotonic Sequence Theorem (page 698). Every bounded, monotonic sequence
is convergent. ( )

Figure 2: Monotonic sequence theorem.

 , :

 , :
 (completeness axiom)
11.1-4

Example 15 (page 699). Investigate the sequence {an }


n=1 defined by the recurrence
relation (): a1 = 2, an+1 = 12 (an + 6) for n = 1, 2, 3, . . ..
Solution. Monotone: We claim: an+1 > an for all n N.
(1) When n = 1,
(2) Assume that it is true for n = k, that is, ak+1 > ak .
(3) When n = k + 1,
(4) By

, we know {an } is monotone.

Bounded: We claim: an < 6 for all n N.


(1) When n = 1,
(2) Assume that it is true for n = k, that is, ak < 6.
(3) When n = k + 1,
(4) By
Limit: By
Since

, we know {an } is bounded above by 6.


, we know lim an exists. Let lim an = L.
n

Example (TA) 16 (page 701). A sequence {an }


is
given
by
a
=
2, an+1 =
1
n=1

2 + an . Show that {an } is increasing, bounded above by 3, lim an exists, and find
n

lim an .
n

Solution.

11.1-5

Homework (page 701). Show that the sequence defined by a1 = 1, an+1 = 3 a1n
is increasing and an < 3 for all n. Deduce that {an } is convergent and find its limit.
n
Example (TA) 17 (page 702). Let an = 1 + n1 . Show that lim an exists.
n

Solution.

Homework (page 702). Let a and b be positive numbers with a > b. Let a1 be
their arithmetic mean () and b1 their geometric mean ():

a+b
a1 =
,
b1 = ab.
2
Repeat this process so that, in general
p
an + bn
an+1 =
,
bn+1 = an bn .
2
(a) Use mathematical induction () to show that an > an+1 > bn+1 > bn .
(b) Deduce that both {an } and {bn } are convergent.
(c) Show that lim an = lim bn . Gauss called the common value of these limits
n
n
the arithmetic-geometric mean of the numbers a and b.
11.1-6

11.2

Series (page 703)

Definition 1 (page 704705). Let {an }


n=1 be an infinite sequence.
(1) The partial sums () of the sequence {an }
n=1 is defined as
sn =

n
X

ak = a1 + a2 + + an .

k=1

These partial sums form a new sequence {sn }


n=1 ().
(2) An infinite series (or just a series ) is denoted by

def.

an = lim
n

n=1

n
X

ak = lim sn = lim (a1 + a2 + + an ),


n

k=1

which means the limit of the partial sums of the sequence {an }
n=1 .
(3) If the limit lim sn = s exists (or convergent) as a finite number, then we say
n

P
the series
an convergent, and the number s is called the sum of the infinite
n=1

series

an ().

n=1

(4) If the sequence {sn }


n=1 is divergent, then the series

an is called divergent.

n=1

Example 2 (page 704). In this chapter, we are not interested in the infinite arithmetic series ( ):

def.

(a + (n 1)d) = a + (a + d) + (a + 2d) + + (a + (n 1)d) + ,

n=1

where each term is obtained from the preceding one by adding it by the common
difference () d. This is because the arithmetic series is convergent if and only if
a = 0 and d = 0.
Example 3 (page 705). The geometric series ( ) is an infinite
series

def.

ar n1 = a + ar + ar 2 + ar 3 + + ar n1 + ,

a 6= 0.

n=1

Each term is obtained from the preceding one by multiplying it by the common ratio
() r. We will discuss the convergence or divergence of the geometric series in
the following theorem.
11.2-1

Theorem 4 (page 706). The geometric series

ar n1 = a + ar + ar 2 + ar 3 + + ar n1 + ,

a 6= 0.

n=1

is convergent if |r| < 1 and its sum is

X
n=1

ar n1 =

a
1r

if

|r| < 1.

If |r| 1, the geometric series is divergent.


Proof.

Homework (page 707). Discuss the series

def.

xn = 1 + x + x2 + x3 + + xn +

n=0

for x R. If the series is convergent, find the sum of the series.


Example 5. Write the number 0.142857 = 0.142857142857 . . . as a ratio of integers
(fraction).
Solution.

Homework. Write the number 0.285714, 2.317, and 0.9 as a ratio of integers.
11.2-2

Homework (page 707).


(a) Show that the series

1
n(n+1)

is convergent, and find its sum.

n=1

(b) Show that the Euler series

1
n2

is convergent.

n=1

Hint: (a)

1
n(n+1)

. (b) For n > 2,

Theorem 6 (page 708). If the series

1
n2

an is convergent, then lim an = 0.


n

n=1

Proof.

Test for Divergence (page 709). If lim an does not exist or if lim an 6= 0, then
n
n

P
an is divergent.
the series
n=1

Example 7 (page 708). The harmonic series () is an infinite series

X
1 1 1
1
1 def.
= 1+ + + ++ + .
n
2 3 4
n
n=1

Show that it is divergent.


Proof.

 n
lim an = 0,
:

n=1

an

n=1
1

ar n1

n=1

11.2-3

n=1

n2

Theorem 8 (page 709). If

an and

n=1

series

c an (where c is a constant),

(an + bn ), and

n=1

c an = c

n=1

(b)
(c)

bn are convergent series, then so are the

n=1

n=1

(a)

(an bn ), and

n=1

an .

n=1

(an + bn ) =

n=1

n=1

(an bn ) =

n=1

an

an

n=1

bn .

n=1

n=1

an +

bn .

n=1

bn

n=1

P
P
P
 ! an bn 6= an bn !
n=1

n=1

n=1

P
P
P
 an bn , (an + bn ) ( 11.2, #83)

n=1

n=1

an

n=1

n=1

bn ,

n=1

(an +bn ) ( 11.2, #84)

n=1

Example (TA) 9 (page 711). Determine whether the series is convergent or divergent. If it is convergent, find its sum.
(a)

n
n=1



X
1
1
(b)
+
en n(n + 1)
n=1

Solution.

11.2-4

(c)

X
n=2

n3

1
.
n

Homework (page 711). Determine whether the series is convergent or divergent.


If it is convergent, find its sum.




X
X
X
n
3
2
1 + 2n
(c)
ln
.
(b)
+
(a)
3n
5n n
n+1
n=1
n=1
n=1
Example (TA) 10 (page 714).

(a) A sequence {an } is defined recursively by the equation an = 21 (an1 + an2 )


for n 3, where a1 and a2 can be any real numbers. Experiment with various
values of a1 and a2 and use your calculator to guess the limit of the sequence.
(b) Find lim an in terms of a1 and a2 by expressing an+1 an in terms of a2 a1
n
and summing a series.
Solution.

Homework (page 714). Consider the series

n
.
(n+1)!

n=1

(a) Find the partial sums s1 , s2 , s3 , an s4 . Do you recognize the denominators?


Use the patten to guess a formula for sn .
(b) Use mathematical induction to prove your guess.
(c) Show that the given infinite series is convergent, and find its sum.
11.2-5

11.3

The Integral Test and Estimates of Sums


(page 714)

The Integral Test (page 716). Suppose f (x) is a continuous, positive, decreasing

P
function on [1, ) and let an = f (n). Then the series
an is convergent if and
n=1
R
only if the improper integral 1 f (x) dx is convergent. In other words,
(a) If

(b) If

f (x) dx is convergent, then

f (x) dx is divergent, then

an is convergent.

n=1

an is divergent.

n=1

Figure 1: The integral test.


Theorem 1 (page 717). The p-series

1
np

(p-) is convergent if p > 1 and

n=1

divergent if p 1.
Proof. If p < 0,
If p = 0,
If p > 0, consider f (x) =
Since

1
,
xp

which is continuous, positive and decreasing on [1, ).

 f (x) ,
 n = 1, x = 1 ;
 ,
11.3-1

Example 2 (page 717). Determine whether the series

ln n
n

converges or diverges.

n=1

Solution.

 x ,
Homework. Determine the value of p 0 such that the series

1
n(ln n)p

converges.

n=2

Homework. Test the following two series

ln n
np

(89, 97 )

where p = 1 and p =

3
,
2

for

n=1

convergence.

(101 )

Estimating the Sum of a Series, page 718


Suppose a series

an is convergent by the Integral Test. We can also estimate the

n=1

size of the remainder ()


Rn = s sn = an+1 + an+2 + an+3 + =

ak .

k=n+1

Remainder Estimate for the Integral Test (page 718). Suppose f (k) = ak ,

P
where f (x) is a continuous, positive, decreasing function for x n and
an is
n=1

convergent. If Rn = s sn , then
Z
Z
f (x) dx Rn
n+1

f (x) dx.

If we add sn to each side of the inequalities (1), because sn + Rn = s, we get


Z
Z
sn +
f (x) dx s sn +
f (x) dx.
n+1

11.3-2

(1)

Figure 2: Remainder estimate for the Integral Test.


Example 3 (page 718-719). Approximate the sum of the series

1
.
n3

How many

n=1

terms are required to ensure that the sum is accurate to within 0.005?
Solution.

Homework (page 721). How many terms of the series

1
n(ln n)2

would you need

n=2

to add to find its sum to within 0.01?

Appendix
Example (TA) 4. Determine the values p such that the series

1
n(ln n)(ln(ln n))p

n=3

converges.
Solution.

11.3-3

(90 )

Example (TA) 5 (page 722). Find all values of c for which the following series
converges:

X
c
n=1

n n+1

Solution.

Example (TA) 6 (page 721). Show that if we want to approximate the sum of the

P
1
series
so that the error is less than 5 in the ninth decimal place, then we
n1.001
n=1

need to add more than 1011301 terms!


Solution.

11.3-4

11.4

The Comparison Tests (page 722)

The Comparison Test (page 722). Suppose that

an and

n=1

positive terms and an bn for all n.


(a) If

bn is convergent, then

n=1

(b) If

bn are series with

n=1

an is also convergent.

n=1

an is divergent, then

n=1

bn is also divergent.

n=1

Proof. Let sn =

n
P

ak , tn =

n
P

bk , and t =

tk .

k=1

k=1

k=1

(a) Monotone: Since both series have positive terms, the sequences {sn }
n=1 and

{tn }n=1 are increasing.


Bounded: Since ak bk for all k, we have sn tn t.

P
By the
,
an converges.
n=1

(b) If

an is divergent, then sn , thus tn . Therefore

n=1

bn diverges.

n=1

Most of time we use p-series and geometric series for the purpose of comparison.
(1) p-series:

1
.
np

It is convergent if

and divergent if

n=1

(2) geometric series:

ar n1 . It is convergent if

and divergent if

n=1

Example 1. Show that the series

n!
nn

is convergent.

(90, 94 )

n=1

Solution.

Homework. Determine the convergence of the following series:


(a)

n+1n .
n+1+ n

(Hint: )

n=0

(b)

q
P
sin n13 . (Hint:

n=1

11.4-1

(99 )

Homework.
(a) Prove that ln(n + 1) < 1 + 12 + +
(b) Test for convergence of

n=1

1
n

< 1 + ln n.

1
1 .
1+ 21 ++ n

The Limit Comparison Test (page 724). Suppose that

(102 )

n=1

with positive terms. If

an and

bn are series

n=1

an
= c,
bn
where c is a finite number and c > 0, then either both series converge or both diverge.
lim

Proof. Let m and M be positive numbers such that m < c < M. Since abnn is close
to c for large n, there is an integer N such that
an
m<
< M mbn < an < Mbn
when n > N.
bn
, we know both series converge or both diverge.
By the


Example 2 (page 725-726). Determine whether the following series converges or
diverges.

X
X
2n2 + 3n
9n

(b)
.
(a)
n
5
3
+
10
5
+
n
n=1
n=1
Solution.

Homework (page 726). Determine whether the following series converges or diverges.
2
1


X
X
X
en
1
1
n
(c)
.
e
(b)
(a)
1+
1
1+ n
n
n
n=1
n=1 n
n=1
11.4-2

Estimating Sums, page 725


If we have used the Comparison Test to show that a series

an converges by

n=1

comparison with a series

bn , then we may be able to estimate the sum

n=1

an by

n=1

comparing remainders.
Consider the remainder Rn = s sn = an+1 + an+2 + and Tn = t tn =
bn+1 + bn+2 + . Since an bn for all n, we have Rn Tn .

P
(1) If
bn is a p-series, we can estimate its remainder Tn as in Section 11.3.
n=1

(2) If

bn is a geometric series, we can sum it exactly.

n=1

Example 3 (page 725). Use the sum of the first 100 terms to approximate the sum

P
1
. Estimate the error involved in this approximation.
of the series
n3 +1
n=1

Solution.

Example 4 (page 726). Use

10
P

cos2 n
5n

.
= 0.07393 to estimate the error of the sum of

n=1

the series

cos2 n
.
5n

n=1

Solution.

Homework (page 726). Use

10
P

1
3n +4n

.
= 0.19788 to estimate the error of the sum

n=1

of the series

1
.
3n +4n

n=1

11.4-3

Appendix
Example (TA) 5 (page 727).
(a) Suppose that

an and

n=1

n=1

an
n bn

an and

n=1

= 0, then

an is also convergent.

n=1

bn are series with positive terms and

n=1

divergent. Show that if lim

an
n bn

n=1

= , then

an is also divergent.

n=1

(c) Determine whether the following series converges or diverges.


(1)

X
ln n
n=1

n3

(2)

bn is

n=1

convergent. Show that if lim


(b) Suppose that

bn are series with positive terms and

X
ln n
n
ne
n=1

Solution.

11.4-4

(3)

X
1
ln n
n=2

(4)

X
ln n
n=1

bn is

11.5

Alternating Series (page 727)

Definition 1 (page 727). An alternating series () is a series whose terms


are alternately positive and negative.
Example 2 (page 727). Two examples of alternating series are

(1)n1

n=1

(1)n

n=1

1 1 1 1 1
1
= 1 + + +
n
2 3 4 5 6

n
1 2 3 4 5 6
= + + +
n+1
2 3 4 5 6 7

Alternating Series Test (page 727). If the alternating series

(1)n1 bn = b1 b2 + b3 b4 + b5 b6 + ,

where bn > 0,

n=1

satisfies
(a) bn+1 bn for all n
(b) lim bn = 0,
n

then the series is convergent.

Figure 1: Alternating series test.

 ,
Example 3 (page 729). Determine whether the following series converges or diverges.
(a)

X
(1)n1
n=1

(b)

Solution.

11.5-1

X
(1)n 3n
n=1

4n 1

Example 4 (page 729). Test the series


gence.

(1)n+1 n3n+1 for convergence or diver-

n=1

Solution.

Homework. Test the series

(1)

n=1

1
n

e 1 for convergence or divergence.


(97 )

Estimating Sums, page 730


Alternating Series Estimation Theorem (page 730). If s =

(1)n1 bn is

n=1

the sum of an alternating series that satisfies


(a) bn+1 bn
(b) lim bn = 0,
n

then |Rn | = |s sn | bn+1 .

 ( (a) (b)),
 ,
Example 5 (page 730). Find the sum of the series

(1)n
n!

correct to three decimal

n=0

place.
1
=
Solution. Since (n+1)!

P (1)n
converges by the
n!

1
(n+1)n!

<

1
n!

1
n n!

and 0 lim

1
n n

lim

= 0, the series

. By the Alternating Series Es-

n=0

timation Theorem we hope |s sn | bn+1 < 0.0005, so (n + 1)! > 2000 and n 6.
.
Hence s s6 = 0!1 1!1 + 2!1 3!1 + 4!1 5!1 + 6!1 = 0.368056 correct to three decimal
places.
Homework (page 731). How many terms of the series do we need to add in order

P
to find the sum of the series
(1)n+1 n16 correct to four decimal place?
n=1

11.5-2

11.6

Absolute Convergence and the Ratio and Root


Test (page 732)

Definition 1 (page 732-733).

P
(1) A series
an is called absolutely convergent () if the series of absolute
n=1

values

|an | is convergent.

n=1

(2) A series

an is called conditionally convergent () if it is convergent

n=1

but not absolutely convergent.


Example 2 (page 732).

P
(1)n
is absolutely convergent.
(a) The series
n2
n=1

(b) The series

(1)n
n

is conditionally convergent.

n=1

Example 3. Determine the series


tionally convergent, or divergent.

(1)n sin n1 is absolutely convergent, condi-

n=1

(95 )
Solution.

Homework. Determine the series (a)

(1)n1

n=1

n+1 n1
n

and (b)

(1)n
n(ln n)

is

n=2

absolutely convergent, conditionally convergent, or divergent.


(94 )

P
Theorem 4 (page 733). If a series
an is absolutely convergent, then it is convergent.

n=1

Proof.

11.6-1

The Ratio Test (page 734).



P


=
L
<
1,
then
the
series
an is absolutely convergent (and
(a) If lim an+1

an
n

n=1

therefore convergent).



P
an+1
an+1
(b) If lim an = L > 1 or lim an = , then the series
an is divergent.
n

n=1



an+1
(c) If lim an = 1, the Ratio Test is inconclusive (); that is, no
n

P
conclusion can be drawn about the convergence or divergence of
an .
n=1

 , r , L < r < 1

Example 5 (page 735737). Determine whether the series is absolutely convergent,

P
P
3
2 n
conditionally convergent, or divergent. (a)
(1)n 3nn
(b)
(1)n+1 n n!2 .
n=1

n=1

Solution.

 , (Ratio Test)
 ,

P
P
1
1
 :

n
n
2

n=1

n=1

Homework (page 737). Determine whether the series is absolutely convergent,

n2
P
P
(n!)2
2
(b)
. (101 )
conditionally convergent, or divergent. (a)
n!
(5n)!
n=1

n=1

The Root Test (page 736).


(a) If lim
n

p
n

|an | = L < 1, then the series

p
n

an is absolutely convergent (and

n=1

therefore convergent).
(b) If lim

|an | = L > 1 or lim


n

p
P
n
|an | = , then the series
an is divergent.
n=1

p
(c) If lim n |an | = 1, the Root Test is inconclusive ().
n

11.6-2

Example 6 (page 736). Test the convergence of the series

n=1

Solution.


2n+3 n
.
3n+2

(an )n (Root Test)

n=1

 (11.8 )
Homework (page 737). Determine whether the series is absolutely convergent, con

 n2

P
P
ditionally convergent, or divergent. (a)
1 + n1
(b)
( n n 1)2n .
n=1

n=1

(101 )

Rearrangements, page 737


If we rearrange the order of the terms in a finite sum, then the value of the sum
remains unchanged. But it is not always the case for an infinite series.

P
By a rearrangement of an infinite series
an () we mean a series obn=1
P
tained by simply changing the order of the terms. Formally, we will write
a(n)
(n)

where (n) is an one-to-one map from the natural number N to itself. For instance,
P
a rearrangement of
a(n) could start as follows:
(n)

a2 + a7 + a3 + a32 + a15 + a10 + a200 + .


It turns out that
Theorem 7 (page 737).
(a) If
of

n=1

an is an absolutely convergent series with sum s, then any rearrangement


an has the same sum s.

n=1

(b) If

an is a conditionally convergent series and r is any real number whatso-

n=1

ever, then there is a rearrangement of

n=1

11.6-3

an that has a sum equal to r.

Example 8 (page 737). Consider the alternating harmonic series


1 1 1 1 1 1 1
(1)
S = 1 + + + +
2 3 4 5 6 7 8
If we multiply this series by 21 and insert 0 between the terms of new series, we get
1
1
1
1
1
S = 0+ +0 +0+ +0 +
(2)
2
2
4
6
8
Now we add the series in (1) and (2) to get
3
1 1 1 1 1
S = 1+ + + +
(3)
2
3 2 5 7 4
Notice that the series in (3) contains the same terms as in (1).

P
P
Theorem 9. If
an and
bn are two absolutely convergent series with sum A
n=0

n=0

and B, respectively, then the product series

P
n
P

ak bnk and any rearrangement of

n=0 k=0
P
n
P

ak bnk has a sum equal to AB.

n=0 k=0

Appendix
Proof of Ratio Test, page 734.
(a) Since L <
1, we can choose a number
r such that L < r < 1. Since
an+1
a
lim
will eventually be less than r;
= L and L < r, the ratio n+1
an
n an
that is, there exists an integer N such that


an+1


an < r |an+1 | < |an |r whwnever n N.
In general, we get

|aN +k | < |aN +k1|r < |aN +k2 |r 2 < < |aN |r k

for all k 1.

By the Comparison Test, we know

X
X
X
|aN |
|aN |r k =
|aN +k | <
.
|an | =
1

r
n=N +1
k=1

k=1

Hence

|an | is convergent, and

n=1

an is absolutely convergent.

n=1








an+1
an+1

(b) If an+1

L
>
1
or

,
then
the
ratio
an
an will eventually be
an
greater than 1; that is, there exists an integer N such that


an+1
> 1 |an+1 | > |an | whwnever n N.
lim
n an
Since lim an 6= 0, the series
n

an diverges by the Test for Divergence.

n=1

11.6-4

11.7

Strategy for Testing Series (page 739)

We have several ways of testing a series for convergence or divergence. There are no
fast rules about which test to apply to a given series, but you may find the following
advice of some use.
(1) If lim an 6= 0, then the Test for Divergence should be used.
n

(2) If the series is of the form

1
,
np

it is a p-series (p-), which we know to

n=1

be convergent if

and divergent if

(3) If the series has the form

ar n1 , it is a geometric series (,

n=1

), which converges if

and diverges if

(4) If the series has a form that is similar to a p-series or a geometric series, then
one of the comparison tests should be considered.
(a) If an is a rational function or algebraic function of n (involving roots of
polynomials), then the series should be compared with a p-series. The
value of p should be chosen by keeping only the highest powers of n in
the numerator and denominator.
(b) The comparison tests apply only to series with positive terms.

P
(c) If
an has some negative terms, then we can apply the Comparison
n=1

Test to

|an | an test for absolute convergence.

n=1

 (Comparison Test)
(5) If the series is of the form

(1)n bn or

n=1

Series Test is an obvious possibility.

(1)n1 bn , then the Alternating

n=1

(6) Series that involve factorials or other products (including a constant raised to
the n-th
power)
are often conveniently tested using the Ratio Test. Notice


an+1
that an as n for all p-series and therefore all rational or algebraic
functions of n. Thus the Ratio Test should note be used for such series.

 (Ratio Test)

(7) If an is of the form (bn )n , the the Root Test may be useful.
R
(8) If an = f (n), where 1 f (x) dx is easily evaluated, then the Integral Test is
effective (assuming the hypotheses of this test are satisfied).
11.7-1

Example (TA) 1. Show that

P
n

n=1

Solution.


2 1 is divergent.

(90 )

Example (TA) 2. Determine the following series are absolutely convergent, conditionally convergent, or divergent.

P
(1)n

.
(a)
n+1+ n
n=1

(b)

(1)n

.
n+1 n

n=1

(c)

n=1

(1)n
.

n3 +1 n3

(96 )

Solution.

11.7-2

Homework. Discuss the following series converges or diverges when a = 1 and


a = 2:

X
n=1

123n
.
(a + 1)(a + 2)(a + 3) (a + n)
(89 )

Homework. Find the values of p for the convergence of the series below
(a)

en(p

2 p2)

n=0

(b)

e n 1
.
np

(102 )

n=1

Homework. Determine the value of x for which the series

n=2

converges absolutely, converges conditionally or diverges.

11.7-3

1
n ln n

1+


1 n
x

(100 )

11.7 Exercises and 11 Review


Determine whether the series is conditionally convergent, absolutely convergent, or
divergent. (page 740-741, page 779)

X
X
X
n
(2n + 1)n
1
2.
3.
(1)n
1.
n
2n
n+3
n
n+2
n=1
n=1
n=1

n
4.
(1) 2
n +2
n=1
7.

X
n=1

10.

8.

n=1

13.
16.
19.
22.

X
3n n2
n=1

n=1

n=1

X
k=1

n2 + 1
n3 + 1
n ln n

1
2 + sin k

31.

26.

X
en

n=1

k=1

29.

n2

5k
3k + 4k

1
34.
n + n cos2 n
n=1

n
X

n
21
37.

32.
35.
38.

n=1

40.
43.

X
n2 + 1
n=1

X
n=2

46.

X
n=1

n3 + 1
1

n ln n

X
n2 + 1

n=1

n=1

5n

(1)n
cosh n

(n!)n
n4n
1

n=2 n

X

21

41.
44.

X
n3

n=1

X
n=1

47.

5n

ln

n
3n + 1

12.

1
2n + 1

n=1

k 2 ek

X
k=1

18.
21.

X
2k1 3k+1

k=1

n=1

5n n!
11.7-4

kk

(1)n1

n1

(1) cos
n

 
1
24.
n sin
n
n=1

X
k ln k
27.
(k + 1)3
k=1

X
j
j
30.
(1)
j+5
j=1
 n2

X
n
33.
n+1
n=1

n=2

X
n=1

1
k k2 + 1

n=1

39.

X
1 3 5 (2n 1)
n=1

k=1

36.

1
1+ n

n=1

2n

n
(1 + 2n2 )n

n=1

9.

15.

n!
2 5 8 (3n + 2)
n=1

X
n2 1
20.
n3 + 2n2 + 5
n=1
 

X
1
23.
tan
n
n=1
17.

X
n!
25.
en2
n=1

28.

X
sin 2n
14.
1 + 2n
n=1

n!

(1)

n=1

6.

(5)n

2 k!
(k + 2)!
k=1


X
1
1
11.
+ n
3
n
3
n=1

n ln n
n2 en

5.

X
n2 2n1

1
(ln n)ln n
n3

n
+1

X
(1)n

42.
n+1
n=1

45.

X
n=1

48.

cos 3n
1 + (1.2)n

X
(5)2n
n=1

n2 9n

1
n2

49.

(1)

n1

n=1

52.

n
n+1

(1)n1 n3

n=1


n

50.

X
n=1

53.

n+1
n

X
(1)n (n + 1)3n

22n+1

n=1

cos 3
55.
n!
n=1
5n

X
2n
58.
n+1
n=2

(2)n
56.
nn
n=1
 n2

X
1
59.
1+
n
n=1

61.

62.

X
n=1

100

100
n!

n1

X
n=1

n2

2
n!

51.

(1)n1 n 3

n=1

54.
57.
60.

X
(1)n n

n=1

X
n=1

X
n=1

ln n

n2 + 1
2n2 + 1

n

(2n)!
(n!)2

13 135 1357
1 3 5 (2n 1)
+

+ + (1)n1
+
3!
5!
7!
(2n 1)!
2 2 6 2 6 10 2 6 10 14
+
+
+
64. +
5 5 8 5 8 11 5 8 11 13

X
2 4 6 (2n)
65.
n!
n=1
63. 1

66.

X
n=1

(1)n

2n n!
5 8 11 (3n + 2)

11.7-5

11.8

Power Series (page 741)

Definition 1 (page 741). A power series () is a series of the form

cn xn = c0 + c1 x + c2 x2 + c3 x3 + ,

n=0

where x is a variable and the cn s are constants called the coefficients () of the
series.
A power series may converge for some values of x and diverge for other values of
x. The sum of the series is a function
f (x) = c0 + c1 x + c2 x2 + c3 x3 + + cn xn +
whose domain () is the set of all x for which the series converges.

 () ()
Example 2 (page 741). If cn 1, the power series becomes the geometric series

xn = 1 + x + x2 + + xn + ,

n=0

which converges when

and diverges when

Definition 3 (page 741). A series of the form

cn (x a)n = c0 + c1 (x a) + c2 (x a)2 +

n=0

is called a power series in (xa) ( (xa) ) or a power series centered


at a ( a ) or power series about a ( a ).

 (x a)0 = 1, x = a
 a , x = a ,
Theorem 4 (page 743). For a given power series

cn (x a)n there are only three

n=0

possibilities:
(a) The series converges only when x = a.
(b) The series converges for all x.

(c) There is a positive number R such that the series converges if |x a| < R and
diverges if |x a| > R. (, )
11.8-1

Definition 5 (page 743).


(1) The number R in case (c) is called the radius of convergence () of the
power series.
(2) By convention, the radius of convergence is R = 0 in case (a) and R = in
case (b).
(3) The interval of convergence () of a power series is the interval that
consists of all values of x for which the series converges. When x is an endpoint
() of the interval, that is, x = a R, anything can happen the interval
of convergence could be
(a R, a + R)

(a R, a + R]

[a R, a + R)

[a R, a + R].

Example 6 (page 742). Find the interval of the convergence of the following series:
(a)

n!x

n=0

(b)

X
(1)n x2n
n=0

22n (n!)2

X
1
(Bessel function of order 0) (c)
(x 3)n .
n
n=1

(90 )
Solution.

11.8-2

Example (TA) 7.
(a) Evaluate lim
x0

(b) Evaluate lim

1cos x
.
x2
1
1cos( n
)

1
n 1cos( n+1 )

(c) Find the interval of convergence of the power series

1 cos

n=1

1
n



xn .

(97 )

Solution.

Homework. Find the radius of convergence of the power series

X
n!(3n + 1)
xn .
1 3 5 (2n + 1)
n=0
Homework. Consider the power series

(95 )

1
xn .
n ln n

n=2

(a) Find the radius of the convergence r.

(b) Discuss whether the power series is convergent or divergent at x = r and


x = r.
(96 )

 n
P
x .
Homework. Find the interval of convergence of the series
ln n+1
n
n=1

(101 )

11.8-3

11.9

Representations of Function as Power Series


(page 747)

In this section, we learn how to represent certain types of functions as sums of


power series. We will see that it is useful for integrating functions that dont have
elementary antiderivatives, for solving differential equations, and for approximating
functions by polynomials.
Example 1 (page 747). Recall that the geometric series:

X
1
= 1 + x + x2 + x3 + + xn + =
xn
1x
n=0

if |x| < 1.

We can express the following functions by manipulating geometric series:


(1)

1
=
1 + x2

(2)

x
=
2+x

Differentiation and Integration of Power Series, page 748


Theorem 2 (page 748). If the power series

cn (x a)n has radius of convergence

n=0

R > 0, then the function f (x) defined by

f (x) = c0 + c1 (x a) + c2 (x a) + =

cn (x a)n

n=0

is differentiable (and therefore continuous) on the interval (a R, a + R) and


2

(a) f (x) = c1 + 2c2 (x a) + 3c3 (x a) + =

ncn (x a)n1 .

n=1

(b)

f (x) dx = C+c0 (xa)+c1

X
(x a)2
(x a)n+1
(x a)3
cn
+c2
+ = C+
.
2
3
n+1
n=0

 , ,
(term-by-term differentiation and integration)
11.9-1

 (a), (b), :
d
(a)
dx
(b)

cn (x a)n

n=0

cn (x a)

n=0

X
d
(cn (x a)n )
dx
n=0

dx =

Z
X

cn (x a)n dx

n=0

 (),
Example 3 (page 749). Express the following function as a power series and find
its interval of convergence.
(1) f (x) =

1
(1 x)2

(2) g(x) = ln(1 + x)

(3) h(x) = tan1 x. (93 )

Solution.

Homework (page 751). Find a power series representation for the following function
and determine the interval of convergence.
(1) f (x) =

3
.
x2 x2

(2) g(x) =

x2 +x
.
(1x)3

(partial fraction first)

(3) h(x) = x2 tan1 (x3 ).


11.9-2

Homework.
(a) Find the radius of convergence and the interval of convergence of the power

P
(x1)n
.
series
(2)n n
n=1

(b) Let f (x) =

(x1)n

(2)n n

when the power series is convergent. Evaluate f (3) (1).

n=1

(102 )

Example 4 (page 752). Find the sum of each of the following series.

X
X
n
n
.
(1)
nx , |x| < 1
(2)
2n
n=1
n=1
Solution.

Homework (page 753). Find the sum of each of the following series.

X
X
X
n2 n
n2
n
n(n 1)x , |x| < 1
(2)
(1)
,
(3)
.
n
n
2
2
n=1
n=1
n=1
R 1
Example 5 (page 750). Evaluate 1+x7 dx as a power series and approximate
R 0.5 1
dx correct to within 107.
0
1+x7

Solution. We express the integrand and then integrate term by term:


1
=
1 + x7
Z

1
dx =
1 + x7

This series converges for


Z 0.5
1
dx =
1 + x7
0

, that is

By the Alternating Series Theorem, the error is smaller than the term with n = 4:
1
6.4 1011 , so we have
29229
Z 0.5
1
dx
1 + x7
0
11.9-3

Example (TA) 6. Let


f (x) =

X
xn
n=1

n2

Find the intervals of convergence for f (x), f (x), and f (x).


Solution.

11.9-4

11.10

Taylor and Maclaurin Series (page 753)

In this section, we will answer two questions: Which functions have power series
representation? How can we find such representation?
We observe a smooth function f (x) that can be represented by a power series:
f (x) = c0 + c1 (x a) + c2 (x a)2 + c3 (x a)3 +

|x a| < R.

(1)

Put x = a, then we get

We differentiate the series (1) term by term:


f (x) =

(2)

Put x = a in (2) and get

If we continue to differentiate and substitute x = a, we obtain


f (n) (a) =

cn =

Theorem 1 (page 754). If f (x) has a power series representation (expansion) at a:


f (x) =

cn (x a)n

|x a| < R

n=0

then its coefficients are given by the formula cn =

f (n) (a)
.
n!

Definition 2 (page 754). If f (x) has a power series expansion at a, then it must
be of the following form:
f (x) =

X
f (n) (a)
n=0

n!

(x a)n = f (a) +

f (a)
f (a)
(x a) +
(x a)2 + .
1!
2!

(3)

The series in equation (3) is called the Taylor series of the function f (x) at a (or
about a or centered at a) ( f (x) x = a ).

 : f (x) , f (x)
 (): ?()
Definition 3 (page 754). For the special cases a = 0 the Taylor series becomes
f (x) =

X
f (n) (0)
n=0

n!

xn = f (0) +

f (0)
f (0) 2
x+
x + .
1!
2!

This case is given the special name Maclaurin series ().


11.10-1

Example 4 (page 754). Find the Maclaurin series of the function f (x) = ex and
its radius of convergence.
, so f (n) (0) =
Solution. We compute f (n) (x) =
Therefore the Maclaurin series of f (x) = ex is

X
f (n) (0)
n=0

n!

for all n N.

xn =

To find the radius of convergence, we let an =




an+1


an =

, then

, the radius of convergence is

By the

Question 5 (page 755). Under what circumstances is a function equal to the sum
of its Taylor series? In other words, if f (x) has derivatives of all orders, when is it
true that

X
f (n) (a)
def.
?
(x a)n = lim Tn (x),
f (x) =
n
n!
n=0
where

Tn (x) =

n
X
f (k) (a)
k=0

k!

(x a)k = f (a) +

f (n) (a)
f (a)
(x a) + +
(x a)n .
1!
n!

(4)

Definition 6 (page 755). The polynomial Tn (x) in (4) is called n-th degree Taylor
polynomial of f (x) at a (f (x) x = a n-).

 : ,
Solution. Let the remainder () of the Taylor series be
def,

Rn (x) = f (x) Tn (x) f (x) = Tn (x) + Rn (x) Tn (x) = f (x) Rn (x).


If lim Rn (x) = 0, then lim Tn (x) = lim (f (x)Rn (x)) = f (x) lim Rn (x) = f (x).
n

Conversely, if f (x) = lim Tn (x), then lim Rn (x) = lim (f (x) Tn (x)) = 0.
n

Theorem 7 (page 756). If f (x) = Tn (x) + Rn (x), where Tn (x) is the n-th degree
Taylor polynomial of f (x) at a, then f (x) is equal to the sum of its Taylor series on
the interval |x a| < R if and only if
lim Rn (x) = 0
n

for |x a| < R.

 :
11.10-2

Question 8 (page 756). How do we show that lim Rn (x) = 0 for a specific function
n
f (x)?
Theorem 9. Suppose that f (x) has continuous derivative at x = a up to n + 1
order, then
f (x) = f (a) +
where Rn (x) =

f (n) (a)
f (a)
(x a) + +
(x a)n + Rn (x)
1!
n!

f (n+1) (c)
(x a)n+1 , c is a number between a and x.
(n + 1)!

Proof. Without loss of generality, we assume a < x. Consider the function


g(t) = f (x) f (t)

f (n) (t)
f (t)
(x t)
(x t)n ,
1!
n!

then g(t) is continuous on [a, x], and


g (t) =

n
X
f (k+1) (t)
k=0

=
=

k!

n
X
f (k+1) (t)
k=0
n
X
k=0

k!

(x t)
k

n
X
f (k) (t)
k=1

k!

k(x t)k1 (1)

n
X
f (k) (t)
k
(x t) +
(x t)k1
(k

1)!
k=1
n1

X f (k+1) (t)
f (k+1) (t)
f (n+1) (t)
(x t)k +
(x t)k =
(x t)n .
k!
k!
n!
k=0

Let h(t) = (x t)n+1 , by the Cauchy Theorem (generalized Mean Value Theorem),
then there exists c (a, x) such that
(n+1)

(c)(xc)
f
g(x) g(a)
0 Rn (x)
g (c)
n!
=

=
,
h (c)
h(x) h(a)
(n + 1)(x c)n
0 (x a)n+1

Rn (x) =

so

f (n+1) (c)
(x a)n+1 .
(n + 1)!

 , , c
Once we have this expression of the remainder, we can estimate it by the following
theorem.
Taylors Inequality (page 756). If |f (n+1) (x)| M for |x a| d, then the
remainder Rn (x) of the Taylor series satisfies the inequality

(n+1)

f
M
(c)
n+1
(x a)
|x a|n+1 for |x a| d.
|Rn (x)| =
(n + 1)!
(n + 1)!
11.10-3

Example 10 (page 757758).


(1) Prove that ex is equal to the sum of Maclaurin series.
(2) Find the Taylor series for f (x) = ex at x = 2.
Solution.
(1) If f (x) = ex , then f (n+1) (x) = ex for all n N. If d is a positive number and
|x| d. then |f (n+1) (x)| = ex ed , so
(n+1)

f

(c)
n+1
for |x| d.
|Rn (x)| =
x
(n + 1)!
Notice that ed is a number independent of n, so we have
lim |Rn (x)|
n

and therefore lim Rn (x) = 0 for all x R.


n

(2) We have f (n) (2) = e2 , so the Taylor series for f (x) = ex at x = 2 is

Example 11 (page 758). Find the Maclrurin series for f (x) = sin x. Prove that it
represents sin x for all x.
Solution. We compute for k = 0, 1, 2, . . .,
f (4k) (x) =
f (4k) (0) =

f (4k+1) (x) =
f (4k+1) (0) =

f (4k+2) (x) =
f (4k+2) (0) =

f (4k+3) (x) =
f (4k+3) (0) =

so the Maclaurin series for f (x) = sin x is

Since f (n+1) (x) is sin x or cos x, we know that |f (n+1) (x)| 1 for all x R. By
Taylors Inequality:
|Rn (x)| =
and
lim |Rn (x)|
n

By the Squeeze Theorem, it follows that Rn (x) 0 as n , so sin x is equal to


the sum of its Maclaurin series.
11.10-4

Example 12 (page 758759).


(1) Represent f (x) = sin x as the sum of its Taylor series centered at x = 3 .
(90 )

(2) Find the Maclaurin series for cos x.


(3) Find the Maclaurin series for x cos x.
Solution. We have for k = 0, 1, 2, 3, . . .
f (4k) (x) =

f (4k+1) (x) =

f (4k+2) (x) =

f (4k+3) (x) =

f (4k) ( 3 ) =

f (4k+1) ( 3 ) =

f (4k+2) ( 3 ) =

f (4k+3) ( 3 ) =

(1) The Taylor series at x =

is

(2) Instead of computing derivatives and substituting in the Maclaurin series for
cos x, we can differentiate the Maclaurin series for sin x:
cos x =
Since the Maclaurin series for sin x converges for all x, the differential series
for cos x also converges for all x.
(3) We can multiply the series for cos x by x:
x cos x =
Example 13 (page 760). Find the Maclaurin series for f (x) = (1 + x)m , where m
is any real number.
Solution.

Therefore the Maclaurin series of f (x) = (1 + x)m is

11.10-5

Example 14 (page 760). Find the radius of convergence of the binomial series (

P
m(m1)(mn+1) n
x .
, )
n!
n=0

Solution. If m is a nonnegative integer, then the terms are eventually 0 and so the
series is finite. For other values of m, if the n-th term is an , then


an+1


an =

By the

, the binomial series converges if


, and the radius of convergence is
.

and diverges if

The Binomial Series (page 761). If m is any real number and |x| < 1, then
(1 + x) =
m

Cnm xn = 1 + mx +

n=0

m(m 1) 2 m(m 1)(m 2) 3


x +
x + .
2!
3!

The interval of convergence depends on m: (1, 1) if m 1; (1, 1] if 1 < m <


0; [1, 1] if m > 0.

 ,
Definition 15 (page 769). Numbers Cnm =
called binomial coefficients ().

m(m 1)(m 2) (m n + 1)
are
n!

Example 16 (page 761). Find the Maclaurin series for g(x) =


of convergence.

1
4x

and its radius

Solution. We rewrite f (x) in a form where we can use the binomial series:

1
=
4x

Using the binomial series with m =

and with x replaced by

, we have

1
=
4x

The series converges if

, so the radius of convergence is


11.10-6

Important Maclaurin series and their radii of convergence

X
1
=
xn = 1 + x + x2 + x3 +
(1)
1 x n=0
(2) e =
x

X
xn
n=0

(3) sin x =

= 1+

n!

X
(1)n x2n+1

(2n + 1)!

n=0

(4) cos x =

x
x2 x3
+
+
+
1! 2!
3!

X
(1)n x2n

(2n)!

n=0

(5) tan1 x =

=1

X
(1)n x2n+1

2n + 1

n=0

(6) ln(1 + x) =

X
(1)n xn
n=0

(7) (1 + x) =
m

= x

x3 x5 x7
+

+
3!
5!
7!

x2 x4 x6
+

+
2!
4!
6!

=x

=x

x2 x3 x4
+

+
2
3
4

Cnm xn = 1 + mx+

n=0

x3 x5 x7
+

+
3
5
7

R=1

R=

R=

R=

R=1

R=1

m(m 1) 2 m(m 1)(m 2)


x +
+ R = 1
2!
3!

Example 17 (page 762). Find the sum of the series


1
1
1
1

+ .
2
3
12 22
32
4 24
Solution.

ex 1 x
.
x0
x2
Solution. Using the Maclaurin series for ex , we have
Example 18 (page 763). Evaluate lim

ex 1 x
=
x0
x2
lim

=
because power series are continuous functions.
11.10-7

Multiplication and division of power series, page 763


Example 19 (page 763). Find the first three nonzero terms in the Maclaurin series
for (1) ex sin x and (2) tan x.
Solution.

11.10-8

Example 20 (page 762).


Z
2
(1) Evaluate ex dx as an infinite series.
(2) Evaluate

1
2

ex dx correct to within an error of 0.001.

(89 )

Solution.

(1) We replace x with x2 in the series for ex and get, for all x R,
2

ex =
We integrate term by term:
Z
2
ex dx =
The series converges

because ex converges

(2) We compute
Z 1
2
ex dx =
0

The Alternating Series Estimation Theorem shows that the error is less than

Example (TA) 21. Let f (x) = ln(5 x).


(a) Find the power series representation for f (x) at x = 0.
(b) Find f (n) (0).

(98 )

Solution.

11.10-9

Example (TA) 22.


(a) Write down the general terms the MacLaurin series of sin x and sin1 x.
(b) Find their radii of convergence.
sin x sin1 x x2
.
x0
x6

(102 )

(c) Find lim


Solution.

11.10-10

Homework. Let f (x) = sin2 x.


(a) Find the Maclaurin series for f (x). (Hint: sin2 x = 21 (1 cos 2x). page 765)
(b) Find f (94) (0).

(94 )

Homework.

(a) Find the Taylor series for f (x) = (x2 + x + 1) x + 1 at x = 0 up to the third
power of x.
q
2
(b) Let f (x) = ln 1+x
. Find f (10) (0).
(101 )
1x2
1

Homework. Find the first three terms of the Maclaurin series of ((1x2 )(1+x)) 4 .
(99 )
(x tan1 x)(e3x 1)
.
x0
2x2 1 + cos 2x

(100 )

Homework. Evaluate lim

sin x tan1 x x2 +
Homework. Find lim
x0
x6

x4
2

(95 )

Homework.
(a) Find the Maclaurin series for f (y) = sin y.
Z
2
(b) Evaluate
sin(cos x) dx correct to within an error of 0.01.

(98 )

Homework*.
(a) Find the Taylor expansion of

2 cos x at x = 4 .

(b) Show that the above Taylor expansion converges to

2 cos x for all x R.


(89 )

Homework*. Find the Taylor series representation for f (x) = ln(2 + 3x) about
x = 1. For what values of x is this representation valid?
(100 )
Homework*. Let f (x) = x tan1 x. Find f (16) (0).

(93 )

Homework* (page 781). Let f (x) = sin(x3 ). Find f (15) (0).

(97 )

Homework*.
(a) Write down the general terms of the Maclaurin series of f (x) = ln
(b) Find the values of f (10) (0).

1 + x2 .

(99 )
11.10-11

Homework*. Express
with error less than

1
.
1000

Homework*. Estimate

cos x2 dx as an alternative series, and estimate the value

(90 )
Z

sin(x2 ) dx with error less than 0.001.

(95 )

Homework*.
(a) Find the Maclaurin series for f (x) = tan1 x and its radius of convergence.
Z 1
4 tan1 x

(b) Estimate the integral


dx with error less than 1014 . (96 )
x
0

11.10-12

11.11

Applications of Taylor Polynomials


(page 768)

In this section we explore some applications of Taylor polynomials. Computer scientists like them because polynomials are the simplest of functions. Then we investigate how physicists and engineers use them in such fields as relativity, optics,
blackbody radiation, electric dipoles, the velocity of water waves, and building highways across a desert.

Approximating Functions by Polynomials, page 768


Recall that the linearization approximation of f (x) at x = a (in section 3.10):
f (x) f (a) + f (a)(x a)

(1)

Right hand side of (1), called the linearization of f (x) at x = a, is the first-degree
Taylor polynomial T1 (x). If f (x) is the sum of its Taylor series, then Tn (x) f (x)
as n , and so Tn (x), nth-degree Taylor polynomial of f (x) at x = a, can be
used as an approximation to f (x):
f (x) Tn (x) =

n
X
f (k) (a)
k=0

k!

(x a)k .

When using a Taylor polynomial Tn (x) to approximate a function f (x), we have to


ask that how good an approximation is it? How large should we take n to be in
order to achieve a desired accuracy? To answer these questions we need to look at
the absolute value of the remainder |Rn (x)| = |f (x) Tn |.
There are three possible methods for estimating the size of the error:
(1) If the series is an alternating series, we can use the Alternating Series Estimation Theorem.
(2) In all cases we can use Taylor Inequality: If |f (n+1) (x)| M for |x a| d,
then

(n+1)

f
M
(c)
n+1
|Rn (x)| =
(x a)
|x a|n+1 for |x a| d.
(n + 1)!
(n + 1)!
(3) If a graphing device is available, we can use it to graph (estimate) |Rn (x)|.

Example 1. Desmos Graphing Calculator is a free, online, graphing calculator:


https://www.desmos.com/calculator
https://desmos.s3.amazonaws.com/Desmos_User_Guide.pdf
We will illustrate Taylor polynomial approximations by Desmos.
11.11-1

Example 2 (page 769).


(a) Approximate f (x) =

x by a Taylor polynomial of degree 2 at x = 8.

(b) How accurate is this approximation when 7 x 9?


Solution.
(a) We compute
f (x) =

f (x) =

f (x) =

f (8) =

f (8) =

f (8) =

f (x) =

So the desired approximation is

x T2 (x) =
=

(b) We can use Taylors Inequality with n = 2 at x = 8:


|R2 (x)|

Thus, if 7 x 9, the approximation in (a) is accurate to within

Homework. Approximate 5 240 with error less than 0.0001.

Example 3 (page 770). What is the maximum error possible in using the approx5
3
imation sin x x x3! + x5! when 0.3 x 0.3? Use this approximation to find
sin 12 correct to six decimal places.
5

Solution. Notice that the Maclaurin series sin x = x x3! + x5! x7! + is alternating
for all x 6= 0, and the successive terms decrease in size because |x| < 1, so we can use
the
. The error in approximating sin x
by the first three terms of its Maclaurin series is at most
7
7
x
= |x|
7! 5040
To find sin 12 , we first convert to radian measure:




sin 12 = sin 12
= sin
180
15

Thus, correct to six decimal places, sin 12


11.11-2

Applications to Physics, page 772


Example 4 (page 772). In Einsteins theory of special relativity the mass of an
object moving with velocity v is
m0
m= q
1

,
v2
c2

where m0 is the mass of the object when at rest and c is the speed of light. The
kinetic energy of the object is the difference between its total energy and its energy
at rest: E = mc2 m0 c2 .
(a) Show that when v is very small compared with c, this expression for E agrees
with classical Newtonian physics: E = 21 m0 v 2 .
(b) Use Taylor Inequality to estimate the difference in these expressions for E
when |v| 100 m/s.
Solution.
(a) Using the expressions given for E and m, we get
m0 c2
E = mc2 m0 c2 = q
m0 c2 = m0 c2
v2
1 c2
2

!

 12
v2
1 2
1 .
c
1

With x = vc2 , the Maclaurin series for (1 + x) 2 is a binomial series with


m = 21 . Therefore we have
 3
 3 5
1
1

2 2 3
1
1
2
2
2
(1 + x) 2 = 1 x +
x2 +
x +
2
2!
3!
1
3
5
= 1 x + x2 x3 + ,
2
8
16
and



5 v6
1 v2 3 v4
+ 1
1+ 2 + 4 +
E = m0 c
2c
8c
16 c6

 2
3 v4
5 v6
1v
2
+ 4 +
+
= m0 c
2 c2
8c
16 c6
2

If v is much smaller than c, then all terms after the first are very small when
compared with the first term. If we omit them, we get
 2
1v
1
2
E = m0 c
= m0 v 2 .
2
2c
2
11.11-3



1
2
(b) Let f (x) = m0 c2 (1 + x) 2 1 with x = vc2 . We can use Taylors Inequality to write
|R1 (x)| =

|f (
c)| 2
x,
2!

where |
c|

v2
.
c2

Since f (x) = 43 m0 c2 (1 + x) 2 and we are given that |v| 100 m/s, so


|f (
c)| =

3m0 c2

4 (1 c) 2

3m0 c2
4 1

Thus, with c = 3 108 m/s,


|R1 (x)| =

1002
c2

 52 .

1
3m0 c2
1004

< (4.17 1010 )m0 .


2 4 1 1002  25 c4
c2

So when |v| 100 m/s, the magnitude of the error in using the Newtonian
expression for kinetic energy is at most (4.17 1010 )m0 .
Homework. If a surveyor measures differences in elevation when making plans for
a highway across a desert, corrections must be made for the curvature of the earth.
(a) If R is the radius of the earth and L is the length of the highway, show that
the correction is
 
L
R.
C = R sec
R
(b) Use a Taylor polynomial to show that
C

L2
5L4
+
.
2R 24R3

(c) Compare the corrections given by the formulas in parts (a) and (b) for a
highway that is 100 km long. (Take the radius of the earth to be 6370 km.)

L
R

Figure 1: Surveyors measures differences in elevation of highway.


Homework. Find the sum of the series (a)

n=0

11.11-4

(1)n n
32n (2n)!

(b)

(ln 2)2n
.
(2n)!

n=0

(100 )

Appendix
Example 5. Consider the function
f (x) =

e x2
0

if x 6= 0
.
if x = 0

(a) The function f (x) is continuous on (, ) because


1

1
2 = 0 = f (0),
y ey

lim e x2 = lim ey = lim


x0

and for x 6= 0, f (x) is a composition of two continuous functions g(x) = ex


and h(x) = x12 , that is, f (x) = (g h)(x).
2

(b) We will show that: For x 6= 0, f (n) (x) = Pn (y)ey , where y = x1 , and Pn (y) is
a polynomial of y with degree 3n.
(1) When n = 1, we compute
f (x) =

df
df dy
2
2
2
=
= ey (2y) (y 2 ) = 2y 3ey = P1 (y)ey ,
dx
dy dx

where P1 (y) = 2y 3 is a polynomial of y with degree 3.


(2) Assume that it is true for n = k, that is, f (k) (x) =
where Pk (y) is a polynomial with degree 3k.

dk f
dxk

= Pk (y)ey ,

(3) When n = k + 1, we compute


f

(k+1)




d dk f
d dk f dy
d 
dk+1 f
y 2
(y 2 )
P
(y)e
=
=
(x) = k+1 =
k
dx
dx dxk
dy dxk dx
dy


dPk (y) y2
y 2
=
e
+ Pk (y)e (2y) (y 2 )
dy


2
3
2 dPk (y)
+ 2y Pk (y) ey .
= y
dy

k (y)
Let Pk+1 (y) = y 2 dPdy
+ 2y 3Pk (y), which is a polynomial of y with
degree 3 + 3k = 3(k + 1).
2

(4) By mathematical induction, we know that for x 6= 0, f (n) (x) = Pn (y)ey ,


where y = x1 , and Pn (y) is a polynomial of y with degree 3n.
(c) Now, we will show that f (n) (0) = 0 for all n N.
(1) When n = 1, we compute
1

f (x) f (0)
e x2
ey
f (0) = lim
= lim
= lim 1
x0
x0 x
y
x0
y

y
2
y ey

= lim

),L
(

1
2 = 0.
y 2y ey

11.11-5

lim

(2) Assume that it is true for n = k, that is, f (k) (0) = 0.


(3) When n = k + 1, we compute
f

(k+1)

f (k) (x)
Pk (y)ey
f (k) (x) f (k) (0)
=
lim
=
lim
(0) = lim
1
x0
y
x0
x0
x
y

yPk (y)
= 0.
y
ey2

= lim

Remark that we can apply L Hospital Rule


is 0.

 3n1 
2

times to get the limit

(4) By mathematical induction, we know that f (n) (0) = 0 for all n N.


(d) Since f (0) = 0 and f (n) (0) = 0 for all n N, the Maclaurin series of f (x) is
T (x) =

X
f (n) (0)
n=0

n!

xn = f (0) +

f (1)
f (0) 2
f (n) (0) n
x+
x ++
x + = 0.
1!
2!
n!

This is a zero function, so the interval of convergence of T (x) is (, ). We


compute the remainder
Rn (x) = f (x) Tn (x) = f (x).
1

We get for any x 6= 0, lim Rn (x) = e x2 6= 0. Therefore, f (x) is not equal to


n
its Maclaurin series.
(e) For any integer k 0, let C k (R) be the set (in fact, it is a vector space)
consisting of all functions f (x) that the derivatives f (x), f (x), . . . , f (k) (x)
exist and are continuous on R. So C 0 (R), which is also denoted by C(R),
k
consists of all continuous functions on R, and C (R) =
k=0 C () consists
of all smooth functions (continuous derivatives of all orders) on R ().

Denote C (R) be the set consisting of all smooth functions f (x) that for all
x R, there exists R > 0 such that f (x) equals its Taylor series expansion on
(x R, x + R). We say a function f (x) C (R) is analytic ().
(f) The above discussion shows that the function f (x) is a smooth function, but
not an analytic function because f (x) is not analytic at x = 0. So the conclusion is C (R) ( C (R).
Remark that we have the following relations:
C 0 (R) ( C 1 (R) ( C 2 (R) ( ( C (R) ( C (R).

11.11-6

Example 6. Recall that the binomial series is

Cnm xn

n=0

X
m(m 1)(m 2) (m n + 1)

n!

n=0

xn .

We will check the convergence of the binomial series at the endpoints.


(a) If m 1, then
|Cnm xn |



m(m 1)(m 2) (m n + 1)

=
=
=

n!
|m||(m 1)||(m 2)| |(m n + 1)|
1 2 3n
=

= 1.
n!
n!

So the series

|Cnm (1)n |

|Cnm |

Cnm xn is divergent at x = 1.

n=0

(b1 ) If 1 < m < 0 and x = 1, then 0 < m < 1, and


m(m 1)(m 2) (m n + 1)
(1)n
n!
(m)(1 m)(2 m) (n 1 m)
=
n!
(n 1 m)
(m)
(m) (1 m) (2 m)

.
=
n
1
2
n1
n

Cnm xn =

Since

n=1

(m)
n

is divergent,

Cnm xn is divergent.

n=0

(b2 ) If 1 < m < 0 and x = 1, then

Cnm xn =

n=0

m(m1)(m2)(mn+1)
n!

is an al-

n=0

ternating series. We compute




m(m 1)(m 2) (m n + 1)
m


|Cn | =

n!



m(m 1)(m 2) (m n + 1) m n
m





n + 1 = |Cn+1 |,
n!

so it is a decreasing sequence. Next, we calculate




m(m 1)(m 2) (m n + 1)
m

|Cn | =

n!



m (m 1) (m 2)
(m

n
+
1)

=

1
2
3
n




 Y

n 


m
+
1
m
+
1
m
+
1
m
+
1
=
= 1
1
1
1
.

1
2
n
k
k=1
11.11-7

Since

 X
 X

n
n
n
Y
m+1
m+1
m
+
1


m
<

=
ln
1

1
ln |Cn | = ln



k
k
k
k=1
k=1
k=1
= (m + 1)

n
X
1
k=1

and lim

n
P

n k=1

1
k

1
n

= , we get

n=1

ln


lim |Cnm | = lim ln |Cnm | = lim |Cnm | = 0.

By the alternating series test,

Cnm xn is convergent.

n=0

(c) Before we check the case m > 0, we introduce the Raabes Test:
The Raabes Test. Suppose a series

an satisfies

n=1



an+1

= 1 and
lim
n an




an+1


lim n
1 < 1,
n
an

then the series is absolutely convergent.


(d) If m > 0, then

m(m1)(mn)


 m




Cn+1
an+1
n!
1 = lim n
1 = lim n
1
lim n



m


m(m1)(mn+1)
n
n
n
an
Cn


n!




|m n|
nm
= lim n
1 = lim n
1
n
n
n+1
n+1


n
= (m + 1) lim
= (m + 1) < 1.
n n + 1
By the Raabes Test,

Cnm xn is convergent.

n=0

11.11-8

Example 7. We will prove (1 + x)m =

Cnm xn on |x| < 1.

n=0

(a) Let g(x) =

Cnm xn on the interval of convergence (1, 1). We will show that

n=0

(1 + x)g (x) = mg(x) on the interval of convergence (1, 1).


We compute g (x) =

Cnm nxn1 on the interval of convergence (1, 1), and

n=1

(1 + x)g (x) = (1 + x)

Cnm nxn1

n=1

m
Cn+1
(n + 1)xn +

n=0

Cnm nxn1

n=1

X
Cnm nxn
n=0

Cnm nxn

n=1

X
m(m 1)(m 2) (m n + 1)(m n)(n + 1)
n=0

n=0

(n + 1)!

m(m 1)(m 2) (m n + 1)n n


x
n!

X
m(m 1)(m 2) (m n + 1)((m n) + n)
n=0

=m

xn

n!

xn

Cnm xn = mg(x).

n=0

(b) Let h(x) =


compute
h (x) =

g(x)
(1+x)m

for x (1, 1). We will show that h(x) 1 on (1, 1). We

(1 + x)m g (x) g(x)m(1 + x)m1


(1 + x)g (x) mg(x)
=
= 0.
(1 + x)2m
(1 + x)m+1

(c) From (b) we see that h(x) = constant. Since h(0) = 1, we get

X
g(x)
m
Cnm xn on (1, 1).

1
on
(1,
1)

(1
+
x)
=
h(x) =
(1 + x)m
n=0

11.11-9

12.6

Cylinders and Quadric Surfaces (page 827)

Cylinders, page 827


Definition 1 (page 827). A cylinder () is a surface that consists of all lines
(called rulings, ) that are parallel to a given line and pass through a given plane
curve.

 , ,
Example 2 (page 827). The following surfaces are cylinders:
(a) Circular cylinder: x2 + y 2 = 1. The rulings are parallel to the z-axis.
(b) parabolic cylinder: z = x2 . The rulings are parallel to the y-axis.
z

y
x

y
x

(a)

(b)

Figure 1: (a) Circular cylinder. (b) Parabolic cylinder.

Quadric Surfaces, page 827


Definition 3 (page 827). A quadric surfaces () is the graph of a seconddegree equation in three variables x, y, an z. The most general such equation is
Ax2 + By 2 + Cz 2 + Dxy + Eyz + F xz + Gx + Hy + Iz + J = 0,
where A, B, C, . . . , J are constants.
(a) If A = B = C = D = E = F = 0 and one of G, H, I is nonzero, then the
surface is a plane.
(b) If one of A, B, C, D, E, F is nonzero, by translation and rotation, it can be
brought into one of the two standard forms
Ax2 + By 2 + Cz 2 + J = 0

or

Ax2 + By 2 + Iz = 0.

 , D, E, F ,
12.6-1

Six types of quadric surfaces in standard form, page 830


z

y
x

Figure 2: Ellipsoid ()

x2
a2

y2
b2

z2
c2

= 1 and cone ()

z2
c2

x2
a2

y2
.
b2

y
y
x

x
Figure 3: Elliptic paraboloid ()
2
2
2
() zc2 = xa2 yb2 .

2
2
x2
+ yb2 + zc2
a2

= 1 and hyperbolic paraboloid


z

Figure 4: Hyperboloid of one sheet ()


2
2
2
of two sheets () xa2 yb2 + zc2 = 1.

x2
a2

y2
b2

z2
c2

= 1 and hyperboloid

Homework (page 833, 836). Classify the following surfaces.


(a) 4x2 + y 2 + 4z 2 4y 24z + 36 = 0.
(b) x2 y 2 + z 2 4x 2y 2z + 4 = 0.
(c) 4y 2 + z 2 x 16y 4z + 20 = 0.
(d) z = x2 y 2.

(e) y 2 + z 2 = 1 + x2 .
12.6-2

(f) 4x2 + y 2 4z 2 = 4.

Chapter 13
13.1

Vector Functions

Vector Functions and Space Curves


(page 840)

We now study functions whose values are vectors because such functions are needed
to describe curves and surfaces in space.
Definition 1 (page 840). A vector-valued function, or vector function (), is
a function whose domain is a set of real numbers and whose range is a set of vectors.
Here we will focus on vector functions r whose values are three-dimensional
vectors. This means that for every number t in the domain of r there is a unique
vector in R3 denoted by r(t). We can write
r(t) = (f (t), g(t), h(t)) = f (t) i + g(t) j + h(t) k,
where f, g, h are real-valued functions of t called the component functions (
) of r.

 i = (1, 0, 0), j = (0, 1, 0), k = (0, 0, 1) R3


 r(t) = hf (t), g(t), h(t)i ,
Example 2 (page 840). If r(t) = (t3 , ln(3 t),

t), then the component functions

are
f (t) = t3 ,

g(t) = ln(3 t),

and h(t) =

t.

By the usual convention, the domain () of r consists of all values of t for which
the expresstion for r(t) is defined. Therefore the domain of r is [0, 3).
Definition 3 (page 840). The limit of a vector function r is defined by taking the
limits of its components functions as follows. If r(t) = (f (t), g(t), h(t)), then


lim r(t) = lim f (t), lim g(t), lim h(t) = lim f (t) i + lim g(t) j + lim h(t) k
ta

ta

ta

ta

ta

ta

ta

provided the limits of the component functions exist.

Definition 4 (page 841). A vector function r is continuous at a if lim r(t) = r(a).


ta


 r ( t = a) f (t), g(t), h(t) ( t = a)
13.1-1

There is a closed connection between vector-valued functions and space curves.


Definition 5 (page 841). The set C of all points (x, y, z) in space, where
x = f (t),

y = g(t),

z = h(t),

(1)

and t varies throughout the interval I, is called a space curve (). The
equations in (1) are called parametric equations of C () and t is called a
parameter ().

 r(t) (position vector)


 R3 ,
Example 6 (page 841). The curve r(t) = cos t i + sin t j + t k is called a helix (
).
z

Figure 1: A helix r(t) = cos t i + sin t j + t k, 0 t 6.

Example 7.
(a) Find a vector equation and parametric equations for the line that join the
point A(a1 , a2 , a2 ) to the point B(b1 , b2 , b3 ).
(b) Find a vector equation and parametric equations that represents the curve of
intersection of the cylinder x2 + y 2 = 1 and the plane y + z = 2.
Solution.

13.1-2

Using computers to draw space curves, page 843


(a) Toroidal spiral:
x = (4 + sin 7t) cos t, y = (4 + sin 7t) sin t, z = cos 7t, 0 t 2.
(b) Trefoil knot:
x = (2 + cos 1.5t) cos t, y = (2 + cos 1.5t) sin t, z = sin 1.5t, 0 t 4.
(c) Twisted cubic: x = t,

y = t2 ,

z = t3 .
z

Figure 2: (a) Toroidal spiral. (b) Trefoil knot. (c) Twisted cubic.

Homework (page 847). Find a vector function that represents the curve of intersection of the two surfaces.
p
(a) The cone z = x2 + y 2 and the plane z = 1 + y.
(b) The semiellipsoid x2 + y 2 + 4z 2 = 4, y 0, and the cylinder x2 + z 2 = 1.

13.1-3

13.2

Derivatives and Integrals of Vector Functions


(page 847)

Derivatives, page 847


Definition 1 (page 847). The derivative () r (t) of a vector function r(t) is
r(t + h) r(t)
dr
= r (t) = lim
.
h0
dt
h
Definition 2 (page 848).
(a) The vector r (t0 ) is called the tangent vector () to the curve C defined
by r(t) at the point P = r(t0 ), provided that r (t0 ) exists and r (t0 ) 6= 0.
(b) The tangent line () to the curve C at P = r(t0 ) is defined to be the line
through P parallel to the tangent vector r (t0 ).
(c) The unit tangent vector () is
T(t) =

r (t)
.
|r(t)|

Theorem 3 (page 848). If r(t) = (f (t), g(t), h(t)) = f (t) i + g(t) j + h(t) k, where
f, g, and h are differentiable functions, then
r (t) = (f (t), g (t), h (t)) = f (t) i + g (t) j + h (t) k.
Theorem 4 (page 850). Suppose u(t) and v(t) are differentiable vector functions,
c is a scalar, and f (t) is a real-valued function. Then
(1)

d
(u(t)
dt

(2)

d
(c u(t))
dt

(3)

d
(f (t) u(t))
dt

(4)

d
(u(t)
dt

v(t)) = u (t) v(t) + u(t) v (t).

(5)

d
(u(t)
dt

v(t)) = u (t) v(t) + u(t) v (t).

(6)

d
(u(f (t)))
dt

+ v(t)) = u (t) + v (t).


= c u (t).
= f (t) u(t) + f (t) u (t).

= u (f (t))f (t).

Proof. Let u(t) = u1 (t) i + u2 (t) j + u3 (t) k and v(t) = v1 (t) i + v2 (t) j + v3 (t) k.

13.2-1

(1)
u(t + h) + v(t + h) (u(t) + v(t))
d
(u(t) + v(t)) = lim
h0
dt
h
v(t + h) v(t)
u(t + h) u(t)
+ lim
= u (t) + v (t).
= lim
h0
h0
h
h
(2)
d
c u(t + h) c u(t)
u(t + h) u(t)
(c u(t)) = lim
= c lim
= c u (t).
h0
h0
dt
h
h
(4)
d
(u(t) v(t))
dt
d
= (u1 (t)v1 (t) + u2 (t)v2 (t) + u3 (t)v3 (t))
dt
= u1 (t)v1 (t) + u2 (t)v2 (t) + u3 (t)v3 (t) + u1 (t)v1 (t) + u2 (t)v2 (t) + u3 (t)v3 (t)
= u (t) v(t) + u(t) v (t).
(6)
d
d
(u(f (t))) = (u1 (f (t)) i + u2 (f (t)) j + u3 (f (t)) k)
dt
dt
= u1 (f (t))f (t) i + u2 (f (t))f (t) j + u3 (f (t))f (t) k
= (u1 (f (t)) i + u2 (f (t)) j + u3 (f (t)) k)f (t) = u (f (t))f (t).

Homework. Show that


(3)

d
(f (t) u(t))
dt

(5)

d
(u(t)
dt

= f (t) u(t) + f (t) u (t).

v(t)) = u (t) v(t) + u(t) v (t).

Example 5 (page 850). If |r(t)| = c (a constant), then r(t) r(t) = c2 and


d
(r(t) r(t)) =
dt
.

Thus r (t) r(t) = 0, which says that


Homework (page 853). If r(t) 6= 0, show that

13.2-2

d
r(t) r (t)
|r(t)| =
.
dt
|r(t)|

Integrals, page 851


The definite integral () of a continuous vector function r(t) is
Z

r(t) dt = lim
n
a

n
X

r(ti )t

i=1

= lim
n

n
X
i=1

Z

f (ti )t i +

n
X

g(ti )t j +

i=1

n
X
i=1

! !

h(ti )t k


Z b

Z b

f (t) dt i +
g(t) dt j +
h(t) dt k
a

We can extend the Fundamental Theorem of Calculus to continuous vector functions


as follows:
Z

h
i t=b

= R(b) R(a),
r(t) dt = R(t)
t=a

where R is an antiderivative of r, that is, R (t) = r(t). We use the notation


for indefinite integrals () (antiderivatives) ().

13.2-3

r(t) dt

13.3

Arc Length and Curvature (page 853)

Concept of a Curve
Definition 1 (page 855). Suppose that C is a space curve given by a vector function
r(t) = f (t) i + g(t) j + h(t) k,

a t b.

We say r(t) is a smooth parametrization () if


( f (t), g(t), h(t) C 1 [a, b])

(a) r (t) is continuous on [a, b].


(b) r (t) 6= 0 for all t [a, b].

( at least one of f (t), g (t), h (t) 6= 0)

The variable t is called the parameter () of the representation.

 () ()
Definition 2 (page 855). A space curve C is called smooth curve () if it
has a smooth parametrization.
A smooth curve has no sharp corners or cusps; when the tangent vector turns,
it does so continuously.
Definition 3 (page 854). A space curve C can be smooth parametrized by more
than one vector function. We say all vector functions are parametrizations (
) of the curve C.

Example 4 (page 854). The twisted cubic r(t) = (t, t2 , t3 ), 1 t 2 could also
be represented by the function r(u) = (eu , e2u , e3u ), 0 u ln 2, where t = eu .
Example 5 (page 854). A plane curve can be thought as a special case of a space
curve. So we have many parametrizations to represent a unit circle x2 +y 2 = 1, z = 0.
For example, r1 (t) = (cos t, sin t, 0), 0 t 2, or r2 (u) = (cos 2u, sin 2u, 0), 0
u , where t = 2u.

 (regular curve) (smooth curve)


 , gauge invariance, diffeomorphism
 , ? ?

13.3-1

Arc Length as a Parameter, page 853


Definition 6.
(a) A smooth curve r(t), a t b is said to be simple if there are no multiple
points; that is, if t1 6= t2 implies r(t1 ) 6= r(t2 ).
(b) The points r(a) and r(b) are called the end points () of the curve.
Definition 7 (page 854). Suppose that C is a simple smooth curve given by
r(t) = f (t) i + g(t) j + h(t) k,

a t b.

We define its arc length function s () by


Z t
Z tp

(f (u))2 + (g (u))2 + (h (u))2 du.


s = s(t) =
|r (u)| du =
a

(1)

Thus s(t) is the length of the part of C between r(a) and r(t).
By the Fundamental Theorem of Calculus, we obtain
ds
=
dt

(2)

We know s(t) is an increasing function, and it is a one-to-one function, so its inverse


function t(s) exists and
1
dt
= ds
ds
dt

(3)

is a continuous function. Hence for a space curve C given by a vector function


r(t) = f (t) i + g(t) j + h(t) k,

a t b,

it can be reparametrized by arc length function


r(s) = r(t(s)) = f (t(s)) i + g(t(s)) j + h(t(s)) k,

c s d,

where t(c) = a and t(d) = b. We will show that r(s) is a smooth parametrization:
dr dt
df (t) dt
dg(t) dt
dh(t) dt
dr
=
=
i+
j+
k,
ds
dt ds
dt ds
dt ds
dt ds
so r(s) is a nonzero, continuous vector function.
Thus the arc length s can be introduced along the curve as a parameter. It is
often useful to parametrize a curve with respect to arc length () because
arc length arises naturally from the shape of the curve and does not depend on a
particular coordinate system.


13.3-2

Definition 8 (page 853). The length of a space curve C is the limit of lengths of
inscribed polygons. Suppose that a simple smooth curve has the vector equation
r(t) = (f (t), g(t), h(t)), a t b, where f (t), g (t), and h (t) are continuous. Then
its length () is
Z b
Z bp

(f (t))2 + (g (t))2 + (h (t))2 dt.


L=
|r (t)| dt =
a

Because a space curve C has many different parameterizations, we have to show


that the length is well-defined: Suppose that r(s) is another smooth parametrization, and s = s(t), t = t(s), c s d, t(c) = a, and t(d) = b, then

Z b
Z d
Z d
dr
dr ds dt
dr
dt =


ds.
L=
dt
ds dt ds ds =
ds
a
c
c

 (well-defined); ,

Example 9 (page 855). Reparametrize the helix r(t) = cos t i + sin t j + t k with
respect to arc length measured from (1, 0, 0) in the direction of increasing t. Find
the length of the arc length of the helix from the point (1, 0, 0) to the point (1, 0, 2).
Solution.

Curvature, page 855


If C is a smooth curve defined by the vector function r(t), recall that the unit
tangent vector is
r (t)
T(t) =
|r (t)|
and indicated the direction of the curve.
13.3-3

Remark that if a space curve is parametrized by arc length r(s) = r(t(s)), then



dr dr dt dr 1
= = = dr 1 = 1.
ds dt ds dt ds dt dr
dt
dt

That is, if a space curve C is parametrized by arc length r(s), then r (s) is unit
tangent vector.

 , 1,
Definition 10 (page 855). The curvature () of a space curve is


dT


=
ds
where T is the unit tangent vector.

 , , (chain rule)
 ,
The curvature of C at a given point is a measure of how quickly the curve changes
direction at that point. Specifically, we define it to be the magnitude of the rate of
change of the unit tangent vector with respect to arc length.
Theorem 11 (page 855861).
(a) If t is another parameter instead of the arc length s, then
(t) =

|T (t)|
.
|r(t)|

(b) The curvature of the curve given by the vector function r(t) is
(t) =

|r (t) r (t)|
.
|r (t)|3

(c) If a plane curve is given as the graph of a function y = f (x), then the curvature
of the curve is
(x) =

|f (x)|
3

(1 + (f (x))2 ) 2

(d) If a plane curve is given as a plane parameter x = x(t), y = y(t), then the
curvature of the curve is
=

|x (t)y (t) y (t)x (t)|


3

((x (t))2 + (y (t))2 ) 2


13.3-4

(e) If a plane curve is given as r = r(), a b in polar coordinates, then the


curvature of the curve is
() =

|2(r ())2 r()r () + r 2 ()|


3

((r ())2 + (r())2 ) 2

Proof.
(a) By chain rule, we have




dT(t(s))
def. dT(s)



=
=
=
ds ds

(b) Since r (t) = T (t)|r (t)|, we have


r (t) =
and
r (t) r (t) =
Notice that |T(t)| = 1, and it implies T(t) and T (t) are orthogonal, so
|r (t) r (t)|
=
|r(t)|3
(c) A plane curve can be parametrized by (x, f (x), 0) in R3 , that is,
r(x) = x i + f (x) j + 0 k.
We compute
r (x) = 1 i + f (x) j + 0 k

|r (x)| =

r (x) = 0 i + f (x) j + 0 k

1 + (f (x))2

r (x) r (x) =
so the curvature is
(x) =

|r (x) r (x)|
=
|r (x)|3

(d) A plane curve can be parametrized by (x(t), y(t), 0) in R3 , that is,


r(t) = x(t) i + y(t) j + 0 k.

13.3-5

We compute
r (t) = x (t) i + y (t) j + 0 k

|r(t)| =

r (t) = x (t) i + y (t) j + 0 k

(x (t))2 + (y (t))2

r (t) r (t) =
so the curvature is
|r (t) r (t)|
(t) =
=
|r (t)|3
(e) A plane curve can be parametrized by (r() cos , r() sin , 0) in R3 , that is,
r() = r() cos i + r() sin j + 0 k.
We compute
r () = (r cos r sin ) i + (r sin + r cos ) j + 0 k |r ()| =

p
r 2 + (r )2

r () = (r cos 2r sin r cos ) i + (r sin + 2r cos r sin ) j + 0 k,


so k-component of r () r (t) is
(r cos r sin )(r sin + 2r cos r sin )
(r sin + r cos )(r cos 2r sin r cos )
= r ()r + 2(r ())2 + r 2 ().
The curvature is
() =

|2(r ())2 r()r () + r 2 ()|


3

((r ())2 + (r())2 ) 2

Example 12 (page 856). Show that the curvature of a circle of radius r is 1r .


Solution.

13.3-6

Example 13. Find the curvature of the circular helix r(t) = cos t i + sin t j + t k.
Solution.

The Normal and Binormal Vectors, page 858


Definition 14 (page 858).
(a) We define the principal unit normal vector N(t) (or simply unit normal
) as
N(t) =

T (t)
.
|T (t)|

(b) The vector B(t) = T(t) N(t) is called the binormal vector (). It is
perpendicular to both T and N and is also a unit vector.
Example 15 (page 858). Find the unit normal and binormal vectors for the circular
helix r(t) = cos t i + sin t j + t k.
Solution.

13.3-7

Definition 16 (page 859).


(a) The plane determined by the normal vector N and binormal vector B at a
point P on a curve C is called the normal plane () of C at P .
(b) The plane determined by the tangent vector T and normal vector N is called
the osculating plane () of C at P .
(c) The circle that lies in the osculating plane of C at P , has the same tangent as
C at P , lies on the concave side of C (toward which N points), and has radius
= 1 (the reciprocal of the curvature) is called the osculating circle ()
(or the circle of curvature) () of C at P . The radius of the osculating
circle is called the radius of curvature () of C at P .

Torsion and Theory of Curve (Appendix)


Let r be a smooth curve parametrized by arc length s such that r (s) 6= 0. The
number (s) defined by B (s) = (s)N(s) is called the torsion of r(s).
To each value of the parameter s, we have associated three orthogonal unit vector
T(s), N(s), B(s). The trihedron is called Frenet trihedron at s. Since N = B T,
we have
N (s) = B (s) T(s) + B(s) T (s) = (s)B(s) (s)T(s).
Hence we get the Frenet formulas:

T
0 0
T
d


N = 0 N
ds
B
0 0
B

Physically, we can think of a space curve as being obtained from a straight line
by bending (curvature) and twisting (torsion). The following theorem states that
and describe completely the local behavior of the curve.
Fundamental Theorem of the Local Theory of Curves. Given a smooth function (s) > 0 and (s), there exists a regular parametrized curve r(s) such that s is
the arc length, (s) > 0 is the curvature, and (s) is the torsion of r(s). Moreover,
any other curve
r, satisfying the same condition, differs from r by a rigid motion;
that is, there exists an orthogonal linear map T of R3 , with positive determinant,
and a vector c such that
r = T r + c.

13.3-8

Example (TA) 17. Let r(t) =

5
t4
, t, 45 t 2
2

for t 0.

(a) Find the length of the arc 0 t 2 of r(t).


(b) Find the curvature (t).
(c) Find the unit tangent vector T(1), the principal unit normal vector N(1), and
the binormal unit vector B(1) when t = 1.
(102 )
Solution.

13.3-9

Homework. Consider a curve r(t) = (2t t3 ) i + 3t2 j + (3t + t3 ) k, t R.


(a) Find the unit tangent T(t).
(b) Find the arc length function s(t).
(c) Find the unit normal N(t).
(100 )

(d) Find the curvature (t).

Homework. Find the curvature , the unit tangent vector T, the unit normal vector
N, and binormal vector B of the curve r(t) = (cos t, sin t, ln(cos t)) at r(0) = (1, 0, 0).
(97 )
Homework. Find the curvature of the graph of the function y = ln(cos x) at x = 4 .
(95 )
Homework*. A particle moves on the circular helix r(t) = (a cos t, a sin t, b t).
(a) Is the speed of the particle constant?
(b) Find the angle of the velocity vector and z-axis.
(c) Find the curvature of the circular helix.
t
Homework*. Let r(t) = (et t, 2 6 e 2 , 3t), 1 t 1.

(92 )

(a) Find the length of the curve r(t).


(b) Find the curvature of r(t) at r = 0.

(92 )

Homework*. Let r(t) = (t2 , 32 t3 , t).


(a) Find the arc length of r(t) from t = 0 to t = 5.
(98 )

Homework*. Find the curvature (t) of the curve r(t) = et i + et j + 2t k.


(101 )
(b) Find the curvature of r(t) at t = 4.

Homework*. Let r(t) = (t2 , sin t + t cos t, cos t + t sin t), t > 0. Find an equation
of the osculating plane of the curve r(t) at the point ( 2 , , 1).
(92 )
Homework*. Consider the curve r(t) = (t+sin t cos t) i+(sin2 t) j+(2 cos t) k, t R.
(a) Find the curvature of the curve (t).
(b) Find all points where the curvature (t) is maximal, and find the unit tangent
T and principal unit normal N at those points.
(96 )
Homework*. Find the curvature of the curve r(t) = 3t i + sin(3t) j + cos(3t) k at
t = .
(99 )
13.3-10

Chapter 14
14.1

Partial Derivative

Functions of Several Variables, page 878

Functions of Two Variable, page 878


Definition 1 (page 878). A function f of two variables () is a rule that
assigns to each ordered pair of real numbers (x, y) in a set D R2 a unique real
number denoted by f (x, y). The set D is the domain () of f and its range (
) is the set of values that f takes on, that is, {f (x, y)|(x, y) D}.
We often write z = f (x, y) to make explicit the value taken on by f at the general
point (x, y). The variables x and y are independent variables () and z is the
dependent variable ().
Example 2 (page 878880).
(a) Function: f (x, y) =

x+y+1
.
x1

Domain: D = {(x, y)|x + y + 1 0, x 6= 1}.

(b) Function: g(x, y) = x ln(y 2 x). Domain: D = {(x, y)|x < y 2 }.


y

x+y+1=0

x=1
0
1

x = y2
0

(b)

(a)
Figure 1: (a) Domain of f (x, y) =

x+y+1
.
x1

(b) Domain of g(x, y) = x ln(y 2 x).

Homework. Find and sketch the domain of the function f (x, y) = sin1 (x2 +y 2 2).

Graphs, page 880


One way of visualizing the behavior of a function of two variables is to consider its
graph.
Definition 3 (page 880). If f is a function of two variables with domain D, then
the graph of f () is the set of all points (x, y, z) R3 such that z = f (x, y) and
(x, y) is in D.
14.1-1

The graph of a function f of two variables is a surface S with equation z =


f (x, y). We can visualize the graph S of f as lying directly above or below its
domain D in the xy-plane.
y

z
3

1
4

0
-1

3
2

2
3

Figure 2: The graph of f (x, y) = sin xy, 0 x 4, 0 y 4 and its level curves.

Level Curves, page 883


Another method for visualizing functions, borrowed from mapmakers, is a contour
map on which points of constant elevation are joined to form contour lines (
), or level curves ( ).
Definition 4 (page 883). The level curves of a function f of two variables are the
curves with equations f (x, y) = k, where k is a constant (in the range of f ).
The level curves f (x, y) = k are just the traces of the graph of f in the horizontal
plane z = k projected down to the xy-plane. The surface is steep where the level
curves are close together. It is somewhat flatter where they are farther apart.


z
y

0
r=

5k

x
Figure 3: Level curves of the function f (x, y) = 5 (x 3)2 (y 3)2 .

14.1-2

Homework (page 891). Match the function (a),(b),(c) with its graph (A),(B),(C)
and its contour map (I), (II), (III). Give reasons for your choices.
(a) f (x, y) = sin x sin y

60
40
20
0
-20
-4

(b) g(x, y) =

xy
1 + x2 + y 2

2
1
0
-1
-2

-2

-2

y0

0
2

-4

-4
-2

44

0
2

-4

(B)

-2

-2

-2

-4
-4

-2

(C)

(I)

0
2
44

-2

y0

-2

-4

-2

-4

44

(A)

-4

0.5
0
-0.5

-2

y0

(c) h(x, y) = ex cos y.

(II)

-4
-4

-2

(III)

Figure 4: Match functions, graphs, and contour maps.

Functions of Three or More Variables, page 887


A function of three variables (), f , is a rule that assigns to each ordered
triple (x, y, z) in a domain D R3 a unique real number denoted by f (x, y, z) R.
For instance, the temperature T at a point on the surface of the earth depends
on the longitude x and latitude y of the point and on the time, so we could write
T = f (x, y, t).
In general, a function of n variables (n-) is a rule that assigns a number z = f (x1 , x2 , . . . , xn ) to an n-tuple (x1 , x2 , . . . , xn ) of real numbers. Sometimes are will use vector notation to write such functions more compactly: If
x = (x1 , x2 , . . . , xn ), we often write f (x) in place of f (x1 , x2 , . . . , xn ).

14.1-3

14.2

Limits and Continuity, page 892

Definition 1 (page 893). Let f be a function of two variable whose domain D


includes points arbitrary close to (a, b). Then we say that the limit of f (x, y) as
(x, y) approaches (a, b) is L and we write
lim

(x,y)(a,b)

f (x, y) = L

if for every number > 0 there is a corresponding number > 0 such that if
p
(x, y) D and 0 < (x a)2 + (y b)2 < , then |f (x, y) L| < .
Other notations for the limit are
lim f (x, y) = L

xa
yb

and

f (x, y) L as (x, y) (a, b).

The definition refers only to the distance between (x, y) and (a, b). It does not
refer to the direction of approach. Therefore, if the limit exists, then f (x, y) must
approach the same limit no matter how (x, y) approaches (a, b). Therefore, we get
Property 2 (page 894). If f (x, y) L1 as (x, y) (a, b) along a path C1 and
f (x, y) L2 as (x, y) (a, b) along a path C2 , where L1 6= L2 , then lim f (x, y)
(x,y)(a,b)

does not exist.


Example 3 (page 894). Show that

x2 y 2
2 +y 2
x
(x,y)(0,0)

lim

does not exist.

Solution.

Example 4 (page 895). If f (x, y) =

xy
,
x2 +y 2

does

Solution.

14.2-1

lim

(x,y)(0,0)

f (x, y) exist?

Example 5 (page 895). If f (x, y) =

xy 2
,
x2 +y 4

does

lim

(x,y)(0,0)

f (x, y) exist?

Solution.

 ,
We can use polar coordinates to find the limit. Note that if (r, ) are polar
coordinates of the point (x, y) with r 0, then r 0+ as (x, y) (0, 0).
Example 6 (page 896). Find

3x2 y
2 +y 2
x
(x,y)(0,0)

lim

if it exists.

Solution.

Homework (page 899900). Find the limit, if it exists, or show that the limit does
not exist.
(a)
(b)
(c)
(d)

lim

xy

x2 +y 2

(x,y)(0,0)

(94 )

x2 y ey
4 +4y 2 .
x
(x,y)(0,0)

lim

x2 +y 2

lim

lim

(x2 + y 2 ) ln(x2 + y 2 ).

(x,y)(0,0)

(x,y)(0,0)

x2 +y 2 +11

14.2-2

Continuity, page 896


Definition 7 (page 897). A function f of two variables is called continuous at (a, b)
if


lim f (x, y) = f (a, b) = f
lim x, lim y .
(x,y)(a,b)

(x,y)(a,b)

(x,y)(a,b)

We say f is continuous on D if f is continuous at every point (a, b) in D.


A polynomial function of two variables (or polynomial , for short) is
a sum of terms of the form cxm y n , where c is a constant and m and n are nonnegative
integers. A rational function () is a ratio of polynomials. All polynomials
are continuous on R2 . Any rational function is continuous on its domain because it
is a quotient of continuous functions.
Example 8 (page 897). Where is the function f (x, y) =

x2 y 2
x2 +y 2

continuous?

Solution. The function f (x, y) is discontinuous at (0, 0) because it is not defined


there. Since f (x, y) is a rational function, it is continuous on its domain D =
{(x, y)|(x, y) 6= (0, 0)}.
Example 9 (page 897). Let
f (x, y) =

x2 y 2
x2 +y 2

if (x, y) 6= (0, 0)

if (x, y) = (0, 0)

Here f (x, y) is defined at (0, 0) but f (x, y) is still discontinuous there because
lim f (x, y) does not exist. (See Example 3.)
(x,y)(0,0)

Example 10. Let


f (x, y) =

3x2 y
x2 +y 2

if (x, y) 6= (0, 0)

if (x, y) = (0, 0)

We know f (x, y) is continuous for (x, y) 6= (0, 0) since it is equal to a rational


function there. From Example 6, we have

Therefore,

, and so it is continuous on

Property 11 (page 898). If f is a continuous function of two variables and g is a


continuous function of a single variable that is defined on the range of f , then the
composite function h = g f defined by h(x, y) = g(f (x, y)) is also a continuous
function.
14.2-3

Example 12 (page 898). Where is the function h(x, y) = tan1

y
x

continuous?

Solution. The function f (x, y) = xy is a rational function and therefore continuous


. The function g(t) = tan1 t is continuous everywhere, so the
except on

composition function h(x, y) = g(f (x, y)) = tan1 xy is continuous except where
.
Homework (page 900). Determine the set of points at which the function is continuous.
f (x, y) =

x2 y 3
2x2 +y 2

if (x, y) 6= (0, 0)

if (x, y) = (0, 0)

Functions of Three or More Variables, page 898


Everything that we have done in this section can be extended to functions of three
or more variables. The notation
lim

(x,y,z)(a,b,c)

f (x, y, z) = L

means that the values of f (x, y, z) approach the number L as the point (x, y, z)
approaches the point (a, b, c) along any path in the domain of f . The function f is
continuous at (a, b, c) if
lim

(x,y,z)(a,b,c)

f (x, y, z) = f (a, b, c) = f

lim

x,

(x,y,z)(a,b,c)

lim

(x,y,z)(a,b,c)

y,

lim

(x,y,z)(a,b,c)

z .

For a function of n variables, we can write these definitions in a single compact


form by vector notation. For instance, let x = (x1 , x2 , . . . , xn ), a = (a1 , a2 , . . . , an ),
and f (x) is a function of n variable, The function f is continuous at a if


lim f (x) = f (a) = f lim x .
xa

xa

Homework. Let

f (x, y, z) =

xy+yz 3
x2 +z 6

if (x, y, z) 6= (0, 0, 0)

if (x, y, z) = (0, 0, 0).

Determine the set of points at which f (x, y, z) is continuous.

14.2-4

(92 )

Example (TA) 13. Consider the function


(
|x|
|x| y 2
if y 6= 0
e
2
y
f (x, y) =
0
if y = 0.
Does

lim

(x,y)(0,0)

(92 )

f (x, y) exist?

Solution.

Homework*. Consider the function


(
f (x, y) =

2x2 y
x4 +y 2

if (x, y) 6= (0, 0)

if (x, y) = (0, 0).

Consider two curves: (t) = (t, at), a 6= 0, and (t) = (t, t2 ).


(a) Is lim f ((t)) = lim f ((t))?
t0

t0

(92 )

(b) Is f (x, y) continuous at (0, 0)?


Homework*.
(a) Find
(b) Find

2yx3
2
6.
y
(x,y)(0,0) +x

lim

lim

(x,y)(0,0)

xy

x2 +y 2

(94 )

Homework*.
(a) Find

xy 3
2 +y 6 .
x
(x,y)(0,0)

(b) Find

x2 y
2 +y 2 .
x
(x,y)(0,0)

lim

(95 )

lim

Homework*. Given a > 1, let Ca be the curve y = xa , x > 0. Find the limit of
as (x, y) tends towards (0, 0) along the curve Ca .
(99 )
f (x, y) = x3xy
y 3
14.2-5

14.3

Partial Derivative, page 900

Definition 1 (page 902). If f is a function of two variables x and y, suppose we


let only x vary while keeping y fixed, say y = y0 , then g(x) = f (x, y0 ) is a function
of a single variable x. If g(x) has a derivative at x = x0 , then we call it the partial
derivative () of f with respect to x at (x0 , y0 ) and denote it by fx (x0 , y0 ). Thus
g(x0 + h) g(x0 )
f (x0 + h, y0) f (x0 , y0)
= lim
.
h0
h0
h
h

fx (x0 , y0 ) = g (x0 ) = lim

Similarly, the partial derivative () of f with respect to y at (x0 , y0 ) and denote


it by fy (x0 , y0 ), is obtained by keeping x fixed, say x = x0 , and finding the ordinary
derivative at y = y0 of the function g(y) = f (x0 , y):
f (x0 , y0 + h) f (x0 , y0)
g(y0 + h) g(y0 )
= lim
.
h0
h0
h
h

fy (x0 , y0) = g (y0 ) = lim

Definition 2 (page 902). If f is a function of two variables, its partial derivatives


() are the functions fx and fy defined by
f (x + h, y) f (x, y)
,
h0
h
f (x, y + h) f (x, y)
.
fy (x, y) = lim
h0
h

fx (x, y) = lim

Notations for Partial Derivatives. If z = f (x, y), we write

f
=
f (x, y) =
x
x
f

fy (x, y) = fy =
=
f (x, y) =
y
y

fx (x, y) = fx =

z
= f1 = Dx f = D1 f,
x
z
= f2 = Dy f = D2 f.
y

Rule for Finding Partial Derivative of z = f (x, y).


(1) To find fx , regard y as a constant and differentiate f (x, y) with respect to x.
(2) To find fy , regard x as a constant and differentiate f (x, y) with respect to y.

 , , ,
Example 3 (page 903). If f (x, y) = x3 + x2 y 3 2y 2 , then
(a) fx (x, y) =
fx (2, 1) =
(b) fy (x, y) =
fy (2, 1) =
14.3-1

Interpretations of Partial Derivatives, page 903


The partial derivatives fx (x0 , y0 ) and fy (x0 , y0 ) can be interpreted geometrically as
the slopes of the tangent lines at P (x0 , y0, f (x0 , y0)) to the trace C1 and C2 of the
surface S in the planes y = y0 and x = x0 .
z
C1
S
P
C2
y
x

(x0 , y0 , 0)

Figure 1: Geometric meaning of partial derivatives.


Example 4 (page 905). If f (x, y) = sin
Solution.

x
1+y

, calculate

f
x

and

f
.
y

z
z
and y
if z is defined implicitly as a function of x
Example 5 (page 905). Find x
and y by the equation x3 + y 3 + z 3 + 6xyz = 1.

Solution.

Homework (page 915). If f (x, y) =

x esin(x

2 y)
3

(x2 + y 2) 2

, find fx (1, 0).

Functions of More Than Two Variables, page 905


If z = f (x1 , x2 , . . . , xn ) is a function of n variables, its partial derivative with respect
to the i-th variable xi is
z
f (x1 , . . . , xi1 , xi + h, xi+1 , . . . , xn ) f (x1 , . . . , xi , . . . , xn )
= lim
.
xi h0
h
We also write
z
f
=
= fxi = fi = Di f.
xi
xi
14.3-2

High Derivatives, page 906


If f is a function of two variables, then its partial derivatives fx and fy are also functions of two variables, so we can consider their partial derivatives (fx )x , (fx )y , (fy )x ,
and (fy )y , which are called the second partial derivatives () of f . If
z = f (x, y), we use the following notation:
 
2f
2z
f
=
=
,
(fx )x = fxx = f11 =
x x
x2
x2
 
2f
2z
f
=
=
,
(fx )y = fxy = f12 =
y x
yx
yx
 
2f
2z
f
=
=
,
(fy )x = fyx = f21 =
x y
xy
xy
 
2f
2z
f
= 2 = 2.
(fy )y = fyy = f22 =
y y
y
y

 fxy
Homework. Let r(x, y) =
and ryy .
Example 6. Let f (x, y) =

2f
yx

p
x2 + y 2 . For (x, y) 6= (0, 0), compute rx , ry , rxx , rxy , ryx ,
x3 xy 2
.
x2 + y 2

(a) Determine the value f (0, 0) such that f (x, y) is continuous at (0, 0).
(b) Find fx (x, y), fx(x, y), fx (0, 0) and fy (0, 0).
(c) Compute fxy (0, 0) and fyx (0, 0).
Solution.

14.3-3

(89 )

 ,
 (c) , ;
Clairauts Theorem (page 907). Suppose f is defined on a disk D that contains
the point (x0 , y0 ). If the functions fxy and fyx are both continuous on D, then
fxy (x0 , y0 ) = fyx (x0 , y0 ).

 fxy fyx ,
Homework (page 915). Let f (x, y) =

x3 yxy 3
x2 +y 2

if (x, y) 6= (0, 0)

if (x, y) = (0, 0)

(a) Find fx (x, y) and fy (x, y) when (x, y) 6= (0, 0).


(b) Find fx (0, 0) and fy (0, 0).
(c) Find fxy (x, y) and fyx (x, y).
(d) Find fxy (0, 0) and fyx (0, 0).
(e) Does the results of (c) and (d) contradict Clairauts Theorem?

Partial Differential Equations, page 908


Partial derivatives occur in partial differential equations () that express
certain physical laws. For instance,
(a)

2u
x2

(b)

u
t

(c)

2u
t2

+
=
=

2y
y 2

= 0: Laplaces equation ().

2u
:
x2
2u
:
x2

heat equation. ().


wave equation ()

14.3-4

14.4

Tangent Planes and Linear Approximations,


page 915

Tangent Planes, page 915


Definition 1 (page 915). Suppose that a surface S has equation z = f (x, y), where
f has continuous partial derivatives, and let P (x0 , y0 , z0 ) be a point on S. Let C1
and C2 be the curves obtained by intersecting the vertical planes y = y0 and x = x0
with the surface S. Let T1 and T2 be the tangent lines to the curves C1 and C2 at
P . Then the tangent plane () to the surface S at the point P is defined to be
the plane that contains both tangent lines T1 and T2 .
z
T1 C1
P

T2
S

C2
y

(x0 , y0 , 0)

Figure 1: The tangent plane contains the tangent lines T1 and T2 .


An equation of the tangent plane to the surface z = f (x, y) at P (x0 , y0 , z0 ) is
z z0 = fx (x0 , y0)(x x0 ) + fy (x0 , y0)(y y0 ), or
fx (x0 , y0 )(x x0 ) + fy (x0 , y0 )(y y0 ) (z z0 ) = 0

()
()

Remark 2. Since tangent vectors to S at P are e1 = (1, 0, fx (x0 , y0)) and e2 =


(0, 1, fy (x0 , y0 )), the normal vector to S at P is
n = e1 e2 = (fx (x0 , y0 ), fy (x0 , y0), 1)//(fx (x0 , y0), fy (x0 , y0 ), 1).

 (fx fy ),
Example 3. Where does the tangent plane of the surface z = exy at (1, 1, 1)
intersect the z-axis?
(97 )
Solution.

14.4-1

Homework. Find the tangent plane to the surface x2 + y 2 + z 2 = 6xzy 3 at the


point (1, 1, 1).
(99 )
Homework. Let z = y ex + cos xy . Find the equation of the tangent plane and the
normal line equation to the surface at (x, y, z) = (1, 2 , e
).
(96 )
2

Linear Approximations, page 917


Definition 4 (page 917). An equation of the tangent plane to the graph of the
function z = f (x, y) at P (x0 , y0, z0 ) is z z0 = z f (x0 , y0) = fx (x0 , y0 )(x x0 ) +
fy (x0 , y0 )(y y0 ). The linear function whose graph is this tangent plane, namely,
L(x, y) = f (x0 , y0 ) + fx (x0 , y0)(x x0 ) + fy (x0 , y0)(y y0 )
is called linearization () of f at (x0 , y0 ) and the approximation
f (x, y) f (x0 , y0 ) + fx (x0 , y0)(x x0 ) + fy (x0 , y0)(y y0 )

(1)

is called the linear approximation () or tangent plane approximation of f at


(x0 , y0 ).
(
xy
if (x, y) 6= (0, 0)
x2 +y 2
.
Example 5 (page 917). Consider the function f (x, y) =
0
if (x, y) = (0, 0)
(a) fx (0, 0) =
(b) fy (0, 0) =
(c) We take the path C1 (x) = (x, x), the function f (x, y)|C1 =
(d) A function of two variables can behave badly even through both of its partial
derivatives exist. To rule out such behavior, we will define a differentiable
function () of two variable.
Definition 6 (page 918). If z = f (x, y), then f is differentiable () at (x0 , y0 )
if z can be expressed in the form
z = fx (x0 , y0 )x + fy (x0 , y0 )y + 1 x + 2 y
where 1 0 and 2 0 as (x, y) (0, 0).
Sometimes it is hard to use the definition to check the differentiability of a
function, but the next theorem provides a convenient sufficient condition for differentiability.
Theorem 7 (page 918). If the partial derivatives fx and fy exist near (x0 , y0 ) and
are continuous at (x0 , y0 ), then f is differentiable at (x0 , y0 ).

 ,
14.4-2

Differentials, page 919


For a differentiable function of two variables, z = f (x, y), we define the differentials
() dx and dy to be independent variables; that is, they can be given any values.
Then the differential dz, also called the total differential (), is defined by
dz = df = fx (x, y) dx + fy (x, y) dy =

f
z
z
f
dx +
dy =
dx +
dy.
x
y
x
y

(2)

If we take dx = x = x x0 and dy = y = y y0 in (2), then the differential of


z is dz = fx (x0 , y0)(x x0 ) + fy (x0 , y0 )(y y0 ), so in notation of differentials, the
linear approximation (1) can be written as f (x, y) f (x0 , y0) + dz.
Figure 2 shows the geometric interpretation of the differential dx and the increment z: dz represents the change in height of the tangent plane, whereas z
represents the change in height of the surface z = f (x, y) when (x, y) changes from
(x0 , y0 ) to (x0 + x, y0 + y).
z

(x0 + x, y0 + y, f (x0 + x, y0 + y))


z = f (x, y)

z
dz

(x0 , y0, f (x0 , y0 ))

x = dx
x

(x0 , y0 , 0)

y
(x0 + x, y0 + y, 0)

y = dy

Figure 2: Geometric interpretation of the differential dz and the increment z.


Example 8 (page 921). The base radius and height of a right circular cone are
measured as 10 cm and 25 cm, respectively, with a possible error in measurement of
as much as 0.1 cm in each. Use differentials to estimate the maximum error in the
calculated volume of the cone.
Solution.

14.4-3

Functions of Three or More Variables, page 921


Linear approximations, differentiability, and differentials can be defined in a similar
manner for functions of more than two variables.

Example (TA) 9. Let f (x, y) =

x2 y
x2 +y 2

if (x, y) 6= (0, 0)

if (x, y) = (0, 0)

(101 )

(a) f (x, y) is continuous at (0, 0) because

(b) fx (0, 0) =
(c) fy (0, 0) =
(d) For (x, y) 6= (0, 0),
(e)

f
x

f
(x, y)
x

is not continuous at (0, 0) because we take the path C1 (x) = (x, x), x 6=
0, then the function fx (x, y)|C1 =

(f) Compute for (x, y) 6= (0, 0)


f (x, y) fx (0, 0)x fy (0, 0)y =
and take the path C1 (x) = (x, x), x 6= 0, we find
f (x, y) fx (0, 0)x fy (0, 0)y|C1(x) =
(g) Form (e) and (f), we know that L(x, y) = fx (0, 0)x + fy (0, 0)y 0 is not a
good linear approximation of f (x, y) at (0, 0).
Homework*. Consider the surface with equation f (x, y, z) = x2 yz + 3y 2 2xz 2 +
8z = 0 and a point p = (1, 2, 1) on the surface.
(a) Find the tangent plane to the surface at p.
(b) Find the equation of the normal line to the surface at p.
Homework*. Consider the surface given by the equation

(89 )

16
(x3 +y 2 )+xyz 2 +8z
3

= 0.

(a) Find zx , zy , zxx and zxy at (1, 1, 0).


(b) Find the tangent plane to the surface at (1, 1, 0).

14.4-4

(92 )

14.5

The Chain Rule, page 924

The Chain Rule, Case 1 (page 924). Suppose that z = z(x, y) is a differentiable
function of x and y, where x = x(t) and y = y(t) are both differentiable function of
t. Then z is a differentiable function of t and
z dx z dy
dz
=
+
.
dt
x dt
y dt

 z(t) = f (x(t), y(t)) (tree diagram) :


Example 1 (page 925). If z = x2 y + 3xy 4 , where x = sin 2t and y cos t, find
when t = 0.

dz
dt

Solution.

Example 2. Find the second derivative

d2 z
.
dt2

Solution. We compute
d2 z
=
dt2

The Chain Rule, Case 2. Suppose that z = z(x, y) is a differentiable function of


x and y, where x = x(s, t) and y = y(s, t) are differentiable functions of s and t.
Then
z x z y
z
=
+
,
s
x s y s

z
z x z y
=
+
.
t
x t
y t

 z(s, t) = f (x(s, t), y(s, t)) :


Case 2 of the Chain Rule contains three types of variables: s and t are independent variables, x and y are intermediate variables, and z is the dependent variable.
14.5-1

Example 3. Let z = e x2 +1 cos y.


(a) Find

z
x

(b) Let x =

and

z
y

u
u2 +v3 +1

at x = 0 and y = 21 .
and y =

v
.
u+v4 +1

Find

z
u

and

z
v

at the point u = 0 and v = 1.


(99 )

Solution.

Homework (page 928). If g(s, t) = f (s2 t2 , t2 s2 ) and f is differentiable, show


that g satisfies the equation t g
+ s g
= 0.
s
t
The Chain Rule, General Version. Suppose that u is a differentiable function
of the n variables x1 , x2 , . . . , xn , and each xi is a differentiable function of the m
variables t1 , t2 , . . . , tn . Then u is a function of t1 , t2 , . . . , tm and
u
u x1
u x2
u xn
=
+
++
.
ti
x1 ti
x2 ti
xn ti


Example 4. Let z = y+f (x2 y 2 ) and f be a differentiable function in one variable.
z
z
+ x y
when x = a and y = b.
Find the value of y x

Solution.

14.5-2

(98 )

Coordinate Changes
In R2 , denote (x, y) as the Cartesian coordinates and (r, ) as the polar coordinates.
We know relations between these coordinates are
(
(
p
x = r cos
r = x2 + y 2
 .
y = r sin .
= tan1 xy
So we know x = x(r, ), y = y(r, ) and r = r(x, y), = (x, y), and hence
x = x(r(x, y), (x, y)) and y = y(r(x, y), (x, y)).
Since x and y are independent variables, we have

We write down each term:

So they are inverse matrices. We check that

14.5-3

Example 5. Let z = f (x, y) such that all the second partial derivative of f are
continuous. Let x = r cos and y = r sin .
(a) Evaluate

r 2 r
,
, ,
x x2 x

and

2
.
x2

Express the results in functions of r and .

(b) Express fx and fxx in terms of (functions of) r, , fr , f , frr , fr , and f .


(97,101 )
Solution.

Homework. Suppose that w = f (x, y) satisfies the Laplace equation: wxx +wyy = 0.
Let x = 12 (u2 v 2 ) and y = uv. Show that w also satisfies the Laplace equation:
wuu + wvv = 0.
(90 )
Homework (page 932). Let z = f (x, y) be a differentiable function. From relations
x = r cos and y = r sin , we get the identity:
 2
 2
 2  2
z
z
z
z
+
= A(r, )
+ B(r, )
.
x
y
r

(96 )

Find A(r, ) and B(r, ).


14.5-4

Implicit Differentiation ()
The Chain Rule can be used to give a more complete description of the process of
implicit differentiation. Suppose that an equation of the form F (x, y) = 0 defines y
implicitly as a differentiable function of x, that is, y = f (x), where F (x, f (x)) = 0
for all x in the domain of f . If F is differentiable, we can apply the Chain Rule to
differentiate both side of the equation F (x, y) = 0 with respect to x to get

If

F
y

6= 0, we solve

dy
dx

and obtain

dy
=
dx
Implicit Function Theorem (page 929). If F (x, y) is defined on a disk containing
(x0 , y0 ), where F (x0 , y0 ) = 0, Fy (x0 , y0 ) 6= 0, and Fx and Fy are continuous on the
disk, then the equation F (x, y) = 0 defines y as a function of x near the point (x0 , y0 )
dy
and the derivative of this function is dx
= FFxy .
Now we suppose that z is given implicitly as a function z = f (x, y) by an equation
of the form F (x, y, z) = 0. This means that F (x, y, f (x, y)) = 0 for all (x, y) in the
domain of f . If F and f are differentiable, then we can use the Chain Rule to
differentiable the equation F (x, y, z) = 0 as follows:

If

F
z

6= 0, we solve

z
x

and obtain

Implicit Function Theorem (page 930). If F (x, y) is defined within a sphere


containing (x0 , y0 , z0 ), where F (x0 , y0) = 0, Fy (x0 , y0 , z0 ) 6= 0, and Fx , Fy and Fz are
continuous inside the sphere, then the equation F (x, y, z) = 0 defines z as a function
of x and y near the point (x0 , y0 , z0 ) and the partial derivatives of this function are
z
z
= FFxz and y
= FFyz .
x
Example 6 (page 930). Find

z
x

and

z
y

if F (x, y, z) = x3 + y 3 + z 3 + 6xyz 1 = 0.

Solution.

14.5-5

Homework (page 932). If f (x, y) = 0 define y as a function of x, show that


fxx fy2 2fxy fx fy + fyy fx2
d2 y
.
=
dx2
fy3
(92 )
Example (TA) 7. Suppose that f (x, y) satisfies

f
x

f
.
y

(a) Find a polynomial f (x, y) with degree 3 satisfies the equation.


(b) Let u = x + y and v = x y. Find

(91 )

f
.
v

Solution.

Homework*. Suppose f (r, , z ) is a smooth function in cylindrical coordinates.


Cylindrical coordinates (r, , z ) are related to Cartesian coordinates (x, y, z) by the
equations:
x = r cos ,

y = r sin ,

z = z.

(a) Express fx , fy , and fz in terms of (functions of) r, , z , fr , f , and fz .


(b) Find the maximum rate of change of f (r, , z ).

(89 )

Homework*. Suppose that f (x, y) is a smooth function and let x = u2 v 2 , and


y = 2uv. Express fx and fy in terms of (functions of) u, v, fu, and fv . (91 )
Homework*. Let x, y, u and v be related by the equation
(
xyuv = 1
x + y + u + v = 0.

y
.
Find x
u
14.5-6

(100 )

14.6

Directional Derivatives and the Gradient Vector, page 933

Directional Derivatives, page 933


Definition 1 (page 934). The directional derivative () of f (x, y) at (x0 , y0 )
in the direction of a unit vector u = (a, b) is
f (x0 + ha, y0 + hb) f (x0 , y0 )
h0
h

Du f (x0 , y0) = lim


if this limit exists.
z

y
x

ha

(x0 , y0 , 0)
hb

Figure 1: Directional derivative.

u ; ,

Theorem 2 (page 935). If f is a differentiable function of x and y, then f has a


directional derivative in the direction of any unit vector u = (a, b) and
Du f (x, y) = fx (x, y)a + fy (x, y)b = (fx (x, y), fy (x, y)) (a, b)
Proof. Define g(h) = f (x0 + ha, y0 + hb), then by the definition of a directional
derivative and the Chain Rule, where x = x0 + ha and y = y0 + hb such that
g = f (x, y), we have


g dx g dy

+
Du f (x0 , y0 ) = g (0) =
x dh y dh h=0
= fx (x0 , y0)a + fy (x0 , y0 )b = (fx (x0 , y0 ), fy (x0 , y0)) (a, b).

If the unit vector u makes an angle with the positive x-axis, then we can write
u = (cos , sin ) and the directional derivative becomes
Du f (x, y) = fx (x, y) cos + fy (x, y) sin = (fx (x, y), fy (x, y)) (cos , sin )
14.6-1

The Gradient Vector, page 936


Definition 3 (page 936). If f is a function of two variables x and y, then the
gradient () of f is the vector function f or grad f defined by
def.

f (x, y) = grad f (x, y) = (fx (x, y), fy (x, y)) =

f
f
i+
j.
x
y

Theorem 4 (page 935). If f is a differentiable function of x and y, then f has a


directional derivative in the direction of any unit vector u = (a, b) and
Du f (x, y) = f (x, y) u.


Maximizing the Directional Derivative
Theorem 5 (page 939). Suppose f is a differentiable function of two variables. The
maximum value of the directional derivative Du f (x, y) is |f (x, y)| and it occurs
when u has the same direction as the gradient vector f (x, y).
Proof. Since |u| = 1, we have
Du f = f u = |f ||u| cos = |f | cos ,
where is the angle between f and u. The maximum value of cos is 1 and this
occurs when = 0. Therefore the maximum value of Du f is f when u is the same
direction as f .


Example 6. Let f (x, y) = 2x2 xy + y 2 2x + y.
(a) Find the directional derivative Du f (p), where p = (0, 0) and u =

3 1
,
2 2


.

(b) Find the unit vector v that the directional derivative of Dv f (p) is maximal.
(91 )
Solution.

14.6-2

Homework. Let f (x, y) = x ey + cos(xy).


(a) Find the direction (a unit vector u) in which f (x, y) increases most rapidly at
(2, 0) (that is, Du f (2, 0) is maximal).
(b) Find the direction in which f (x, y) decreases most rapidly at (2, 0).
(c) What are the direction of zero change in f (x, y) at (2, 0)?

(101 )

Functions of Three Variables, page 937


Definition 7 (page 937). The directional derivative of f at (x0 , y0, z0 ) in the direction of a unit vector u = (a, b, c) is
f (x0 + ha, y0 + hb, z0 + hc) f (x0 , y0 , z0 )
.
h0
h
For a function f of three variable, the gradient vector, denoted by f or grad f ,
Du f (x0 , y0 , z0 ) = lim

is
f = (fx , fy , fz ) =

f
f
f
i+
j+
k
x
y
z

If we use vector notation, then we can write the directional derivative in the
compact form:
f (x0 + hu) f (x0 )
= f (x0 ) u.
h0
h
where x0 = (x0 , y0) if n = 2 and x0 = (x0 , y0 , z0 ) if n = 3.
Du f (x0 ) = lim

Example 8. Let T (x, y, z) = ex

2 3y 2 9z 2

(a) Find the directional derivative of T (x, y, z) at P0 = (2, 1, 2) toward the point
(3, 3, 3).
(b) Find the maximum of directional derivative of T (x, y, z) at P0 = (2. 1, 2).
(98 )
Solution.

z
attain the maximum rate of
Homework. In what direction does f (x, y, z) = xy
change at the point (1, 2, 4)? What is this maximum rate of change? (99 )

14.6-3

Tangent Planes to Level Surfaces, page 940


Suppose S is a level surface with equation F (x, y, z) = k, and let P (x0 , y0 , z0 ) be
a point on S. Let C be any curve that lies on the surface S and passes through
the point P , that is, C is parametrized by r(t) = (x(t), y(t), z(t)) and r(t0 ) =
(x(t0 ), y(t0 ), z(t0 )) = (x0 , y0 , z0 ). Since C lies on S, we know
F (x(t), y(t), z(t)) = k.

(1)

If x, y, and z are differentiable functions of t and F is also differentiable, then we


can use the Chain Rule to differentiate both sides of equation (1) as follows:
F dx F dy F dz
+
+
= 0 F r (t) = 0.
x dt
y dt
z dt
In particular, when t = t0 , we have F (x0 .y0 , z0 ) r (t0 ) = 0
The gradient vector at P , F (x0 , y0 , z0 ), is perpendicular to the tangent
vector r (0) to any curve C on S that passes through P .
Definition 9 (page 940). If F (x0 , y0 , z0 ) 6= 0, it is therefore natural to define the
tangent plane to the level surface F (x, y, z) = k at P () as the plane
that passes through P and has normal vector F (x0 , y0, z0 ). The equation is
Fx (x0 , y0, z0 )(x x0 ) + Fy (x0 , y0 , z0 )(y y0 ) + Fz (x0 , y0 , z0 )(z z0 ) = 0.

(2)

 ,
Definition 10 (page 941). The normal line () to S at P is the line passing
through P and perpendicular to the tangent plane. The direction of the normal line
is the gradient vector F (x0 , y0, z0 ), and so its symmetric equations are
x x0
y y0
z z0
=
=
.
Fx (x0 , y0 , z0 )
Fy (x0 , y0 , z0 )
Fz (x0 , y0 , z0 )

(3)

 z = f (x, y), F (x, y, z) = z f (x, y) = 0


Example 11 (page 941). Find the equations of the tangent plane and normal line
3
2
at P (2, 1, 3) to the ellipsoid x4 + y 2 + z9 = 3.
Solution.

14.6-4

Homework (page 945). Find the equations of the tangent plane and normal line
at P (0, 0, 1) to the surface x + y + z = exyz .

Significance of the Gradient Vector, page 942


(1) The gradient vector f gives the direction of fastest increases of f .
(2) The gradient vector f is orthogonal to the level surface.
y
f (p)
p

f (x, y) = l
x

curve of steepest ascent

Figure 2: The gradient vector is orthogonal to the level surface.

Intersection of Two Surfaces


Example 12. Find the parametric equation of the tangent line to the curve of
intersection of the surfaces x2 + 2y 2 + z 2 = 4 and x2 + y 2 z 2 = 1 at the point
(94 )

(1, 1, 1).
Solution.

Homework. Two surfaces x3 + 3x2 y 2 + y 3 + 4xy z 2 = 0 and x2 + y 2 + z 2 11 = 0


intersect a curve. Find the parametric equations for the tangent line to the surface
(95 )

at the point (1, 1, 3).


14.6-5

Example (TA) 13. Let f (x, y) = sin(x y)ex

2 y 2


and p = ( 2, 2).

(a) Find the maximum rate of change of f at p.


(b) Find the direction in which the maximum rate of change occurs.
(c) Find the directional derivative Du f (p), where u = ( 21 ,

3
).
2

(102 )

Solution.

Homework*. Let f (x, y) = x2 + xy and p = (2, 3). Find u = (cos , sin ) such
that Du f (p) = 0.
(90 )
Homework* (page 945). Show that the ellipsoid 3x2 + 2y 2 + z 2 = 9 and the sphere
x2 + y 2 + z 2 8x 6y 8z + 24 = 0 are tangent to each other at the point (1, 1, 2).
(92 )
Homework*. For f (x, y, z) = ln(x2 + y 2 ) + z.
(a) Find f .
(b) Consider the cylinder of radius 5 with axis along the z-axis. Find the normal
vector to the cylinder at the point (3, 4, 4).
(c) Find the rate of change of f in the direction normal to the cylinder at the
point (3, 4, 4).
(94 )
Homework*. Let T (x, y, z) = x2 + y 2 + z 2 xyz xz.
(a) Find Du T (1, 2, 3) where u is the direction i + 2 j + 3 k.
(b) In what direction does T (x, y, z) at (1, 2, 3) have the maximum rate of change?
What is this maximum rate of change?
(95 )

14.6-6

Homework*. Suppose that the function z = f (x, y) satisfies the following equation
sin(x + y) + sin(y + z) + sin(z + x) = 0
in the neighborhood of the point (x, y, z) = (, , ). In what direction does z =
f (x, y) at (, ) have the maximum rate of change? What is this maximum rate of
change?
(96 )
Homework*. You are wandering around in a strange desert where the temperature
2
at the point (x, y) is given by the function T (x, y) = eyx .
(a) You have stopped at the point (2, 1). Suddenly you are feeling chilled and
want to warm up - in what direction should you go to warm up as rapidly as
possible?
(b) Figure out the coordinates of all points where there is no increase or decrease
in temperature in the (1, 1) direction.
(97 )
Homework*. Consider the function
(
f (x, y) =

(a) Find

lim

(x,y)(0,0)

f
y

(d) Find

if (x, y) 6= (0, 0)

if (x, y) = (0, 0).

f (x, y). Is f continuous at (0, 0)?

(b) Find the partial derivatives


(c) Is

x3
x2 +y 2

f
x

and

f
.
y

continuous at (0, 0)?


2f
f (0, 0)
xy

and

2f
(0, 0).
yx

(e) Find the directional derivative of f at (1, 1) in the direction of 5 i + 12 j.


(f) Find the maximal rate of change of f at (1, 1) and the direction in which it
occurs.
(100 )
Homework*. Let
f (x, y) =

x2 (x2 + y 2) 2 esin(x
0

2 y)

if (x, y) 6= (0, 0)
if (x, y) = (0, 0).

(a) Let u = (cos , sin ). Find Du f (0, 0).


(b) Prove that f (x, y) is continuous (0, 0).

14.6-7

(92 )

Homework*. Let C be the curve of intersection of the paraboloid z = x2 + 21 xy + y4


and the circular cylinder x2 + y 2 = 13.
(a) Find a parametric equation for the tangent line to C at the point (3, 2, 13).

(b) An object is moving along C. If the x-coordinate is increasing at the rate of


4 cm/ sec, how fast is the z-coordinate changing at the instant when x = 3 cm
and y = 2 cm?
(100 )
Homework*.
(a) Suppose fx (0, 0) = 2, fy (0, 0) = 3. Suppose v = g(u) satisfies f (u, v 2 + v) = 0
and g(0) = 0. Find g (0).
(b) Find all points at which the direction of fastest increase of the function f (x, y) =
x2 + y 2 2x 4y is 2 i + j.

(94 )

14.6-8

14.7

Maximum and Minimum Values, page 946

Definition 1 (page 946). A function of two variables has a local maximum (

) at (x0 , y0 ) if f (x, y) f (x0 , y0 ) when (x, y) is near (x0 , y0). (This means
that f (x, y) f (x0 , y0 ) for all points (x, y) in some disk with center (x0 , y0).) The
number f (x0 , y0) is called a local maximum value (). If f (x, y) f (x0 , y0 )
when (x, y) is near (x0 , y0), then f has a local minimum () at (x0 , y0 )
and f (x0 , y0 ) is a local minimum value ().
Definition 2 (page 946). If the inequalities in Definition 1 hold for all points (x, y)
in the domain of f , then f has an absolute maximum () or absolute minimum
() at (x, y0 ).
z

y
x

y
x

(a)

(b)

Figure 1: (a) Local and absolute minimum. (b) Local and absolute maximum.
Theorem 3 (page 946). If f has a local maximum or minimum at (x0 , y0 ) and the
first-order partial derivatives of f exist there, then fx (x0 , y0) = 0 and fy (x0 , y0 ) = 0.
Proof. Let g(x) = f (x, y0 ). If f has a local maximum (or minimum) at (x0 , y0), then
g(x) has a local maximum (or minimum) at x0 , so by Fermats Theorem, we get
g (x0 ) = 0 = fx (x0 , y0 ). Similarly, by applying Fermats Theorem to the function
g(y) = g(x0 , y0), we obtain g (y0 ) = 0 = fy (x0 , y0 ).

 (x0 , y0) , f (x0, y0) = (fx (x0 , y0), fy (x0 , y0)) = (0, 0) = 0
Definition 4 (page 946). A point (x0 , y0 ) is called a critical point () or stationary point ( ) of f if fx (x0 , y0 ) = 0 and fy (x0 , y0 ) = 0, or if one of
these partial derivatives does not exist.

 fx (x0, y0) = fy (x0 , y0) = 0 ,


 ,
14.7-1

Example 5 (page 946). Consider f (x, y) = x2 + y 2 2x 6y + 14, then


fx (x, y) = 2x 2

fy (x, y) = 2y 6.

These partial derivative are equal to 0 when x = 1 and y = 3, so the only critical
point is (1, 3). Since f (x, y) = 4 + (x 1)2 + (y 3)3 4 for all x and y, f (1, 3) = 4
is a local minimum, and in fact it is the absolute minimum of f .
with vertex (1, 3, 4).
The graph of f is the
Example 6 (page 947). Consider the function f (x, y) = y 2 x2 . Since fx = 2x and
. For points on the x-axis (x 6= 0) we have
fy = 2y, the only critical point is
2
f (x, 0) = x < 0 and for points on the y-axis (y 6= 0) we have f (x, 0) = y 2 > 0.
Thus every disk with center (0, 0) contains points where f takes positive values and
negative values. Therefore, f has no extreme value.
The graph of f is the
.
Definition 7 (page 947). The graph of z = y 2 x2 has a horizontal tangent plane
z = 0 at the origin. f (0, 0) = 0 is a maximum in the direction of x-axis but a
minimum in the direction of the y-axis. Near the origin the graph has the shape of
a saddle and so (0, 0) is called a saddle point () of f .
z

y
x
Figure 2: (0, 0) is a saddle point of f (x, y) = y 2 x2 .
For a function of one variable f (x), we use second derivative of f (x) to detect
the critical points are local maximum or local minimum. Here we will introduce the
Second Derivative Test for functions of two variables to investigate the properties
of critical points.
Definition 8. The Hessian matrix or Hessian () of f (x, y) at (x0 , y0 ) is
"
#
fxx (x0 , y0) fxy (x0 , y0 )
Hess (f )(x0 , y0) =
.
fyx (x0 , y0 ) fyy (x0 , y0 )

14.7-2

Second Derivative Test (page 947). Suppose the second partial derivatives of f
are continuous on a disk with center (x0 , y0), and suppose that fx (x0 , y0 ) = 0 and
fy (x0 , y0 ) = 0 (that is, (x0 , y0 ) is a critical point of f ). Let
D = D(x0 , y0 ) = det (Hess(f )(x0 , y0 )) = fxx (x0 , y0 )fyy (x0 , y0 ) (fxy (x0 , y0))2 .
(a) If D > 0 and fxx (x0 , y0) > 0, then f (x0 , y0 ) is a local minimum.
(b) If D > 0 and fxx (x0 , y0) < 0, then f (x0 , y0 ) is a local maximum.
(c) If D < 0 and f (x0 , y0), then f (x0 , y0 ) is not a local maximum or minimum.

 (c)
 D = 0, ( ) ;
Remark 9.
(1) , Hess(f ) = P DP 1, D ; P

(2) det(AB) = det(BA), ; det(Hess(f )) =


det(P DP 1) = det(P P 1D) = det(ID) = det(D)
(3) fxx > 0 x , () ; fxx < 0 x

, ()
Example 10. Find the extreme value (local maximum and minimum values and
saddle points) of the function f (x, y) = 2x3 4xy + 3y 2.
Solution.

14.7-3

(90 )

Example 11 (page 950). Find the shortest distance from the point (1, 0, 2) to the
plane x + 2y + z = 4.
Solution.

Example 12 (page 950). A rectangular box without a lid is to be made from 12 m2


of a cardboard. Find the maximum volume of such a box.
Solution.

14.7-4

Absolute Maximum and Minimum Values, page 951


Recall that for one variable function f (x), the Extreme Value Theorem says that if
f is continuous on a closed interval [a, b], then f has an absolute maximum value
and an absolute minimum value. Absolute maximum and absolute minimum points
are happened at the critical points or endpoints.
We will introduce the Extreme Value Theorem of two variables in this section.
Definition 13 (page 951).
(a) A boundary point () of a set D R2 is a point (x0 , y0 ) such that every
disk with center (x0 , y0) contains points in D and also points not in D.
(b) A closed set () D in R2 is one that contains all its boundary points.
(c) A bounded set () D in R2 is one that is contained within some disk.

(a)

(b)

Figure 3: (a) Closed sets. (b) Sets that are not closed.

Extreme Value Theorem for Functions of Two Variables (page 951). If f is


continuous on a closed, bounded set D in R2 , then f attains an absolute maximum
value f (x1 , y1 ) and an absolute minimum value f (x2 , y2 ) at some points (x1 , y1) and
(x2 , y2 ) in D.
To find the absolute maximum and minimum values of a continuous function f
on a closed, bounded set D:
(a) Find the values of f at the critical points of f in D.
(b) Find the extreme values of f on the boundary of D.
(c) The largest of the values from step 1 and 2 is the absolute maximum value;
the smallest of these values is the absolute minimum value.

 , ,
14.7-5

Example 14. Find the extreme values of f (x, y) = x2 + xy + y 2 4x + 3y in the


region bounded by x = 0, y = 0, and x + y = 4.
(90 )
Solution.

Example 15. Find the absolute maximum and minimum values of f (x, y) = 4x +
6y x2 y 2 in the region x2 + y 2 1.
(92 )
Solution.

14.7-6

Example (TA) 16. Find and classify the critical points of the function f (x, y) =
2
2
(x2 + y 2 )ey x .
(98 )
Solution.

Example (TA) 17. Find the extreme value of the function f (x, y) = e3x 2y in
the region D = {(x, y) : x2 + y 2 1, y x 1, y x 1}.
(89 )
Solution.

14.7-7

Homework. Find and classify the critical points of f (x, y) = xyex

2 y 2

(97 )
Homework. Find the local maximum, and local minimum values and saddle point(s)
of f (x, y) = y 3 + 3x2 y 3x2 3y 2 + 3.

(102 )

Homework (page 955). Find the maximum volume of a rectangular box that is
inscribed in a sphere of radius r.

2
2
Homework*. Find the extreme value and critical points of f (x, y) = x2 3xy y2
in the region x2 + y 2 1.
(91 )
Homework*. Let f (x, y) = y 3 + 2x2 + 6xy + 6y. Find all critical points of the
function, and determine they are local maximum, local minimum, or saddle points.
(95 )
Homework*. Find all critical points of the function z = x3 2x + xy 2 + y + 1 and
determine they are local maximum, local minimum or saddle points. (96 )
Homework*. Find the critical points of f (x, y) = x3 + y 2 2xy + 7x 8y + 2.
Which of them give rise to maximum values, minimum values and saddle points?
(99 )
Homework*. Find and classify the critical points of the function f (x, y) = x4 +
y 4 4xy.

(100 )

Appendix
Proof of the Second Derivative Test. We compute the second-order directional derivative of f in the direction of u = (h, k). The first-order derivative is
Du f = fx h + fy k.
Apply this theorem a second time, we have



2
(fx h + fy k), (fx h + fy k) (h, k)
Du f = Du (Du f ) = (Du f ) u =
x
y
= (fxx h + fyx k)h + (fxy h + fyy )k = fxx h2 + 2fxy hk + fyy k 2 .

If we complete the square in this expression, we obtain



2

fxy
k2
2
2
Du f = fxx h +
fxx fyy fxy
.
k +
fxx
fxx
14.7-8

We are given that fxx (x0 , y0 ) > 0 and D(x0 , y0 ) > 0. But fxx and D are continuous
functions, so there is a disk B with center (x0 , y0) and radius > 0 such that
fxx (x, y) > 0 and D(x, y) > 0 whenever (x, y) is in B. Therefore, we see that
Du2 (x, y) > 0 whenever (x, y) is in B. This means that if C is the curve obtained
by intersecting the graph of f with the vertical plane through P (x0 , y0 , f (x0 , y0 )) in
the direction of u, then C is concave upward on an interval of length 2. This is
true in the direction of every vector u, so if we restrict (x, y) to lie in B, the graph
of f lies above its horizontal tangent plane at P . Thus f (x, y) f (x0 , y0) whenever
(x, y) is in B. This shows that f (x0 , y0 ) is a local minimum.

14.7-9

14.8

Lagrange Multipliers, page 957

Method of Lagrange Multipliers (page 958). To find the maximum and minimum values of f (x, y, z) subject to the constraint g(x, y, z) = k (assuming that these
extreme values exist and g 6= 0 on the surface g(x, y, z) = k):
(a) Find all values of x, y, z, and such that
f (x, y, z) = g(x, y, z)

and

g(x, y, z) = k.

(b) Evaluate f at all the points (x, y, z) that result from step (a). The largest
(smallest) of these values is the maximum (minimum) value of f .
Proof. For R, consider F (x, y, z, ) = f (x, y, z) (g(x, y, z) k), then on the
level set g(x, y, z) = k, the function F (x, y, z, ) and f (x, y, z) take the same value.
So to find the extreme value of f (x, y, z) on the surface g(x, y, z) = k is equivalent
to find the extreme value of F (x, y, z, ), and it implies
F (x, y, z, ) = 0

f = g

and g(x, y, z) = k.

Example 1 (page 959). A rectangular box without a lid is to be made from 12 m2


of a cardboard. Find the maximum volume of such a box.
(89 )
Solution.

14.8-1

Example 2. Find the maximum and minimum values of the function f (x, y) =
x3 + 3x2 y in the region x2 + 4xy + 5y 2 5.
(95 )
Solution.

Example 3 (page 960). Find the points on the sphere x2 + y 2 + z 2 = 4 that are
closest to and farthest from the point (3, 1, 1).
Solution.

14.8-2

Example 4. The temperature at the point (x, y, z) is given by T (x, y, z) = 6xy+8yz.


Use Lagrange multipliers to fine the highest temperature and lowest temperature
on the sphere x2 + y 2 + z 2 = 50.
(99 )
Solution.

Two Constraints, page 961


We want to find the maximum and minimum values of a function f (x, y, z) subject
to two constraints of the form g(x, y, z) = k and h(x, y, z) = c. Let
F (x, y, z, , ) = f (x, y, z) (g(x, y, z) k) (h(x, y, z) c).
For , R, under these two constraints, F (x, y, z, , ) and f (x, y, z) take the
same value. So to find the extreme values of f (x, y, z) with g(x, y, z) = k and
h(x, y, z) = c is equivalent to find the extreme values of F (x, y, z, , ). That is,
F = 0

f = g + h and g(x, y, z) = k
14.8-3

and h(x, y, z) = c.

Example 5 (page 962). Find the maximum value of the function f (x, y, z) = x +
2y + 3z on the curve of intersection of the plane x y + z = 1 and the cylinder
x2 + y 2 = 1.
Solution.

14.8-4

Example 6. The plane x + y + 2x = 2 intersects the paraboloid z = x2 + y 2 in


an ellipse. Find the points on this ellipse that are nearest to and farthest from the
origin.
(98 )
Solution.

14.8-5

Example (TA) 7. Suppose that the temperature of a metal plate is given by the
function T (x, y) = 4x2 4xy + y 2. An ant walks around the circle x2 + y 2 = 5.
What are the highest and lowest temperatures that ant is received? (90 )
Solution.

Homework. Find the absolute minimum and maximum values of f (x, y) = xy 2 on


the curve x2 + 7xy + y 2 = 45, x 0, y 0.
(94 )
Homework. Let be the curve which is the intersection of 2y + 4z = 5 and
4x2 + 4y 2 + z 2 = 0. Find the point on that is closest from the origin (0, 0, 0).
(91 )

2
2
2
Homework. The plane x + y + 2z = 0 and the ellipsoid x4 + y4 + z2 = 1 intersect
an ellipse.
(a) Find the farthest distance and the closest distance from the point on the ellipse
to the origin.
(96 )

(b) Find the area of the ellipse.

Homework. The plane 2x + y + z = 10 intersects the paraboloid z = x2 + y 2 in an


ellipse. Find the highest point on the ellipse.
(92 )
14.8-6

Homework*. Find the maximum value of f (x, y, z) = xy + xz + yz subject to the


constraints x 0, y 0, z 0 and x + y + z = 1.

(92 )

Homework*. Use the method of Lagrange multipliers to find the extreme values
of z on the curve of intersection x2 + y 2 = 1 and y 2 + z 2 + z = 1.
2

(97 )

Homework*. If the ellipse xa2 + yb2 = 1(a, b > 0) is to enclose the circle x2 +y 2 = 2y,
what values of a and b minimize the area of the ellipse?
(100 )
Homework*. Find the extreme value of f (x, y) = x2 y xy + xy 2 on x2 + xy + y 2
x y = 1.
(102 )

14.8-7

14.9

Midterm Review

Taylor and Maclaurin Series


Definition 1. Given a smooth function f (x),
. (f (x) x = a )

(a) Taylor series of f (x) at x = a is


(b) Maclaurin series of f (x) is

(f (x) x = 0 )

Remark 2.
(1) Taylor series or Maclaurin series of f (x) is a power series (). We need
.
to know the domain of a power series, that is, the
(2) Most of time, the orders of the coefficients and xn () are different, we
can use the
to find the interval of convergence.
, then the series is convergent.

(3) The Ratio Test says that if

, the Ratio Test is inconclusive (). That

(4) If

is, we have to use other Tests to prove the series is convergent or divergent at
.
the




(5) If we use the Ratio Test to know that lim an+1
= C|x a|, C > 0, then the
an
n

radius of convergence of the power series is

(6) In mathematics, one important question is that


Is f (x) equal to its Taylor series in the interval of convergence?
Answer: f (x) = Taylor or Maclaurin lim Rn (x) = 0 f (x) is analytic.
n

(7) We have learned seven analytic functions: (, , )


1
=
1x

R=

ex =

R=

ln(1 + x) =

R=

sin x =

R=

tan1 x =

R=

cos x =

R=

(1 + x)m =

R=

Cnm =

C0m = 1

14.9-1

(8) Applications:
(a)

x2
1 + x3

(b) ln(5 x)
(c) sin x at x =

(d) sin2 x
(e) tan x
(f) sin1 x

(9) Applications:
(a) Find the limit: It is difficult to use L Hospital Rule.
(b) Find higher derivatives f (n) (0): Compare coefficients of xn .

P
(1)n n
.
(c) Find the sum: Find correct function and take the value.
32n (2n)!
n=0

(d) Approximate the value and estimate the errors: 5 240, sin 12 .

Space Curve
Given a space curve parametrized by r(t), and s(t) =
parameter, then

Rt
0

|r (u)|du is the arclength

(1) unit tangent: T(t) =


(2) unit normal: N(t) =

(|T(t)| = 1 T(t) T (t) = 0).

(3) unit binormal: B(t) =


(4) curvature: (t) =

(5) normal plane: the plane spanned by


(6) osculating plane: the plane spanned by
(7) radius of curvature:
14.9-2

and
and

.
.

The Chain Rule


Example 3. Find radial solutions of the harmonic equation u = 0 in B1 (0) {0}.
Solution. We try to rewrite the harmonic equation u = uxx + uyy = 0 in polar
coordinates x = r cos , y = r sin :
ux =
uy =
uxx =
uyy =
u =
Since
"
#"
# "
#
"
#"
# "
#
rx ry
1 0
cos r sin
rx ry
1 0
xr x
=

=
,
yt y
x y
0 1
sin r cos
x y
0 1
we have
"
# "
"
# "
#1
#
cos sin
rx ry
cos r sin
1 r cos r sin
=
,
=
=
r sin cos
sinr cosr
x y
sin r cos
and
rxx = (rx )x = (cos )x = sin x =
ryy = (ry )y = (sin )y = cos y =


r cos x sin rx
r sin cos + sin cos
sin
=
=
xx = (x )x =
2
r
r
r2

 x
cos
r sin y cos ry
r sin cos cos sin
yy = (y )y =
=
=
,
2
r
r
r2
y
so we get
u =
Suppose that u = u(r) is a function only depends on r, then we get u (r) + u r(r) = 0.
= 0 is a first order linear ordinary differential
Let v(r) = u (r), then v (r) + v(r)
r

equation.
(rv(r)) = 0 rv(r) = ru(r) = c1 ,
The integrating factor is
and u (r) = cr1 u(r) = c1 ln r + c2 , where c1 and c2 are constants.
14.9-3

Approximating Functions or Series


Some methods for estimating the size of the error:
(1) Remainder Estimate for the Integral Test. (11.3)
Suppose f (k) = ak , where f (x) is a continuous, positive, decreasing function

P
for x n and
an is convergent. If Rn = s sn , then
n=1

f (x) dx Rn

n+1

f (x) dx.

(1)

If we add sn to each side of the inequalities (1), because sn + Rn = s, we get


Z
Z
sn +
f (x) dx s sn +
f (x) dx.
n+1

(2) If the series is controlled by a geometric series, we can sum the geometric series
exactly. (11.4)
(3) If the series is an alternating series, we can use the Alternating Series Estimation Theorem. (11.5)
(4) Power series approximation. (11.9)
(5) We can use Taylor Inequality: If |f (n+1) (x)| M for |x a| d, then

(n+1)

f
M
(c)
n+1
(x a)
|x a|n+1 for |x a| d.(11.11)
|Rn (x)| =
(n + 1)!
(n + 1)!

Example 4 (page 770). What is the maximum error possible in using the approx3
5
imation sin x x x3! + x5! when 0.3 x 0.3? Use this approximation to find
sin 12 correct to six decimal places.
3

Solution. Notice that the Maclaurin series sin x = x x3! + x5! x7! + is alternating
for all x 6= 0, and the successive terms decrease in size because |x| < 1, so we can
use the Alternating Series Estimation Theorem. The error in approximating sin x
by the first three terms of its Maclaurin series is at most
7
7
7
x
= |x| (0.3) 4.3 108
7! 5040
5040
To find sin 12 , we first convert to radian measure:



= sin
sin 12 = sin 12
180
15

1  3 1  6

+
0.20791169.
15 3! 15
5! 15

Thus, correct to six decimal places, sin 12 0.207912.


14.9-4

Example 5 (Homework 4, #1). Approximate

240 with error less than 0.0001.

P
Solution. From the binomial series f (x) = (1 + x)m =
Cnm xn with m = 15 and
n=0

x = 314 , we have

240 =

243 3 =

s
5



1
n



X
1
1 5
3
1
5
=3 1 4
,
243 1
=3
Cn 4
243
3
3
n=0

where C05 = 1, C15 = 15 and for n N, n 2,





4
9
1
1
1

(n

1)
1 4 9
5n 6
5
5
5
5
= (1)n1

Cn5 =
n!
5 10 15
5n

n 
n1 Y
(1)
6
=
1
5
5k
k=2

so

!

n 

X
6 (1)n
1 1
(1)n1 Y
240 = 3
Cn
1
=3 1 4 +
5
3
5
5k
34n
n=0
n=2
k=2
!

n 

X
6
1 1
1 Y
1
.
=3 1 4
4n
5 3
5

3
5k
n=2
k=2


If we approximate 5 240 3 1 15 314 2.9926, then the error is

!


n 
n 


X
X
6
6
1 Y
1 Y


1
1
|error| = 3
=3


5 34n k=2
5k
5 34n k=2
5k
n=2
n=2

1
5

X
n=2

1
4
3

n

1
1
1
4
6 X 1
6
1
38
=

=
=

<
.
1
5 34n 10
25 n=2 34n
25 1 34
25 27 40
10000

Solution 2. From the binomial series f (x) = (1 + x) 5 =

Cn5 xn with m =

n=0

1
5

and

 1
n
P
x = 314 , we have 5 240 = 3 1 + 314 5 = 3
Cn5 314 . If we approximate
n=0 

5
240 by first two terms, that is, 5 240 3 1 51 314 2.9926, the remainder is




f (c) 2 3|f (c)| 2
1
1


3|R1 (x)| = 3
x
|x| , for |x| 4 and c 4 , 0 .
2!
2!
3
3
4

4
(1 + x) 5 , then
We compute f (x) = 51 (1 + x) 5 and f (x) = 25


  95
 2


4
4
1
81
81
4
4

|f (c)| =

,
9
9 =

25(1 + c) 5 25 1 1 5
25
80
25
80
34

so




1 3

3 R1 4
3

4
25

2!


81 2
80



1 2
= 3 = 0.0000375 < 0.0001.
34
27 54
14.9-5

Example 6 (Homework 1 #12). How many terms of the series

1
,
x(ln x)2

1
n(ln n)2

would

n=2

you need to add to find its sum to within 0.01?


Solution. Let f (x) =

which is a positive and continuous function on [2, ).

Furthermore, we compute
f (x) =

(ln x)2 2x(ln x) x1


ln x + 2
= 2
< 0,
2
4
x (ln x)
x (ln x)3

so f (x) is decreasing on [2, ). By the Integral Test, we can estimate the remainder
of the series
Z
Z
Rn
f (x) dx =
n

Z t
Z t
1
1
1
dx = lim
dx = lim
d ln x
2
2
2
t
t
x(ln x)
n x(ln x)
n (ln x)
n


 x=t

1
1
1
1
1
= lim
= lim
=
+
<
.

t
ln x x=n t
ln t ln n
ln n
100

So we require ln n > 100, which is equivalent to n > e100 .

Example 7 (Homework 2 #2). How many terms of the series do we need to add

P
in order to find the sum of the series
(1)n+1 n16 correct to four decimal place?
n=1

1
Solution. By the Alternating Series Estimation Theorem, we require bn+1 = (n+1)
6 <

6
1
0.00005 = 20000 , and it implies (n + 1) > 20000 5.21. So we need to add at least

5 terms.
Example 8 (Homework 1 #16). Use

10
P

1
3n +4n

.
= 0.19788 to estimate the error

n=1

of the sum of the series

1
.
3n +4n

n=1

Solution. Since an =

1
3n +4n

<

1
3n +3n

R10 T10

1
23n

= bn , so the error is

1
1
1
2311
=
1 = 10 .
n
23
3
1 3
n=11

A better estimate is using the Integral Test:


R10

 x=t

Z t x
1
3
1 3x
T10
dx = lim
dx = lim
x
t 10 2
t
2 ln 3 x=10
10 2 3


1
310
3t
=
+
.
lim
10
t
2 ln 3 2 ln 3
2 3 ln 3
Z

14.9-6

15 16

James Stewart: Calculus, Early Transcendentals, Seventh
Edition, International Metric Version, c2012.

Chapter 15
15.1

Multiple Integrals

Double Integrals over Rectangles, page 974

Review of the Definite Integral, page 974


Suppose that f (x) is defined for a x b.
(1) Divide the interval [a, b] into n subintervals [xi1 , xi ] of equal width x =

ba
.
n

(2) In each subinterval [xi1 , xi ], choose a sample point xi [xi1 , xi ].


(3) Define the Riemann sum () =

n
P

f (xi )x.

i=1

(4) Define the definite integral () of f (x) from a to b by


Z

b
def.

f (x) dx = lim

n
X

f (xi )x.

i=1

Figure 1: Definition of a definite integral.

 : ; x-;
Volumes and Double Integrals, page 974
Similarly, we consider a function f (x, y) defined on a closed rectangle
R = [a, b] [c, d] = {(x, y) R2 |a x b, c y d},
and we first suppose that f (x, y) 0. The graph of f is a surface with equation
z = f (x, y). Let S be the solid that lies above R and under the graph of f , that is
S = {(x, y, z) R3 |0 z f (x, y), (x, y) R},
The goal is to find the volume of S.
15.1-1

(1) Divide the rectangle R into subrectangles:


Divide [a, b] into m subinterval [xi1 , xi ] with width x =
Divide [c, d] into n subinterval [yi1 , yi ] with width y =

ba
.
m

dc
.
n

We form the subrectangles:


Rij = [xi1 , xi ] [yi1 , yi ] = {(x, y)|xi1 x xi , yj1 y yj }.
Each Rij with area A = xy.
(2) Choose a sample point (xij , yij ) in each Rij .
(3) We get an approximation to the total volume of S by double Riemann sum
():
V

m X
n
X

f (xij , yij )A.

i=1 j=1

(4) Define the volume () of the solid S by


V = lim
m,n

n
m X
X

f (xij , yij )A.

i=1 j=1

Definition 1 (page 977). The double integral (, ) of f (x, y) over the


rectangle R is
ZZ
m X
n
X
def.
f (x, y) dA = lim
f (xij , yij )A
m,n

i=1 j=1

if this limit exists.

 : ; xy-
 dA ,
 dx dy () (dy dx = dx dy)
The Midpoint Rule, page 978
The method that we used for approximating single integrals (the Midpoint Rule,
the Trapezoidal Rule, Simpsons Rule) all have counterparts for double integrals.
Here we introduce the Midpoint Rule for double integrals.
Midpoint Rule for Double Integrals (page 978).
ZZ
m X
n
X
f (x, y) dA
f (
xi , yj )A,
R

i=1 j=1

where xi is the midpoint of [xi1 , xi ] and yj is the midpoint of [yj1 , yj ].


15.1-2

Average Value, page 979


Recall that the average value of a function f of one variable defined on an interval
[a, b] is
fave

1
=
ba

f (x) dx.

In a similar fashion we define the average value () of a function f of two


variables defined on a rectangle R to be
fave

1
=
Area(R)

ZZ

f (x, y) dA.
R


Properties of Double Integrals, page 981
We assume that all of the integrals exist.
ZZ
ZZ
ZZ
(a)
(f (x, y) + g(x, y)) dA =
f (x, y) dA +
g(x, y) dA.
R

(b)

ZZ

cf (x, y) dA = c
R

ZZ

f (x, y) dA, where c is a constant.

(a) and (b) are called the linearity () of the integral.


ZZ
ZZ
(c) If f (x, y) g(x, y) for all (x, y) R, then
f (x, y) dA
g(x, y) dA.
R

15.1-3

15.2

Iterated Integrals, page 982

Goal: Compute the double integrals by iterated integrals.


Recall that it is usually difficult to evaluate single integrals directly from the definition of an integral. The evaluation of double integrals from the definition is even
more difficult. In this section, we will see how to express a double integral as an
iterated integral, which can be evaluated by calculating two single integrals.
Suppose that f (x, y) is integrable on the rectangle R = [a, b] [c, d].
Definition 1 (983). Define the partial integration of f (x, y) with respect to y, deRd
noted by c f (x, y) dy to mean that x is fixed and f (x, y) is integrated with respect
to y from y = c to y = d.
Rd
After partial integration, c f (x, y) dy depends on x, so we denote it by A(x).
Rd
Definition 2 (page 983). If we integrate A(x) = c f (x, y) dy with respect to x
from x = a to x = b, we get the iterated integral ( y x ):

Z b
Z b Z d
Z bZ d
A(x) dx =
f (x, y) dy dx =
f (x, y) dy dx
a

 , , dy dx dx dy
Similarly, the iterated integral ( x y )

Z dZ b
Z d Z b
Z d
f (x, y) dx dy =
f (x, y) dx dy =
B(y) dy.
c

means that we first integrate with respect to x (fixed y) from x = a to x = b and


Rb
then integrate the resulting function B(y) = a f (x, y) dx from y = c to y = d.

Fubinis Theorem (page 984). If f (x, y) is continuous on the rectangles R =


{(x, y)|a x b, c y d}, then


ZZ
Z b Z d
Z d Z b
f (x, y) dA =
f (x, y) dy dx =
f (x, y) dx dy.
R

In general, this is true if we assume that f (x, y) is bounded on R, f (x, y) is discontinuous only on a finite number of smooth curves, and the iterated integrals exist.
RR
Example 3 (page 985). Evaluate R sin(xy) dA, where R = [1, 2] [0, ].
Solution. If we first integrate with respect to x, we get
ZZ
y sin(xy) dA =
R

15.2-1

Solution 2. If we reverse the order of integration, we get


ZZ
y sin(xy) dA =
R

We use

and get

So

 ,
Example 4 (page 986). Find the volume of the solid S that is bounded by the
elliptic paraboloid x2 + 2y 2 + z = 16, the plane x = 2 and y = 2, and the three
coordinate planes.
Solution.

In the special case, where f (x, y) = g(x)h(y) is the product of a function of x


only and a function of y only, by Fubinis Theorem, we get

Example 5 (page 987). If R = [0, 2 ] [0, 2 ], then


ZZ
sin x cos y dA =
R

15.2-2

Homework (page 987). Calculate the integral.



Z 1 Z 1
ZZ

1
dA, R = [1, 3] [1, 2].
(1)
x + y dx dy.
(2)
R 1+x+y
0
0

Homework (page 988). Find the volume of the solid in the first octant bounded
by the cylinder z = 16 x2 and the plane y = 5.

Review: Integration (TA)


Table of Basic Indefinite Integrals:
Z
xn dx =
(n 6= 1)
Z
ex dx =
Z
sin x dx =
Z
sec2 x dx =
Z
1
dx =
2
x + a2

Substitution Rules:
Z
tan x dx =

1
dx =
Z x
ax dx =
Z
cos x dx =
Z
sec x tan x dx =
Z
1

dx =
2
a x2
Z

Integration by Parts:
Z
x sin x dx =
Z
ln x dx =
Z
x ex dx =
Z
sin1 x dx =
Z

tan1 x dx =

Trigonometric Integrals: (study all types of trigonometric integrals by yourself)


Z
sin2 x dx =
Z
sec x dx =
15.2-3

Trigonometric Integrals:
Expression

Substitution

Identity

1 sin2 = cos2

a2 x2

a2 + x2

x2 a2
Z

1 + tan2 = sec2
sec2 1 = tan2

1
dx =
x2 a2

Partial Fractions: (study all types of partial fractions by yourself)


Z
1
dx =
2
x a2

Example (TA) 6. Find the integral


the value in both iterated integrals?

ZZ

y exy dA, R = [0, 2] [0, 3]. Can we find


R

Solution.

15.2-4

15.3

Double Integrals over General Regions, page


988

Goal: We will learn how to integrate a function f (x, y) over a bounded region D.
Define a new function F (x, y) with a rectangular region R D by
(
f (x, y) if (x, y) is in D
F (x, y) =
0
if (x, y) is in R but not in D.

(1)

Figure 1: Double integral of f over D.

Definition 1 (page 989). If F (x, y) is integrable over R, then we define the double
integral of f (x, y) over D ( D f (x, y) ) by
ZZ
ZZ
f (x, y) dA =
F (x, y) dA, where F is given by (1).
D

Definition 2 (page 989).


(1) A plane region D is said to be of type I if it lies between the graphs of two
continuous functions of x, that is, D = {(x, y)|a x b, g1 (x) y g2 (x)},
where g1 (x) and g2 (x) are continuous on [a, b].
(2) A plane region D is said to be of type II if it lies between the graphs of two
continuous functions of y, that is, D = {(x, y)|h1 (y) x h2 (y), c y d},
where h1 (y) and h2 (y) are continuous on [c, d].

Figure 2: Type I and Type II region.

15.3-1

Theorem 3 (page 990).


(a) If f (x, y) is continuous on a type I region D, then
ZZ

f (x, y) dA =

y=g2 (x)

f (x, y) dy dx.

y=g1 (x)

(b) If f (x, y) is continuous on a type II region D, then


ZZ

f (x, y) dA =

d
c

x=h2 (y)

f (x, y) dx dy.

x=h1 (y)

Proof of (a). We choose a rectangle R = [a, b] [c, d], where c and d are constants
satisfy c min g1 (x) and d max g2 (x). Let F (x, y) be the function given by (1).
x[a,b]

x[a,b]

By Fubinis Theorem, we have


ZZ
ZZ
Z b Z
f (x, y) dA =
F (x, y) dA =
D

F (x, y) dy dx.

For fixed x [a, b], since F (x, y) = 0 if y < g1 (x) or y > g2 (x), the lower limit can
be replaced by g1 (x), and the upper limit can replaced by g2 (x). Therefore,
!
!
ZZ
Z b Z g2 (x)
Z b Z g2 (x)
f (x, y) dA =
F (x, y) dy dx =
f (x, y) dy dx
D

g1 (x)

g1 (x)

because F (x, y) = f (x, y) when g1 (x) y g2 (y).

 ,
 y , y , y = g2(x) y = g1(x)
 ();
RR
Example 4 (page 990). Evaluate D (x + 2y) dA, where D is the region bounded
by the parabolas y = 2x2 and y = 1 + x2 .

15.3-2

Example 5 (page 991). Find the volume of the solid that lies under the paraboloid
z = x2 + y 2 and above the region D in the xy-plane bounded by the line y = 2x and
the parabola y = x2 .
Solution.

Solution 2.

RR
Example 6 (page 992). Evaluate D xy dA, where D is the region bounded by the
line y = x 1 and the parabola y 2 = 2x + 6.
Solution.

15.3-3

Example 7 (page 440, 996). Find the volume common to two circular cylinders,
each with radius r, if the axis of the cylinder intersect at right angles.
Solution.

Solution 2.

Example 8 (page 993). Evaluate the iterated integral


Solution.

15.3-4

R1R1
0

sin(y 2 ) dy dx.

Properties of Double Integrals, page 993


We assume that all of the integrals exist.
ZZ
ZZ
ZZ
(a)
(f (x, y) + g(x, y)) dA =
f (x, y) dA +
g(x, y) dA.
D

(b)

ZZ

cf (x, y) dA = c
D

ZZ

f (x, y) dA.
D

(c) If f (x, y) g(x, y) for all (x, y) D, then

ZZ

f (x, y) dA

ZZ

g(x, y) dA.

(d) If D = D1 D2 , where D1 and D2 dont overlap except on boundaries, then


ZZ
ZZ
ZZ
f (x, y) dA.
f (x, y) dA +
f (x, y) dA =
D

D2

D1

This property can be used to evaluate double integrals over regions D that are
neither type I nor type II but can be expressed as a union of regions of type I
or type II.

Figure 3: D is neither type I nor type II. D1 is type I, D2 is type II.


(e)

ZZ

1 dA = Area(D).
D

(f) If m f (x, y) M for all (x, y) in D, then


ZZ
mArea(D)
f (x, y) dA MArea(D).
D

Homework. Find the integral

Z 8Z
0

Homework. Find the integral

ZZ

1 + x4 dx dy.

(95 )

|y x2 | dA, R = [1, 1] [0, 2]. (96 )

Homework. Evaluate the integral

sin(y 2) dy dx.

(97 )

x
2

Homework (page 996). Find the volume of the solid bounded by the cylinder
y 2 + z 2 = 4 and the planes x = 2y, x = 0, z = 0 in the first octant.
Z 4Z 2
2
ex dx dy.
(98 )
Homework*. Evaluate the integral
0

y
2

15.3-5

15.4

Double Integrals in Polar Coordinates, page


997

RR
Goal: We want to evaluate a double integral R f (x, y) dA, where R is easily
described using polar coordinates.
Recall that relations between Cartesian coordinates and polar coordinates:
(
(
p
x = r cos
r = x2 + y 2

= tan1 xy .
y = r sin
Definition 1 (page 997). Define the polar rectangle ()
R = {(r, )|a r b, }.
Here we use the definition of double integral to find the formula of double integrals in polar rectangles.
(1) Divide the polar rectangle into small polar rectangles:
Dividing [a, b] into m subinterval [ri1 , ri ] with width r =

ba
.
m

Dividing [, ] into n subinterval [i1 , i ] with width =

.
n

We get small polar rectangles: Rij = {(r, )|ri1 r ri , j1 j }.


(2) Choose the center of the polar subrectangle (ri , j ) as the sample point in
each Rij , where
1
j = (j1 + j ).
2

1
ri = (ri1 + ri ),
2
We compute the area of Rij :

1
1 2
1
Ai = ri2 ri1
= (ri + ri1 )(ri ri1 ) = ri r.
2
2
2
(3) We get the double Riemann sum in polar rectangles:
n
m X
X

f (ri cos j , ri sin j )Ai =

n
m X
X

f (ri cos j , ri sin j )ri r.

i=1 j=1

i=1 j=1

(4) When m, n , we get


ZZ
m X
n
X
f (x, y) dA = lim
f (ri cos j , ri sin j )ri r
m,n

i=1 j=1

Z Z

f (r cos , r sin )rdrd.


a

15.4-1

Change to Polar Coordinates in a Double Integral (page 999). If f is continuous on a polar rectangle R given by 0 a r b, , where 0 2,
then
ZZ
Z Z b
f (x, y) dA =
f (r cos , r sin )r dr d.
R

 r dr d; dr rd:

 r dr d r Jacobian: (x,y)
(r,)
RR
Example 2 (page 999). Evaluate R (3x + 4y 2 ) dA, where R is the region in the
upper half-plane bounded by the circles x2 + y 2 = 1 and x2 + y 2 = 4.
Solution.

Example 3 (page 1000). Find the volume of the solid bounded by the plane z = 0
and the paraboloid z = 1 x2 y 2 .
Solution.

15.4-2

Theorem 4 (page 1000). If f (x, y) is continuous on a polar region of the form


D = {(r, )|h1 () r h2 (), }, then
ZZ

f (x, y) dA =
D

Z Z

r=h2 ()

f (r cos , r sin )r dr d.

r=h1 ()

 r , r = h2 (), r = h1(),
Example 5 (page 1000). Use a double integral to find the area enclosed by one
loop of the four-leaved rose r = cos 2.
Solution.

Example 6 (page 1001). Find the volume of the solid that lies under the paraboloid
z = x2 + y 2 , above the xy-plane, and inside the cylinder x2 + y 2 = 2x.
Solution.

RR

|y| dA, where D is the region enclosed by r =

ZZ

1
dA, where R is the region enclosed
(1 + x2 + y 2 )
(95 )

Homework. Find the integral


4 + 3 cos .
Homework. Find the integral
by the Lemniscate r 2 = cos 2.

(94 )

15.4-3

Homework. Find the volume bounded by the sphere x2 + y 2 + z 2 = 4 and the


cylinder (x 1)2 + y 2 = 1.
(95 )

Z 2 Z 4x2
2
2
Homework. Evaluate the integral
e(x +y ) dy dx
(99 )
2 0

Homework*. Find the integral

Z 1Z 1y2
2

ln(x2 + y 2) dx dy.

(96 )

3y

Example (TA) 7 (page 1003). Find the integral


Solution.

ZZ

e(x

2 +y 2 )

dA.

R2

Appendix
The area form (or volume form) has the following structure: Since
(
"
# "
#"
#
dx = dr cos r sin d
dx
cos r sin
dr

=
.
dy = dr sin + r cos d
dy
sin r cos
d
We compute the determinant of the matrix, called Jacobian:


(x, y) cos r sin
=
= r cos2 + r sin2 = r.
sin r cos
(r, )

Thus the area form is dx dy = r dr d.

15.4-4

15.6

Surface Area, page 1013

Goal: Find the formula of the surface area of the graph of f (x, y).
Let S be a surface with equation z = f (x, y), where f (x, y) has continuous partial
derivatives. We assume that f (x, y) 0 and the domain D of f is a rectangle. The
idea is to approximate the surface area by the tangent plane areas.
(1) Divide D into small rectangles Rij with area A = xy.
(2) If we choose (xi , yi ), the corner of Rij closest to the origin, as a sample point,
then the tangent plane to S at Pij = (xi , yj , f (xi , yj )) is an approximation to
S near Pij . The area Tij of the part of this tangent plane that lies directly
above Rij is an approximation to the area Sij of the part of S that lies
directly above Rij .
Tij = |ui vj | = |(x i + fx (xi , yi )x k) (y j + fy (xi , yj )y k)|
= | fx (xi , yi )xy i fy (xi , yi )xy j + xy k|
q
= 1 + (fx (xi , yi ))2 + (fy (xi , yi ))2 A.
z
Pij
u

Tij
y
x
y

Figure 1: The area of a parallelogram Tij = |ui vj |.


(3) The sum

m P
n
P

Tij is an approximation to the total area of S.

i=1 j=1

(4) We define the surface area () of S to be


m X
n q
X
1 + (fx (xi , yi ))2 + (fy (xi , yi ))2 A.
Area(S) = lim
m,n

i=1 j=1

Theorem 1 (page 1014). The area of the surface with equation z = f (x, y), (x, y)
D, where fx and fy are continuous, is
ZZ q
Area(S) =
1 + (fx (x, y))2 + (fy (x, y))2 dA.
D

 :

L=

Rbp
a

1 + (y (x))2 dx.
15.6-1

Example 2 (page 1015). Find the surface area of the part of the surface z = x2 +2y
that lies above the triangular region T in the xy-plane with vertices (0, 0), (1, 0),
and (1, 1).
Solution.

Example 3 (page 1015). Find the area of the part of the sphere x2 + y 2 + z 2 = 4
that lies within the cylinder x2 + y 2 = x.
Solution.

Homework (page 1016). Find the area of the surface created by the cylinder y 2 +
z 2 = r 2 that lies within another cylinder x2 + z 2 = r 2 .
(91 )
2

Homework (page 1016). Find the area of the surface z = 23 (x 3 + y 3 ), 0 x, y 1.


(98 )
Homework. Find the area of the surface {x2 + y 2 + z 2 = 4, 1 x2 + y 2 3, z 0}.
(102 )
15.6-2

15.7

Triple Integrals, page 1017

Goal: Define and compute triple integrals of f (x, y, z) over a bounded region.
We first deal with the case where f (x, y, z) is defined on a rectangular box:
B = {(x, y, z)|a x b, c y d, r z s}.
(1) Divide the box B into lmn sub-boxes: Bijk = [xi1 , xi ] [yj1, yj ] [zk1 , zk ].
Each Bijk has area V = xyz.

(2) Choose a sample point (xijk , yijk


, zijk
) in each Bijk .

(3) We get the triple Riemann sum ():

l P
m P
n
P

f (xijk , yijk
, zijk
)V .

i=1 j=1 k=1

(4) Definition 1 (page 1017). The triple integral () of f over B is


ZZZ
l X
m X
n
X

f (xijk , yijk
, zijk
)V
f (x, y, z) dV = lim
l,m,n

i=1 j=1 i=k

if this limit exists.

 (, )
 :
Just as for double integrals, the practical method for evaluating triple integrals
is to express them as iterated integrals as follows.
Fubinis Theorem for Triple Integrals (page 1017). If f is continuous on the
rectangular box B = [a, b] [c, d] [r, s], then
ZZZ
Z sZ dZ b
f (x, y, z) dV =
f (x, y, z) dx dy dz.
B
r c a
RRR
Example 2 (page 1018). Evaluate the triple integral
xyz 2 dV , where B =
B
[0, 1] [1, 2] [0, 3].
Solution. Direct computation gives
ZZZ
Z 3Z 2 Z 1
Z 3Z
2
2
xyz dV =
xyz dx dy dz =
B

yz 2
dy dz =
2

3
0

27
3z 2
dz = .
4
4

Now we define the triple integral over a general bounded region E (


) in three dimensional space (a solid). We enclosed E in a box B, and
then define F (x, y, z) on B that it agrees with f (x, y, z) on E but is 0 in B that
outside E. By definition,
ZZZ
ZZZ
f (x, y, z) dV =
F (x, y, z) dV.
E

This integral exists if f (x, y, z) is continuous and the boundary of E is reasonably


smooth.
15.7-1

Definition 3 (page 1018).


(a) Region E is called type x if E = {(x, y, z)|(x, y) D, u1(y, z) x u2 (y, z)},
where D is the projection of E onto the yz-plane.
(b) Region E is called type y if E = {(x, y, z)|(x, y) D, u1(x, z) y u2 (x, z)},
where D is the projection of E onto the xz-plane.
(c) Region E is called type z if E = {(x, y, z)|(x, y) D, u1(x, y) z u2 (x, y)},
where D is the projection of E onto the xy-plane.
Now we will change triple integrals to iterated integrals.

If E is type z and D is type I, then


ZZZ
Z bZ y=g2 (x)Z
f (x, y, z) dV =
E

y=g1 (x)

z=u2 (x,y)

f (x, y, z) dz dy dx.
z=u1 (x,y)

z
z
u2 (x, y)

y
(0, 0, 1)

z =1xy

u1 (x, y)

y =1x

(0, 1, 0)
(x

g2

(1, 0)

(1, 0, 0)

g 1 (x )

(0, 1)

x
(a)

(b)

Figure 1: (a) Triple Integrals. (b) Solid E in Example 4.


RRR
Example 4 (page 1019). Evaluate
z dV , where E is the solid tetrahedron
E
bounded by the four planes x = 0, y = 0, z = 0, and x + y + z = 1.
Solution.

15.7-2

Example 5 (page 1020). Evaluate

RRR
E

x2 + z 2 dV , where E is the region bounded

by the paraboloid y = x2 + z 2 and the plane y = 4.


Solution.

Example 6 (page 1021). Evaluate the iterated integral

R 1R x2R y
0 0

f (x, y, z) dz dy dx

as a triple integral and then rewrite it as an iterated integral in a different order,


integrating first with respect to x, then z, and then y.
Solution.

15.7-3

Applications of Triple Integrals, page 1022


If f (x, y, z) = 1 for all points in E. Then the triple integral does represent the
volume E:
ZZZ
V (E) =
dV.
E

Example 7 (page 1022). Use a triple integral to find the volume of the tetrahedron
T bounded by the plane x + 2y + z = 2, x = 2y, x = 0, and z = 0.
Solution.

Definition 8 (page 1023).


(a) If the density function of a solid object that occupies the region E is (x, y, z)
(in units of mass per unit volume), then its mass () is
ZZZ
m=
(x, y, z) dV.
E

(b) The moments () of region E about the three coordinate planes are
ZZZ
ZZZ
Myz =
x(x, y, z) dV,
Mxz =
y(x, y, z) dV,
E
Z Z ZE
Mxy =
z(x, y, z) dV.
E

(c) The center of mass () is located at the point (


x, y, z), where
x =

Myz
,
m

y =

Mxz
,
m

z =

Mxy
.
m

(d) The moments of inertia () about the three coordinate axes are
ZZZ
ZZZ
2
2
Ix =
(y + z )(x, y, z) dV, Iy =
(x2 + z 2 )(x, y, z) dV,
E
Z Z ZE
Iz =
(x2 + y 2)(x, y, z) dV.
E

15.7-4

Definition 9 (page 1023). The total electric charge on a solid object occupying a
region E and having charge density (x, y, z) is
ZZZ
Q=
(x, y, z) dV.
E

Example 10 (page 1024). Find the center of mass of a solid of constant density
that is bounded by the parabolic cylinder x = y 2 and the planes x = z, z = 0, and
x = 1.
Solution.

Homework (page
1026).
Write five other iterated integrals that are equal to the
Z 1Z
1Z y
iterated integral
f (x, y, z) dz dx dy.
0

Homework. Evaluate the triple integral

Z 2Z
0

4x2Z x
0

sin 2z
dy dz dx.
4z

Homework (page 1025). Use a triple integral to find the volume of the solid enclosed by the cylinder y = x2 and the planes z = 0 and y + z = 1.
15.7-5

Homework. The region E is bounded by z = (x2 + y 2 ) 2 , x2 + y 2 = 9, and z = 0.


Suppose the density function (x, y, z) = z. Find the mass of E and the center of
mass (
x, y, z).
(92 )
Example (TA) 11. Figure 2 shows the region of integration for the integral
Z 1Z
0

1x2Z 1x

f (x, y, z) dy dz dx.

Rewrite this integral as an equivalent iterated integral in the five other orders.
z

x
Figure 2: Integration of f (x, y, z) on the solid.

Solution.

15.7-6

Example (TA) 12. Express the integral

RRR

f (x, y, z) dV as an iterated integral

in six different ways, where E is the solid bounded by y = x2 , z = 0, and y + 2z = 4.


Solution.

15.7-7

Appendix

If E is type z and D is type II, then


ZZZ

f (x, y, z) dV =

f (x, y, z) dV =

Z bZ

z=g2 (x)Z y=u2 (x,z)

z=g1 (x)

f (x, y, z) dV =
E

f (x, y, z) dy dz dx.

y=u1 (x,z)

x=h2 (z)Z z=u2 (x,y)

Z sZ
r

x=h1 (z)

f (x, y, z) dy dx dz.

z=u1 (x,y)

If E is type x and D is type I (z = gi (y)), then


ZZZ

f (x, y, z) dz dx dy.

z=u1 (x,y)

If E is type y and D is type II (x = hi (z)), then


ZZZ

x=h1 (y)

If E is type y and D is type I (z = gi (x)), then


ZZZ

x=h2 (y)Z z=u2 (x,y)

Z dZ

f (x, y, z) dV =
E

Z dZ
c

z=g2 (y)Z x=u2 (y,z)

z=g1 (y)

f (x, y, z) dx dz dy.

x=u1 (y,z)

If E is type x and D is type II (y = hi (z)), then


ZZZ

f (x, y, z) dV =

Z sZ
r

y=h2 (z)Z z=u2 (x,y)

y=h1 (z)

f (x, y, z) dx dy dz.

z=u1 (x,y)

Homework*. Find the total electric charge over the region


R = {(x, y)| 1 x + y 1, 1 x y 1}
with charge density (per unit area) (x, y) = |x| + |y|.
Z 1Z 1xZ 1
sin(z)
dz dy dx.
Homework*. Evaluate
0 0
y z(2 z)

15.7-8

(96 )
(100 )

15.8

Triple Integrals in Cylindrical Coordinates,


page 1027

Goal: Compute triple integrals in cylindrical coordinates.

Cylindrical Coordinates, page 1028


In the cylindrical coordinate system (), a point P in three-dimensional space
is represented by the ordered triple (r, , z), where r and are polar coordinates of
the projection of P onto the xy-plane and z is the directed distance from the xyplane to P .
z

P (r, , z)
z

x
Figure 1: Cylindrical coordinate system.
To convert from cylindrical to rectangular coordinate, we use the equations
x = r cos ,

y = r sin ,

z = z,

whereas to convert from rectangular to cylindrical coordinates, we use


y
r 2 = x2 + y 2,
tan = ,
z = z.
x
Example 1 (page 1028).
(a) Find rectangular coordinates of the point with cylindrical coordinates (2, 32 , 1).
(b) Find cylindrical coordinates of the point with rectangular coordinates (3, 3, 7).
Solution.
(a) We compute
x=
Thus the point is

y=

z=

in rectangular coordinates.

(b) We compute
r=

tan =

z=

, and another is
Therefore one of cylindrical coordinates is
. As with polar coordinates, there are infinitely many choices.
15.8-1

Example 2 (page 1029). Describe the surface whose equation in cylindrical coordinates is z = r.
Solution. Since z 2 = r 2 = x2 + y 2 , it is a

whose axis is the z-axis.

Evaluating Triple Integrals with Cylindrical Coordinates, page


1029
Suppose that E is a type z region whose projection D onto the xy-plane is conveniently described in polar coordinates. Suppose that f (x, y, z) is continuous and
E = {(x, y, z)|(x, y) D, u1(x, y) z u2 (x, y)},
where D is given in polar coordinates by
D = {(r, )| , h1 () r h2 ()}.
We get the formula for triple integration in cylindrical coordinates:
ZZZ
Z Z r=h2 ()Z z=u2 (r cos ,r sin )
f (x, y, z) dV =
f (r cos , r sin , z)r dz dr d.
E

r=h1 ()

z=u1 (r cos ,r sin )

z
z
z = u2

z = u1

h1
r=

dz
rd
dr

D
=

r = h2

x
Figure 2: Cylindrical coordinate system.

Example 3. Evaluate

4x2

4x2

(x2 + y 2) dz dy dx.

(98 )

x2 +y 2

Solution. The region E has a simpler description in cylindrical coordinates:

Therefore we have

15.8-2

Example 4 (page 1030). A solid E lies within the cylinder x2 + y 2 = 1, below the
plane z = 4, and above the paraboloid z = 1 x2 y 2. The density at any point is
proportional to its distance from the axes of the cylinder. Find the mass of E.
Solution.

Homework (page 1031). Evaluate the integral by changing to cylindrical coordinates.


Z

Z 4y2 Z

4y 2

xz dz dx dy.
x2 +y 2

Homework (page 1032). Find the volume of the solid enclosed by the three cylinders x2 + y 2 = 1, x2 + z 2 = 1, and y 2 + z 2 = 1.

15.8-3

15.9

Triple Integrals in Spherical Coordinates,


page 1033

Goal: Define and compute triple integrals in spherical coordinates.

Spherical Coordinates, page 1028


The spherical coordinates (, , ) () of a point P in space are shown in Figure 1, where = |OP | is the distance from the origin to P, is the same angle as in
cylindrical coordinates, and is the angle between the positive z-axis and the line
segment OP . Note that we assume 0 and 0 .
z

x
Figure 1: Spherical coordinate system.

 ( r ) ( )
The spherical coordinate system is useful in problems where there is symmetry
about a point, and the origin is placed at this point. Figure 2 shows the surfaces of
= c, = c, and = c.
z

c
y

c
x

Figure 2: (a) = c is a sphere. (b) = c is a half-plane. (c) = c is a half-cone.


Relations between rectangular coordinates and spherical coordinates are
x = sin cos ,
2 = x2 + y 2 + z 2 ,

y = sin sin ,
tan =

y
,
x

15.9-1

z = cos .
x2 + y 2
tan =
.
z2
2

Example 1 (page 1034). The point (x, 4 , 3 ) is given in spherical coordinates. Plot
the point and find its rectangular coordinates.
Solution.

Example 2 (page 1034). The point (0, 2 3, 2) is given in rectangular coordinates.


Find spherical coordinates for this point.
Solution.

Evaluating Triple Integrals with Spherical Coordinates, page


1034
Consider the spherical wedge
E = {(, , )|a b, , c d},
where a 0 and 2, and d c .
(1) Divide E equally into Eijk by = i , = j , and = k .
(2) Eijk is approximately a rectangular box with dimensions , i , and i sin k .
So an approximation to the volume of Eijk is given by
Vijk ()(i )(i sin k ) = 2i sin k .
In fact, by the Mean Value Theorem (see the Appendix), the volume of Eijk
is given exactly by
Vijk = 2i sin k ,

where (
i , j , k ) is some point in Eijk . Let (xijk , yijk
, zijk
) be the rectangular
coordinates of the sample point (
i , j , k ).

15.9-2

(3) We get the Riemann sum


l X
m X
n
X

f (
i sin k cos j , i sin k sin j , i cos j )
2i sin k .

i=1 j=1 k=1

(4) When l, m, n , we get the formula for triple integration in spherical


coordinates:
ZZZ
Z dZ Z b
f dV =
f ( sin cos , sin sin , cos )2 sin d d d.
E

z
d
sin d

y
d

x
Figure 3: Volume element of the spherical coordinate system.
This formula can be extended to include more general spherical regions such as
E = {(, , )| , c d, g1 (, ) g2 (, )}, and in this case the
triple integration will be
ZZZ
Z dZ Z g2 (,)
f dV =
f ( sin cos , sin sin , cos )2 sin d d d,
E

g1 (,)

Example 3 (page 1036). Evaluate


B = {(x, y, z)|x2 + y 2 + z 2 = 1}.

ZZZ

e(x

3
2 +y 2 +z 2 ) 2

Solution.

15.9-3

dV , where B is the unit ball:


(93 )

Example 4 (page 1036). Use spherical coordinates to find the volume of the solid
p
that lies above the cone z = x2 + y 2 and below the sphere x2 + y 2 + z 2 = z.

Solution.

Example (TA) 5. Evaluate the integral


Z 1 Z 1x2 Z 2x2 y2 p
x2 + y 2 + z 2 dz dy dx.

x2 +y 2

(94 )
Solution.

15.9-4

Homework. Evaluate the integral


Z Z 2Z

16x2 y 2

4x

4x2

3(x2 +y 2 )

x2 + y 2 + z 2 dz dy dx.
(94 )

Homework. Evaluate the integral

ZZZ

z3

p
x2 + y 2 + z 2 dV , where E is the solid

that lies above the xy-plane and below the sphere x2 + y 2 + z 2 = 1. (92 )

Homework. Find the volume of the solid bounded below by the cone z 2 = 4(x2 +y 2 )
and below by the ellipsoid 4(x2 + y 2 ) + z 2 = 8.

(101 )

Appendix
)
in E such that
Claim: For each spherical wedge E, there exists (
, ,

V = 2 sin .
z

z
r 2 + z 2 = a2
r

a
y

E
2

1 2

Figure 4: Volume element of the spherical coordinate system.


(a) First of all, we will get the volume of the solid bounded above by the sphere
r 2 + z 2 = a2 and below by the cone z = r cot 0 , 0 < 0 < 2 , and 1 2 ,
3

is V = a 3 (1cos 0 ), where = 2 1 . Using the cylindrical coordinates


to compute
V =

2Z a sin 0Z

a2 r 2

r dz dr d =

r cot 0

a sin 0



2
2
2
r a r r cot 0 dr

i r=a sin 0
h

3
2
2 32
=
(a r ) r cot 0
3
r=0

3

=
(a2 a2 sin 0 ) 2 + a3 a3 sin3 0 cot 0
3
 a3
a3
=
1 cos3 0 sin2 0 cos 0 =
(1 cos 0 ).
3
3
15.9-5

(b) Next, we will show that the volume of the spherical wedge given by 1
2 , 1 2 , 1 2 is
V =

3
( 31 )(cos 1 cos 2 ).
3 2

Denote Vij by the volume of the region bounded by the sphere of radius i and
the cone with angle j , and from 1 to 2 . and letting V be the volume of
the wedge E, then we have
V = (V22 V21 ) (V12 V11 )
3
=
( (1 cos 2 ) 32 (1 cos 1 ) 31 (1 cos 2 ) + 31 (1 cos 1 ))
3 2
3
((2 31 )(1 cos 2 ) (32 31 )(1 cos 1 ))
=
3
3
=
( 31 )(cos 1 cos 2 ).
3 2
(c) By the Mean Value Theorem with f () = 3 , there exists some (1 , 2 )
such that f (2 ) f (1 ) = f (
)(2 1 ) 32 31 = 3
2 . Similarly, for
2
g() = cos , there exists (1 , 2) such that g(2) g(1 ) = g ()(

1 ) cos 1 cos 2 = sin .


Hence for each spherical wedge E, there
)
in E such that
exists (
, ,
Vijk = 2i sin k .

15.9-6

15.10

Change Variables in Multiple Integrals,


page 1040

Goal: Find relations of change of variable in double and triple integrals.


Recall that
(1) For a function of one variable f (x), we have the Substitution Rule:
Z

f (x) dx =

f (x(u))x (u) du,

where x = x(u) and a = x(c), b = x(d).


(2) In section 15.4, we get the formula of double integrals in polar coordinates.
Supppose that x = r cos , y = r sin , then
ZZ
ZZ
f (r cos , r sin )r dr d,
f (x, y) dA =
R

where S is the region in the r-plane that corresponds to the region R in the
xy-plane.
More generally, we consider a change of variables that is given by a C 1 and one-toone transformation T form the uv-plane to the xy-plane (
):
T (u, v) = (x, y),
where x and y are related to u and v by the equations
(
(
x = x(u, v)
u = u(x, y)
T :
T 1 :
.
y = y(u, v),
v = v(x, y)
v

y
T
(u0 , v0 )

T 1

S
u

R
(x0 , y0 )
x

Figure 1: Transformation T and inverse transformation T 1 .

15.10-1

Definition 1 (page 1041).


(a) The terminology C 1 means that x(u, v) and y(x, y) have continuous first-order
partial derivatives.
(b) If T (u1, v1 ) = (x1 , y1 ), then (x1 , y1) is called the image of (u1 , v1 ).
(c) T is called one-to-one if no two points have the same image.
(d) T transforms S into a region R in the xy-plane called the image of S, consisting
of the images of all points in S.
Example 2 (page 1041). A transformation is defined by the equations x = u2
v 2 , y = 2uv. Find the image of the square S = {(u, v)|0 u 1, 0 v 1}.
Solution.

Now we will see how a change of variables affects a double integral. We start
with a small rectangle S in the uv-plane whose lower corner is the point (u0 , v0 )
and whose dimensions are u and v. The image of S is a region R in the xyv

(u0 , v0 ) u

v = v0

r(u
0,

v)

u = u0

(x0 , y0 ) r(u, v )
0

Figure 2: Transformation T from a rectangle S to a region R.


plane, one of whose boundary points is (x0 , y0 ) = T (u0 , v0 ). The vector r(u, v) =
x(u, v) i+y(u, v) j is the position vector of the image of the point (u, v). The equation

15.10-2

of the lower side of S is v = v0 , whose image curve is given by the vector function
r(u, v0 ). The tangent vector at (x0 , v0 ) to this image curve is
ru = xu (u0 , v0 ) i + yu (u0 , v0 ) j
Similarly, the tangent vector at (x0 , y0) to the image curve of the left side of S
(namely, u = u0 ) is
rv = xv (u0 , v0 ) i + yv (u0 , v0 ) j
We can approximate the image region R = T (S) by a parallelogram determined by
the secant vectors
a = r(u0 + u, u0 ) r(u0 , v0 )

and

b = r(u0 , v0 + v) r(u0 , v0 ).

Since
r(u0 + u, v0 ) r(u0 , v0 )
u0
u

ru = lim
and so

r(u0 + u, v0 ) r(u0, v0 ) uru

and r(u0 , v0 + v) r(u0 , v0 ) vrv .

This means that we can approximate R by a parallelogram determined by the vectors


uru and vrv . Therefore we can approximate the area of R by the area of this
parallelogram


x x
u v
|(u)ru (v)rv | = |ru rv |uv = y y uv.


u

Definition 3 (page 1043). The Jacobian of the transformation T given by x =


g(u, v) and y = h(u, v) is



x
x
(x, y) u v x y x y
= y y =

.
u v v u
(u, v) u
v


(x,y)
With this notation we can get A (u,v)
uv, where the Jacobian is evaluated at (u0 , v0 ).
For the general region S in the uv-plane we divide S into rectangles Sij and
call their images in the xy-plane Rij . Applying the approximation to each Rij , we
approximate the double integral of f over R as follows:
ZZ
m X
n
X
f (x, y) dA = lim
f (xi , yi )A
R

m,n

i=1 j=1

m X
n
X



(x, y)


uv + H.O.T.
f (x(ui , vi ), y(ui, vi ))
= lim

m,n
(u,
v)
(u
,v
)
i i
i=1 j=1


ZZ
(x, y)
du dv.
=
f (x(u, v), y(u, v))
(u, v)
S
15.10-3

Change to Variables in a Double Integral (page 1045). Suppose that T is a C 1


transformation whose Jacobian is nonzero and that maps a region S in the uv-plane
onto a region R in the xy-plane. Suppose that f is continuous on R and that R
and S are type I or type II plane regions. Suppose also that T is one-to-one, except
perhaps on the boundary of S. Then


ZZ
ZZ
(x, y)
du dv.
f (x, y) dA =
f (x(u, v), y(u, v))
(u, v)
R
S
RR x+y
Example 4 (page 1045). Evaluate the integral R e xy dA, where R is the trapezoidal region with vertices (1, 0), (2, 0), (0, 2), and (0, 1).
Solution.

RR
Example 5 (page 1045). Use x = u2 v 2 , v = 2uv to evaluate the integral R y dA,
where R is the region bounded by the x-axis and the parabolas y 2 = 4 4x and
y 2 = 4 + 4x, y 0.
Solution.

15.10-4

Example 6. Evaluate
Solution.

RR

x2 +xy+y 2 1

e(x

2 +xy+y 2 )

(98 )

dA

Triple Integrals, page 1046


The Jacobian of the transformation T is the following 3 3 determinant:


x x x
u v w
(x, y, z)
y y y
= u
.
w
(u, v, w) z v
z
z

u

We have the following formula for triple integrals:




ZZ
ZZ
(x, y, z)
du dv dw.

f (x, y, z) dV =
f (x(u, v, w), y(u, v, w), z(u, v, w))

(u,
v,
w)
R
S
Example 7. Elliptic cylindrical coordinate system is
x = ar cos ,

y = br sin ,

z = cz,

where a, b, c > 0 are constants. The volume element is

Example 8. Ellipsoidal coordinate system is


x = a sin cos ,

y = b sin sin ,

z = c cos ,

where a, b, c > 0 are constants. The volume element is

 , Jacobian (Homework 7)

15.10-5

RR

Homework. Evaluate the integral

sin(x + y) cos(2x y) dA, where R is the

region bounded by y = 2x 1, y = 2x + 3, y = x, and y = x + 1. (90 )


RRR

Homework. Evaluate the integral

(x2 y + 3xyz) dV , where E is the region


(91 )

1 x 2, 0 xy 2, and 0 z 1.
Homework. Evaluate the integral
y 2 a2 }.

RR

ex

2 xy+y 2

dA, where R = {(x, y)|x2 xy +


(92 )

RR
Homework*. Evaluate the integral R sin(x2 +2xy +y 2) dA, where R is the region
bounded by x + y = 0, x = 0, and y = 0.
(93 )
RR
x2
Homework*. Evaluate the integral R x2 +4y
2 dA, where R is the region bounded
2
2
2
2
by two ellipses x + 4y = 1 and x + 4y = 4.
(94 )
Homework*. Evaluate the integral
Homework. Evaluate the integral
3y 2 + 5z 2 + 6yz + 2xz 1}.

R 3 R 1 y
2

RRR

(2x + y)eyx dx dy.

(95 )

(x + y + z)2 dV , where E = {(x, y, z)|2x2 +


(96 )

Homework*. Compute the area of the domain in the first quadrant bounded by
the four curves xy = 1, xy = 4, xy2 = 1, and

y
x2

= 2.

(97 )

RRR
Homework*. Find the region E R3 for which the triple integral
(4 x2
E
4y 2 9z 2 ) dV is a maximum, and compute this maximum value.
(97 )
Homework*. Evaluate
Homework*. Find
16y 2 + 9z 2 144}.

e(x

x2 +xy+y 2 1

RRR

Homework*. Evaluate

RR

2 +xy+y 2 )

2y and y 0.

(98 )

xyz dV , where E = {(x, y, z)|x 0, y 0, 0, 36x2 +


(99 )

RR

RR

e4x

sin
R

2yx
2y+x

dA, where R is the region bounded by 2y +

x = 1, 2y + x = 2, 2y x = 0, and 2y + 5x = 0.
Homework*. Evaluate

dA

2 +12xy10y 2

(99 )

dA, where R is the region satisfying x


(100 )

2 9y 2

e4x

and x 0.

RR

Homework*. Evaluate
4, y = 1, and y = 3.

RR

exy dA, where R is the region bounded by xy = 1, xy =


(102 )

Homework*. Evaluate

dA, where R is the region satisfying 2x 3y


(101 )

15.10-6

Chapter 16
16.1

Vector Calculus

Vector Fields, page 1056

Definition 1 (page 1057).


(1) Let D be a set in R2 . A vector field on R2 () is a map F that assigns to
each point (x, y) in D a two-dimensional vector F(x, y).
(2) Let E be a subset of R3 . A vector field on R3 is a map F that assigns to each
point (x, y, z) in E a three-dimensional vector F(x, y, z).
The best way to picture a vector field is to draw the arrow representing the
vector F(x, y) starting at the point (x, y) for a few representative points in D.
y
z
F(x, y, z)
(x, y)
(x, y, z)

F(x, y)

x
Figure 1: Vector fields on R2 and on R3 .
Since F(x, y) is a two-dimensional vector, we can write it in terms of its component functions () P and Q as follows:
F(x, y) = P (x, y) i + Q(x, y) j = (P (x, y), Q(x, y)).
Functions P (x, y) and Q(x, y) are called scalar function () or scalar fields.
Example 2 (page 1057). A vector field on R2 is defined by F(x, y) = y i + x j.
Denote x = x i + y j by the position vector.

Figure 2: Vector Fields F = y i + x j = (y, x) and x = x i + y j = (x, y).


Remark that x F(x) = (x i + y j) (y i + x j) = xy + xy = 0, so two vector
fields are orthogonal ().
16.1-1

Example 3 (page 1059). Newtons Law of Gravitation states that the magnitude
of the gravitational force between two objects with masses m and M is
mMG
,
r2
where r is the distance between the objects and G is the gravitational constant.
Let the position vector of the object with mass m be x = (x, y, z), then r 2 = |x|2 .
Therefore the gravitational force acting on the object at x is
|F| =

F(x) =

mMG x
mMG
=
x,
2
|x| |x|
|x|3

(1)

and we say the equation (1) is gravitational field ().


Example 4 (page 1060). Suppose an electric charge Q is located at the origin.
According to Coulombs Law, the electric force F(x) (or electric field ) exerted
by this charge on a charge q located at a point (x, y, z) with position vector x =
(x, y, z) is
F(x) =

qQ x
qQ
=
x,
2
|x| |x|
|x|3

where is a constant. For like charges, we have qQ > 0 and the force is repulsive;
for unlike charges, we have qQ < 0 and the force is attractive.
Instead of considering the electric force F, physicists often consider the force per
unit charge ():
E(x) =

Q
1
F(x) =
x.
q
|x|3

Gradient Fields (), page 1060


Recall that for a smooth function f (x, y), the gradient f , or grad f , is defined by
f (x, y) = grad f = fx (x, y) i + fy (x, y) j.
Likewise, if f (x, y, z) is a scalar function of three variables, its gradient is a vector
field on R3 given by f (x, y, z) = fx (x, y, z) i + fy (x, y, z) j + fz (x, y, z) k.
Definition 5 (page 1060).
(a) For a scalar function f , we say f is a gradient vector field ().
(b) A vector field F is called a conservative vector field () if it is the
gradient of some scalar function, that is, if there exists a function f such that
F = f . In this situation f is called a potential function () for F.
Not all vector fields are conservative, but such fields do arise frequently in physics.
16.1-2

Example 6 (page 1061). The gravitational field F is conservative because if we


define a function
mMG
,
f (x, y, z) = p
x2 + y 2 + z 2

then
f =

Example 7. Let f (x, y) be a smooth function, then the gradient vector field f (x, y)
is perpendicular to the level curves f (x, y) = k.
4

-2

-4
-4

-2

Figure 3: Level sets of f (x, y) = x2 y 2 and the gradient field f = 2x i 2y j.

16.1-3

Homework (page 1062). Match the functions f1 , f2 , f3 , and f4 with the plots of
their gradient vector fields labeled I - IV. Give reasons for your choices.
(a) f1 (x, y) = x2 + y 2 .
(b) f2 (x, y) = x(x + y).
(c) f3 (x, y) = (x + y)2 .
p
(d) f4 (x, y) = sin x2 + y 2 .
1
1

0.5

0.5

-0.5

-0.5

-1

-1

III

-1
-0.5

0.5

II

-1.5

0.5

0.5

-0.5

-0.5

-1

-1

-1

-0.5

0.5

IV

-1

-1

Figure 4: Gradient vector fields.

16.1-4

-0.5

-0.5

0.5

0.5

1.5

16.2

Line Integrals, page 1063

Line Integrals of Scalar Functions (), page 1063


Suppose that a smooth plane curve C is given by the parametric equations r(t) =
x(t) i + y(t) j, a t b. Recall that a curve is smooth means that r (t) is continuous
and r (t) 6= 0. We will define an integral over a curve C.
y
Pi1
P2
P1

Pi (xi , yi)
Pi
Pn

ti

P0

a t1 t2 ti1 ti tn1 b

Pn1
x

Figure 1: Line integrals.


(1) We divide the parameter interval [a, b] into n subintervals [ti1 , ti ] of equal
width. We let xi = x(ti ) and yi = y(ti ), then the corresponding points Pi (xi , yi )
divide C into n subarcs with lengths s1 , s2 , . . . , sn .
(2) Choose any point P (xi , yi) (corresponding to ti [ti1 , ti ]) in the i-th subarc.
(3) If f (x, y) is any function of two variables whose domain includes the curve C,
n
P
we form the sum (similar to a Riemann sum)
f (xi , yi)si .
i=1

(4) Define the line integral of f (x, y) along C ( f (x, y) C ) is


Z
n
X
f (xi , yi )si
f (x, y) ds = lim
n

i=1

if this limit exists.


Recall that the arc length function of C is
s
s 
 2
Z t  2  
2
dx
dx
dy
dy
s(t) =
du ds =
+
+
dt,
du
du
dt
dt
a
so the line integral has the following expression:
s 
 2
Z
Z b
2
dy
dx
+
dt.
f (x, y) ds =
f (x(t), y(t))
dt
dt
C
a
16.2-1

Well-defined problem: (copy from Wikipedia) In mathematics, an expression is


called well-defined () if its definition assigns it a unique interpretation
or value. For example, a function is well-defined if it gives the same result when
the representation of the input is changed without changing the value of the input.
That is, if f (x) is a well-defined function defined on R, then f (0.5) must be equal
to f ( 21 ), and f (1) must be equal to f (0.9).
Suppose that r(v) = x(v) i + y(v) j, c v d is another parametrization of the
plane curve C. To show the line integral is well-defined, we have to check that
s 
 2
Z
Z d
2
dy
dx
+
dv.
f (x, y) ds =
f (x(v), y(v))
dv
dv
C
c
Check: This is because the arc length is
s 
s
 2
Z v  2  2
2
dx
dx
dy
dy
+
du ds =
+
dv.
s(v) =
du
du
dv
dv
a
The geometric meaning of the line integrals is to compute the area of one side
of the fence or curtain, whose base is C an whose height at the point (x, y)
is f (x, y).

 ds , , ,
 , ,
We can define the line integrals on piecewise smooth curve () C,
which is a union of a finite number of smooth curves C1 , C2 , . . . , Cn . Then we define
the integral of f (x, y) along C as the sum of the integrals of f along each of the
smooth pieces of C:
Z
Z
Z
Z
f (x, y) ds.
f (x, y) ds + +
f (x, y) ds +
f (x, y) ds =
C

C2

C1

Cn

f (x, y)

y
(x, y)

C
x
Figure 2: Line integrals: The area of the fence. The total calories of running.

16.2-2

Example 1 (page 1064). Evaluate


unit circle x2 + y 2 = 1.

(2 + x2 y) ds, where C is the upper half of the

Solution.

R
Example 2 (page 1064). Evaluate C 2x ds, where C consists of the arc C1 of the
parabola y = x2 from (0, 0) to (1, 1) followed by the vertical line segment C2 from
(1, 1) to (1, 2).
Solution.

R
Any physical interpretation of a line integral C f (x, y) ds depends on the physical
interpretation of the function f . Suppose that (x, y) represents the linear density
at a point (x, y) of a thin wire shaped like a curve C. Then the mass of the part of
the wire from Pi1 to Pi in Figure 1 is approximately (xi , yi)si and so the total
n
P
mass of the wire is
(xi , yi )si . When n tends to infinity, we obtain the mass
i=1

() m of the wire:

m = lim
n

n
X

(xi , yi)si

(x, y) ds.

i=1

The center of mass () of the wire with density function (x, y) is located at the
point (
x, y), where
Z
Z
1
1
x(x, y) ds and y =
y(x, y) ds.
x =
m C
m C
16.2-3

Example 3 (page 1065). A wire takes the shape of the semicircle x2 +y 2 = 1, y 0,


and is thicker near its base than near the top. Find the center of mass of the wire
if the liner density at any point is proportional to its distance from the line y = 1.
Solution.

Definition 4 (page 1066).


(a) Define line integrals of f along C respect to x ( f C x ):
Z
Z b
n
X

f (x, y) dx = lim
f (xi , yi )xi =
f (x(t), y(t))x (t) dt.
n

i=1

(b) Define line integrals of f along C respect to y ( f C y ):


Z b
Z
n
X

f (xi , yi )yi =
f (x, y) dy = lim
f (x(t), y(t))y (t) dt.
n

i=1

(c) The original line integral is line integrals of f along C respect to are length:
Z
Z b
n
X
p

f (x, y) dx = lim
f (xi , yi )si =
f (x(t), y(t)) (x (t))2 + (y (t))2 dt.
n

i=1

 f C x , dx ,
It frequently happens that line integrals with respect to x and y occur together.
When this happens, its customary to abbreviate by writing
Z
Z
Z
P (x, y) dx +
Q(x, y) dy =
P (x, y) dx + Q(x, y) dy.
C

16.2-4

Example 5 (page 1067). Evaluate

y 2 dx + x dy, where

(a) C = C1 is the line segment from (5. 3) to (0, 2).


(b) C = C2 is the arc of the parabola x = 4 y 2 from (5, 3) to (0, 2).
Solution.

Line Integrals in Space, page 1068


Suppose that C is a smooth space curve given by the parametric equation x =
x(t), y = y(t), z = z(t), a t b or vector equation r(t) = x(t) i + y(t) j + z(t) k. If
f (x, y, z) is a smooth function that is continuous on some region containing C, then
we define the line integral of f along C as
Z
n
X
f (x, y, z) ds = lim
f (xi , yi, zi )si
n

i=1

=
=

a
b

p
f (x(t), y(t), z(t)) (x (t))2 + (y (t))2 + (z (t)) dt
f (r(t))|r(t)| dt.

Line integrals along C with respect to x, y, and z can also be defined. For example,
Z
Z b
n
X


f (x, y, z) dz = lim
f (xi , yi , zi )zi =
f (x(t), y(t), z(t))z (t) dt.
n

i=1

Therefore, we evaluate integrals of the form


Z
P (x, y, z) dx + Q(x, y, z) dy + R(x, y, z) dz
C

by expressing everything (x, y, z, dx, dy, dz) in terms of the parameter t.


16.2-5

Line Integrals of Vector Fields (), page 1070


Suppose that F(x, y, z) = P (x, y, z) i + Q(x, y, z) j + R(x, y, z) k is a continuous force
field on R3 such as gravitational field, electric force field, etc. We want to compute
the work () done by this force in moving a particle along a smooth curve C.
(1) We divide C into subarcs Pi1 Pi with lengths si by dividing the parameter
interval [a, b] into subintervals of equal width.
(2) Choose Pi (xi , yi, zi ) on the i-th subarc corresponding to ti .
(3) If si is small, then as the particle moves from Pi1 to Pi along the curve, it
proceeds approximately in the direction T(ti ), the unit tangent vector at Pi.
Thus the total work done by the force F in moving the particle along C is
approximately
n
X

F(xi , yi, zi ) T(xi , yi, zi )si .

i=1

(4) When n tends to infinity, we define the work () W done by the force field F:
Z
Z
W =
F(x, y, z) T(x, y, z) ds =
F T ds.
(1)
C

Hence, work is the line integral with respect to arc length of the tangential component
of the force.
z
P0

P1
P2

Pi

Pn1

Pn

Pi1
Pi (xi , yi , zi )

T(ti )

F(xi , yi , zi )

Figure 3: Line integrals of vector fields.


If the curve C is given by the vector equation r(t) = x(t) i + y(t) j + z(t) k, then

we get the unit tangent vector is T = |rr (t)


, so we can rewrite (1) in the form
(t)|

Z
Z 
r (t)

|r (t)| dt =
F(r(t)) r (t) dt
W =
F(r(t))
|r
(t)|
C
ZC
Z
Z
=
F(r(t)) dr =
F(x(t), y(t), z(t)) dr =
F dr.
C

16.2-6

Definition 6 (page 1071). Let F be a continuous vector field on a smooth curve C


given by a vector function r(t), a t b. Then the line integral of F along C is
Z
Z b
Z

F dr =
F(r(t)) r (t) dt =
F T ds.
C

 (, )
R
R
 C C , C F dr = C F dr

Example 7. Find the work done by the force field F (x, y) = y 2 i + x2 j in moving

a particle along arc of the circle x2 + y 2 = 2 traversed counterclockwise from ( 2, 0)

to ( 2, 0).
(90 )
Solution.

Connection between line integrals of vector fields and line


integral of scalar functions, page 1072
Suppose that the vector field F on R3 is given in component form by the equation
F(x, y, z) = P (x, y, z) i + Q(x, y, z) j + R(x, y, z) k. Then
Z
F dr
C
Z b
Z b

=
F(r(t)) r (t) dt =
(P i + Q j + R k) (x (t) i + y (t) j + z (t) k) dt
a
a
Z b
=
(P (x(t), y(t), z(t))x (t) + Q(x(t), y(t), z(t))y (t) + R(x(t), y(t), z(t))z (t)) dt
Za
=
P (x, y, z) dx + Q(x, y, z) dy + R(x, y, z) dz.
C

16.2-7

R
Example (TA) 8. Evaluate C F dr, where F = 3z i + 23 x j and C is the space
curve which is the intersection of x2 + y 2 + z 2 = 1 and y = 1 x2 in the first octant
(95 )
from ( 12 , 12 , 21 ) to (1, 0, 0).
Solution.

R
Homework. Evaluate C xy dx+yz dy +zx dz, where C consists of the line segment
C1 from (0, 0, 0) to (1, 1, 0) followed by another line segment C2 from (1, 1, 0) to
(1, 1, 1).
(90 )
Homework. Find the work done by the force field F(x, y) = x2 y i xy 2 j on a
particle that starts at (2, 0) and moves once around the circle x2 + y 2 = 4 oriented
in the counterclockwise direction.
(92 )
R
Homework. Evaluate C x ds, where C is part of unit circle from ( 12 , 12 ) to
(0, 1).
(92 )
Homework. Find the work done by the force field F(x, y) = z i + y j x k in
moving a particle along the circular helix given by the equations x(t) = cos t, y(t) =
sin t, z(t) = t, 0 t 2.
(92 )
R
Homework. Find the line integral C 6y 2 dx + 4x3 dy, where C is the arc of the
parabola y = 1 x2 from (1, 0) to (0, 1) and then to (1, 0).
(99 )
16.2-8

16.3

The Fundamental Theorem for Line Integrals, page 1075

Recall that Part 2 of the Fundamental Theorem of Calculus is


Z b
F (x) dx = F (b) F (a),

(1)

where F (x) is continuous on [a, b]. We also called equation (1) the Net Change
Theorem: The integral of a rate of change is the net change.
Here we will introduce the Fundamental Theorem for line integrals, where we
think of the gradient vector f of a function f as a sort of derivative of f .
Theorem 1 (page 1075). Let C be a smooth curve given by the vector function
r(t), a t b. Let f be a differentiable function of two or three variables whose
gradient vector f is continuous on C. Then
Z
f dr = f (r(b)) f (r(a)).
C

 (),
R
R
 , C f dr = C f dr
y

A(x1 , y1 )

A(x1 , y1, z1 )
B(x2 , y2, z2 )
y

B(x2 , y2 )
x

C
x

Figure 1: The fundamental theorem for line integrals.


Proof. By the Chain Rule and the Fundamental Theorem of Calculus, we have

Z
Z b
Z b
f dx f dy f dz

f dr =
f (r(t)) r (t) dt =
dt
+
+
x dt
y dt
z dt
C
a
a
Z b
d
f (r(t)) dt = f (r(b)) f (r(a)).
=
a dt
Theorem 1 is also true for piecewise smooth curves. This can be seen by subdividing C into a finite number of smooth curves and adding the resulting integrals.
16.3-1

Example 2 (page 1076). Find the work done by the gravitational field F(x) =
G
x in moving a particle with mass m from the point (3, 4, 12) to the point
mM
|x|3
(2, 2, 0) along a piecewise smooth curve C.
Solution.

Independence of Path, page 1066


Suppose that C1 and C2 are two piecewise smooth curves, which are also called paths
() with have the same initial point A and terminal point B.
Definition 3 (page 1076). Suppose that F is a continuous vector field with domain
R
D, we say the line integral C F dr is independent of path () if
Z
Z
F dr
F dr =
C2

C1

for any two path C1 and C2 in D with the same initial and terminal points.
In general vector field F,

F dr 6=
C1

C2

F dr. (See 16.2, Example 5).

For conservative vector field F = f , the Fundamental Theorem for line inteR
R
grals tells us C1 f dr = C2 f dr = f (r(b)) f (r(a)).

The following discussion will say that the only vector fields that are independent of path are conservative vector fields.


Definition 4 (page 1077). A curve is called closed () if its terminal point
coincides with its initial point, that is, r(b) = r(a).

Figure 2: Closed curve (left) and non-closed curve (right).

16.3-2

R
Theorem 5 (page 1077). The line integral C F dr is independent of path in D if
R
and only if C F dr = 0 for every closed path C in D.
Proof. () We choose any two points A and B on C and regard C as being composed

of the path C1 from A to B followed by the path C2 from B to A. Then


Z
Z
Z
Z
Z
Z
F dr = 0.
F dr
F dr =
F dr +
F dr =
F dr =
C

C1

C2

C1

C1 C2

C2

() For any paths C1 and C2 from A to B in D, we define C to be the curve


consisting of C1 followed by C2 . Then we get
Z
Z
Z
Z
F dr =
F dr +
0=
F dr =
C

and hence

C1

F dr =

C1

C2

C2

C1

F dr

F dr,

C2

F dr.


 ()
Definition 6 (page 1077).
(a) A domain D is open () if for every point P in D, there is a disk with
center P that lies entirely in D. (D doesnt contain any of its boundary points.)
(b) A domain D is path connected () if any two points in D can be joined
by a path that lies in D.

Figure 3: Open set; non-open set; path connected region; non-path connected region.

16.3-3

Theorem 7 (page1077). Suppose F is a vector field that is continuous on an open,


R
path connected region D. If C F dr is independent of path in D, then F is a
conservative vector field on D; that is, there exists a function f such that f = F.
Proof. Let A(a, b) be a fixed point in D. We construct the potential function f by
Z (x,y)
f (x, y) =
F dr
(a,b)

for (x, y) D. Since C F dr is independent of path, the function is well-defined.


Now we will show that f = F:
y

y
(x, y)

(x, y)
(x1 , y)

C2

C2

(x, y1 )
C1

C1

(a, b)

(a, b)

Figure 4: Choose suitable paths to prove f = F.


Since D is open, there exists a disk contained in D with center (x, y). Choose
any point (x1 , y) in the disk with x1 < x and let C consists of any path C1 from (a, b)
to (x1 , y) followed by the horizontal line segment C2 from (x1 , y) to (x, y). Then
Z (x1 ,y)
Z
Z
Z
F dr =
F dr +
F dr.
f (x, y) =
F dr +
C1

(a,b)

C2

C2

Notice that the first of these integrals does not depend on x, so


Z

F dr.
(2)
f (x, y) = 0 +
x
x C2
R
R
If we write F = P (x, y) i + Q(x, y) j, then C2 F dr = C2 P (x, y) dx + Q(x, y) dy.
On C2 , y is constant, so dy = 0. Let x1 t x, then (2) implies
Z
Z x

P (x, y) dx + Q(x, y) dy =
P (t, y) dt = P (x, y).
f (x, y) =
x
x C2
x x1
Similarly, using a vertical line segment, we have
Z
Z y

P (x, y) dx + Q(x, y) dy =
Q(x, t) dt = Q(x, y).
f (x, y) =
y
y C2
y y1

Thus

F(x, y) = P (x, y) i + Q(x, y) j =

f
f
i+
j = f.
x
y

 Theorem 7 :
16.3-4

Next, we will determine whether or not a vector field F is conservative. Suppose


that F = P i + Q j is conservative, where P and Q have continuous first order
partial derivatives. Then there is a function f such that F = f , that is, P = f
x
.
By
Clairauts
Theorem,
we
know
and Q = f
y
P
2f
2f
Q
=
=
=
.
y
yx
xy
x
Hence Py = Qx is a necessary condition () that F = P i+Q j is conservative.
Theorem 8 (page 1078). If F(x, y) = P (x, y) i + Q(x, y) j is a conservative vector
field, where P and Q have continuous first order partial derivatives on a domain D,
then throughout D we have
P
Q
=
.
y
x
The condition Py = Qx is a sufficient condition for a simply connected region.
Definition 9 (page 1078).
(a) We say C is a simple curve () if it doesnt intersect itself anywhere
between its endpoints. (r(t1 ) 6= r(t2 ) when a < t1 < t2 < b).

Figure 5: (Left to right) Simple, not closed; simple closed; not simple, not closed;
not simple, closed.
(b) D is a simply connected region () in a plane if it is path connected
and every simple closed curve in D enclosed only points that are in D.

Figure 6: Simply connected region; non simply connected region.

16.3-5

Theorem 10 (page 1079). Let F(x, y) = P (x, y) i + Q(x, y) j be a vector field on an


open simply-connected region D. Suppose that P and Q have continuous first-order
derivatives and
Q
P
=
y
x

throughout D,

then F is conservative.
We will prove Theorem 10 in the next section.

 , (genus);
Finally, we will use partial integration to find the potential functions.
Example 11 (page 1079).
(a) If F(x, y) = (3 + 2xy) i + (x2 3y 2) j, find a function f such that F = f .
R
(b) Evaluate the line integral C F dr, where C is the curve given by r(t) =
et sin t i + et cos t j, 0 t .
(92 )
Solution.

Example 12 (page 1083). Show that if the vector field F = P i + Q j + R k is


conservative and P, Q, R have continuous first order partial derivatives, then
Q
P
=
y
x

R
P
=
z
x

R
Q
=
.
z
y

Proof. Since F = f , we have P = fx , Q = fy , and R = fz . By Clairauts Theorem,


we know that Py = (fx )y = fxy = fyx = (fy )x = Qx , Pz = (fx )z = fxz = fzx =
(fz )x = Rx , and Qz = (fy )z = fyz = fzy = (fz )y = Ry .
16.3-6

Example 13. If F(x, y, z) = y 2 i + (2xy + e3z ) j + 3ye3z k, find a function f such


that f = F.
Solution.

Example 14 (page 1083). Let F(x, y) = P (x, y) i + Q(x, y) j =


(a) Show that

P
y

y
x2 +y 2

i+

x
x2 +y 2

j.

Q
.
x

(b) Let f = tan1 ( xy ), (which is function in polar coordinates). Compute f .


R
(c) Show that C F dr is not independent of path.

Solution.

(a) Direct computation gives


(x2 + y 2 ) + y(2y)
x2 + y 2
(x2 + y 2 ) x(2x)
Q
P
=
=
=
=
.
2
2
2
2
2
2
2
2
2
y
(x + y )
(x + y )
(x + y )
x
(b) Direct computation gives
f =

1
xy2
f
y
f
x
x
i
+
i+
j=
i+ 2
j.
y 2
y 2 j = 2
2
x
y
1 + (x)
1 + (x)
x +y
x + y2

(c) Choose C1 : r1 (t) = cos t i+sin t j, 0 t and C2 : r2 (t) = cos t i+sin t j, 0


t . Two paths start from (1, 0) to (1, 0), and
Z
Z
Z
F dr =
sin t( sin t) + (cos t)(cos t) dt =
dt =
C1
0
0
Z 0
Z
Z 0
F dr =
sin t( sin t) + (cos t)(cos t) dt =
dt = .
C2

So

F dr is not independent of path.

Example 14 ? ?
16.3-7

Appendix: Conservation of Energy, page 1081


We will apply these ideas to a continuous force field F that moves an object along
a path C given by r(t), a t b, where r(a) = A is the initial point and r(b) = B
is the terminal point of C.
According to Newtons Second Law of Motion, the force F(r(t)) at a point on C
is related to the acceleration a(t) = r (t) by the equation F(r(t)) = m r (t), so the
work done by the force on the object is
Z
Z b
Z b

F dr =
W =
F(r(t)) r (t) dt =
m r (t) r (t) dt
C
a
a
Z b
Z b
m
m h i b
d
d 2
m

|r (t)|
(r (t) r (t)) dt =
|r (t)| dt =
=
a
2 a dt
2 a dt
2
m 2
1
1
= (|r (b)| |r(a)|2 ) = m|v(b)|2 m|v(a)|2 ,
(3)
2
2
2
where v(t) = r (t) is the velocity.
The quantity 21 m|v(t)|2 , is called the kinetic energy () of the object. Therefore we can rewrite Equation (3) as W = K(B) K(A), which says that the work
done by the force field along C is equal to the change in kinetic energy at the
endpoints of C.
Now lets further assume that F is a conservative force field; that is, we can write
F = f . In physics, the potential energy () of an object at the point (x, y, z) is
defined as P (x, y, z) = f (x, y, z), so we have F = f = P . Then we have
Z
Z
W =
F dr = P dr = (P (r(b)) P (r(a))) = P (A) P (B).
C

Comparing this equation with W = K(B) K(A), we see that


P (A) + K(A) = P (B) + K(B),
which says that if an object moves from one point A to another point B under the
influence of a conservative force field, then the sum of its potential energy and its
kinetic energy remains constant. This is called the Law of Conservation of Energy
() and it is the reason the vector field is called conservative.
Homework*. Find the potential function of the following vector field:
!
!
z
x
y
y
1
k.
i+
+p
j+ p
F=
+
1 + x2 y 2
1 + x2 y 2
1 y2z2
1 y2z2 z

(90 )

16.3-8

Homework*.
(a) Find a the potential function of the following vector field:


1
1
2x sin y j + k.
F = 2 cos y i +
y
z
R
(b) Find C F dr, where C is any curve from (0, 2, 1) to (1, 2 , 2). (91 )

Homework*. Let F(x, y) = (3 + 2xy) i + (x2 3y 2) j.

(a) Find a function f (x, y) such that f = F.


R
(b) Find C F dr, where C : r(t) = cos t i + sin t j, 0 t 2 .

(92 )

Homework. Let
F(x, y, z) =

y
i+
1 + x2 y 2

z
x
+p
2
2
1+x y
1 y 2z 2

j+

y
p
+z
1 y 2z 2

k.

(a) Find a potential function of F.


R
(b) Find C F dr, where C is the upper cardioid r = 1 + cos , 0 .
(93 )
Homework*. Let
F(x, y, z) =

xi + yj + zk
.
x2 + y 2 + z 2

(a) Find a smooth function f (x, y, z) such that f = F.


R
(b) Find C F dr, where C is the line segment from (1, 1, 1) to (2, 3, 4).
(94 )
Homework*.
(a) Let F = eyz i+(xzeyz +z cos y) j+(xyeyz +sin y) k. Find the potential function
of F.
(b) Let C be a curve consists of the line segment C1 from (1, 0, 1) to (1, 2 , 1)
followed by another line segment C2 from (1, 2 , 1) to (1, 2 , 2). Find the work
done by F in moving a particle along C.
(95 )
Homework. Let

F = yz i + (xz + ze ) j + 2xyz + p(y, z) +
2

yz

1
1+z

k,

where p(y, z) is a smooth functions with p(0, z) = 0. Suppose that F is a conservative


field.
16.3-9

(a) Find p(y, z).


(b) Find the potential function of F.
(c) Let C be a curve r(t) = t i + t2 j + t3 k. Find
Homework*. Let


y
F(x, y, z) = yz i + xz + 2
y + z2 + 1

(96 )

F dr.


j + xy +

z
+ cos z
2
y + z2 + 1

k.

(a) Find a function f such that F = f .


R
(b) Compute the line integral C F dr, where C is the curve starting from the
origin given by r() = cos i + sin j + k, 0 .
(97 )
Homework. Let F(x, y, z) = y 2 i + (2xy + e3z ) j + 3ye3z k.
(a) Find the potential function of F.
R
(b) Compute the line integral C Fdr, where C : r(t) = 4t i+3 cos t j+3 sin t k, 0
t 2 .
(98 )
Homework*. Let F = cos y i + (z 2 x sin y) j + 2(y + 1)z k. Find a scalar function
R
f (x, y, z) such that f = F and then evaluate C F dr, where C is the line segment
from (1, 0, 0) to (2, 2, 1).
(99 )
Homework*. Find the work done by the force field
F = (2xyz + z 2 2y 2 + 1) i + (x2 z 4xy + x) j + (x2 y + 2xz + 2z) k
in moving a particle along the curve C parameterized by r(t) = 2t i + sin1 t j +
t k, 0 t 1.
(100 )
Homework. Find the line integral
Z
(2x sin(y) ez ) dx + (x2 cos(y) 3ez ) dy xez dz
C

along the curve C = {(x, y, z)|z = ln

1 + x2 , y = x, 0 x 1}.

(101 )

Homework*.
(a) Find a scalar function f (x, y, z) such that f = sin y i + x cos y j sin z k.
R
(b) Find the line integral C sin y dx + x cos y dy + (y sin z) dz, where C : r(t) =
(102 )
t i + 2 cos t j + 2 sin t k, 0 t .
16.3-10

16.4

Greens Theorem, page 1084

Greens Theorem gives the relationship between a line integral around a simple
closed curve C and a double integral over the plane region D bounded by C.
Definition 1 (page 1084). We say a simple closed curve C is positive orientation
() if the curve is traverses counterclockwise.
If C is given by the vector function r(t), a t b, then the region D is always
on the left at the point r(t) traverses C.
y
y

Figure 1: Positive orientation (left) and negative orientation (right).


Greens Theorem (page 1084). Let C be a positive oriented, piecewise smooth,
simple closed curve in the plane and let D be the region bounded by C. If P (x, y)
and Q(x, y) have continuous partial derivatives on an open region that contains D,
then


 
Q P
dA.

P dx + Q dy =
x
y
C
D
Remark 2. Sometimes we use the following notations


P dx + Q dy,
C

P dx + Q dy,

or

P dx + Q dy
D

to indicate that the line integral is calculated in the positive orientation.


Remark 3. In differential geometry, we define the wedge product or exterior
operator on vectors or differential forms. Given two differential forms dx, dy, their
wedge product dx dy means the positive oriented area element, so we have
dA = dx dy = dy dx,

and dx dx = 0,

and d(dx) = 0.

Greens Theorem can be regarded as the relationship between the integral, differential forms, and wedge product:


P dx + Q dy =

d(P dx + Q dy)



P
Q
Q
P
=
dx +
dy dx + P d(dx) +
dx +
dy dy + Q d(dy)
x
y
x
y
D



 
Q P
P
Q
dA.
dy dx +
dx dy =

=
x
y
D y
D x
D
C

16.4-1

Example 4 (page 1086). Evaluate

x4 dx + xy dy, where C is the triangular curve

consisting of the line segments from (0, 0) to (1, 0), form (1, 0) to (0, 1), and from
(0, 1) to (0, 0).
Solution.

Example 5 (page 1086). Evaluate


is the circle x2 + y 2 = 9.

(3y esin x ) dx + (7x +

y 4 + 1) dy, where C

Solution.

Example 6 (page 1086). If P (x, y) = Q(x, y) = 0 on a simple closed curve C, then


Greens Theorem gives

no matter what values P and Q assume in the region D.


Example 7 (page 1087). If we take (P, Q) = (0, x), (P, Q) = (y, 0), and (P, Q) =
( 12 y, 12 x), then Greens Theorem gives

16.4-2

Extended Versions of Greens Theorem


Greens Theorem can be extended to apply to regions with holes (genus), that is,
regions that are not simply connected.
y
y
D

C2
D

C1

(a)

(b)

Figure 2: Region D is not simply connected.


See Figure 2 (a). Observe that the boundary C of the region D consists of two
simple closed curves C1 and C2 . We assume that these boundary curves are oriented
so that the region D is always on the left as the curve C is traversed. Thus the
positive direction is counterclockwise for the outer curve C1 but clockwise for the
inner curve C2 .
If we divide D into two region D and D by means of the lines shown in Figure 2
(b), then we applying Greens Theorem to each of D and D to get



 
 
 
Q P
Q P
Q P
dA =
dA +
dA

x
y
x
y
x
y
D
D
D


P dx + Q dy +
D

P dx + Q dy.
D

Since the line integrals along the common boundary lines are in opposite directions,
they cancel and we get

 



Q P
dA =

P dx + Q dy +
P dx + Q dy =
P dx + Q dy.
x
y
D
C1
C2
C
Example 8 (page 1090).
(a) If C is the line segment connecting the point (x1 , y1) to the point (x2 , y2), then


x dy y dx = x1 y2 x2 y1 .
C

(b) If the vertices of a polygon, in counterclockwise order, are (x1 , y1 ), (x2 , y2), . . .,
(xn , yn ), then the area of the polygon is
1
A = ((x1 y2 x2 y1 ) + (x2 y3 x3 y2 ) +
2
+ (xn1 yn yn1 xn ) + (xn y1 yn x1 )).

16.4-3

Example 9 (page 1089). Let F =

y
x2 +y 2

i+

x
x2 +y 2

j.

(a) Show that C F dr = 0 for every positively oriented simple closed path that
does not encloses the origin.


(b) Show that C F dr = 2 for every positively oriented simple closed path that
encloses the origin.
Solution.

16.4-4

Appendix, page 1085


Proof of Greens Theorem in which D is a simple region. It suffices to show that


P (x, y) dx =
C

P
dA and
y

Q(x, y) dy =
C

Q
dA.
x

y
y = g2 (x)
C4

C3

C2

y = g1 (x) C1

Figure 3: Simple Region D.


We express D as a type I region D = {(x, y)|a x b, g1 (x) y g2 (x)},
where g1 (x) and g2 (x) are continuous functions. By the Fundamental Theorem of
Calculus, we have

D

P
dA =
y

b
a

g2 (x)
g1 (x)

P
(x, y) dy dx =
y

(P (x, g2(x)) P (x, g1 (x))) dx.


a

On the other hand, we know C = C1 C2 C3 C4 . On C1 we write the


parametric equations as x = x, y = g1 (x), a x b and get


P (x, y) dx =

P (x, g1(x)) dx.

C1

Observe that C3 goes from right to left but C3 goes from left to right, so we can
write the parametric equations of C3 as x = x, y = g2 (x), a x b. Therefore


P (x, y) dx =
C3

P (x, y) dx =

P (x, g2(x)) dx.


a

C3

On C2 or C4 , x is constant, so dx = 0 and hence




P (x, y) dx = 0 =
C2

Hence


C1
 b
a

P (x, y) dx +
C3

P (x, g2(x)) dx =
a

Q(x, y) dy =

Equality

P (x, y) dx +
C2

P (x, g1 (x)) dx

=


P (x, y) dx +

P (x, y) dx =
C

P (x, y) dx.
C4

Q
dA can be proved similarly.
x
16.4-5

P (x, y) dx
C4

P
dA.
y

Proof of Theorem 10 in section 16.3. If C is any simple closed path in D and R is


the region that encloses, then Greens Theorem gives



 

Q P
F dr =
P dx + Q dy =
0 dA = 0.
dA =

x
y
C
C
R
R
A curve that is not simple crossed itself at one or more points and can be broken up
into a number of simple curve. We have shown that the line integral of F around

these simple curves are all 0 and, adding these integrals, we see that C F dr = 0

for any closed curve C. Therefore C F dr is independent of path in D, and F is a
conservative vector field.
Homework*. Find the work done by the force field F(x, y) = x2 y i xy 2 j on a
particle that starts at (2, 0) and moves once around the circle x2 + y 2 = 4 oriented
in the counterclockwise direction.
(92 )
Homework. Find the positive-oriented simple closed curve C such that the integral
(y 3 y) dx x3 dy is maximum. Find the maximum value.
(93 )
C

Homework*. Find the integral C (4y + 3x+ sin y) dx+ (3y + 2x+ x cos y) dy, where
C is the circle (x 2)2 + (y 5)2 = 9.
(94 )

Homework*. Compute the line integral C (3xy + 1) dx + (x2 + x) dy, where the
closed curve C is the cardioid given by the polar equation r = 1 + sin , and is
oriented counterclockwise.
(97 )
Homework*. Let P =

y3
(x2 +y 2 )2

and Q = (x2xy
. Let C be the counterclockwise
+y 2 )2

x2 y 2
+ 16
9

= 1, and D be the region inside C but outside the unit circle x2 +y 2 = 1.



  Q
P
(a) Evaluate D x y dA.

ellipse

(b) Evaluate

P dx + Q dy.

(98 )

Homework*. Evaluate the line integral C (x2 y) dx + (1 + y 2) dy, where C is the


loop of the four leaved rose r = cos 2, 4 4 , oriented counterclockwise.
(99 )

Homework. Evaluate the line integral C F dr counterclockwise around the region


x2
bounded by 0 x 1 and x2 y 1, where F(xy) = sin(x3 ) i + 1+y
2 j.
(100 )
Homework. Evaluate
counterclockwise.

r=1cos

(x2 y + y) dx (xy 2 x) dy with the curve oriented


(101 )

Homework. Let D be the bounded region in the first quadrant enclosed by y =

0, x = 1, andy = x with positively oriented


C (i.e. counter clockwise.)
 boundary



1
3
3
3
2
3
3
2
3
Evaluate C 9x y(x + 1) 2 xy (x + 1) 2 dx + 2(x + 1) 2 + 2(y 3 + 1) 2 dy.
(102 )
16.4-6

16.5

Curl and Divergence, page 1091

Curl (), page 1091


Definition 1 (page 1091). If F = P i + Q j + R k is a vector field on R3 and the
partial derivatives of P (x, y, z), Q(x, y, z), and R(x, y, z) all exist, then the curl of
F () is the vector field on R3 defined by






R Q
P
R
Q P
curl F =
i+
j+
k.

y
z
z
x
x
y

 rot F
We introduce the vector differential operator as
=

i+
j+
k.
x
y
z

(1) When it operates on a scalar function, we get the gradient of f :


f =

f
f
f
i+
j+
k.
x
y
z

, y
, z
, then curl F is the
(2) If we think of as a vector with components x
formal cross product of with the vector field F as follows:


i

j
k







P
Q P
R
R Q

i+
j+
k
F = x y z =



y
z
z
x
x
y
P Q R

= curl F.

Theorem 2 (page 1092). If f (x, y, z) is a function that has continuous second order
partial derivatives, then curl(f ) = 0.
Proof. By Clairauts Theorem, we have

i
j k


curl(f ) = (f ) = x y z
f f f

x

Corollary 3. If F is a conservative vector field, then curl F = 0.


Theorem 4 (page 1093). If F is a vector field defined on all of R3 (more generally if
the domain is simply-connected) whose component functions have continuous partial
derivatives and curl F = 0, then F is a conservative vector field.
16.5-1

Divergence (), page 1094


, Q ,
Definition 5 (page 1094). If F = P i + Q j + R k is a vector field on R3 and P
x y
exist, then the divergence of F () is the function of three variables
and R
z
defined by
div F =

P
Q R
+
+
.
x
y
z

(1) curl F is a vector field but div F is a scalar field.


(2) The divergence of F is the dot product of and F: div F = F.
Theorem 6 (page 1094). If F = P i + Q j + R k is a vector field on R3 and P, Q,
and R have continuous second order partial derivatives, then div curl F = 0.
Proof. Using the definition of divergence and curl, we have
div curl F = ( F)
=
=
because the terms cancel in pairs by Clairauts Theorem.
Example 7 (page 1095). Show that the vector field F(x, y, z) = xz i + xyz j y 2 k
cant be written as the curl of another vector field, that is, F 6= curl G.
Solution.

Definition 8 (page 1095).


(a) Define the operator 2 = .
(b) The Laplace operator of a function f is defined as:
f = 2 f = f = (f ) =

2f
2f
2f
+
+
.
x2
y 2
z 2

(c) We can apply the Laplace operator to a vector field F:


2 F = 2 P i + 2 Q j + 2 R k.

16.5-2

Vector forms of Greens Theorem, page 1096


The curl and divergence operators allow us to rewrite Greens Theorem in versions
that will be useful in our later work. We suppose that the plane region D, its
boundary curve C, and the functions P and Q satisfy the hypotheses of Greens
Theorem. Then we consider the vector field F = P i + Q j. Its line integral is
I
I
F dr =
P dx + Q dy,
C

and, regarding F as a vector field on R with third component 0, we have





i
j
k 


Q P


curl F =
=
k.

x
y
z

x
y
P (x, y) Q(x, y) 0
So we can rewrite Greens Theorem in the vector form
I
ZZ
F dr =
(curl F) k dA.
C

Now we will derive a similar formula involving the normal component of F. If C


is given by the vector equation r(t) = x(t) i + y(t) j, a t b, then the unit tangent
vector T(t) and outward unit normal vector n(t) to C are given by
T(t) =

y (t)
x (t)
i
+
j,
|r(t)|
|r (t)|

and n(t) =

y (t)
x (t)
i

j.
|r (t)|
|r (t)|

y
T(t)

n(t)
r(t)

D
C

Figure 1: Unit tangent vector T(t) and unit normal vector n(t) of the curve C : r(t).
So we have
I
Z b
F n ds =
(F n)|r (t)| dt
C
a

Z b
P (x(t), y(t))y (t) Q(x(t), y(t))x (t)
=
|r(t)| dt

(t)|
(t)|
|r
|r
a
Z b
=
P (x(t), y(t))y (t) dt Q(x(t), y(t))x (t) dt
a

Z b
ZZ 
Q
P
dA.
=
+
P dy Q dx =
x
y
a
D
16.5-3

Appendix
Physics meaning of curl and divergence, page 1094 - 1095
The reason for the name curl occurs when F represents the velocity field in fluid
flow. Particles near (x, y, z) in the fluid tend to rotate about the axis that points
in the direction of curl F(x, y, z), and the length of this curl vector is a measure of
how quickly the particles move around the axis.
C

(x, y, z)

Figure 2: Curl of a vector field F.


If curl F = 0 at a point P , then the fluid is free from rotations at P and F is
called irrotational () at P .
If curl F 6= 0, the paddle wheel rotates about its axis.
The reason for the name divergence can be understood in the context of fluid
flow. If F(x, y, z) is the velocity of a fluid or gas, then div F(x, y, z) represents the
net rate of change (with respect to time) of the mass of fluid flowing from the point
(x, y, z) per unit volume.
If div F = 0, then F is said to be incompressible ().
Homework (page 1097). Let f be a scalar field and F a vector field. State whether
each expression is meaningful. If not, explain why. If so, state whether it is a
scalar field or a vector field. (a) curl f . (b) grad f . (c) div F. (d) curl(grad f ). (e)
grad F (f) grad(div F) (g) div(grad f ) (h) grad(div f ) (i) curl(curl F) (j) div(div F)
(k) (grad f ) (div F) (l) div(curl(grad f )).
p

H
x
x2 + y 2 1+y
Homework. Let F =
i + (ex tan1 y) j. Find C F n ds,
2

where C is the cardioid r = 1 + cos .


(95 )




H
2)
tan1 x
i + yln(1+y
j.
Find
F n ds, where C is
Homework. Let F = x+2y1+y
2
2
1+x
C 1
the boundary of the rectangular region R = {(x, y)|1 x 3, 3 y 1}
(96 )

16.5-4

16.6

Parametric Surfaces and Their Areas,


page 1099

Here we use vector functions to describe more general surfaces, called parametric
surfaces, and compute their areas.

Parametric surfaces (), page 1099


We can describe a surface by a vector function r(u, v) of two parameters u and v:
r(u, v) = x(u, v) i + y(u, v) j + z(u, v) k,
which is a vector-valued function defined on a region D in the uv-plane. So x(u, v),
y(u, v), and z(u, v), called the component functions () of r(u, v).
Definition 1 (page 1099). The set of all points (x, y, z) in R3 such that
x = x(u, v),

y = y(u, v),

z = z(u, v),

(1)

where (u, v) varies throughout D, is called parametric surfaces S (). Equations (1) are called parametric equations of S ().
v

z
S

r
(u, v)

y
u

Figure 1: Parametric surface.


Example 2 (page 1101). Find a parameter representation of the following surfaces.
(1) Plane passes through P0 with position vector r0 and contains vectors u and v.
(2) Sphere x2 + y 2 + z 2 = r02 .
(3) Cylinder x2 + z 2 = r02 .
Solution.

16.6-1

Definition 3 (page 1100). If we keep u constant by putting u = u0 , we get a curve


Cv given by r(u0, v). If we keep v constant by putting v = v0 , we get a curve Cu
given by r(u, v0 ). These curves are called grid curves ().
v

z
D

S
(u0 , v0 )

v = v0

Cu

Cv
y

u = u0

Figure 2: Grid curves.


Example 4 (page 1101). Find a parameter representation of the following surfaces.
(1) Elliptic paraboloid z = x2 + 2y 2.
p
(2) The top half of the cone z = 2 x2 + y 2 .

Solution.

Surfaces of revolution (), page 1103


Consider the surface of revolution S obtained by rotating the curve y = f (x), a
x b, about the x-axis, where f (x) 0. Let be the angle of rotation. If (x, y, z)
is a point on S, then
x = x,

y = f (x) cos ,

z = f (x) sin .

(2)

Therefore we take x and as parameters and regard equation (2) as parameter


equations of S. The parameter domain is given by a x b, 0 2.

16.6-2

Example 5 (page 1103). Find parametric equations for the surface generated by
rotating the curve y = sin x, 0 x 2, about x-axis.
Solution.

Tangent planes, page 1103


We now find the tangent plane to a parametric surface S traced out by
r(u, v) = x(u, v) i + y(u, v) j + z(u, v) k
at a point P0 with position vector r(u0 , v0 ).
If we keep v constant by putting v = v0 , then r(u, v0 ) becomes a vector function
of the single parameter u and defines a grid curve Cu lying on S. The tangent vector
to C1 at P0 is obtained by taking the partial derivative of r with respect to u:
ru =

y
z
x
(u0 , v0 ) i +
(u0 , v0 ) j +
(u0 , v0 ) k.
u
u
u

z
P0
D

ru
(u0 , v0 )

v = v0

rv

Cu S Cv

y
u = u0

u
x

Figure 3: Tangent plane is spanned by ru and rv .


If we keep u constant by putting u = u0 , then r(u0, v) becomes a vector function
of the single parameter v and defines a grid curve Cv lying on S. The tangent vector
to C1 at P0 is obtained by taking the partial derivative of r with respect to v:
rv =

x
y
z
(u0 , v0 ) i +
(u0 , v0 ) j +
(u0 , v0 ) k
v
v
v

If ru rv is not 0, then the surface S is called smooth (), and it has no


corners. For a smooth surface, the tangent plane () is the plane that contains
the tangent vectors ru and rv , and the vector ru rv is a normal vector to the
tangent plane.
16.6-3

Surface area, page 1104


Definition 6 (page 1105). If a smooth parametric surface S is given by the equation
r(u, v) = x(u, v) i + y(u, v) j + z(u, v) k,

(u, v) D,

and S is covered just cone as (u, v) ranges throughout the parametric domain D,
then the surface area of S is
ZZ
A(S) =
|ru rv | dA,
D

where
ru =

z
x
y
z
x y
i+ j +
k and rv =
i+
j+
k.
u u
u
v
v
v

Example 7 (page 1105). Find the surface area of a sphere of radius R.


Solution.

Surface area of the graph of a function, page 1106


Example 8 (page 1106). If a surface S is a graph of z = f (x, y), then the parameter
equation of S is
x=x

y=y

We compute

16.6-4

z = f (x, y).

Example 9 (page 1107). For surfaces of revolution, prove that the surface area
formula is consistent with the surface area formula from single-variable calculus.
Solution.

Homework (page 1110).


(a) Find a parametric representation for the torus obtained by rotating about the
z-axis the circle in the xz-plane with center (b, 0, 0) and radius a < b.
(b) Find the surface area of the torus.
16.6-5

(95 )

16.7

Surface Integrals, page 1110

The relationship between surface integrals and surface area is much the same as the
relationship between line integrals and arc length.
Suppose f is a function of three variables whose domain includes a surface S.
First, we will define the surface integral of f over S.

Surface integrals of functions (), page 1110


Suppose that a surface S has a vector equation
r(u, v) = x(u, v) i + y(u, v) j + z(u, v) k,

(u, v) D.

(1) We first assume that the parameter domain D is a rectangle and we divide it
into subrectangles Rij with dimensions u and v. The surface S is divided
into corresponding patches Sij .
(2) We evaluate f at a point Pij in each patch, multiply by the area Sij .
(3) We form the Riemann sum

n
m P
P

f (Pij )Sij .

i=1 j=1

(4) Taking the limit as the number of patches increasing and define the surface
integral of f over the surface S () as
ZZ

f (x, y, z) dS = lim

 :

m,n

m X
n
X

f (Pij )Sij

f (x, y, z) ds =

f (r(u, v))|ru rv | dA.

i=1 j=1

ZZ

Rb
a

f (r(t))|r (t)| dt.

Example 1 (page 1111). Compute the surface integral


unit sphere x2 + y 2 + z 2 = 1.
Solution.

16.7-1

RR

x2 dS, where S is the

Surface integrals have applications. For example, if a thin sheet has the shape
of a surface S and density (x, y, z), then the total mass () of the sheet is
ZZ
m=
(x, y, z) dS.
S

The center of mass () of the sheet is


ZZ
ZZ
1
1
x(x, y, z) dS y =
y(x, y, z) dS
x =
m S
m S

1
z =
m

ZZ

z(x, y, z) dS.

Graph, page 1112


Example 2 (page 1112). Any surface S with equation z = g(x, y) can be regarded
as a parameter surface with parametric equations
x=x

y=y

z = g(x, y).

So we have

Similar formulas apply when we project S onto the yz-plane or xz-plane.


(a) If S is a surface with equation y = h(x, z) and D is its projection onto the
xz-plane, then
s
 2  2
ZZ
ZZ
y
y
f (x, y, z) dS =
f (x, h(x, z), z) 1 +
+
dA.
x
z
S
D
(a) If S is a surface with equation x = k(y, z) and D is its projection onto the
xz-plane, then
s
 2  2
ZZ
ZZ
x
x
+
dA.
f (x, y, z) dS =
f (k(y, z), y, z) 1 +
y
z
S
D
If S is a piecewise smooth surface, that is, a finite union of smooth surfaces
S1 , S2 , . . . , Sn that intersect only along their boundaries, then the surface integral of
f over S is defined by
ZZ
ZZ
ZZ
f (x, y, z) dS.
f (x, y, z) dS + +
f (x, y, z) dS =
S

Sn

S1

16.7-2

Example 3 (page 1113). Evaluate


2

RR

z dS, where S is the surface whose sides S1

are given by the cylinder x + y = 1, whose bottom S2 is the disk x2 + y 2 1 in


the plane z = 0, and whose top S3 is the part of the plane z = 1 + x that lies above
S2 .
Solution.

16.7-3

Oriented surfaces, page 1115


To define surface integrals of vector fields, we need to rule out nonorientable surfaces
such as the Mobius strip. The Mobius strip is a one side surface.

From now on we consider only orientable (two-side) surfaces. We start with


a surface S that has a tangent plane at every point (x, y, z) on S except at any
boundary point. There are two unit normal vectors n1 and n2 = n1 at (x, y, z).
Definition 4 (page 1115). If it is possible to choose a unit normal vector n at every
such point (x, y, z) so that n varies continuously over S, then S is called an oriented
surface () and the given choice of n provides S with an orientation (
). There are two possible orientations for any orientable surface.

(a) For a surface z = g(x, y) given as a the graph of g, we can get the unit normal
vector
j+k
 2 .

g
g 2
1 + x + y

n= r

g
i
x

g
y

The unit normal vector gives the upward orientation of the surface.
(b) If S is a smooth orientable surface given by r(u, v), then the orientation of the
unit normal vector is
n=

ru rv
.
|ru rv |

and opposite orientation is given by n.


16.7-4

Surface integrals of vector fields (), page 1116


Suppose that S is an orientated surface with unit normal vector n, and imagine a
fluid with density (x, y, z) and velocity field v(x, y, z) flowing through S. Then the
rate of flow (mass per unit time) per unit area is v. If we divide S into small patches
Sij , then Sij is nearly planar and so we can approximate the mass of fluid per unit
time crossing Sij in the direction of the normal n by the quantity (v n)A(Sij ),
where , v, and n are evaluated at some point on Sij . By summing these quantities
and taking the limit, we get the surface integral of the function v n over S:
ZZ
ZZ
v n dS =
(x, y, z)v(x, y, z) n(x, y, z) dS
S

and this is interpreted physically as the rate of flow through S.


If we write F = v, then F is also a vector field on R3 and the integral becomes
RR
F n dS. A surface integral of this form occurs frequently in physics, even when
S

F is not v, and is called the surface integral (), or flux integral (


) of F over S.
Definition 5 (page 1117). If F is a continuous vector field defined on an oriented
surface S with unit normal vector n, then the surface integral of F over S is
ZZ
Z
F dS =
F n dS.
S

This integral is also called the flux () of F across S.


(a) The surface integral of a vector field over S is equal to the surface integral of
its normal component over S.
(b) If S is given by a vector function r(u, v), then we have

ZZ 
ZZ
ZZ
ru rv
ru rv
|ru rv | dA
dS =
F(r(u, v))
F dS =
F
|ru rv |
|ru rv |
D
S
S
ZZ
=
F(r(u, v)) (ru rv ) dA
D

(c) If a surface S is given by a graph z = g(x, y), then we can use parameter
equation r(x, y) = x i + y j + g(x, y) k and get
ZZ
ZZ
F dS =
F (rx ry ) = (P i + Q j + R k) (gx i gy j + k) dA
S
D
ZZ
=
(P gx Qgy + R) dA.
D

16.7-5

Example 6 (page 1117). Find the flux of the vector field F(x, y, z) = z i + y j + x k
across the unit sphere x2 + y 2 + z 2 = 1.
Solution.

RR
Example 7 (page 1118). Evaluate S F dS, where F = y i + x j + k and S is the
boundary of the solid region E enclosed by the paraboloid z = 1 x2 y 2 and the
plane z = 0.
Solution.

16.7-6

Appendix
Definition 8 (page 1119). If E is an electric field, then the surface integral
is called the electric flux of E () through the surface S.

RR

EdS

One of the important laws of electrostatics is Gausss Law (), which says
that the net charge enclosed by a closed surface S is
ZZ
Q = 0
E dS,
S

where 0 is a constant (called the permittivity of free space) that depends on the
units used. Therefore, if the vector field F = y i + x j + k represents an electric field,
we can conclude that the charge enclosed by S is Q = 34 0 .
Another application of surface integrals occurs in the study of heat flow. Suppose
the temperature at a point (x, y, z) in a body is u(x, y, z). Then the heat flow is
defined as the vector field
F = Ku,
where K is an experimentally determined constant called the conductively of the
substance. The rate of heat flow across the surface S in the body is then given by
the surface integral
ZZ
ZZ
F dS = K
u dS.
S

RR
Homework. Find the surface integral S y 2z dS, where S is the part of the sphere
p
x2 + y 2 + z 2 = 4 between the cone z = x2 + y 2 .
(90 )
RR
Homework. Find the surface integral x2 +y2 +z 2 =R2 z 4 dS.
(91 )
RR
Homework. Evaluate S (x2 + y 2 )z dS, where S is the upper hemisphere x2 + y 2 +
z 2 = 1, z 0.
(93 )
p
RR
Homework. Evaluate S 1 + x2 + y 2 dS, where S is the helicoid parameterized
as r(u, v) = u cos v i + u sin v j + v k, 0 u 1, 0 v .
(97 )
RR
Homework. Evaluate the surface integral S (x2 + y 2 )z dS, where S is the part of
the plane z = 4 + x + y that lies inside the cylinder x2 + y 2 = 4.
(101 )
RR
Homework. Evaluate S F dS, where F = (x2 + y 2)k and S is the surface (z +
xy)3 = x2 + y 2 lies in 1 x2 + y 2 4 with upward orientation.
(90 )
RR
Homework. Evaluate S F dS, where F = xz i + yz j + z 2 k, and S is the part of
(91 )
sphere x2 + y 2 + z 2 = 1, z 12 with upward orientation.
16.7-7

Homework. Let F = x i + y j + z k and S be the surface generated by the line


RR
segments from the point (cos t, sin t, t) to (0, 0, t), 0 t 2. Find S F dS.
(94 )
RR
Homework. Evaluate S F dS, where F (x, y, z) = 2y 2 j z k and S is the surface
of the solid enclosed by y = x2 + z 2 and y = 1 with outward normal vector.
(98 )
Homework. Find the flux of F = yz i xz j + (x2 + y 2) k upward through the
surface r(u, v) = eu cos v i + eu sin v j + u k, where 0 u 1, 0 v .
(100 )
Homework. Let F(x, y, z) = 3xy 2 i + y 3 j + ex
2

2 +y 2

k. Let S be the part of the

surface z = 1 x y that lies above xy-plane oriented upwards (that is, with
RR
normal having k-component 0). Calculate the flux S F dS of F across S. Note
(102 )

that S is not closed.

16.7-8

16.8

Stokes Theorem, page 1110

Stokes Theorem can be regarded as a generalization of Greens Theorem.


Greens Theorem relates a double integral over a plane region D to a line
integral around its plane boundary curve.
Stokes Theorem relates a surface integral over a surface S to a line integral
around the boundary curve (space curve) of S.
y

x
x

Figure 1: Greens Theorem (left) and Stokes Theorem (right).


Figure 1 (right) shows an oriented surface with unit normal vector n. The
orientation of S induces the positive orientation of the boundary curve C (
) shown in the figure.

 n C
Stokes Theorem (page 1122). Let S be an oriented piecewise smooth surface
that is bounded by a simple, closed, piecewise smooth boundary curve C with positive orientation. Let F be a vector field whose components have continuous partial
derivatives on an open region on R3 that contains S. Then
Z
ZZ
F dr =
curl F dS.
C

(a) Since

F dr =

F T ds and

ZZ

curl F dS =
S

ZZ

curl F n dS,

Stokes Theorem says that the line integral around the boundary curve of S
of the tangential component of F is equal to the surface integral over S of the
normal component of the curl of F.
(b) Greens Theorem is the special case of Stokes Theorem, where S is flat and
lies in the xy-plane with upward orientation, and the unit normal is k, so

Z
Z
ZZ 
Q P
dA

F dr =
P dx + Q dy =
x
y
C
C
D
ZZ
ZZ
=
curl F dS =
(curl F) k dA.
S

16.8-1

R
Example 1 (page 1124). Evaluate C F dr, where F(x, y, z) = y 2 i + x j + z 2 k
and C is the curve of intersection of the plane y + z = 2 and the cylinder x2 + y 2 = 1.
(Orient C to be counterclockwise when viewed from above.)
Solution.

RR
Example 2 (page 1125). Use Stokes Theorem to compute the integral S curl F
dS, where F(x, y, z) = xz i+yz j+xy k and S is the part of the sphere x2 +y 2 +z 2 = 4
that lies inside cylinder x2 + y 2 = 1 and above the xy-plane.
Solution.

 ,

RR

16.8-2

curl F dS ()

Example 3. Suppose that S consists of the part of cylinder x2 + y 2 = 1, 0 z 1


RR
and the lid x2 + y 2 1, z = 1. Let F = y i + x j + x2 k. Evaluate S F n dS.
(90 )
Solution.

Example 4. Suppose that C is the circle that is the intersection of the plane passing
through the origin and the sphere x2 + y 2 + z 2 = 4. Let F = z i + x j + y k. Find
R
the equation of the plane such that the line integral C F dr attains the maximum.
(91 )

Solution.

16.8-3

Appendix
Proof of a special case of Stokes Theorem. We assume that the equation of S is
z = z(x, y), (x, y) D, where z(x, y) has continuous second order partial derivatives
and D is a simple plane region whose boundary curve C1 corresponds to C. If the
orientation of S is upward, then the positive orientation of C corresponds to the
positive orientation of C1 . So we have
r(x, y) = x i + y j + z(x, y) k
z
z
rx (x, y) = 1 i + 0 j +
k
ry (x, y) = 0 i + 1 j +
k
x
y
z
z
rx ry (x, y) = i
j + 1 k.
x
y
z

n
z = z(x, y)
S
C
y
D

C1

Figure 2: Proof of Stokes Theorem.


Let F = P i + Q j + R k. We first compute
ZZ
curl F dS
S





ZZ  
P
R z
Q P
R Q z
dA,

y
z x
z
x y
x
y
R

where the partial derivative of P, Q, and R are evaluated at (x, y, z(x, y)). On the
other hand, if x = x(t), y = y(t), a t b is a parametric representation of C1 , then
a parametric representation of C is x = x(t), y = y(t), z = z(x(t), y(t)), a t b.
By Greens Theorem, we have

Z
Z b
dx
dy
dz
F dr =
P
dt
+Q +R
dt
dt
dt
C
a


Z b
dy
z dx z dy
dx
dt
+Q +R
+
=
P
dt
dt
x dt
y dt
a

 

Z b 
z dx
z dy
=
P +R
dt
+ Q+R
x dt
y dt
a



Z 
z
z
dx + Q + R
dy
=
P +R
x
y
C



ZZ  

z
=
Q+R
dx
P +R
dA.
x
y
y
x
D
16.8-4

Using the Chain Rule carefully, that is, P, Q, and R are functions of x, y, and z and
z itself a function of x and y, we will get

Z
ZZ 
2z
Q Q z R z R z z
+
+
+
+R
F dr =
x
z x
x y
z x y
xy
C
D


P z R z R z z
2z
P
dA
+
+
+
+R

y
z y
y x
z y x
yx





ZZ  
R Q z
P
R z
Q P
=

dA.

y
z x
z
x y
x
y
D
Homework*. Suppose that S consists of the part of cylinder x2 + y 2 = 1, 0 z 1
RR
and the lid x2 + y 2 1, z = 1. Let F = y i + x j + x2 k. Evaluate S F n dS.
(90 )
Homework. Suppose that S is the part of the paraboloid z = x2 + y 2 that lies
below the plane z = 1 and oriented upward. Let F = x2 y i xy 2 j + zex k. Evaluate
RR
F n dS.
(91 )
S

Homework*. Suppose that C is the circle that is the intersection of the plane
passing through the origin and the sphere x2 + y 2 + z 2 = 4. Let F = z i + x j + y k.
R
Find the equation of the plane such that the line integral C F dr attains the
maximum.
(91 )
H
Homework*. Evaluate C FT ds, where F(x, y, z) = xy i+yz j+zx k and C is the
triangle with vertices (1, 0, 0), (0, 1, 0), (0, 0, 1) oriented counterclockwise as viewed
from above.
(92 )
Homework*. Let F = x2 yz i + yz 2 j + z 3 exy k.
(a) Find the curl of F.
RR
(b) Evaluate S curl F n dS, where S is the part of sphere x2 + y 2 + z 2 = 5, z 1
that lies above the plane z = 1, oriented upward.
(92 )
RR
Homework. Evaluate S curl F n dS, where F = yz 5 i + sin(xyz) j + x k and S is
the part of paraboloid y = 1 x2 z 2 that lies to the right of the xz-plane, oriented
in the direction of the positive y-axis.
(93 )
Homework*. Let a2 + b2 + c2 = 1. Let C be the curve that is the intersection
of the plane ax + by + cz = 54 and the sphere x2 + y 2 + z 2 = 1, and the curve is
R
oriented clockwise as viewed from the center. Evaluate C F dr, where F(x, y, z) =
(2y z) i + (3x + 2z) j + (2x y) k.
(93 )
16.8-5

RR

Homework*. Evaluate

curl F dS, where F(x, y, z) = y i + z j + x k and S is

the upper hemisphere x + y + z 2 = 1, z 0, oriented upward.


RR

Homework*. Evaluate

(94 )

curl F ndS, where F = y i + xz j + (x + 2y) k and S is

the part of the plane z = x + 2 lies inside the cylinder x2 + y 2 = 1, oriented upward.
(95 )
Homework. Let closed curve C be the intersection of the plane x + y + z = 1
and the cylinder x2 + y 2 = 9, oriented counterclockwise as viewed from above, and
H
F(x, y, z) = x2 z i + xy 2 j + z 2 k. Find C F dr.
(97 )

RR
2
Homework*. Compute the integral S curl F dS, where F(x, y, z) = (ez + y) i +
(4z y) j + (8x sin y) k and S is the part of z = 4 x2 y 2 above the xy-plane with
(98 )

orientation given by the upward unit normal vector.


Homework*.

RR
2
(a) Find curl F, where F = y 3 i+x3 j +ez k and evaluate S curl FdS, where S
is the portion of the surface of z = x3 +y 3 3xy within the cylinder x2 +y 2 = a2
with upward normal.
R
(b) Use Stokes Theorem to evaluate C F dr, where C is the boundary of S
oriented counterclockwise when viewed from above.
(99 )
Homework. Evaluate

F dr around the curve

r(t) = cos t i + sin t j + (sin t + cos t) k,


where F = (ex y 3) i + (ey + x3 ) j + (ez + x + y) k.

0 t 2,
(100 )

Homework*. Let F = (2x y) i + (2y + z) j + x2 y 2z 2 k and let S be the upper half


2

z
= 1. Find the flux of curl F in the direction of the upper
of the ellipsoid x4 + y9 + 25
unit normal n (pointing away from the origin).
(101 )

Homework*. Let S be the part of the sphere x2 + y 2 + (z 2)2 = 8 that lies


above the xy-plane and that has outward normal (i.e. with k-component 0). Let
RR
F(x, y, z) = y 3 cos xz i + x3 eyz j exyz k. Find S curl F dS.
(102 )

16.8-6

16.9

The Divergence Theorem, page 1127

In section 16.5, we have discussed the vector version of the following line integral

ZZ
Z
ZZ 
Q
P
dA =
div F dA,
+
F n ds =
x
y
D
C
D
where C is the positively oriented boundary curve of the plane region D. Here we
will generalize this result to vector fields on R3 , and this is called the Divergence
Theorem ().
y

n
S

n
r(t)

D
C

n
x

Figure 1: Normal line integral (left) and the Divergence Theorem (right).

The Divergence Theorem (page 1129). Let E be a simple solid region and let
S be the boundary surface of E, given with positive outward orientation. Let F be
a vector field whose component functions have continuous partial derivatives on an
open region that contains E. Then
Z
ZZZ
F dS =
div F dV.
S

The Divergence Theorem relates the integral of a derivative of a function (div F)


over a region to the integral of the integral of the origin function F over the boundary
of the region.
Example 1 (page 1131). Find the flux of the vector field F(x, y, z) = z i + y j + x k
over the unit sphere x2 + y 2 + z 2 = 1.
Solution.

16.9-1

RR
2
Example 2 (page 1131). Evaluate S FdS, where F(x, y, z) = xy i+(y 2 +exz ) j+
sin(xy) k, and S is the surface of the region E bounded by the parabolic cylinder
z = 1 x2 and the planes z = 0, y = 0, and y + z = 2.
Solution.

Example 3. Let

 3


p

z
x2 +y 2
1
2
2
2
4
k.
e
F(x, y, z) = xy + y + z i + tan x + x y j +
3
RR
Find S F dS, where the surface S is the top half of the sphere x2 + y 2 + z 2 = 1
with the unit normal vectors pointing away from the origin.
(97 )
Solution.

16.9-2

Example 4. Let S be the sphere x2 + y 2 + z 2 = 1.


(a) Find a vector field such that F n = x4 + y 4 + z 4 on the sphere S.
(b) Find
Solution.

RR

(x4 + y 4 + z 4 ) dS.

(94 )

General version of the Divergence Theorem, page 1132


Lets consider the region E that lies between the closed surfaces S1 and S2 , where
S1 lies inside S2 .

n2
n1

S1

n1

S2
Figure 2: General version of the divergence theorem.
Let n1 and n2 be outward normals of S1 and S2 . Then the boundary surface of
E is S = S1 S2 and its normal n is given by n = n1 or n = n2 on S2 . Applying
the Divergence Theorem on S, we get
ZZZ
ZZ
ZZ
div FdV =
F dS =
F n dS
E
S
S
ZZ
ZZ
ZZ
ZZ
F dS.
F dS +
F n2 dS =
F (n1 ) dS +
=
S1

S2

16.9-3

S1

S2

Example 5 (page 1132). Show that the electric flux of E through any closed surface
RR
S2 that encloses the origin is S2 E dS = 4Q.

Solution. We let S1 be a small sphere with radius r0 and center the origin. For the

electric field E(x) =


ZZ

E dS =
S2

Q
x,
|x|3

ZZ

we have div E = 0. So

E dS +
S1

ZZZ

div E dV =
E

ZZ

E dS =

ZZ

E n dS.

S1

S1

We can compute the surface integral over S1 because S1 is a sphere. The normal
vector at x is

x
.
|x|

Therefore
Q
x
En=
|x|3

x
|x|4

Q
Q
Q
xx=
= 2.
2
|x|
|x|
r0

Thus we have
ZZ
ZZ
ZZ
Q
Q
Q
E dS = 2
E dS =
1 dS = 2 Area(S1 ) = 2 4r02 = 4Q.
r0
r0
r0
S1
S2
S1

Appendix
Proof of the Divergence Theorem for simple solid regions. Let F = P i + Q j + R k.
+ Q
+ R
, so
Then div F = P
x
y
z
ZZZ

divF dV =

ZZZ

P
dV +
x

ZZZ

Q
dV +
y

ZZZ

R
dV.
z

If n is the unit outward normal of S, then we have


ZZ
ZZ
ZZ
F dS =
F n dS =
(P i + Q j + R k) n dS
S
S
Z ZS
=
P i n dS + Q j n dS + R k n dS.
S

So, to prove the Divergence Theorem, it suffices to prove the following equations:
ZZ
ZZZ
ZZ
ZZZ
P
Q
P i n dS =
dV
Q j n dS =
dV
S
E x
S
E y
ZZ
ZZZ
R
dV.
(1)
R k n dS =
S
E z
Here we only prove (1) and E is a type z region:
E = {(x, y, z)|(x, y) D, u1(x, y) z u2 (x, y)},

16.9-4

By the Fundamental Theorem of Calculus, we have


ZZZ

ZZ

R
dV =
z

ZZ

z=u2 (x,y)
z=u1 (x,y)

R
(x, y, z) dz
z

dA

(R(x, y, u2(x, y)) R(x, y, u1(x, y))) dA.


D

On the other hand, the boundary surface S consists of three pieces: the bottom surface S1 , the top surface S2 , and a vertical surface S3 , which lies above the boundary
curve of D.
z
S2 : z = u2 (x, y)
S3 E

S1 : z = u1 (x, y)
y

D
x
Figure 3: Proof of the Divergence Theorem.
We write
ZZ
ZZ
ZZ
ZZ
R k n dS.
R k n dS +
R k n dS +
R k n dS =
S

S2

S1

S3

On S3 , we have k n = 0, because k is vertical and n is horizontal, and so


ZZ
ZZ
0 dS = 0.
R k n dS =
S3

S3

The equation of S2 is z = u2 (x, y), (x, y) R, and the outward normal n points
upward, so we have
ZZ
ZZ
R k n dS =
R(x, y, u2(x, y)) dA.
D

S2

On S1 we have z = u1 (x, y), but here the outward normal n points downward, so
we multiply by 1:
ZZ
ZZ
R k n dS =
R(x, y, u1(x, y)) dA.
D

S1

Therefore,
ZZ

R k n dS =

ZZ

(R(x, y, u2(x, y)) R(x, y, u1(x, y))) dA.

16.9-5

Homework*. Evaluate the surface integral

RR

Fn dS, where F = 3xy 2 i+3x2 y j+

z 3 k, S is the sphere x2 + y 2 + z 2 = 1, and n is the unit normal vector pointing


outward.
(90 )
RR
Homework. Evaluate the surface integral S Fn dS, where F = 2y 2 i+xez j+3 k,
S is the upper hemisphere x2 + y 2 + z 2 = 1, z 0, and n is the unit normal pointing
upward.
(91 )
Homework.
(a) Find the flux integral of the vector field F(x2 4xy) i 6yz j + 12z k on the
surface of the box 0 x a, 0 y b, 0 z 1.
(91 )

(b) Find a, b such that the flux integral attains maximum.

RR
Homework*. Evaluate S F n dS, where F = x2 i + xy j + z k and S is the surface
of the solid bounded by the paraboloid z = x2 + y 2 and the plane z = 1.
(92 )
Homework. Evaluate

RR

F dS, where F(x, y, z) = xy i

y2
2

j + z k, and S =

S1 S2 S3 , S1 = {(x, y, z)|z = 4 3x2 3y 2 , 1 z 4}, S2 = {(x, y, z)|x2 + y 2 =


1, 0 z 1}, S3 = {(x, y, z)|x2 + y 2 1, z = 0}.
(93 )
Homework*. Let S be the sphere x2 + y 2 + z 2 = 1.
(a) Find a vector field such that F n = x4 + y 4 + z 4 on the sphere S.
(b) Find

RR

(x4 + y 4 + z 4 ) dS.

(94 )

Homework*. Let S be the surface of the box E : {(x, y, z)|0 x a, 0 y


b, 0 z 1}. Let F = (x2 4xy) i 6yz j + 12 k. Find the values a, b such that
RR
the flux S F n dS attains its maximum. Find the maximum values.
(95 )
Homework*. Suppose that S is the surface of the solid bounded by the paraboloid
z = x2 + y 2 and the planes z = 0 and z = 1. Let F = 4x i + 4y j + 2 k. Evaluate the
outward flux of F on S.
(96 )
Homework*. Let S be the surface of the solid E + {(x, y, z)|1 x2 + y 2 2, 1
p
z 2}. Suppose that F = ln(x2 + y 2 ) i xyz j + z 2 x2 + y 2 k. Compute the
(96 )

outward flux of F on S.

16.9-6

Homework*. Let

 3


p

z
2 +y 2
x
1
2
2
e
k.
F(x, y, z) = xy + y 2 + z 4 i + tan x + x y j +
3
(a) Find div F.

RR
(b) Find S FdS, where the surface S is the top half of the sphere x2 +y 2 +z 2 = 1
with the unit normal vectors pointing away from the origin. Warning: S is
(97 )

not a closed surface!


Homework*. Evaluate

RR

FdS, where F(x, y, z) = 2y 2 jz k and S is the surface

of the solid enclosed by y = x2 + z 2 and y = 1 with outward normal vector.


(98 )
RR
Homework*. Find the surface integral S F dS, where F = x2 y i + xy 2 j + 2xyz k
and S = {(x, y, z) : x2 + y 2 + z 2 = 2, x 0, y 0, z 1} with the normal pointing
(99 )

upwards.

Homework*. Let S1 be the upper hemisphere x2 + y 2 + z 2 = 1 with z 0, S2 the


unit disk x2 + y 2 1 on xy-plane and V the region enclosed by S1 S2 . Endow
S1 S2 with outward normal. Let F = xz 2 i + (yx2 + ez ) j + (y 2z + cos(x2 + y 2 )) k.
(a) Find div F and evaluate

RRR

div F dV .

(b) Use Divergence Theorem to evaluate

Homework*. Evaluate the integral

RR

RR

RR
F

dS
and
then
find
F dS.
S1 S2
S1
(99 )

F n dS, where
2

F = (x + y) i + (y + ez ) j + (2x sin y + 2z) k,


S is the surface of the region D inside the sphere x2 + y 2 + z 2 = 4 and the cylinder
x2 + y 2 = 1, and n is the unit outward normal of S.
(100 )
Homework*. Evaluate the flux of
2

F(x, y, z) = (z x + y z) i +



1 3
y + z tan x j + x2 z + 2y 2 + 1 k
3

across S: the upper half sphere x2 + y 2 + z 2 = 1, z 0 with normal pointing away


(101 )

from the origin.

16.9-7

16.10

Summary, page 1135

The main results of this chapter are all higher dimensional versions of the Fundamental Theorem of Calculus. In each case, we have an integral of a derivative
over a region on the left side, and the right side involves the values of the original
function only on the it boundary of the region.
Fundamental Theorem of Calculus. Suppose F (x) is continuous on [a, b]. Then
Z b
F (x) dx = F (b) F (a).
a

Fundamental Theorem of Line Integrals. Let C : r(t), a t b be a smooth


curve. Let f (x) be a function whose gradient f (x) is continuous on C. Then
Z
f dr = f (r(b)) f (r(a)).
C

r(a)
a

r(b)

Figure 1: Fundamental theorem of calculus and line integrals.


Greens Theorem. Let C be a positively oriented, piecewise smooth, simple closed
where
curve in the plane and let D be the region bounded by C. If P, Q C 1 (D),
D is open, then
D

ZZ 
I
Q P
dA =
P dx + Q dy.

x
y
D
C
Stokes Theorem. Let S be an oriented piecewise smooth surface that is bounded
by a simple, closed, piecewise smooth boundary curve C with positive orientation.
Let F be a C 1 vector field in an open region S S. Then
ZZ
Z
curl F dS =
F dr.
S

n
D

n
S

Figure 2: Greens theorem and Stokes theorem.

16.10-1

The Divergence Theorem. Let E be a simple solid region and let S be the boundary surface of E, given with positive outward orientation. Let F be a C 1 vector field
in an open region S E. Then
ZZZ
ZZ
div F dV =
F dS.
E

n
S
n

n
Figure 3: The divergence theorem.

16.10-2


(1) C , r(t) = x(t) i + y(t) j + z(t) k, a t b,
Z
Z b
f (x, y, z) ds =
f (r(t))|r(t)| dt
C
a
Z b
p
=
f (x(t), y(t), z(t)) (x (t))2 + (y (t))2 + (z (t))2 dt.
a

(2) C , C z(t) 0

(3) : f 1; f = ; (moment) f = x, y, z ;
(4) ds (),

(1) C , r(t) = x(t) i + y(t) j + z(t) k, a t b; r(a)


, r(b) , T(t) F(x, y, z) =
P (x, y, z) i + Q(x, y, z) j + R(x, y, z) k R3 ,
Z
Z
Z b
Z

F T ds =
F dr =
F(r(t)) r (t) dt =
P dx + Q dy + R dz
C
C
a
C
Z b
=
(P (x(t), y(t), z(t))x (t) + Q(x(t), y(t), z(t))y (t) + R(x(t), y(t), z(t))z (t)) dt.
a

(2) C , C z(t) 0
(3) : (work done by the force F); (circulation)
(4) dr ,
() C , F C n :
Z
Z
Z b

F n ds = (F n)|r (t)|dt =
Q dx + P dy.
C

(1) S , r(u, v) = x(u, v) i + y(u, v) j + z(u, v) k, D =


{(u, v)|a u b, c v d},
ZZ
ZZ
f (x, y, z) dS =
f (r(u, v))|ru rv | dA.
S

(2) S , S z(u, v) 0
(3) : f 1; f = ; (moment) f = x, y, z ;
(4) dA (),
16.11-1


(1) S , r(u, v) = x(u, v) i + y(u, v) j + z(u, v) k, D =
{(u, v)|a u b, c v d} F(x, y, z) = P (x, y, z) i + Q(x, y, z) j +
R(x, y, z) k R3 ,
ZZ
ZZ
ZZ
F n dS =
F dS =
F(r(u, v)) (ru rv )(u, v) dA.
S

(2) S , S z(u, v) 0
(3) : (flux integral)
(4) dS = n dS , n
(5) , n ; , n

: () ()
(1)

f (x) dx = f (b) f (a)

f dr = f (r(b)) f (r(a))

(2)

(3)
(4)

ZZ

curl F dS =
S

ZZZ

div F dV =

F dr

Green, Stoke ()

ZZ

F dS

()

( )
(1) Green ():

I
I
ZZ
ZZ 
Q P
G.T.
dA =
P dx + Q dy =
F dr.

curl F k dA =
x
y
C
C
D
D
(2) Green ():

ZZ
ZZ 
I
I
P
Q
G.T.
div F dA =
dA =
Q dx + P dy =
F n ds.
+
x
y
D
D
C
C
ZZ
ZZ
ZZ
ZZ
(3)
curl F dS =
curl F n dS =
( F) dS =
( F) n dS.
S

(4)

ZZZ

div F dV =

ZZZ

F dV .

16.11-2


(1) F (conservative vector field): f f = F
(2) :
, : curl F = curl (f ) = 0


(3) : Stoke : S C (),
ZZ
I
I
curl F dS =
F dr =
f dr = f (r(b)) f (r(a)) = 0.
S

(4) F : curl F = 0 ,
F = P i + Q j:
F = P i + Q j + R k:

Q
P
=

x
y
Q P
R Q
P
R
=
,
=
,
=

y
z z
x x
y

(5) (simply connected), curl F = 0, F

(1) , , Green
(2) , , ,
Stoke
(3) , ,
(4) F , F1 F = F1 + F2 ,

F1

: r(t), r(u, v)
(1)

(2)

ZZ

F dr =

F(r(t)) r (t) dt.

F dS =
S

ZZ

F(r(u, v)) (ru rv )(u, v) dA.

16.11-3

16.12

1+ 212 + 312 + + n12 + =

1 +

1
22

1
32

+ +

1
n2

2
6

+ ;

(Euler) 1734 (),

2
6

,
,
:
Z x=1Z y=1
Z x=1Z y=1
f (x, y) dy dx =
x=0

y=0

x=0

y=0

dy dx = lim
t1
1 xy

x=tZ y=t

x=0

y=0

1
dy dx
1 xy

: (improper integral)
, ,

X
X
1
f (x, y) =
=
(xy)n =
xn y n ,
1 xy
n=0
n=0

|xy| < 1 ,

!
Z x=1Z y=1 X
Z x=1Z y=1

1
dy dx =
xn y n dy dx
x=0 y=0
x=0 y=0 1 xy
n=0
!
 y=1 !

Z x=1 X
Z y=1
Z x=1 X


1
n+1
n
n
n
dx
y
=
x
y dy dx =
x

n+1
x=0
y=0
x=0
y=0
n=0
n=0

 x=1 X
Z 1

X
X
X

1
1
1
1
1
n
n+1
=
=
x dx =
x
=


2
2
n
+
1
n
+
1
n
+
1
(n
+
1)
n
0
x=0
n=0
n=0
n=0
n=1
, x = u v, y = u + v,





(x, y)
1 1

= 2,
(u, v) = det

1 1

R = {(x, y)|0 x 1, 0 y 1} u v = 0, u v =

1, u + v = 0 u + v = 1 S
y

v
v=u

1
S

R
1

v = u1
u
v = u + 1

v = u

Figure 1: x = u v y = u + v R S
16.12-1

x=1Z y=1

x=0

y=0

u= 21

u=0

1
dy dx
1 xy

v=u

v=u

2
dv du +
1 u2 + v 2

u=1
u= 21

v=u+1

v=u1

2
dv du = I + II
1 u2 + v 2

1
1
dx =
2
2
a +x
a

1
1+

I=

=4

u= 21
u=0

1
2

 d
x 2
a

x
a

x
1
+ C,
tan1
a
a


 v=u

2
v
1

du
tan
1 u2
1 u2 v=u


1
u
1

tan
du
1 u2
1 u2


u = sin , du = cos d, = 6 , = 0,
I=4

Z
II =
=4

1
1
2

sin
cos

cos d = 4

 =
1
1 2 6
= 2
d = 4
2
18
=0



 v=u+1

2
v
1

du
tan
1 u2
1 u2 v=u1




Z 1
1 sin
1u
1
1
1
1

cos d
tan
tan
du = 4
1 cos
cos
1 u2
1 u2
2

u=1

u= 21

1
tan1
cos



sin2 2 + cos2 2 2 sin 2 cos 2
cos 2 sin 2
1 tan 2
1 sin

,
=

=
=
= tan
cos
4 2
cos2 2 sin2 2
cos 2 + sin 2
1 + tan 2

II = 4

4 2

x=1Z y=1

x=0

y=0

 =
i = 2
h
1

1 2 2
2
= = 2
d = 4

4
4
9
= 6
=


1
1
2
1
dy dx = I + II = 2 + 2 =
1 xy
18
9
6

16.12-2

Calculus A (1) Homework 1 ANSWER


1.3

New Functions from Old Functions

Homework 1. Starting from f (x) = sin x, x [0, 2], plot the function g(x) =
sin(2x + ).
Solution. See the Figure 1. The black curve is the graph of f (x) = sin x. To get
the graph of g(x), first we shift the graph of f (x) a distance units to the left.
Denote the new function f1 (x). Next, we shrink the graph of f1 (x) horizontally by
a factor of 2. It turns out that the graph of g(x) is the red curve.
y
1

1
Figure 1: f (x) = sin x and g(x) = sin(2x + ).

1.6

Inverse Functions and Logarithms

Homework 2. Find a formula for the inverse of the function.

(a) f (x) = 1 + 2 + 3x.


(b) f (x) = e2x1 .
(c) f (x) = x2 x, x 21 .
(d) f (x) =

ex
.
1+2ex

Solution.
(a) Let y = 1 +

2 + 3x. We want to solve x in terms of y. So we get

y1=

2 + 3x (y 1)2 = 2 + 3x x =

Hence the inverse function of f (x) = 1 +


1

(y 1)2 2
.
3

2 + 3x is f 1 (x) =

(x1)2 2
.
3

(b) Let y = e2x1 . We want to solve x in terms of y. So we get


y = e2x1 ln y = 2x 1 x =

1 + ln y
.
2

Hence the inverse function of f (x) = e2x1 is f 1 (x) =

1+ln x
.
2

(c) Let y = x2 x, x 21 . We want to solve x in terms of y. So we get


2

x xy = 0 x=

1+ 1+4y
.
2

1+ 1+4x
.
2

Since x 12 , we choose x =
x2 x is f 1 (x) = x =
(d) Let y =

ex
.
1+2ex

1 + 4y
.
2

Hence the inverse function of f (x) =

We want to solve x in terms of y. So we get

ex
y
y=
y(1 + 2ex ) = ex ex =
x = ln
x
1 + 2e
1 2y
Hence the inverse function of f (x) =

ex
1+2ex

y
1 2y

x
is f 1 (x) = ln 12x
.

Homework 3. Find the exact value of each expression.


(a) sin1 (

2
)
2

(c) tan1

))
(e) sin1 (sin( 7
3

(d) tan(tan1 (10))

1
3
1 3
(f) sin(2 sin ( 5 )).

(b) cos1 (1)

Solution.
(a) sin1 (

2
)
2

= 4 .

(b) cos1 (1) = .


 
1 1
(c) tan
= 6 .
3

(d) Let = tan1 (10), then tan = 10 and 2 < < 2 . Hence tan(tan1 (10)) =
tan = 10.
(e) Since sin

7
3

= sin

3
,
2

(f) Let = sin1 ( 53 ), then sin =

we have sin

3
5

(sin( 7
))
3

= sin

 
3
2

= 3 .

and 2 < < 2 , which implies cos = 45 . So

sin(2 sin1 ( 35 )) = sin(2) = 2 sin cos = 2 35

4
5

24
.
25

2.3

Calculating Limits Using the Limit Laws

Homework 4. Find the limit lim


x0

(94 )

|2x1||2x+1|
.
x

Solution. Since near x = 0 (for example, we only need to consider x ( 41 , 41 )),


|2x 1| = (2x 1) and |2x + 1| = 2x + 1, we have
lim
x0

|2x 1| |2x + 1|
(2x 1) (2x + 1)
4x
= lim
= lim
= lim 4 = 4.
x0
x0
x0
x
x
x

Homework 5. Evaluate the limit lim


x2

x 2x
.
2
|x 4|

(101 )

Solution. For 0 < x < 2, we know |x2 4| = (x2 4) = 4 x2 , so

x 2x
x 2x
x2 2x

lim 2
= lim
=
lim
x2
x2
x2 (4 x2 )(x +
|x 4|
4 x2
2x)
x
1
x(x 2)

= lim
= lim
= .
x2 (x + 2)(x 2)(x +
8
2x) x2
(x + 2)(x + 2x)
Homework 6. Find the limit lim
x0

sin(|x|)
x

and lim+
x0

sin(|x|)
.
x

(101 )

Solution.
sin(x)
sin x
sin x
sin(|x|)
= lim
= lim
= lim
= 1.
x0
x0
x0
x0
x
x
x
x
sin x
sin(|x|)
= lim+
= 1.
lim+
x0
x0
x
x
lim

sin 3x
.
x0 |3x2||x+2|

(91 )

Homework 7. Find the limit lim

Solution. When x is close to 0, we have |3x 2| = (3x 2) and |x + 2| = x + 2,


so
sin 3x
sin 3x
sin 3x
= lim
= lim
x0 |3x 2| |x + 2|
x0 (3x 2) (x + 2)
x0 4x
3
3
sin 3x
= lim
= .
4 x0 3x
4
lim

Homework 8. Find the limit lim


x0
1cos x
x2
x0

Solution. Remark that lim


tan x sin x
= lim
x0
x0
x3
lim

sin x
cos x

tan xsin x
.
x3

(Hint: tan x =

2 sin2
x2
x0

= lim

x
2

sin x
.)
cos x

2 x
1 sin ( 2 )
x 2
2
(
)
x0
2

= lim

(92,93 )

= 21 . So we have

sin x
sin x 1 cos x
1
1
1
=
lim

=
1

=
.
x0 cos x
x3
x
x2
2
2

cos x1
.
x0 sin(x sin x)

(101 )

Homework 9. Find the limit lim


Solution.

x sin x
x
1
1
cos x 1
cos x 1
= lim

1
=

.
x0
x0 sin(x sin x)
x2
sin(x sin x) sin x
2
2
lim

Homework 10. Find the limit lim


x0

cos(sin x)1
.
tan2 x

(101 )

Solution. Let y = sin x, then y 0 as x 0, so


cos(sin x) 1
cos(sin x) 1
cos(sin x) 1
= lim
cos2 x = lim
lim cos2 x
2
2
x0
x0
x0
x0
tan x
sin x
sin2 x
2 y
2 y
2
sin
sin
1
1
cos y 1
2
2
lim cos2 x = lim
1 = lim
= lim
y 2 = .
2
2
x0
y0
y0
y0
y
y
2 (2)
2
lim

1sin2 (ax)cos(ax)
.
2
x0 1+sin (bx)cos(bx)

(101 )

Homework 11. Find the limit lim


Solution.

) sin2 (ax)
2 sin2 ( ax
1 sin2 (ax) cos(ax)
2
lim
= lim
x0 1 + sin2 (bx) cos(bx)
x0 2 sin2 ( bx ) + sin2 (bx)
2
a2
2

x0 b2
2

= lim

sin2 ( ax
)
2
2
( ax
)
2
sin2 ( bx
)
2
( bx
)2
2

a2

sin2 (ax)
(ax)2

+ b2

sin2 (bx)
(bx)2

a2
2
b2
2

a2
+ b2

a2
.
3b2

Homework 12. Is there a number a such that


3x2 + ax + a + 3
x2
x2 + x 2
lim

exists? If so, find the value of a and the value of the limit.
Solution. Notice that

3x2 +ax+a+3
x2 +x2

3x2 +ax+a+3
.
(x+2)(x1)

3x2 +ax+a+3
x2 +x2
x2
2

If the limit lim

exists,

the numerator must hace a factor (x + 2). Hence the solution of 3(2) + a(2) +
a + 3 = 0 is a = 15, and 3x2 + 15x + 15 + 3 = 3(x2 + 5x + 6) = 3(x + 2)(x + 3). So
3x2 + ax + a + 3
3(x + 2)(x + 3)
3(x + 3)
lim
= lim
= lim
= 1.
2
x2
x2
x2
x +x2
(x + 2)(x 1)
x1

Calculus A (1) Homework 2 ANSWER


2.4

The Precise Definition of a Limit

Homework 1.
(a) Give an - proof to show that lim f (x) = L implies lim |f (x)| = |L|.
xa

xa

(b) Show that the reversed direction of the above statement is not true.
(96 )
Solution.
(a) By the definition of lim f (x) = L, for any > 0, there exists > 0 such that
xa

if 0 < |x a| < , then |f (x) L| < . We choose the same , whenever


0 < |x a| < , by the triangle inequality, we have
||f (x)| |L|| |f (x) L| < .
Hence lim |f (x)| = |L|.
xa

(b) Consider the function

1 if x is rational
f (x) =
.
1 if x is irrational

Then |f (x)| 1, so we take L = 1 that lim |f (x)| = |L|. However, lim f (x)
xa

does not exist for any a.


Homework 2. Prove that lim+

x = 0.

x0

Solution. Given > 0, choose = 2 . If 0 < x < , then

Hence lim+

x<

x = 0.

x0

2 = .

xa

2.5

Continuity



Homework 3. Find the limit lim sin1 x sin x1 .

(99 )

x0



Solution. Since sin x1 1 for all x 6= 0, we have



1
1
1
| sin x| sin x sin
| sin1 x|.
x

Because sin1 x is continuous at x = 0, we have lim sin1 x = sin1 0 = 0. By the


x0


1
1
Squeeze Theorem, we get lim sin x sin x = 0.
x0

Homework 4. Prove that the equation x 2 sin x = 0 has a root on [ 2 , ].

Solution. Consider the function f (x) = x 2 sin x, which is a continuous on the


interval [ 2 , ]. Since f ( 2 ) = 2 2 sin 2 = 2 21 < 0 and f () = 2 sin = > 0,
by the Intermediate Value Theorem, there is a number c ( 2 , ) such that f (c) = 0.
The number c is a root of x 2 sin x = 0.

2.6

Limits at Infinity (Due 10/8)

3
Homework 5. Find the limit lim x 2 ( x + 2 2 x + 1 + x).

(102 )

Solution.

3
lim x 2 ( x + 2 2 x + 1 + x)
x

3
= lim x 2 (( x + 2 x + 1) ( x + 1 + x))
x


3
x + 2 (x + 1)
x+1x
2

= lim x

x
x+2+ x+1
x+1+ x


3
1
1

= lim x 2

x
x+2+ x+1
x+1+ x




3
x x+2
2

= lim x

x
( x + 2 + x + 1)( x + 1 + x)


3
2

= lim x 2

x
( x + 2 + x + 1)( x + 1 + x)( x + x + 2)
2
q
q
= lim q
 q


x
1 + x2 + 1 + x1
1 + x1 + 1 1 + 1 + x2
=

1
2
= .
(1 + 1)(1 + 1)(1 + 1)
4

Homework 6. Find lim

x2 + 4x + 5

x2 + x + 1.

(98 )

Solution.

(x2 + 4x + 5) (x2 + x + 1)

x2 + x + 1 = lim
x
x
x2 + 4x + 5 + x2 + x + 1
3 + x4
3x + 4
q
q

= lim
= lim
x
x2 + 4x + 5 + x2 + x + 1 x 1 + 4 + 52 1 + 1 + 12
x
x
x
x
lim

x2 + 4x + 5

3
= .
2

Homework 7. Find the limit lim

x2 2x + x.

(93 )

Solution.

(x2 2x) x2
2x
x2 2x + x = lim
= lim
x
x
x
x2 2x x
x2 2x x
2
2
q
= 1.
=
= lim
x
1 1
1 2 1
lim

Homework 8.

(a) Use the Squeeze Theorem to evaluate lim


x

sin x
.
x

(b) How many times does the graph cross the asymptote?
Solution.
(a) Since 1 sin x 1, x > 0, we have

1
sin x
1

.
x
x
x

1
x x

Because lim x1 = 0 and lim


x

lim
x

sin x
x

= 0, by the Squeeze Theorem, we get

= 0.

(b) From (a), we know y = 0 is a horizontal asymptote. Since for xk = k, k Z,


sin xk = 0, we know the graph of f (x) =
infinite times.

sin x
x

crosses the horizontal asymptote

2.7

Derivatives and Rates of Change (Due 10/8)

Homework 9. (parabola) y =

1 2
x,
2p

(focus) F (0, p2 ), , Q(x0 , y0 ),


(a) F (0, p2 ) Q(x0 , y0) L
(b) Q(x0 , y0) (tangent line) (normal line)
(c) L Q(x0 , y0 ) L x = x0
Solution.
(a) The equation L is y

p
2

y0 p2
x0

x.
1 2
x
2p

(b) The slope of the tangent line of y = f (x) =


f (x) f (x0 )
= lim
xx0
xx0
x x0
x0
1
lim (x + x0 ) = .
=
2p xx0
p

f (x0 ) = lim

1 2
x
2p

1 2
x
2p 0

x x0

So the equation of the tangent line is y y0 =


normal line of y = f (x) =
line is y y0 =

xp0 (x

1 2
x
2p

at (x0 , y0 ) is

xp0 ,

at (x0 , y0 ) is
=

1
(x + x0 )(x x0 )
lim
2p xx0
x x0

x0
(x
p

x0 ). The slope of the

so the equation of the normal

x0 ).

(c) From (a) and (b), we know that


p
p 
t,
L : y = + y0
2
2

x = x0 t,

tR

N : px + x0 y = px0 + x0 y0 .

We want to find the reflection of L about N. It suffices to find the reflection


of F (0, p2 ) about the normal line N. Set x = ps, y =
p(ps) +

x0 ( p2

p
2

+ x0 s, s R. Solve

+ x0 s) = px0 + x0 y0 to get
s=

px0
2

+ x0 y0
.
+ p2

x20

So the reflection of F (0, 2p ) about N is


x =

y =

px0
+ x0 y0
p2 x0 + 2py0 x0
p2 x0 + x30
=
=
= x0 ,
2ps = 2p 2 2
p + x20
p2 + x20
p2 + x20
px0
+ x0 y0
p
p px20 + 2x20 y0
p
p
=
= +
+ 2x0 s = + 2x0 2 2
+
2
2
2
2
2
p + x0
2
p + x0
2

Hence the line passing through (x0 , y0 ) and (x , y ) is L : x = x0 .


4

x20
.
p

Calculus A (1) Homework 3 ANSWER


2.8

The Derivative as a Function

Homework 1. Suppose

x2
x1
f (x) =
.
ax + b x > 1

Find a and b such that f is continuous and differentiable at x = 1.

(92 )

Solution. The condition f (x) is continuous at x = 1 is lim f (x) = lim+ f (x) =


x1

x1

f (1), which implies a + b = 1. The condition f (x) is differentiable at x = 1 is


f (1) = f+ (1). which implies a + b = 1. By definition, we know
x2 1
f (x) f (1)
= lim
= lim (x + 1) = 2,
x1 x 1
x1
x1
x1
f (x) f (1)
(ax + b) 1
ax + (1 a) 1
f+ (1) = lim+
= lim
= lim
x1
x1
x1
x1
x1
x1
ax a
= lim a = a.
= lim
x1
x1
x1

f (1) = lim

Hence a = 2, b = 1.
Homework 2.
2

(a) If f (x) = x 3 , show that f (0) does not exist.


(b) If a 6= 0, find f (a).
2

(c) Show that y = x 3 has a vertical tangent line at (0, 0).


Solution.
(a) By definition, we have
2

1
x3
f (x) f (0)
= lim
= lim x 3 .
f (0) = lim
x0 x
x0
x0
x0

The limit does not exist, so f (0) does not exist.


(b) If a 6= 0, we get
2

x3 a3 x3 + x3 a3 + a3
x3 a3
= lim
4
f (a) = lim
2
2
4
xa x a
xa x a
x3 + x3 a3 + a3
x+a
2a
2 1
x2 a2
a 3.
= lim 4
= lim
4
4 =
2
2
4
2
2
4 =
xa x 3 + x 3 a 3 + a 3
xa (x a)(x 3 + x 3 a 3 + a 3 )
3
3a 3

(c) From (b), we know that for x 6= 0, the derivative of f (x) = x 3 is f (x) = 23 x 3 .

2

Since lim |f (x)| = lim 23 11 = , y = x 3 has a vertical tangent line at (0, 0).
x0

3.1

x3

x0

Derivatives of Polynomials and Exponential


Functions

Homework 3. Show that the derivative of f (x) =

x = x 2 is f (x) =

1 1
2 x

= 12 x 2 .

Solution.
1

f (x + h) f (x)
(x + h) 2 x 2
f (x) = lim
= lim
h0
h0
h
h
1
1
1
1
(x + h) 2 x 2 (x + h) 2 + x 2
1
1
1 1

x 2.
= lim
1
1
1 =
1 = lim
1 =
h0
h0 (x + h) 2 + x 2
h
2
(x + h) 2 + x 2
2x 2

Homework 4. Find the derivative of f (x) = x4 + x4 x + 2.

Solution. Using the Power Rule, we have


f (x) = 4x3

3.2

4
1 1
.
2
x
2 x

The Product and Quotient Rules

Homework 5. The curve y =

x
1+x2

is called a serpentine. Find an equation of the

3
tangent line to this curve at the point (3, 10
).

Solution. From the Quotient Rule, the slope of the tangent line of the serpentine
3
) is
at (3, 10




dy
(1 + x2 ) x 2x
1 x2
2
=
=
= .



2
2
2
2
dx x=3
(1 + x )
(1 + x ) x=3
25
x=3

So the equation of the tangent line is y

3
10

2
= 25
(x 3).

Homework 6. If f is a differentiable function, find an expression for


(a) y =

x2
,
f (x)

(b) y =

1 + xf (x)

.
x

Solution.
(a) From the Quotient Rule, we have
2xf (x) x2 f (x)
dy
=
.
dx
(f (x))2
2

dy
.
dx

(b) From the Quotient Rule and Product Rule, we have

x(f (x) + xf (x)) (1 + xf (x)) 12 1x


dy
=
dx
x
xf (x) + 2x2 f (x) 1
2xf (x) + 2x2 f (x) 1 xf (x)
=
.
=
3
3
2x 2
2x 2

3.3

Derivatives of Trigonometric Functions

Homework 7.
(a) (tan2 ) = (tan tan ) = sec2 tan + tan sec2 = 2 tan sec2 .
(b) (sec2 ) = (sec sec ) = sec tan sec + sec sec tan = 2 sec2 tan .
Homework 8. If f (x) = ex cos x, find f (x) and f (x).
Solution. From the Product Rule, we have
f (x) = ex cos x ex sin x.
f (x) = ex cos x ex sin x ex sin x ex cos x
= 2ex sin x.

3.4

The Chain Rule


2x 32
.
x5 x5

(98 )

Homework 9. Find lim

Solution. Let f (x) = 2x , then f (5) = 25 = 32 and f (x) = 2x ln 2, so


2x 32
f (x) f (5)
lim
= lim
= f (5) = 25 ln 2.
x5 x 5
x5
x5
Homework 10. Let f (x) =

x cos x
.
(x+1)(x+2)(x+n)

Find f (0).

(96 )

Solution. From the quotient rule, we have



(x + 1)(x + 2) (x + n)(x cos x) (x cos x)((x + 1)(x + 2) (x + n))
.
f (0) =

((x + 1)(x + 2) (x + n))2
x=0

Notice that (x cos x) |x=0 = (cos x x sin x)|x=0 = 1 and (x cos x)|x=0 = 0, so
f (0) =
Homework 11. Find

d3
dx3

3
1+x

1 2n
1
= .
2
(1 2 n)
n!


.
3

Solution. Notice that

3
1+x

= x(1 + x) 3 and

1
4
1
d
(1 + x) 3 = (1 + x) 3
dx
3
2
d
1
7
7
4
1 4
(1 + x) 3 = (1)2 (1 + x) 3 = (1 + x) 3
2
dx
3 3
9
28
1
d3
4
7
1
10
10
(1 + x) 3 = (1)3 (1 + x) 3 = (1 + x) 3 ,
3
dx
3 3 3
27

From the Leibniz Rule, we have





1
x
d3
d3 
3

x(1
+
x)
=
dx3 3 1 + x
dx3






1
1
1
1

3
3
3
3
+ 3(x) (1 + x)
+ x (1 + x)
= (x) (1 + x) + 3(x) (1 + x)




1
7
10
1
28
4
= 3 (1 + x) 3 + x (1 + x) 3
= (1 + x) 3 x(1 + x) 3 .
3
27

Homework 12. Given g(2) = 4, f (2) = 2 and g (x) = x2 + 5, f (x) = x3 + 1


for all x > 0, find the derivative of g(f (x)) at x = 2.

(99 )

Solution. By the Chain Rule, we know



d
= g (f (2)) f (2) = g (2) 23 + 1 = 22 + 5 3 = 9.
g(f (x))
dx
x=2

1
d
d 3
.
(95 )
sec x + tan x and dx
Homework 13. Find dx
1+x2 (x+ 1+x2 )
Solution. Direct computation gives
1
d
1
1 (sec x tan x + sec2 x)
3
sec x + tan x =
= sec x(sec x + tan x) 3 ,
2
dx
3 (sec x + tan x) 3
3

and

1
d
1
d 
1

x(1 + x2 ) 2 + 1 + x2
=
dx 1 + x2 (x + 1 + x2 )
(1 + x2 )(x + 1 + x2 )2 dx
!
x
1
2x
1

+ 2x
=
(1 + x2 ) 2 +
2
2
2
2 (1 + x2 ) 21
(1 + x )(x + 1 + x )
1

1 + 2x2 + 2x(1 + x2 ) 2
=

3
(1 + x2 ) 2 (x + 1 + x2 )2
Homework 14. Let

Find f (0) and f (0).

x3 sin 1 x 6= 0
x
f (x) =
0
x=0
4

(91 )

Solution. We first to compute




x3 sin x1
1
f (x) f (0)
2
= 0.
= lim
= lim x sin
f (0) = lim
x0
x0
x0
x0
x
x

The above limit exists because of x2 x2 sin x1 x2 and by the Squeeze Theorem.
When x 6= 0, we have


1
1
1
1
1
3
f (x) = 3x sin + x cos 2 = 3x2 sin x cos .
x
x
x
x
x

So we compute


3x2 sin x1 x cos x1
f (x) f (0)
1
1
f (0) = lim
.
= lim
= lim 3x sin cos
x0
x0
x0
x0
x
x
x

We take two sequences xn =

1
2n

and xn =

1
2n+

for n N, then



1
1

lim 3xn sin cos = lim ( cos(2n)) = 1,


n
xn 0
xn
xn


1
1
3xn sin cos = lim ( cos(2n + )) = 1.
lim

n
xn 0
xn
xn

If lim 3x sin x1 cos x1 exists, they must have
x0





1
1
1
1

3xn sin cos ,


lim 3xn sin cos = lim
x
xn 0
xn
xn
xn
xn
n 0

but the above computations give a contradiction. Hence f (0) does not exist.

Calculus A (1) Homework 4 ANSWER


3.5

Implicit Differentiation (Due 10/22)

Homework 1. Calculate derivatives of the other five inverse trigonometric functions.


d
1
X
sin1 x =
dx
1 x2
1
d
cot1 x =
dx
1 + x2

1
d
cos1 x =
dx
1 x2
1
d
sec1 x =
dx
x x2 1

1
d
tan1 x =
dx
1 + x2
1
d
csc1 x =
.
dx
x x2 1

Solution.
d
= 1, so
Let = cos1 x, then cos = x and sin dx

d
1
1
1
cos1 x =
=
=
.
dx
sin
1 cos2
1 x2
d
= 1, so
Let = tan1 x, then tan = x and sec2 dx

d
1
1
1
tan1 x =
=
=
.
2
2
dx
sec
1 + tan
1 + x2
d
= 1, so
Let = cot1 x, then cot = x and csc2 dx

d
1
1
1
cot1 x = 2 =
=
.
2
dx
csc
1 + cot
1 + x2
d
= 1, so
Let = sec1 x, then sec = x and sec tan dx

d
1
1
1

.
sec1 x =
=
=
dx
sec tan
sec sec2 1
x x2 1
d
Let = csc1 x, then csc = x and csc cot dx
= 1, so

d
1
1
1

csc1 x =
=
=
.
dx
csc cot
csc csc2 1
x x2 1
Homework 2. Consider the equation x3 + y 3 = 2xy. Find

dy
dx

and

d2 y
dx2

at (x, y) =

(89 )

(1, 1).

Solution. We use the implicit differentiation with respect to x to get



dy
dy
2y 3x2
dy
23
2
2 dy
= 2y + 2x

= 2

= 1.
=
3x + 3y

dx
dx
dx
3y 2x
dx (x,y)=(1,1) 3 2
2

dy
2y3x
We use the quotient rule on dx
= 3y
2 2x and get


(3y 2 2x)(2y 6x) (2y 3x2 )(6yy 2)
d2 y
=
= 16.

dx2 (x,y)=(1,1)
(3y 2 2x)2
(x,y)=(1,1)

Homework 3. Consider the curve xy + 2x y = 0. Find all points p on the curve


(95 )

such that the normal line is parallel to 2x + y = 0.

Solution. Using the implicit differentiation with respect to x, we get the slope of
the tangent line:
dy
dy
dy
2+y
+2
=0
=
.
dx
dx
dx
1x

y+x

. We want to find p such that 1x


= 2,
So the slope of the normal line is 1x
2+y
2+y
and this equation implies x = 2y 3. We put it into xy + 2x y = 0 to get
y 2 + 4y + 3 = 0, so y = 1 or 3. When y = 1, we get x = 1. When y = 3,
we get x = 3. Hence p = (1, 1) or (3, 3).
Homework 4. Suppose x, y satisfy x2 y 3 xy = 1. Find the tangent line at (2, 1)

d2 y
and dx2
.
(96 )
(2,1)

Solution. We use the implicit differentiation with respect to x to get



41
dy
dy
2x y
dy
3
2 dy
=
yx
=0
= 2

= .
2x 3y

dx
dx
dx
3y + x
dx (2,1) 3 + 2
5

So the tangent line of the curve x2 y 3 xy = 1 at (2, 1) is y 1 = 35 (x 2). We


use the quotient rule on

dy
dx

2xy
3y 2 +x

to get



d2 y
(3y 2 + x)(2 y ) (2x y)(6yy + 1)
34
=
=

.


dx2 (2,1)
(3y 2 + x)2
125
(2,1)

Homework 5. Find the equation of the line normal to the graph of sin(xy) =
x2 cos y at the point (2, 2 ).

(95 )

Solution. We use the implicit differentiation with respect to x to get the slope of
the tangent line of the curve sin(xy) = x2 cos y at (2, 2 ):


dy
dy
cos(xy) y + x
= 2x cos y x2 sin y
dx
dx


2x cos y y cos(xy)

dy
=
= .



2
dx (2, )
2 cos(xy) + x sin y (2, )
4
2

So the slope of the normal line is 4 and the equation of the normal line is y 2 =
4 (x 2).


Homework 6. Find the equation of the tangent line of the curve sin1 xy =

(99 )
tan1 (xy) at ( 2, 26 ).
Solution. We use the implicit differentiation with respect to x to get


dy
y
x dx
dy
1
1
p
.
y+x
=
1 + (xy)2
dx
1 ( xy )2 x2


dy
Put (x, y) = ( 2, 26 ) into the above equation and get dx
= 5 6 3 . So the equation of

the tangent line is y 26 = 5 6 3 (x 2).

3.6

Derivatives of Logarithmic Function


(Due 10/22)

Homework 7. Find the derivative of f (x) = log2 (3x + x4 + 56 ).


Solution. f (x) =

(102 )

3x ln 3+4x3
.
ln 2(3x +x4 +56 )

Homework 8. Let f (x) = ex ln(2 + sin x).


(a) Find f (x).
(98 )

(b) Find f (0).


Solution.
cos x
= ex ln(2 + sin x) +
(a) f (x) = ex ln(2 + sin x) + ex 2+sin
x

cos x
2+sin x

(b) f (0) = ln 2 + 21 .
Homework 9. Differentiate y =

q
3

(x+1)(x+2)
(x+3)(x+4)

by logarithmic differentiation method.

Solution. We take logarithms on both sides of the equation to get


ln y =

1
(ln(x + 1) + ln(x + 2) ln(x + 3) ln(x + 4)) .
3

Then we differentiate with respect to x to get




y
1
1
1
1
1
.
=
+

y
3 x+1 x+2 x+3 x+4
Hence

1
1
1
1
+

x+1 x+2 x+3 x+4


s


1
1
1
1
1 3 (x + 1)(x + 2)
.
+

=
3 (x + 3)(x + 4) x + 1 x + 2 x + 3 x + 4

y
y =
3

Homework 10. Differentiate y = x

Solution. We take logarithms and differentiate with respect to x to get


ln y =

x ln x

1
1
1
1
y
= ln x + x = ln x + .
y
x
2 x
2 x
x

Hence
y =y

1
1
ln x +
2 x
x

=x

1
1
ln x +
2 x
x

.
(96 )

Homework 11. Differentiate y = (ln x) 2 , x > 1.

Solution. We take logarithms and differentiate with respect to x to get


x
y
1
x 1 1
1
1
ln(ln x)
= ln(ln x) +
= ln(ln x) +
.
2
y
2
2 ln x x
2
2 ln x

ln y =
Hence

y =y

1
1
ln(ln x) +
2
2 ln x

Homework 12. Find

d
(sec x)x , 2
dx

= (ln x)

x
2


1
1
.
ln(ln x) +
2
2 ln x
(102 )

< x < 2 .

Solution. We take logarithms and differentiate with respect to x on y = (sec x)x


to get
ln y = x ln(sec x) =

sec x tan x
y
= ln(sec x) + x
= ln(sec x) + x tan x.
y
sec x

Hence
y = y (ln(sec x) + x tan x) = (sec x)x (ln(sec x) + x tan x) .
Homework 13.
(a) Use logarithmic differentiation to find the derivative of the function y =
1

x x , x > 0.
(b) Find the tangent line of the function y = tan1 (ex ) at x = 0.

(97 )

Solution.
1

(a) We take logarithm and differentiate y = x x to get


x x1 ln x 1
ln x
y
1 ln x
ln y =
=
=
2
x
y
x
x2




1
1 ln x
1 ln x

x
=x
.
y =y
x2
x2
4

(b) First, we get the slope of the tangent line at x = 0:


y =

1
1
ex y (0) = .
x
2
1 + (e )
2

Since y(0) = 4 , the equation of the tangent line is y

= 21 x.

Calculus A (1) Homework 5 ANSWER


3.8

Exponential Growth and Decay

Homework 1.
(a) Use the fact that the world population was 2560 million in 1950 and 3040
million in 1960 to model the population of the world in the second half of the
20th century.
(b) What is the relative growth rate?
(c) Use the model to estimate the world population in 1993 and to predict the
population in the year 2020.
Solution.
(a) We use t = 0 to represent the year 1950 and t = 10 to the year 1960. Then from
 .
1
the formula P (t) = P (0) ekt, we have 3040 = 2560e10k . So k = 10
ln 3040
=
2560
0.017185. Hence the population of the world in the second half of the 20th

century is modelled by
P (t) = 2560 e0.017185t .
.
(b) The relative growth rate is k = 0.017185.
(c) In this setting, the year 1993 is t = 43 and the year 2020 is t = 70, so
.
P (43) = 2560 e0.01718543 = 5360 (millions),
.
P (70) = 2560 e0.01718570 = 8525 (millions).
Homework 2. Experiments show that if the chemical reaction
1
N2 O5 2NO2 + O2
2
takes place at 45 C, the rate of reaction of dinitrogen pentoxide is proportional to
its concentration as follows:

d[N2 O5 ]
= 0.0005[N2O5 ]
dt
1

(a) Find an expression for the concentration [N2 O5 ] after t seconds if the initial
concentration is C.
(b) How long will the reaction take to reduce the concentration of N2 O5 to 90%
of its original value?
Solution.
(a) The expression for the concentration [N2 O5 ] after t seconds is
[N2 O5 ](t) = [N2 O5 ](0) e0.0005t = C e0.0005t .
(b) We solve at t = t0 ,
[N2 O5 ](t0 ) =

90
C = C e0.0005t0 .
100

So
1
ln
t0 =
0.0005

90
100

.
= 210.7 (sec).

Homework 3. A roast turkey is taken from an oven when its temperature has
reached 85 C and is placed on a table in a room where the temperature is 22 C.
(a) If the temperature of the turkey is 65 C after half an hour, what is the temperature after 45 minutes?
(b) When will the turkey have cooled to 40 C?
Solution.
(a) Let y(t) = T (t) Ts = T (t) 22. So we have y(0) = 85 22 = 63 and
y(30) = 65 22 = 43. Since y(t) satisfies y(t) = y(0) ekt, we solve 43 = 63 e30k
 .
1
to get k = 30
ln 43
= 0.0127312. Now we want to know the temperature
63
at 45 minutes, that is, t = 45. So

.
y (45) = 63 e0.012731245 = 35.5.
Hence the temperature is 35.5 + 22 = 57.5 (C).

(b) We want to solve t = t0 such that y(t0 ) = T (t0 ) 22 = 40 22 = 18. That is,
we will solve 18 = 63 e0.0127312t0 . Hence
 
1
18 .
t=
= 98.40 (mins).
ln
0.0127312
63
When the turkey puts about 98.4 minutes, it will cool to 40 C.
Homework 4.
(a) If $3000 is invested at 5% interest, find the value of the investment at the end
of 5 years if the interest is compounded (1) annually, (2) semiannually, (3)
monthly, (4) weekly, (5) daily, and (6) continuously.
(b) If A(t) is the amount of the investment at time t for the case of continuous
compounding, write a differential equation and an initial condition satisfied
by A(t).
Solution.
(a) (1) annually: 3000(1 + 0.05)5 = 3828.84.
52
= 3840.25.
(2) semiannually: 3000 1 + 0.05
2
512
= 3850.08.
(3) monthly: 3000 1 + 0.05
12

552
(4) weekly: 3000 1 + 0.05
= 3851.61.
52
5365
= 3852.01.
(5) daily: 3000 1 + 0.05
365
(6) continuously: 3000 e50.05 = 3852.08.

(b)

3.9

dA(t)
dt

= 0.05A(t), A(0) = 3000.

Related Rate

Homework 5. A ladder 13 m long is leaning against a wall when its base starts to
slide away. By the time the base is 12 m from the wall, the base is moving at the
rate of 0.5 m/sec. At what rate is the area of the triangle formed by the ladder, the
(97 )

wall and the ground changing then?

Solution. The area formula is A(t) = 21 x(t)y(t). We take derivative with respect
to t to get
1
dA
=
dt
2

dx
dy
y(t) + x(t)
dt
dt

By the Pythagorean Theorem, we have


x2 (t) + y 2(t) = 132 2x(t)

dy
dx
+ 2y(t)
= 0.
dt
dt

At time t = t0 , x(t0 ) = 12 and dx


= 0.5, it gives y(t0 ) = 5 and dy
= 1.2, so
dt
dt

1
dA
= (0.5 5 + 12 (1.2)) = 5.95 m2 /sec.

dt t=t0
2

Homework 6. A cone of radius r centimeters and height h centimeters is lowered


point first at a rate of 1 cm/s into a tall cylinder of radius R > r centimeters that is
partially filled with water. How fast is the water level rising at the instant the cone
(94 )

is completely submerged?

R
r
h

x(t)

y(t)
z(t)

Figure 1: The cone is submerged into the water.


Solution. As Figure 3.9, we set x(t) the distance from the vertex of cone to the
water level, y(t) the height from the ground to the water level, and z(t) the height
from the ground to the vertex of the cone. At any time t, we have
y(t) = x(t) + z(t) and

dx dz
dx
dy
=
+
=
1.
dt
dt
dt
dt

By similarity, the volume of the cone under the water is


 r 2
V (t) =
(x(t))3 .
3 h

We also have another relation:

 r 2
R y(t) = Vol water + V (t) = Vol water +
(x(t))3 .
3 h
2

(1)

We differentiate this equation with respect to t to get


 r 2
dx
dy
=
.
(x(t))2
R2
dt
h
dt

When the cone is completely submerged, say t = t0 , we have x(t0 ) = h and






2 dy
2 dx
R
=
r
.
dt t=t0
dt t=t0
Then we solve the above equation and equation (1) to get

dy
r2
=
.
dt t=t0
R2 r 2

Homework 7. Car A at the lower level pulls car B, which is located on the upper
lever 9 meters higher, with constant velocity 5 m/ min to the right while a pulley

6 meters above the upper level is used to connect the two cars. Suppose that the
total length of the rope is 35 meters. Let O be the point right underneath pulley P
on the lower level, and = BP O. Find the changing rate of when car A is 20
(98 )

meters away from point O. (See Figure 2, next page.)


P

6
O
9
A 5 m/min

20

Figure 2: Car A pulls Car B with constant velocity 5 m/ min to the right.
Solution. Let OA = x(t), then we have
p
35 152 + (x(t))2 = 6 sec((t)).
We differentiate both sides with respect to t to get
2x(t) dx
dt

d
= 6 sec((t)) tan((t)) .
p
2
2
dt
2 15 + (x(t))


= 5 and x(t0 ) = 20, and it gives sec((t0 )) =
At t = t0 , we know dx
dt t=t0

tan((t0 )) = 34 , so we get

4
d
1 3 3
3
= 5 = (rad/min).

dt t=t0
5
6 5 4
10
5

5
3

and

Homework 8. 4000 ,

3000 6000 /,
?

(92 )

Solution. Set h(t) is the height of the rocket from the ground and (t) is the
angle between the horizontal line and the rocket. We have tan((t)) =

h(t)
4000

or

h(t) = 4000 tan((t)). We differentiate with respect to t to get


dh
d
= 4000 sec2 ((t)) .
dt
dt
When t = t0 , h(t) = 3000,

dh
dt

= 6000, and sec((t)) =

6000 = 4000

p
1 + tan2 ((t)) = 54 , so

25 d
d
24

=
(rad/sec).
16 dt
dt
25

Homework 9. A particle is moving along the curve y = sin1 x2 . As the particle

passes through the point ( 2, 4 ), its y-coordinate increases at a rate 2 cm/s. How
fast is the distance from the particle to the origin changing at this moment? (99

)
Solution. Let x(t) and y(t) be the x-coordinate and y-coordinate of the particle at
time t, respectively. Let the distance from the particle to the origin at time t be
z(t). By the Pythagorean Theorem, we have
(z(t))2 = (x(t))2 + (y(t))2 .
We differentiate with respect to t to get
2z(t)

dx
dy
dz
= 2x(t)
+ 2y(t) .
dt
dt
dt

At t = t0 , we have x(t0 ) = 2 and y(t0) = 4 . Furthermore, we know


 
Since y(t) = sin1 x(t)
, we get
2

So

dx
dt t=t0

= 2 2. Hence

1
dy
1 dx
=r
.



2 2 dt
dt
x(t)
1 2


!
dx
dy
x(t0 )
+ y(t0)
dt t=t0
dt t=t0


4 + 2
1

.
=q
22 2+ 2 = q
4
2
2
2 + 16
2 + 16


dz
1
=

dt t=t0
z(t0 )

dy
dt t=t0

= 2.

Homework 10. The minute hand () on a clock is 8cm long and the hour hand
() is 4cm long. How fast is the distance between the tips of the hands changing
at two oclock? Give your answer in the unit cm/hour.

(102 )

Solution. By the Cosine Law, at two oclock, we first get


cos((t)) =

42 + 82 (x(t))2
248

We differentiate this relation respect to t to get


sin((t))

1
dx
d
= x(t)
dt
32
dt

At two oclock, say t = t0 , we solve x(t0 ):


  42 + 82 (x(t ))2

0
cos
=
x(t0 ) = 4 3.
3
248
and we know

3.10

d
dt

2
12

2 = 11
, and sin( 3 ) = 23 . Hence
6



32 23
dx
22
11
=
(cm/hr).
=


dt t=t0
6
3
4 3

Linear Approximations and Differentials

Homework 11. Find the linear approximation of the function





x1
1
g(x) = sin
tan1 ( x)
x+1
(102 )

at the point x = 3.
Solution. Since

 


1
tan1 ( 3) = = ,
g(3) = sin
2
6
3
6
1
1
(x + 1) (x 1)
1
2
g (x) = q


2
2
(x + 1)
1 + ( x) 2 x
1 x1
1

x+1

1
1
1
=

=
.
x(x + 1) 2 x(x + 1)
2 x(x + 1)
1
g (3) = ,
8 3

The linear approximation of g(x) at x = 3 is


g(x) g(3) + g (3)(x 3)
1

= + (x 3).
6 8 3
7

Homework 12. Let f (x) =

1+cos x
.
1+sin x

Use a differential to estimate f (44 ). (101

)
Solution. Since
 

2
2

2
2

= 1,
1+
(1 + sin x)( sin x) (1 + cos x)(cos x)
f (x) =
(1 + sin x)2
sin x cos x 1
=
(1 + sin x)2

  2 2 1

1
2

2
2
2
3


2

= 2 2 2.
= 2
f
= 3 =
2
4
+ 2
3+2 2 32 2
2
1 + 22
f

1+

The linear approximation of f (x) at x =

is

  

+f
x
f (x) f
4
4
4


= 1 + (2 2 2) x
.
4
 

So we estimate

f (44 ) 1 + (2 2 2)

44

180
4

( 2 1)
=1+
.
90

Homework 13. For a ball with radius r meter, its surface area and volume are
A = 4r 2 meter2 and v = 34 r 3 meter3 , respectively.
(a) When r = 2 meter, what is the instantaneous rate of change of volume with
respect to the radius?
(b) Let h be a very small positive number. When the radius increases from 2 meter
to (2 + h) meter, use differential to give an approximation on the increase of
volume.
(c) Use geometric point of view to explain the relationship between the increase
of volume derived in (b) to the surface area of the ball with radius 2 meter.
(89 )
Solution.
(a) Since V = 43 r 3 , we have

dV
dr

= 4r 2 . At r = 2, we get
8

dV
dr

= 16.

(b) The differential dV = V (r)dr. We put r = 2 and dr = h to get


dV = V (r)dr = 16h.
(c) Since dV = 16h meter3 , and the surface area of the ball with radius 2 meter
is 16 meter2 , the increase of volume dV is approximately the same as the
surface area multiplies the increment of radius h.

4.1

Maximum and Minimum Values

Homework 14. Prove that the function f (x) = x101 + x51 + x + 1 has neither a
local maximum nor a local minimum.
Solution. Since f (x) = 101x100 + 51x50 + 1 1 > 0 for all x R, by Fermats
Theorem, the function has neither a local maximum nor a local minimum.
Homework 15. Find the absolute maximum and absolute minimum values of
f (x) = 2x3 3x2 12x + 1 on [2, 3].
Solution. We compute f (x) = 6x2 6x 12 = 6(x + 1)(x 2). So the solution of
f (x) = 0 is x = 1 and x = 2. We compare the following values:
f (2) = 3,

f (1) = 8,

f (2) = 19,

f (3) = 8.

So the absolute maximum is f (1) = 8 and absolute minimum if f (2) = 19.

4.2

The Mean Value Theorem

Homework 16. Let f (x) = a(x3 + x 2) + b(x3 + 3x + 4), ab > 0. Show that
(98 )

f (x) = 0 has exactly one real solution.

Solution. Since f (1) = 8b and f (1) = 4a, which imply f (1) f (1) = 32ab <
0, by the Intermediate Value Theorem, there exists c (1, 1) such that f (c) = 0,
so f (x) = 0 has at least one real solution.
Suppose x1 and x2 are two solutions of f (x) = 0. Since f (x) is continuous on
[x1 , x2 ], differentiable on (x1 , x2 ) and f (x1 ) = f (x2 ) = 0, by Rolles Theorem, there
exists c (x1 , x2 ) such that f (c) = 0. We compute
f (x) = a(3x2 + 1) + b(3x2 + 3) = 3(a + b)x2 + (a + 3b).
9

If a > 0 and b > 0, then f (x) = 3(a + b)x2 + (a + 3b) > 0. If a < 0 and b < 0,
then f (x) = 3(a + b)x2 + (a + 3b) > 0. In both cases, they imply f (x) 6= 0 and it
contradicts to f (c) = 0. So f (x) = 0 has exactly one real solution.
Homework 17. Use the Mean Value Theorem to prove the inequality
| sin a sin b| |a b| for all a and b.
Solution. Given any two numbers a, b with a < b, consider the function f (x) = sin x
on [a, b]. Then f (x) is continuous on [a, b] and differentiable on (a, b). By the Mean
Value Theorem, there exists c (a, b) such that
f (a) f (b) = f (c)(a b) |f (a) f (b)| = |f (c)||a b|.
Since f (x) = cos x and |f (c)| = | cos c| 1, we have
| sin a sin b| = | cos c||a b| |a b|.
Homework 18. Show that
ln(1 + x)
1
<
<1
1+x
x
(99 )

for x > 0.

Solution. Consider the function f (t) = ln(1 + t) defined on [0, x]. Then f (t) is
continuous on [0, x] and differentiable on (0, x). By the Mean Value Theorem, there
exists c (0, x) such that
f (c) =
Since f (x) =

1
,
1+c

f (x) f (0)
ln(1 + x)
=
.
x0
x

we have f (c) =

1
1+c

and

1
1+x

< f (c) =

1
1+c

< 1. Hence we have

ln(1 + x)
1
<
< 1.
1+x
x
Homework 19. Suppose both f (x), g(x) are continuous on [a, b], and differentiable
on (a, b). Suppose g (x) 6= 0 for any x (a, b). Show that there exists c (a, b)
such that
f (c) f (a)
f (c)
=
.
g (c)
g(b) g(c)
10

Solution. Consider the function F (x) = f (x)g(x) g(b)f (x) f (a)g(x) on [a, b].
Since f (x) and g(x) are continuous on [a, b], so is F (x). Since f (x) and g(x) are
differentiable on (a, b), so is F (x). By the Mean Value Theorem, there exists c
(a, b) such that
F (b) F (a) = F (c)(b a).

(2)

Since
F (x) = f (x)g(x) + f (x)g (x) g(b)f (x) f (a)g (x)
F (b) = f (b)g(b) g(b)f (b) f (a)g(b) = f (a)g(b)
F (a) = f (a)g(a) g(b)f (a) f (a)g(a) = g(b)f (a),
which implies F (b) F (a) = 0, equation (2) gives
F (c) = f (c)g(c)+f (c)g (c) g(b)f (c) f (a)g (c) = 0
and
f (c) f (a)
f (c)
=
.

g (c)
g(b) g(c)
Remark that g(b) g(c) 6= 0. This is because if g(b) = g(c), by the Mean
Value Theorem, there exists d between b and c such that g (d) = 0, and this fact
contradicts to g (x) 6= 0.

11

Calculus A (1) Homework 6 ANSWER


4.4

Indeterminate Forms and lHospitals Rule


x(ex +1)2(ex 1)
.
x3
x0

(96 )

Homework 1. Find the limit lim

Solution. By lHospital Rule, we have


x(ex + 1) 2(ex 1) L
ex + 1 + x ex 2ex
ex + 1 + x ex
=
lim
=
lim
x0
x0
x0
x3
3x2
3x2
x
x
x
x
1
e + e + x e
e
L
= lim
= lim
= .
x0
x0 6
6x
6
lim

Homework 2. Let f (x) =

cot 2 x
,0
ln x

< x < 1. Find lim f (x) and lim f (x).


x1

(100

x1

)
Solution. By lHospital Rule, we have
lim f (x) = lim

x1

x1

cot 2 x L
csc2 ( x)

= lim 2 1 2 = .
x1
ln x
2
x

We compute
(ln x)( 2 csc2 ( 2 x)) (cot( 2 x)) x1
=
f (x) =
(ln x)2


2

x(ln x) + sin( 2 x) cos( 2 x)


x sin2 ( 2 x)(ln x)2

So


x(ln x) + sin( 2 x) cos( 2 x)
1
2
lim f (x) = lim
lim
2
x1 x sin ( x) x1
x1
(ln x)2
2


(ln x) + 2 + 2 cos2 ( 2 x) 2 sin2 ( 2 x)
L
2
= lim
x1
2(ln x) x1

1


ln x + 2 cos2 ( 2 x) L
+ 4 cos( 2 x) sin( 2 x) 2

x
= lim
= lim
1
4 x1
ln x
4 x1
x

= .
4

Homework 3. Compute

lim (sec x tan x).

x( 2 )

Solution. By lHospital Rule, we have






1
sin x
1 sin x
lim (sec x tan x) = lim
= lim

x( 2 )
x( 2 )
x( 2 )
cos x cos x
cos x


cos x
L
= 0.
= lim

x( 2 )
sin x
1

1+tan x
1+sin x

Homework 4. Find the limit lim


x0

1+tan x
1+sin x

Solution. Let y(x) =


Rule, we have
1
lim ln y(x) = lim 3 ln
x0
x0 x
sec2

 13
x

1 + tan x
1 + sin x

 13
x

.
(96 )
1
x3

, then ln y(x) =

= lim
x0

ln

1+tan x
1+sin x

, and by lHospital

ln(1 + tan x) ln(1 + sin x)


x3

cos x
1+sin
1 + sin x cos3 x(1 + tan x)
x
=
lim
= lim
x0 3x2 cos2 x(1 + tan x)(1 + sin x)
x0
3x2
1
1 + sin x cos3 x(1 + tan x)
= lim

lim
x0 3 cos2 x(1 + tan x)(1 + sin x) x0
x2
cos x + 3 cos2 x sin x(1 + tan x) cos3 x sec2 x
L 1
= lim
3 x0
2x
cos x(1 + tan x) + sin x sec2 x
sin x(1 + tan x) L
1
2
= lim
= lim 3 cos x lim
x0
x0
3 x0
2x
2
2
1
cos x + sin x + sin x sec x
= lim
= .
x0
2
2
L

x
1+tan x

By the continuity of logarithmic function, we have


1

1
1
1 + tan x x3
ln lim y(x) = lim ln y(x) = lim y(x) = lim
= e2 .
x0
x0
x0
x0
2
1 + sin x
x
(97 )
Homework 5. Evaluate the limit lim 1 + x3 + x52 .


Solution. Let y(x) = 1 +


lHospital Rule, we have

3
x


5 x
,
2
x

then ln y(x) = x ln 1 +

3
x

5
x2


, and by




ln 1 + x3 + x52
5
3
lim ln y(x) = lim x ln 1 + + 2 = lim
1
x
x
x
x x
x

1
x32 x103
5
3
+
3 + 10
1+
(
)
L
x
x2
x
 = 3.
=
lim
= lim
x 1 + 3 + 52
x
x12
x
x

By the continuity of logarithmic function, we have




ln lim y(x) = lim ln y(x) = 3 lim y(x) = lim


x0

x0

x0

Homework 6. Find the limit lim


x0

Solution. Let y(x) =

tan x
x

 12
x

tan x
x

 12

x

= e3 .

(100 )

, then ln y(x) =

5
3
1+ + 2
x x

1
x2

tan x
x

ln(tan x)ln x
x2

ln( sinx x )ln cos x


,
x2

and by lHospital Rule, we have


x
sin x
x
x cos xsin
+ cos
ln( sinx x ) ln cos x L
sin x
x2
x
= lim
lim ln y(x) = lim
2
x0
x0
x0
x
2x
1
sin x
cos x

+
1
x

sin
x
cos
x
= lim sin x x cos x = lim 2
x0
2x
2 x0 x sin x cos x
2
1 cos x + sin2 x
L 1
= lim
2 x0 2x sin x cos x + x2 cos2 x x2 sin2 x
1
2 sin2 x
= lim
2 x0 2x sin x cos x + x2 cos2 x x2 sin2 x
1
1
1
= lim
= .
=

2
2
2
2
x
x0 2 x cos x +
211+1 1 0
3
cos2 x x2
sin x

sin x

By the continuity of logarithmic function, we have

1
ln lim y(x) = lim ln y(x) = lim y(x) = lim
x0
x0
x0
x0
3


4.3

tan x
x

 12
x

= e3 .

How Derivatives Affect the Shape of a Graph

Homework 7. Suppose a cubic function f (x) has the coefficient of leading order
1, and the graph of f (x) passes through (0, 4). The graph of f (x) is decreasing on
(1, 3) and increasing on (, 1) and (3, ). Find f (x) and its inflection point. (96
)
Solution. Let f (x) = x3 + bx2 + cx + d. Since the graph of f (x) passes through
(0, 4), we have f (0) = d = 4. We compute the derivative of f (x):
f (x) = 3x2 + 2bx + c.
Since The graph of f (x) is decreasing on (1, 3) and increasing on (, 1) and
(3, ), x = 1 and x = 3 are critical point of f (x), so f (1) = 0 and f (3) = 0. This
information gives
3 + 2b + c = 0 and 27 + 6b + c = 0.
So we get b = 6 and c = 9. Thus f (x) = x3 6x2 +9x+4 and f (x) = 3x2 12x+9.
We compute f (x) = 6x 12 and f (x) = 0 implies x = 2. Since f (x) > 0 on
(2, ) and f (x) < 0 on (, 2), the inflection point is (2, 6).
Homework 8. Let f (x) =
(93 )

tan x
.
x

Show that when 0 < x1 < x2 < 2 , f (x1 ) < f (x2 ).

Solution. We compute
f (x) =

x sin x cos x
x sec2 x tan x
=
.
2
x
x2 cos2 x

Let g(x) = x sin x cos x. To prove f (x) is increasing on (0, 2 ), it suffice to show
that g(x) > 0 on (0, 2 ). Since g(0) = 0 and g (x) = 1 cos2 x + sin2 x = 2 sin2 x > 0
on (0, 2 ), we get g(x) > 0 on (0, 2 ). Hence for any 0 < x1 < x2 < 2 , f (x1 ) < f (x2 ).

4.5

Summary of Curve Sketching


2

(96 )

Homework 9. Sketch the curve y = x 3 (x 3) 3 .


Solution.
A. The domain is R.
B. The x-intercepts are x = 0, 3, and y-intercept is 0.
C. Symmetry: none.
D. Since
1

lim x 3 (x 3) 3 = ,
x

there is no horizontal asymptotes. There is no vertical asymptotes. We compute


1

lim (x 3 (x 3) 3 x)
x
2
3

1
3

= lim (x (x 3) x)

= lim

x 3 (x 3) 3 + x 3 (x 3) 3 x + x2
x(x 3)2 x3

= lim

x 3 (x 3) 3 + x 3 (x 3) 3 x + x2

x 3 (x 3) 3 + x 3 (x 3) 3 x + x2
6x2 + 9x

x 3 (x 3) 3 + x 3 (x 3) 3 x + x2
6 + x9
6 + 0
= lim
= 2.
=
4
2
x (1 3 ) 3 + (1 3 ) 3 + 1
1+1+1
x

The graph has a slant asymptote y = x 2.


E. Direct computation gives
2
1
2 1
(x 3) + 2x
1 2
x1
y = x 3 (x 3) 3 + x 3 (x 3) 3 =
2
2
1 =
1 .
3
3
3x 3 (x 3) 3
x 3 (x 3) 3

Since y (x) > 0 when (, 1) and (3, ) and y (x) < 0 when (1, 3), y is
increasing on (, 1) and (3, ) and decreasing on (1, 3).
F. The critical numbers are x = 0, 1, 3. Since y changes from positive to negative

at 1, f (1) = 3 4 is a local maximum by the First Derivative Test. Since y


changes from negative to positive at 3, f (3) = 0 is a local minimum by the
First Derivative Test. Since y doesnt change sign at 0, f (0) = 0 is neither a
local maximum nor local minimum.
5

G. We use the logarithmic differentiation to get the second derivative y :


2
1
ln y = ln(x 1) ln x ln(x 3)
3
3
1
2 1 1
1
6
y
=

=

y
x1 3 x 3 x3
3x(x 1)(x 3)
6
x

1
2
y =
= 5
1
4 .
2
3x 3 (x 3) 3 3x(x 1)(x 3)
x 2 (x 3) 3
We know y (x) > 0 on (, 0) and y (x) < 0 on (0, 3) and (3, ). Thus the
curve is concave upward on the interval (, 0) and concave downward on
(0, 3) and (3, ). The point of inflection is (0, 0).
H. Using this information to sketch the curve in Figure 1.
2
1
2

-1

-2
-3
-4
1

Figure 1: The graph of y = x 3 (x 3) 3 .


Homework 10. Sketch the curve y = (2x2 + 3x)ex .

(99 )

Solution.
A. The domain is R.
B. The x-intercepts are x(2x + 3) = 0 x = 0, 32 and y-intercept is y(0) = 0.
C. Symmetry: none.
D. Since
4x + 3 L
4
2x2 + 3x L
= lim
= lim x = 0,
x
x
x
x e
x
x
e
e
the line y = 0 is a horizontal asymptote. No vertical asymptotes.
lim (2x2 + 3x)ex = lim

E. Direct computation gives


y (x) = 4x + 3ex (2x2 + 3x)ex = (2x2 + x + 3)ex = (x + 1)(2x 3)ex .
Since y (x) > 0 when (1, 32 ) and y (x) < 0 when (, 1) and ( 23 , ), y is
increasing on (1, 32 ) and decreasing on (, 1) and ( 32 , ).
6

F. The critical numbers are x = 1 and x = 23 . Since y changes from negative


to positive at x = 1, f (1) = e is a local minimum by the First Derivative
3
Test. Since y changes from positive to negative at x = 23 , f ( 32 ) = 9 e 2 is a
local maximum by the First Derivative Test.
G. Direct computation gives
y (x) = (4x + 1)ex (2x2 + x + 3)ex = (2x2 5x 2)ex .



, 54 41 and 5+4 41 , , so the curve is con



5 41
5+ 41
cave upward on the interval , 4
and
, Since y (x) < 0
4




on 54 41 , 5+4 41 , the curve is concave downward on 54 41 , 5+4 41 . The
We know y (x) > 0 on

5 41
4

inflection points of y are happened at x =

( 2.85 and 0.35).

H. Using this information to sketch the curve.


8
6
4
2
2

-2

Figure 2: The graph of y = (2x2 + 3x)ex .

4.7

Optimization Problems

Homework 11. See the Figure 3.


x
x

x
x
Figure 3: A square piece of cardboard with 30 cm wide.

A box with cover is to be constructed from a square piece of cardboard, 30 cm wide,


by cutting out a square or a rectangle (shaded region) from each of the four corners
and bending up the remaining cardboard (unshaded region) along the dotted lines.
What is the largest volume that such a box can have? Justify that the volume you
obtain actually is the maximum volume.
(97 )
Solution. From the setting in the Figure 3, we have
2(x + y) = 30 x + y = 15 y = 15 x,

where 0 x, y 15.

The volume of the box is


V (x) = y(30 2x)x2 = 2(15 x)2 x = 2x3 60x2 + 450x.
To find the maximum of V (x), we compute
V (x) = 6x2 120x + 450 = 6(x 15)(x 5).
The critical points and endpoints of V (x) are x = 0, 5, 15. We compare these
three values that V (0) = 0, V (5) = 1000, and V (15) = 0, so the largest volume is
V (5) = 1000.
Homework 12. A cylindrical can is to be made to hold 1 L of oil. Find the dimensions that will minimize the cost of the metal to manufacture the can. (The can
contains top, bottom, and sides.)
Solution. Let r is the radius of the cylindrical can and h the height. Since we need
, 0 < r < . Next,
to hold 1 L of oil, we require r 2 h = 1000, which implies h = 1000
r 2
we compute the total surface area of the cylinder:
A(r) = 2r 2 + 2rh = 2r 2 + 2r

1000
2000
= 2r 2 +
.
2
r
r

To minimize the area of the cylinder A(r), we compute


4(r 3 500)
2000
.
A (r) = 4r 2 =
r
r2
q
 q 
.
Since
A(r)
is
decreasing
on
0, 3 500
The only critical point of A(r) is r = 3 500

q
q

and increasing on 3 500
, , r = 3 500
is the absolute minimum.

Homework 13. A cone-shaped drinking cup is made from a circular piece of paper
of radius R by cutting out a sector and joining the edges CA and CB. Find the
maximum capacity of such a cup.
8

A
R
C

Figure 4: Making a cone-shaped drinking cup.


Solution. Let be the angle of the sector ACB (large part), 0 2. When it
makes a cone-shaped drinking cup, the radius of the cup is
r() =

R
,
2

and the height of the cup is


h(t) =
The volume of the cup is

R2 (r())2 =

R 2
4 2 .
2

1
R3
2
4 2 2 .
V () = (r())2 h() =

3
24 2
To find the maximum value of V (), we compute



2
R3
2

2
2
2 4 +
V () =
24 2
2 4 2 2
3

R

=
2(4 2 2 ) 2
24 2 4 2 2

R3

8 2 32 .
=
24 2 4 2 2
Since 0 2, the critical number and endpoints are =
compare these three values that V
V

)
( 2 2
3

2R3

9 3

)
( 2 2
3

3
2R
,V
9 3

2 2
,0
3

and 2. We

(0) = 0, and V (2) = 0. So

is the maximum capacity of the drinking cup.

Homework 14. What is the smallest possible area of the triangle that is cut off by
the first quadrant and whose hypotenuse is tangent to the parabola y = 1 x2 at
(94 )

some point?

Solution. Let P (a, 1 a2 ) be any point on the parabola y = 1 x2 . The tangent


line of the parabola at P is y (1 a2 ) = 2a(x a) We solve x-intercept and
9

y-intercept to get Q(a +

1a2
, 0)
2a

= ( 1+a
, 0) and R(0, 1 + a2 ). Since we want the
2a

triangle P QR is in the first quadrant, a > 0. So the area of P QR is


A(a) =

(a2 + 1)2
1 a2 + 1

(a2 + 1) =
.
2
2a
4a

To find the smallest area of A(a), we compute


4a 2(a2 + 1) 2a 4(a2 + 1)2
4(a2 + 1)(3a2 1)
A (a) =
=
.
(4a)2
16a2

The critical numbers of A(a) is a = 13 . Since A(a) is decreasing on (0, 13 ) and


increasing on ( 13 , ), the smallest area of P QR is A( 13 ) =

10

4 3
.
9

Calculus A (1) Homework 7 ANSWER


4.9

Antiderivative

Homework 1. Find the most general antiderivative of the function


f (x) = sin x

1
1
+ 4.
x x

Solution. The most general antiderivative of the function f (x) = sin x


F (x) = cos x ln |x| 3x13 + C.

1
x

1
x4

is

Homework 2. Find f , where f (x) = x2 , x > 0, f (1) = 0, f (2) = 0.


Solution. We have f (x) = x1 + C1 and f (x) = ln |x| + C1 x + C2 , where C1
and C2 are constants. Since f (1) = C1 + C2 = 0 and f (2) = ln 2 + 2C1 + C2 = 0,
we have C1 = ln 2 and C2 = ln 2. Hence f (x) = ln |x| + (ln 2)x ln 2.

5.1

Areas and Distances

Homework 3.
(a) Let An be the area of a polygon with n equal sides inscribed in a circle with
radius r. By dividing the polygon into n congruent triangles with central angle
2
, show that
n
 
1 2
2
An = nr sin
.
2
n
(b) Show that lim An = r 2 .
n

Solution.
, so the total area of the polygon
(a) The area of each small triangle is 21 r r sin 2
 n
2
1
2
1
2
is An = n 2 r r sin n = 2 nr sin n .

(b) We compute

1
lim An = lim nr 2 sin
n
n 2

2
n

= lim r 2
n

sin

2
n
2
n

= r 2 2lim
n

sin

2
n
2
n

= r 2 .

Calculus A (1) Homework 8 ANSWER


5.2

The Definite Integral

Homework 1. Show that the Dirichlet function


(
0 if x is rational
f (x) =
1 if x is irrational
on [0, 1] is not integrable.
Hint: Both rational numbers and irrational numbers are dense in [0, 1].
Solution. We divide [0, 1] into n-subintervals with width x = n1 . Since rational
numbers are dense in [0, 1], if we take rational numbers as sample points xi on each
subinterval, then
n
X

f (xi )x =

i=1

n
X

0 x = 0 lim
n

i=1

n
X

f (xi )x = 0.

i=1

On the other hand, since irrational numbers are dense in [0, 1], if we take irrational
numbers as sample points xi on each subinterval, then
n
X

f (xi )x

n
X

1 x = 1 lim
n

i=1

i=1

n
X

f (xi )x = 1.

i=1

Two limits are not equal, so the Dirichlet function is not integrable.
Homework 2. Change the following limits of sums as integrals:
 
n
X
i
.
sin
(a) lim
n
2n
2n
i=1
(b) lim
n

n
X
i=1

(c) lim
n

 2
i
1
.
1
n n

1
+
n3

(d) lim ln
n

1
(e) lim
n n

(93 )

2
++
n3

!
n
.
n3

(94 )

n!
.
n
ln

n+1
n
n+1
n

(93 )

(96, 100 )


ln

n+2
n
n+2
n

++

ln

n+n
n
n+n
n

!

(98 )

Solution.
(a) We have
lim
n

n
X

sin

i=1

i
2n

f (xi )x,
= lim
2n n i=1

where f (x) = sin x, and we divide [0, 2 ] into n-subintervals with width x =
i
i

, and we choose sample points xi = 2n


on each subinterval [ (i1)
, 2n
], i =
2n
2n
1, . . . , n. Hence
 
Z
n
X
2
i
lim
sin
=
sin x dx.
n
2n
2n
0
i=1
Remark that we can view this limit as


 
n
n
n
X
X
X
i 1

i
sin
= lim

= lim
f(xi )x,
lim
sin
n
n
n
2n 2n
2 i=1
2 n n
2
i=1
i=1
where f(x) = sin( 2 x), and we divide [0, 1] into n-subintervals with width
x = n1 , and we choose sample points xi = ni on each subinterval [ i1
, i ], i =
n n
1, . . . , n. Hence
 
Z
n
 
X
1
i
=
sin
x dx.
lim
sin
n
2n
2n
2
2
0
i=1
(b) We have
lim
n

n
X
i=1

 2
n
X
1
i
= lim
f (xi )x,
1
n
n n i=1

where f (x) = 1 x2 , and we divide [0, 1] into n-subintervals with width


, i ], i =
x = n1 , and we choose sample points xi = ni on each subinterval [ i1
n n
1, . . . , n. Hence
s
 2
Z 1
n
X
i
1
lim
=
1
1 x2 dx.
n
n
n
0
i=1
(c) We have
lim
n

1
+
n3

2
++
n3

n
n3

= lim
n

n
X
i=1

= lim
n

n
X
i=1

X
i
=
lim
n3 n i=1

f (xi )x,

i 1

n n

where f (x) =

x, and we divide [0, 1] into n-subintervals with width x = n1 ,

, i ], i = 1, . . . , n.
and we choose sample points xi = ni on each subinterval [ i1
n n
Hence
r
r
r ! Z 1

1
2
n
x dx.
lim
+
+

+
=
n
n3
n3
n3
0
(d) We have

(n (n 1) (n 2) 3 2 1) n
n

1
n n1 n2
3 2 1 n
= lim ln


n
n
n
n
n n n


n
n
X
i
1X
ln
= lim
f (xi )x,
= lim
n
n n
n
i=1
i=1

n!
lim ln
= lim ln
n
n
n

where f (x) = ln x, and we divide [0, 1] into n-subintervals with width x = n1 ,


, i ], i = 1, . . . , n.
and we choose sample points xi = ni on each subinterval [ i1
n n
Hence

Z 1
n
n!
ln x dx.
lim ln
=
n
n
0
Remark that this is an improper integral (since ln x as x 0). We
will show this integral is meaningful (exists), and evaluate it in section 7.8.
(e) We have
1
lim
n n

ln

n+1
n
n+1
n

ln

n+2
n
n+2
n

++

ln

n+n
n
n+n
n

!


n
X
ln 1 + ni
1
= lim

i
n
n
1+ n
i=1
= lim
n

n
X

f (xi )x,

i=1

where f (x) =

ln x
,
x

and we divide [1, 2] into n-subintervals with width x = n1 ,

and we choose sample points xi = 1 + ni on each subinterval [1 + i1


, 1 + ni ], i =
n
1, . . . , n. Hence


! Z 2
n+2
n+n
ln
ln
1 ln n+1
ln x
n
+ n+2n + + n+nn
=
lim
dx.
n+1
n n
x
1
n
n
n

5.3

The Fundamental Theorem of Calculus

Homework 3. Let f (x) = e

g(x)

, where g(x) =

t
dt. Find f (2).
4
1+t
(90 )

Solution. By the chain rule, we have f (x) = eg(x) g (x). The Fundamental Theorem
of Calculus, Part 1, tells us
x
2
g (2) = .
4
1+x
17

g (x) =
We also know

g(2) =

Hence f (2) = eg(2) g (2) = e0


d
Homework 4. Compute
dx

2
17

x3

t
dt = 0.
1 + t4

2
.
17

t sin t dt.

(92 )

Solution. The Fundamental Theorem of Calculus tells us


Z x3
q



1
d
3
2
3
(
t)
sin
t
dt
=
x
sin(x
)

3x

x(sin
x)

dx x
2 x

7
1 1
= 3x 2 sin(x3 ) x 4 (sin x).
2
Z tan x
1
d
du.
(93 )
Homework 5. Compute
dx x
u4 + 1
Solution. We use the Fundamental Theorem of Calculus and Chain Rule to get
Z tan x
1
1
1
d
du =
sec2 x 4
.
4
4
dx x
u +1
(tan x) + 1
x +1
Homework 6. Let f () =

cos

sin(t2 ) dt. Find f () and f ().

(94 )

Solution. We use the Fundamental Theorem of Calculus and Chain Rule to get
f () = sin(cos2 ) ( sin ) = sin sin(cos2 )
f () = cos sin(cos2 ) sin cos(cos2 ) 2 cos ( sin )
= cos sin(cos2 ) + 2 cos sin2 cos(cos2 ).

Homework 7. Find the limit lim+


x0

Rx

sin(t2 ) dt
.
x sin(x2 )

(89 )

Solution. By l Hospital Rule, Product Rule, Chain Rule, and the Fundamental
Theorem of Calculus, we have
Rx
sin(t2 ) dt ( 00 ,L)
sin(x2 )
=
lim
lim+ 0
x0+ sin(x2 ) + 2x2 cos(x2 )
x0
x sin(x2 )
1
1
1
= lim+
=
= .
2
x
x0 1 + 2
1+211
3
cos(x2 )
sin(x2 )
Homework 8. Find the limit lim
x0

R x2
0

t
1+t3
x4

dt

(95 )

Solution. By l Hospital Rule, Chain Rule, and the Fundamental Theorem of Calculus, we have
R x2 t
2

x
dt ( 00 ,L)
2x
1
1
1
0
1+t3
1+x6
= .
lim
= lim
= lim
=
4
3
x0
x0 2 1 + x6
x0
x
4x
2
2 1+0
R1 2
dt x2
.
(97 )
Homework 9. Find the limit lim cos x t 4
x0
x
Solution. By l Hospital Rule, Chain Rule, and the Fundamental Theorem of Calculus, we have
R1 2
2 ( sin x) 2x
dt x2 ( 00 ,L)
tan x x
=
lim
= lim cos x
lim cos x t 4
x0
x0
x0
x
4x3
2x3
2
2
0
( 0 ,L)
sec x 1
tan x
= lim
= lim
2
x0
x0 6x2
6x
2

sin x
1
1
1
1
= 11= .

= lim
2
x0 6
x
cos x
6
6
Homework 10 (page 396). Find a function f and a number a such that
Z x

f (t)
dt
=
2
x for all x > 0.
6+
t2
a
Solution. If we take x = a, then the equation becomes
Z a

f (t)
6+
dt
=
2
a

6
=
2
a a = 9.
t2
a
We take derivative with respect to x on both sides, and by the Fundamental Theorem
of Calculus, Part 1, we get
3
1
f (x)
= f (x) = x 2 .
2
x
x

Calculus A (1) Homework 9 ANSWER


5.5

The Substitution Rule (page 407)


3

x
dx.
(94 )
x+1
0
Solution. Let u = x + 1, then du = dx and x = u 1. Upper limit is u = 4 and
lower limit is u = 1. So we have

 u=4
Z 3
Z 4
Z 4
1
1
x
u1
2 3
12
2
2
2

du =
u 2u
(u u ) du =
dx =
u
3
x+1
Homework 1. Compute the integral

2
8
2
= (8 1) 2(2 1) = 7 2 = .
3
3
3

Z 64
3
4 x+7 x

Homework 2. Compute the integral


dx.
6
x
1

u=1

(97 )

Solution. Let u = x 6 , then du = 16 x 6 dx, which implies dx = 6u5 du. Upper limit
1
1
is u = 2 and lower limit is u = 1. Furthermore, we get x 2 = u3 and x 3 = u2 . Thus


Z 2
Z 64
Z 2 3
4 x+73x
4u + 7u2
5

6u du =
(24u7 + 42u6) du
dx =
6
u
x
1
1
1
 8

7 u=2
= 3(356 1) + 6(128 1) = 1527.
= 3u + 6u
u=1

Solution 2. One can compute the integral directly.

Z 64
Z 64
h 4
i
1
7 x=64
1
4 x+73x
3 + 7x 6 ) dx =
3 + 6x 6

3x
dx
=
(4x
6
x=1
x
1
1
= 3(256 1) + 6(128 1) = 1527.

Homework 3. Find

sec x tan x

dx.
sec x

(92 )

Solution.
Z 0
Z 0
h
i x=0


1
sec x tan x
1

4

dx =
d sec x = 2 sec x
=
2
1

2
.
sec x
sec x
x= 4
4
4
Z ln 7
e2x
(99 )
Homework 4. Compute the integral
1 dx.
(ex + 1) 3
0
Solution. Let u = ex + 1, then du = ex dx. Upper limit is u = 8 and lower limit is
u = 2. Thus
Z ln 7
Z ln 7 x
Z 8
Z 8
2
1
e2x
(e + 1) 1 x
u1
e dx =
du =
(u 3 u 3 ) du
1 dx =
1
1
(ex + 1) 3
(ex + 1) 3
u3
0
0
2
2
 
 u=8 


3
2
3
3
3 2
3 5 3 2
u3 u3
32 4
2 23 23
=
=
5
2
5
2
5
2
u=2

3 2
66
+ 23 .
=
5
10

d
Homework 5. Find
dx

e t (x t)
dt.
t2

(100 )

Solution. By the Fundamental Theorem of Calculus, we have


! Z
Z x 1
Z x 1
Z x 1
1
1
x 1t
d
d
ex
ex x
e t (x t)
et
et t
e
x
dt =
dt
dt =
dt + x 2 2
2
2
dx 1
t2
dx
t2
x
x
1 t
1
1 t
  h
Z x
i
1 t=x
1
1
1
= e t
= e x + e.
=
et d
t
t=1
1
Z
2t
Homework 6 (page 414). Find
dt.
2t + 3

Solution. Let u = 2t + 3, then du = ln 2 2t dt, so


Z
Z
1
1
1
1
2t
dt =
du =
ln |u| + C =
ln(2t + 3) + C
t
2 +3
ln 2
u
ln 2
ln 2
Z
tan1 x
dx.
(96 )
Homework 7. Compute the integral
1 + x2

Solution.

Z
tan1 x
1
1
1
dx
=
tan
x
d(tan
x)
=
(tan1 x)2 + C.
2
1+x
2
Z
4
x2 tan x dx.
Homework 8. Find
Z

Solution. Let f (x) = x2 tan x, then f (x) = (x)2 tan(x) = x2 tan x = f (x),
so f (x) = x2 tan x is an odd function. The value of this integral is 0.

6.1

Areas Between Curves (page 422)

Homework 9 (page 428). The curve with equation y 2 = x2 (x+3) is called Tschirnhausens cubic. If you graph this curve you will see that part of the curve forms a
loop. Find the area enclosed by the loop.
Solution. From the equation y 2 = x2 (x + 3), we know that if (x0 , y0) lies on the
graph, then (x0 , y0 ) lies on the graph as well. That means the graph is symmetric
about the x-axis. We solve y 2 = x2 (x + 3) = 0 that x = 0 or x = 3, so we know
the loop lies between x = 3 and x = 0.
The area enclosed by the loop is
Z 0 p
 p

x2 (x + 3) x2 (x + 3)
dx
A=
3
Z 0
Z 0


=
x x + 3 x x + 3 dx = 2
x x + 3 dx.
3

Let u = x + 3, then x = u 3 and du = dx. Upper limit is u = 3 and Lower limit


is u = 0, so
A = 2

x x + 3 dx = 2

(u 3) u du = 2


 3
1
u 2 3u 2 du


 u=3


3
2 5
2
24
= 2 u 2 2u 2
= 2
9 323 3 =
3.
5
5
5
u=0


6.2

Volumes (page 430)

Homework 10. Show that the volume of a right circular cone with height h and
radius r is V = 13 r 2 h.
Solution. We rotate the region bounded by y = hr x, y = 0, and x = h, about the
x-axis to get the volume of a right circular cone:
V =

 x=h

 r 2
1
r 2 1 3
= r 2 h.
x dx = 2
x

h
h
3
3
x=0

2e
Homework 11. Let be the region enclosed by y = 2x1
, y = 1, x = 1, and x = 4.
(99 )
Find the volume generated by revolving about the line x = 21 .

Solution. Notice that the curve y =


revolving about x =

1
2

2e x
2x1

is above y = 1. The volume generated by

is

  x


Z 4

2e
1
1 dx =
2e x (2x 1) dx
V =
2 x
2
2x 1
1
1
Z 4
Z 4
= 2
e x dx
(2x 1) dx = V1 + V2 .
Z

To deal with V1 , let u =

x, then du =

2 x

dx. Upper limit is u = 2 and lower

limit is u = 1. So

h i
Z 4
Z 2
Z 2
Z 2
u=2
u
x
u
u
u
V1 = 2
e dx = 4

e du
ue du = 4
u de = 4 ue
u=1
1
1
1
1

h i u=2 


2
= 4 2e2 e e2 + e = 4e2 .
= 4 2e e eu
u=1

Next, we evaluate V2 :
Z 4
h
i x=4

2
V2 =
(2x 1) dx = x x
= (42 12 4 + 1) = 12.
x=1

Hence the volume is V = 4e2 12.

Homework 12 (page 436). A solid has a circular base of radius 1. Parallel crosssections perpendicular to the base are equilateral triangles. Find the volume of the
solid.
Solution. We take the circle to be x2 + y 2 = 1. The base and a cross-section of the
solid are shown in Figure 1.
y
A
y=

1 x2

x
A

3y
y

B
(a)

(b)

Figure 1: (a) Its base; (b) A cross-section.


The area of each cross-section is

1
A(x) = 2 1 x2 3 1 x2 = 3(1 x2 ).
2
So the volume of the solid is
Z 1
Z 1
Z 1
2
3(1 x ) dx = 2 3
(1 x2 ) dx
V =
A(x) dx =
1

6.3


 x=1



4
1 3
1
=2 3 1
=
=2 3 x x
3.
3
3
3
x=0

Volumes by Cylindrical Shells (page 441)

Homework 13. The region in the first quadrant enclosed by x = y 2 , x-axis and
x = 1 is rotated about the line y = 3. Use the disk method (Section 6.2) and
cylindrical shell method to find the volume of the solid. The answers should be the
same.
(98 )
Solution. First, we use the disk method to get the volume:
Z 1
Z 1



2
2
V =
3 ( x 3) dx =
x + 6 x dx
0
0

 x=1



3
1
7
1
= + 4 = .
= x2 + 4x 2
2
2
2
x=0
4

Next, we use the cylindrical shell method to find the volume:


Z 1
Z 1
2
V =
2(3 y)(1 y ) dy = 2
(y 3 3y 2 y + 3) dy
0
0

 y=1



1
1 2
1
7
1 4
3

= 2
y y y + 3y
1 + 3 = .
= 2
4
2
4
2
2
y=0

Homework 14 (page 445). Use the method of cylindrical shells to find the volume
generated by rotating the region by x = y 2 + 1, x = 2 about y = 2.
Solution. The volume is
Z 1
Z 1
2
V =
2(y + 2)(2 (y + 1)) dy = 2
(y + 2)(1 y 2 ) dy
1
1
Z 1
Z 1
= 2
(y 3 2y 2 + y + 2) dy = 4
(2y 2 + 2) dy
0


 y=1


2
16
2 3

= 4 + 2 = .
= 4 y + 2y
3
3
3
y=0


Calculus A (1) Homework 10 ANSWER


6.5

Average Value of a Function (page 451)

Homework 1 (page 460). The figure shows a curve with the property that, for
every point P on the middle curve y = 2x2 , the areas A and B are equal. Find an
equation for .
(93 )
y
y = 2x2

y = x2

B
A

x
Figure 1: Find an equation for such that the areas A and B are equal.
Solution. Let P : (x, 2x2 ). First, we compute the area A:
Z x
1
Area of A =
(2t2 t2 ) dt = x3 .
3
0
Suppose : x = g(y) = f 1 (x), then the area of B (integration along the y-axis) is
!
Z 2x2 r
t
1
Area of B =
g(t) dt = x3 .
2
3
0
We require Area of A = Area of B, so
!
Z y
Z 2x2 r
t
1 3
g(t) dt = x
2
3
0
0

t
g(t)
2

dt =

1  y  23
.
3 2

By the Fundamental Theorem of Calculus, we get


 y  12
2

1 3  y  12 1
1  y  21
3  y  21
g(y) =
=
g(y) =
.
3 2 2
2
4 2
4 2

To find its inverse function, we solve


 2
 2
y
4x
4x
32
3  y  21
=
y = f (x) = 2
= x2 .
x=
4 2
2
3
3
9

7.1

Integration by Parts (page 464)

Homework 2. Find

x2 sin(3x) dx.

(89 )

Solution.


Z
Z
Z
1
1
2
2
2
2
x sin(3x) dx =
x d cos(3x) =
x cos(3x) cos(3x) dx
3
3
Z
2
1 2
x cos(3x) dx
= x cos(3x) +
3
3
Z
2
1 2
x d sin(3x)
= x cos(3x) +
3
9


Z
2
1 2
x sin(3x) sin(3x) dx
= x cos(3x) +
3
9
Z
1 2
2
2
= x cos(3x) + x sin(3x)
sin(3x) dx
3
9
9
1
2
2
= x2 cos(3x) + x sin(3x) +
cos(3x) + C.
3
9
27
Z
Homework 3 (page 465). Evaluate (ln x)2 dx.

Solution.
Z
Z
Z
1
2
2
2
2
(ln x) dx = (ln x) x x d(ln x) = x(ln x) 2x(ln x) dx
x


Z
Z
2
2
= x(ln x) 2 ln x dx = x(ln x) 2 (ln x)x x d(ln x)
Z
Z
1
2
2
= x(ln x) 2x(ln x) + 2 x dx = x(ln x) 2x(ln x) + 2 1 dx
x

= x(ln x)2 2x(ln x) + 2x + C.


Z
Homework 4. Evaluate the integral
x ln x dx, R.

(97 )

Solution. If 6= 1,


Z
Z
Z
1
1

+1
+1
+1
x ln x dx =
ln x dx
=
(ln x)x
x
d ln x
+1
+1
Z
Z
1
x+1 ln x
1
x+1 ln x
+1 1
x
dx =
x dx

=
+1
+1
x
+1
+1
x+1 ln x
x+1
=

+ C.
+1
( + 1)2
If = 1,
Z

x ln x dx =

ln x
dx =
x

Z
2

1
ln x d ln x = (ln x)2 + C.
2

Homework 5 (page 465). Evaluate


Solution.
Z

2 x

x e dx =

x2 ex dx.

2 x

dx.

2 x

x de = x e e dx = x e 2xex dx


Z
Z
2 x
x
2 x
x
x
= x e 2 x de = x e 2 xe e dx
Z
2 x
x
= x e 2xe + 2 ex dx = x2 ex 2xex + 2ex + C.
x

Homework 6. Evaluate the integral

esin

(94 )

Solution. Let u = sin1 x, then x = sin u and dx = cos u du, so we have


Z
Z
Z
Z
sin1 x
u
u
u
I= e
dx = e cos u du = e d sin u = e sin u sin u deu
Z
Z
u
u
u
= e sin u e sin u du = e sin u + eu d cos u
Z
Z
u
u
u
u
u
= e sin u + e cos u cos u de = e sin u + e cos u eu cos u du
= eu sin u + eu cos u I.

Hence
Z
1
1
1
1
1
esin x dx = (eu sin u + eu cos u) = (esin x sin(sin1 x) + esin x cos(sin1 x))
2
2


1 sin1 x 
x + 1 x2
= e
2
Z 1
Z 1
1 2
Homework 7. Evaluate
x tan (x ) dx and
x(tan1 x)2 dx (101 )
0

Solution.
Z
Z
Z 1
1 u=1
1 1
1 2
2
1 2
tan (x ) dx =
tan1 u du
x tan (x ) dx =
2 0
2 u=0
0


u=1 Z 1
1
1
1

u d tan u
u tan u
=
2
u=0
0
Z
Z
1 1 1 u
1 1 1
=
du =
d(u2 + 1)
2
2
2 4 2 0 1+u
8 4 0 u +1
u=1 1
1

= ln 2.
= ln(1 + u2 )
u=0
8 4
8 4
Remark that the above calculation gives
Z 1
1
tan1 x dx = ln 2.
4 2
0
3

Z
1 1
(tan1 x)2 dx2
x(tan x) dx =
2 0
0


Z 1

x=1
1
2
1
2
1
2 2

x d(tan x)
=
(tan x) x
2
x=0
0
Z
1  2 1 1 2
1
=
x 2 tan1 x
dx

2 4
2 0
1 + x2


Z 1
Z 1
1  2
1  2
x2
1
1
1
=
dx =
dx

tan x
tan x 1
2 4
1 + x2
2 4
1 + x2
0
0
Z 1
Z 1
1  2
1
1
=
dx

tan x dx +
tan1 x
2 4
1 + x2
0
0
Z 1
1  2 1
=
+ ln 2 +
tan1 x d tan1 x
2 4
4 2
0
 1



2

1
1  2 1
1  2
1
1
1
2
=
=
(tan x)
+ ln 2 +
+ ln 2 +
2 4
4 2
2
2 4
4 2
2 4
x=0
 2 1
+ ln 2.
=
4
4 2
Z

Homework 8. Find the volume of the solid obtained by rotating the region bounded
by the given curves about the specified line.
(95 )

(a) y = 1 + sin x, y = 0, and 0 x 2; about the y-axis.

(96, 97, 102 )

(b) y = cos x, y = 0, and 0 x 2 ; about the y-axis.


Solution.
(a) We use the cylindrical shell method to get the volume:
Z 2
Z 2
Z
Volume =
2x(1 + sin x) dx = 2
x dx + 2
0

x sin x dx = I + II,

where
I = 2

  x=2
x dx = x2 x=0 = 4 3 ,

and
II = 2

x sin x dx = 2
x d cos x
0


Z 2
x=2

2
2
= 4 2 .
= 2 [x cos x]|x=0
cos x dx = 4 + 2[sin x]
0

x=0

Hence we get Volume = 4 3 4 2 .

(b) We use the cylindrical shell method to get the volume:


Z
Z
2
2
2x cos x dx = 2
x d sin x
Volume =
0
0
!
x=
x= Z 2
2
2
sin x dx = 2 + 2[cos x]

= 2 2.
= 2 [x sin x]
x=0

7.2

x=0

Trigonometric Integrals (page 471)

Homework 9 (page 471). Evaluate

sin5 x cos2 x dx.

Solution.
Z
Z
Z
5
2
4
2
sin x cos x dx = sin x cos x d cos x = (1 cos2 x)2 cos2 x d cos x
Z
= (1 2 cos2 x + cos4 x) cos2 x d cos x
Z
= ( cos2 x + 2 cos4 x cos6 x) d cos x
1
2
1
= cos3 x + cos5 x cos7 x + C.
3
5
7
Z
Homework 10 (page 472). Evaluate
sin4 x dx.

Solution. By half-angle formula,


2
Z
Z
Z 
1
1 cos 2x
4
(1 2 cos 2x + cos2 2x) dx
dx =
sin x dx =
2
4

Z 
1
1
1
1 + cos 4x
= x sin 2x +
dx
4
4
4
2
1
1
1
1
3
1
1
= x sin 2x + x +
sin 4x + C = x sin 2x +
sin 4x + C.
4
4
8
32
8
4
32
Z
Homework 11 (page 475). Evaluate
tan3 x dx.
Solution.
Z
Z
Z
Z
3
2
2
tan x dx = (sec x 1) tan x dx = sec x tan x dx tan x dx
Z
Z
1
= sec x d sec x tan x dx = (sec x)2 ln | sec x| + C.
2
Remark that
Z
Z
Z
1
sin x
dx =
d cos x = ln | cos x| + C = ln | sec x| + C.
tan x dx =
cos x
cos x
5

Homework 12 (page 475). Evaluate

sec3 x dx.

(97 )

Solution. Since
Z
Z
Z
3
I = sec x dx = sec x d tan x = sec x tan x tan x d sec x
Z
Z
2
= sec x tan x sec x tan x dx = sec x tan x sec x(sec2 x 1) dx
Z
Z
3
= sec x tan x sec x dx + sec x dx = sec x tan x + ln | sec x + tan x| I,
we have
I=

sec3 x dx =

1
(sec x tan x + ln | sec x + tan x|) + C.
2

Remark that
Z
Z
Z
sec2 x + sec x tan x
sec x + tan x
dx =
dx
sec x dx = sec x
sec x + tan x
sec x + tan x
Z
1
d(sec x + tan x) + ln | sec x + tan x| + C.
=
sec x + tan x
Z
Homework 13 (page 473). Evaluate
tan6 x sec4 x dx.
Solution.
Z
Z
Z
6
4
6
2
tan x sec x dx = tan x sec x d tan x = tan6 x(tan2 x + 1) d tan x
Z
1
1
= (tan8 x + tan6 x) d tan x = tan9 x + tan7 x + C.
9
7
Z
Homework 14 (page 474). Evaluate
tan5 x sec7 x dx.
Solution.
Z
Z
Z
5
7
4
6
tan x sec x dx = tan x sec x d sec x = (sec2 x 1)2 sec6 x d sec x
Z
= (sec4 x 2 sec2 x + 1) sec6 x d sec x
Z
= (sec10 x 2 sec8 x + sec6 x) d sec x
=

1
2
1
sec11 x sec9 x + sec7 x + C.
11
9
7

7.3

Trigonometric Substitution (page 478)


Z

9 x2
dx.
x2
Solution. Let x = 3 sin , 2 2 , then dx = 3 cos d. So we have
Z
Z p
Z
Z
9 x2
9 9 sin2
cos2
dx =
3 cos d =
d = cot2 d
x2
9 sin2
sin2

Z
 
9 x2
1 x
2
+ C.
sin
= (csc 1) d = cot + C =
x
3
Z 2
1

Homework 16. Evaluate the integral


dx.
(94 )
2
1 + x2
1 x

Homework 15 (page 479). Evaluate

Solution. Let x = tan , 2 < x < 2 , then dx = sec2 d. Upper limit is =


tan1 (2), and lower limit is = 4 .
Z tan1 (2)
Z 2
Z tan1 (2)
1
cos
1
2

sec d =
d
dx =
2
2
2

tan sec
sin2
1+x
1 x
4
4


 =tan1 (2)
Z tan1 (2)
1
1
5
=
d sin =
+ 2.
=
2

sin

sin

=
4
4
Z
2

dx, x > 1.
(89 )
Homework 17. Find the integral
x3 x2 1
Solution. Let x = sec , 0 < 2 or < 32 , then dx = sec tan d. So
Z
Z
Z
2
2
2

sec tan d =
d
dx =
3
3
2
sec tan
sec2
x x 1
Z
Z
sin 2
2
= 2 cos d = (1 + cos 2) d = +
+C
2

x2 1
1
2
+ C.
= + sin cos + C = tan ( x 1) +
x2
Z
x

Homework 18. Evaluate the integral


dx.
(98 )
x2 + 2x + 2
Solution. Since
Z
Z
x
x

p
dx,
dx =
2
x + 2x + 2
(x + 1)2 + 1
let x + 1 = tan , 2 < < 2 , then dx = sec2 d, so the integral becomes
Z
Z
Z
x
tan 1
2

sec d = (sec tan sec ) d


dx =
sec
x2 + 2x + 2
= sec ln | sec + tan | + C

2

2
= x + 2x + 2 ln x + 2x + 2 + x + 1 + C.
7

Homework 19 (page 484). Find the area of the crescent-shaped region (called a
lune) bounded by arcs of circles with radii r and R.

r
R

Figure 2: The crescent-shaped region.

Solution. We set up the coordinates as follows:


y

r
x

Figure 3: The crescent-shaped region.

For small circle, the center is (0, R2 r 2 ) and radius is r, so the equation is

x2 + (y R2 r 2 )2 = r 2 . For big circle, the center is (0, 0) and radius is R, so the

equation is x2 + y 2 = R2 . The intersections of two circles are (r, R2 r 2 ) and

(r, R2 r 2 ).
The area of the crescent-shaped region is
Z r

Area =
( R2 r 2 + r 2 x2 R2 x2 ) dx
r
Z r

=2
( R2 r 2 + r 2 x2 R2 x2 ) dx = I+II+III,
0

where
I=2

Z r

R2

r2

dx = 2

R2

r2

  x=r
= 2r R2 r 2 .
x
x=0

Let x = r sin , 2 < < 2 , then dx = r cos d. Upper limit is =


limit is = 0, so
Z r
Z
2
2
II = 2
r x dx = 2

sin r cos d = 2
0
0

 = 2
Z

2
sin
2


= r2
(1 + cos 2) d = r 2 +
= r2.

2
2
0
=0
r2

r2

and lower

r 2 cos2 d

Let x = R sin , 2 2 , then dx = R cos d. Upper limit is = sin1 ( Rr ),


and lower limit is = 0, so
III = 2

r
)
sin1 ( R

R2

R2

sin R cos d = 2

r
)
sin1 ( R

R2 cos d

 =sin1 ( Rr )
sin 2
= R
(1 + cos 2) d = R +

2
0
=0



r r
r
2 r2

R
= R2 sin1
+
= R2 sin1
r R2 r 2 .
R
R
R
R
2

r
sin1 ( R
)

Hence
r

Area = 2r R2 r 2 + r 2 R2 sin1
r R2 r 2
2
 rR

2
2
1
.
= r R2 r 2 + r R sin
2
R

Remark that the crescent-shaped area is half circle ( 2 r 2 ) plus triangular with

base 2r and height R2 r 2 (r R2 r 2 ) minus sector with radius R and angle




sin1 Rr (R2 sin1 Rr ).

Calculus A (1) Homework 11 (Due 12/10)


7.4

Integration of Rational Functions by Partial


Fractions (page 484)

Homework 1. Evaluate

2
1

x2 + 4
dx.
x4 + 3x3 + 2x2

(99 )

Solution.
Z

x2 + 4
dx =
x4 + 3x3 + 2x2

x2 + 4
dx
2
1 x (x + 1)(x + 2)

Z 2
A3
A4
A1 A2
dx,
+ 2 +
+
=
x
x
x+1 x+2
1
Z

We will determine A1 , A2 , A3 , and A4 by comparing


A1 x(x + 1)(x + 2) + A2 (x + 1)(x + 2) + A3 x2 (x + 2) + A4 x2 (x + 1) = x2 + 4
We take x = 0, then 2A2 = 4 A2 = 2.
We take x = 1, then A3 = 5.
We take x = 2, then 4A4 = 8 A4 = 2.
We take x = 1, then 6A1 + 6A2 + 3A3 + 2A4 = 5 A1 = 3.
So
Z

x2 + 4
dx =
x4 + 3x3 + 2x2



2
5
2
3
dx

+ 2+
x x
x+1 x+2

 x=2

2
= 3 ln |x| + 5 ln |x + 1| 2 ln |x + 2|
x
x=1



2 2 2
4
3
= 3 ln + + 5 ln 2 ln
1
2 1
2
3


= 1 12 ln 2 + 7 ln 3.
Z
1
dx.
Homework 2. Evaluate the integral
3
x +1
Solution.
Z

1
dx =
3
x +1

1
dx =
(x + 1)(x2 x + 1)

Z 

(93 )

A2 x + B2
A1
+ 2
x+1 x x+1

We will determine A1 , A2 , an B2 by comparing


A1 (x2 x + 1) + (A2 x + B2 )(x + 1) = 1.
1

dx.

We take x = 1, then 3A1 = 1 A1 = 31 .


We take x = 0, then A1 + B2 = 1 B2 = 32 .
We take x = 1, then A1 + 2(A2 + B2 ) = 1 A2 = 31 .
So
Z


13 x + 23
dx
+
x + 1 x2 x + 1
Z
Z
Z
1
2x 1
1
1
1
1
dx
dx +
dx
=
2
2
3
x+1
6
x x+1
2
x x+1
Z
Z
Z
1
1
1
1
1
1
2
=
dx
d(x

x
+
1)
+
dx
1
3
x+1
6
x2 x + 1
2
(x 2 )2 + 34

1
x

1
1
1 1
= ln |x + 1| ln |x2 x + 1| + q tan1 q 2 + C
3
6
2 3
3
4
4


1
1
1
2x 1
2
1

= ln |x + 1| ln |x x + 1| + tan
+ C.
3
6
3
3
Z
1
dx.
(100 )
Homework 3. Find the integral
3
x + x2 + x
1
dx =
3
x +1

Solution.
Z

Z 

1
3

1
dx =
3
x + x2 + x

1
dx =
2
x(x + x + 1)

Z 

A2 x + B2
A1
+ 2
x
x +x+1

dx.

We will determine A1 , A2 , an B2 by comparing


A1 (x2 + x + 1) + (A2 x + B2 )x = (1 + A2 )x2 + (A1 + B2 )x + A1 = 1.
So A1 = 1, A2 = 1, and B2 = 1. Thus

Z
Z 
1
x 1
1
dx
dx =
+
x3 + x2 + x
x x2 + x + 1
Z
Z
Z
2x + 1
1
1
1
1
dx
dx

dx
=
x
2
x2 + x + 1
2
x2 + x + 1
Z
Z
1
1
1
1
2
= ln |x|
d(x + x + 1)
1 2
2
2
x +x+1
2
(x + 2 ) +

1
x+
1
1 1
= ln |x| ln |x2 + x + 1| q tan1 q 2 + C
2
2 3
3
4
4


1
1
2x + 1
2
1

= ln |x| ln |x + x + 1| tan
+ C.
2
3
3
2

3
4

dx + C

Homework 4. Evaluate the integral


Solution.
Z

1
dx =
3
x 1

x3

1
dx.
1

1
=
(x 1)(x2 + x + 1)

Z 

(98 )

A1
A2 x + B2
+ 2
x1 x +x+1

dx.

We will determine A1 , A2 , an B2 by comparing


A1 (x2 + x + 1) + (A2 x + B2 )(x 1) = 1.
We take x = 1, then 3A1 = 1 A1 = 13 .
We take x = 0, then A1 B2 = 1 B2 = 32 .
We take x = 1, then A1 2(A2 + B2 ) = 1 A2 = 31 .
So
Z


13 x 32
dx
+
x 1 x2 + x + 1
Z
Z
Z
1
1
2x + 1
1
1
1
=
dx
dx
dx
2
2
3
x1
6
x +x+1
2
x +x+1
Z
Z
Z
1
1
1
1
1
1
2
dx
=
dx
d(x + x + 1)
1 2
2
3
x1
6
x +x+1
2
(x + 2 ) + 34

1
x+
1
1
1 1
= ln |x 1| ln |x2 + x + 1| q tan1 q 2 + C
3
6
2 3
3
4
4


1
1
2x + 1
1
2
1

+ C.
= ln |x 1| ln |x + x + 1| tan
3
6
3
3
Z 3
x x2 + x + 1
Homework 5. Evaluate
dx.
(102 )
x3 + x2 + x + 1
1
dx =
3
x 1

Z 

1
3

Solution.

Z
Z 
Z 3
2x2
A1
A2 x + B2
x x2 + x + 1
dx.
dx = 1 +
dx =
1+
+
x3 + x2 + x + 1
(x + 1)(x2 + 1)
x+1
x2 + 1
We will determine A1 , A2 , an B2 by comparing
A1 (x2 + 1) + (A2 x + B2 )(x + 1) = (A1 + A2 )x2 + (A2 + B2 )x + (A1 + B2 ) = 2x2 .
We take x = 1, then 2A1 = 2 A1 = 1.
We take x = 0, then A1 + B2 = 0 B2 = 1.
We take x = 1, then 2A1 + 2(A2 + B2 ) = 2 A2 = 1.
3

So
x3 x2 + x + 1
dx =
x3 + x2 + x + 1


1
x + 1
dx
1+
+ 2
x+1
x +1
Z
Z
Z
Z
2x
1
1
1
dx
dx
+
dx
= 1 dx
x+1
2
x2 + 1
x2 + 1
1
= x ln |x + 1| ln |x2 + 1| + tan1 x + C.
2

Z p
1+ x
Homework 6. Evaluate the integral
dx.
(97 )
x
p

Solution. Let u = 1 + x, then x = u2 1 and


Z

du =

So
Z p

1+
x

Z 

1
1
1
1
2
p
dx = p
dx dx = 4(u 1)u du.
2 1+ x2 x
4 x 1+ x
dx =

4u2 (u2 1)
du =
(u2 1)2

4u2
du =
u2 1

Z 
4

2
2
+
u+1 u1

du

= 4u 2 ln |u + 1| + 2 ln |u 1| + C
q
q


q






= 4 1 + x 2 ln 1 + x + 1 + 2 ln 1 + x 1 + C.
Z
1
dx.
(96 )
Homework 7. Find the integral
2 + sin x

2t
2
Solution. Let t = tan x2 , < x < , then we have sin x = 1+t
2 and dx = 1+t2 dt.
So
Z
Z
Z
Z
1
1
1
1
2
dx =
dt =
dt =
dt
2
2t
2
2
2 + sin x
t +t+1
2 + 1+t2 1 + t
t + 21 + 43



1
2t + 1
1
2
1
1 t + 2
q

+C
= q tan
+ C = tan
3
3
3
3
4
4


2 tan( x2 ) + 1
2
1

= tan
+ C.
3
3
Z
2
1
dx.
(99 )
Homework 8. Evaluate
0 3 5 sin x

2t
2
Solution. Let t = tan x2 , < x < , then we have sin x = 1+t
2 and dx = 1+t2 dt.
Upper limit is t = 1 and lower limit is t = 0. So
Z
Z 1
Z 1
2
1
1
2
2
dx =
dt. =
dt
2t
2
2
1+t
0 3 5 sin x
0 3 5 1+t2
0 3t 10t + 3
Z 1
2
dt.
=
0 (t 3)(3t 1)
4

Be careful that this is an improper integral! (Sorry, I should put this homework in
section 7.8.) This is because there is an infinity discontinuity at t = 13 . So
Z

2
dt
0 (t 3)(3t 1)
Z s
Z 1
2
2
= lim
dt + lim+
dt
1
1
s 3
s 3
0 (t 3)(3t 1)
s (t 3)(3t 1)

1
dx =
3 5 sin x

= I + II.
For the first term, we have

Z s
Z s 1
43
2
4
I = lim
dt
dt = lim
+
t 3 3t 1
s 31
s 31
0 (t 3)(3t 1)
0

 t=s

1
3
= lim
ln |t 3| ln |3t 1|
3
4
s 31
t=0


1
3
1
= lim
ln |s 3| ln |3s 1| ln 3.
1
3
4
3
s 3
The limit does not exist, so the improper integral does not converge. We can check
the limit of the second term does not exist as well:

Z 1
Z 1 1
43
2
4
dt
dt = lim+
+
II = lim+
t 3 3t 1
s 31
s 31
s (t 3)(3t 1)
s

 t=1

1
3
= lim+
ln |t 3| ln |3t 1|
3
4
s 31
t=s


1
1
3
3
= lim+
ln 2 ln |s 3| ln 2 + ln |3s 1| .
3
3
4
4
s 31
The above limit does not exist because lim+ 34 ln |3s 1| does not exist.
s 31

7.5

Strategy for Integration (page 494)

Homework 9. Find the integral

1
p
dx.
x 1 + (ln x)2

(90 )

Solution. Let ln x = tan , then x1 dx = sec2 d. Upper limit is = 4 and lower


limit is = 0. So
Z
Z
Z e
4
4
1
1
2

p
dx =
sec d
sec d =
2
2
1 + tan
0
0
1 x 1 + (ln x)


h
i = 4



2
+
1
= ln | sec + tan |
= ln
.
=0

Homework 10. Find the integral

x
dx.
8 2x2 x4

(95 )

Solution. Let u = x2 , then du = 2x dx, so


Z
Z
Z
1
1
1
1
x

p
du.
dx =
du =
2
4
2
2
2
8 2x x
8 2u u
(u + 1)2 + 9

We use the Substitution Rule again that let u + 1 = 3 sin , then du = 3 cos d, so
Z
Z
Z
1
x
1
1
1

p
p
3 cos d
du =
dx =
2
2
8 2x2 x4
(u + 1)2 + 9
(3 sin )2 + 32


Z
1
1
1 1 u + 1
=
d = + C = sin
+C
2
2
2
3


1 1 x2 + 1
+ C.
= sin
2
3
Homework 11. Find the limit lim
x

R x2
0

etx (2t2 + 1) dt
.
x4

(96 )

Hint: ,
Solution. We first compute
Z

x2

tx2

x2

(2t + 1) dt = e

x2

t 2

e t dt +

x2

e dt .
t

Since
2

x2

et t2 dt = 2

x2

t2 det = 2

t=0

h i t=x2 Z

t2 et

x2

= 2x4 e

x2

x2

t det = 2x4 e

= 2x4 ex 4x2 ex

x2

2tet dt

i t=x2 Z
t

te
t=0

x2

et dt

h it=x2
2
2
2
= 2x4 ex 4x2 ex + 4ex 4
+ 4 et
t=0

and
Z
we have
lim
x

R x2
0

x2

h i t=x2
2

= ex 1,
e dt = et
t

t=0

etx (2t2 + 1) dt
2x4 4x2 + 4 4ex + 1 ex
=
lim
x
x4
x4


5
5
4
= lim 2 2 + 4 4 x2 = 2.
x
x
x
xe
6


and y =
Homework 12. Consider the region bounded by the curves y = sin x
2
6
for 0 x 1. Note that the two curves meet at x = 1. Find the volume
(x2 +3x+2)
of revolving the region (a) about the y-axis; (b) about x-axis.
(101 )
Solution.
(a) We use the method of cylindrical shell:

Z 1
 x 
6
dx
sin
Volume =
2x
x2 + 3x + 2
2
0
Z 1
Z 1
 x 
x
= 12
dx = I + II,
dx 2
x sin
2
0 (x + 1)(x + 2)
0
where

I = 12
= 12

1
0

h

x
dx = 12
(x + 1)(x + 2)

0
i x=1

ln |x + 1| + 2 ln |x + 2|

x=0

2
1
+
x+1 x+2


dx

= 12 ( ln 2 + ln 1 + 2 ln 3 2 ln 2) = 12 (2 ln 3 3 ln 2)
and
Z

 x 

 x 

dx = 4
x d cos
II = 2
x sin
2
2
0
0
h
 x  i x=1 Z 1
 x  

= 4 x cos
dx
cos

2
2
x=0
0
h
 x  i x=1
2
8

= 4 sin
= .

x=0

Hence

Volume = 12 (2 ln 3 3 ln 2)

8
.

(b) We use the disk method to get


!

2
Z 1
 x 
6
Volume =

dx
sin2
2 + 3x + 2
x
2
0
Z 1
Z 1
 
1
2 x
dx = I + II.
dx

sin
= 36
2
2
2
0
0 (x + 1) (x + 2)
We compute the first term:
Z 1
1
dx
I = 36
2
2
0 (x + 1) (x + 2)

Z 1
A2
A3
A4
A1
+
+
+
dx,
= 36
x + 1 (x + 1)2 x + 2 (x + 2)2
0
7

where A1 , A2 , A3 , and A4 are determined by comparing


A1 (x + 1)(x + 2)2 + A2 (x + 2)2 + A3 (x + 1)2 (x + 2) + A4 (x + 1)2 = 1.
We take x = 1, then A2 = 1.
We take x = 2, then A4 = 1.
We take x = 0, then 4A1 + 4A2 + 2A3 + A4 = 1 2A1 + A3 = 2.
We take x = 1, then 18A1 + 9A2 + 12A3 + 4A4 = 1 3A1 + 2A3 = 2
A1 = 1, A3 = 2.
Thus
I = 36

2
1
2
1
+
+
+
2
x + 1 (x + 1)
x + 2 (x + 2)2

dx
 x=1


1
1

+ 2 ln |x + 2|
= 36 2 ln |x + 1|
x+1
x + 2 x=0


1 1
1 1
= 36 2 ln 2 + 2 ln 1 + + 2 ln 3 2 ln 2 +
2 1
3 2


2
.
= 36 4 ln 2 + 2 ln 3 +
3
0

For the second term, we have



Z 1
Z 1
 
1 cos x
2 x
II =
sin
dx =
dx
2
2
0
0
 x=1


sin x
1
= .
x
=

2
2
2
x=0

Hence Volume = 36 4 ln 2 + 2 ln 3 + 23 2 = 72(2 ln 2 + ln 3) +

7.8

47
.
2

Improper Integrals (page 519)

Homework 13. Determine the real number a so that the integral



Z 2
1
1

dx

x x2 + 4 ax
0
(100 )

converges.

1
Solution. The function xx12 +4 ax
has infinity discontinuity at x = 0, so the
integral is improper and


Z 2
Z 2
1
1
1
1

dx = lim
dx

t0 t
x x2 + 4 ax
x x2 + 4 ax
0

Z 2
Z 2
1
1

dx = lim (I + II) .
dx
= lim
2
t0
t0
t ax
t x x +4

Let x = 2 tan , then dx = 2 sec2 d. Upper limit is =


=

tan1 ( 2t ),

and lower limit is

so the first integral becomes


2

1
2 sec2 d
t
tan1 ( 2 ) 2 tan 2 sec
t
Z
i = 4
1 4
1h

=
csc d =
ln | csc + cot |
2 tan1 ( 2t )
2
=tan1 ( 2t )

 




1

t
1
t

1
1

+ cot tan
= ln 2 + 1 + csc tan

2
2
2
2


1 t2 + 4 + 2
1



= ln 2 + 1 + ln
.

2
2
t

I=

dx =
x x2 + 4

For the second integral, we have


Z 2
i x=2
1
1h
1
1

= ln 2 + ln t.
ln |x|
dx =
II =
a
a
a
x=t
t ax
So we get


1
1

dx

x x 2 + 4 ax
0
!

1
1 t2 + 4 + 2 1
1

= lim
+
ln
ln
t

ln
2
+
1


ln 2
t0


2
t
a
2
a





1
1
1
1

1
= lim ln ( t2 + 4 + 2) 2 t a 2 ln 2 + 1 ln 2
t0
2
a

1
 1 1 
1




= lim ln t a 2 + ln 2 ln 2 + 1 ln 2.
t0
2
a
Z

The above limit exists if and only if a = 2, so the integral converges if and only if
a = 2.
Homework 14. Let In =

xn ex dx.
0

(a) Compute I3 .

(95 )

(b) Find the general formula of In .

(98 )

Solution.

(a) We compute the indefinite integral first:




Z
Z
Z
3 x
3
x
3 x
x
3
x e dx = x de = x e e dx
Z
Z
3 x
2 x
3 x
= x e + 3 x e dx = x e 3 x2 dex


Z
Z
3 x
2 x
3 x
2 x
x
2
= x e 3 x e e dx = x e 3x e + 6 xex dx


Z
Z
3 x
2 x
x
3 x
2 x
x
x
= x e 3x e 6 xde = x e 3x e 6 x e e dx
Z
3 x
2 x
x
= x e 3x e 6x e + 6 ex dx
= x3 ex 3x2 ex 6x ex 6ex + C.

So
Z

3 x

x e dx = lim
x3 ex dx
t
0
0
h
i x=t
3 x
2 x
x
x
= lim x e 3x e 6x e 6e
t
x=0

3 t
2 t
t
t
= lim t e 3t e 6t e 6e + 6 = 6.

I3 =

Remark that for every n N, by LHospital Rule, we have


tn
t et

lim tn et = lim
t

(
,L)

(
,L)
ntn1
n!

=
lim t = 0.
t
t
t
e
e

lim

(b) First we show the reduction formula In = nIn1 :


Z
Z t
Z t
n x
n x
In =
x e dx = lim
x e dx = lim
xn dex
t
t
0
0

0

h
Z t
i x=t Z t

n t
x
n
n1 x
n x

e dx = lim t e + n
x e dx
= lim x e
t

x=0

= nIn1 .

We compute
Z
Z t
h
i x=t

x
x
x
= lim et + 1 = 1.
I0 =
e dx = lim
e dx = lim e
0

x=0

Hence In = nIn1 = n (n 1)In1 = = n!I0 = n!.

10


n
Homework 15. Find the limit lim
n

n!
.
n

(96 )

Remark: We have derived the integral expression of this limit in Homework 8,


section 5.2.
Solution. Recall from Homework 8 (section 5.2.) that
!

1
n
n
n!
(n (n 1) (n 2) 3 2 1)
= lim ln
lim ln
n
n
n
n
1

n n1 n2
3 2 1 n
= lim ln


n
n
n
n
n n n


n
n
X
i
1X
f (xi )x,
ln
= lim
= lim
n
n n
n
i=1
i=1
where f (x) = ln x, and we divide [0, 1] into n-subintervals with width x = n1 , and
, i ], i = 1, . . . , n. Hence
we choose sample points xi = ni on each subinterval [ i1
n n

Z 1
n
n!
lim ln
=
ln x dx.
n
n
0
This is an improper integral (since ln x as x 0). So we have to deal with
it carefully:


h
Z 1
Z 1
n
i x=1 Z 1
n!
1

x dx
=
ln x dx. = lim
ln x dx = lim x ln x
lim ln
t0 t
t0
n
x=t
n
x
t
0
= lim (t ln t 1 + t) = 1.
t0

because
lim t ln t = lim
t0

t0

ln t
1
t

,L

= lim
t0

1
t

t12

= lim t = 0.
t0

Hence
!

n
n
n!
n!
= 1 lim
lim
= e1 .
n n
n n

n!
= ln
lim ln
n
n

Homework 16. Is the improper integral

Solution. The function f (x) =


Z

1
dx = lim
t1
(1 x)2

1
(1x)2
t

2
0

1
dx convergent or divergent?
(1 x)2
(90 )

has infinity discontinuity at x = 1, so

1
dx + lim+
t1
(1 x)2
11

2
t

1
dx = I + II.
(1 x)2

Since
1
dx = lim
I = lim
2
t1
t1
0 (1 x)


1
= lim
1 ,
t1
1t
Z

t
0

h 1 i x=t
1

d(1 x) = lim
t1
(1 x)2
1 x x=0

the limit does not exist. So the improper integral divergent.


Homework 17. Find the volume of solid generated by revolving the curve y =
ex sin x, x 0, about the x-axis.
(100 )
Solution. The volume of solid is
Z

(ex sin x)2 dx,

which is an improper integral. We compute the following indefinite integral first:


Z
Z
Z
1
x
2
2x
2
(e sin x) dx = e
sin x dx =
sin2 x de2x
2


Z
Z
1 2 2x
1
2
2x
2x
2
sin x e
e
d sin x = sin x e
+ e2x sin x cos x dx
=
2
2
Z
1 2 2x 1
= sin x e
sin x cos x de2x

2
2


Z
1 2 2x 1
2x
2x
sin x cos xe
e
d sin x cos x

= sin x e
2
2
Z
1
1 2 2x 1
2x
e2x (cos2 x sin2 x) dx
sin x cos x e
+
= sin x e
2
2
2
Z
1 2 2x 1
1
2x
= sin x e
e2x (1 2 sin2 x) dx
sin x cos x e
+
2
2
2
Z
Z
1
1 2 2x 1
2x
2x
e
dx e2x sin2 x dx
sin x cos x e
+
= sin x e
2
2
2
Z
1 2 2x 1
1 2x
2x
= sin x e
sin x cos x e
e
(ex sin x)2 dx + C .
2
2
4
So we get
Z

1
1
1
(ex sin x)2 dx = sin2 x e2x sin x cos x e2x e2x + C.
4
4
8

So
Z

(e sin x) dx = lim
(ex sin x)2 dx
t
0
0

 x=t
1 2 2x 1
1 2x
2x
= lim sin x e
sin x cos x e
e

t
4
4
8
x=0


1
1 2t 1
1 2 2t 1
2t
= .
= lim sin t e sin t cos t e e +
t
4
4
8
8
8
x

12

Remark that because | sin2 t e2t | e2t and | sin t cos t e2t | e2t , by the Squeeze
Theorem, we get lim sin2 t e2t = 0 and lim sin t cos t e2t = 0.
t

13

Calculus A (1) Homework 12 ANSWER


8.1

Arc Length (page 538)

Homework 1. Let f (x) = ln x 81 x2 , 1 x 2. Find the length of the graph of f .


(90 )
Solution. We compute f (x) = x1 41 x and
s
s

2 r
2
p
1 1
1
1 2
1
1 1

2
x = 1+ 2 + x =
+ x
1 + (f (x)) = 1 +
x 4
x
2 16
x 4
1 1
= + x.
x 4
So
Length =

1+

3
= ln 2 + .
8

(f (x))2

dx =

 x=2


1 1
1 2
+ x dx = ln |x| + x
x 4
8
x=1

Homework 2. Find the length of the curve y =

1
x2

t3 + 1 dt,

1
2

x 1.

(94 )

Solution. By the Fundamental Theorem of Calculus, we compute


s 
3
1
2

+1
f (x) = 3
x
x2
s
s

 r
2
p
2
1
4
2
4
4

2
1+ 6
+ 1 = 1 + 6 + 12 =
= 1 + 6.
1 + (f (x)) = 1 + 6
6
x x
x
x
x
x
So

Length =

1
1

Z
p

2
1 + (f (x)) dx =

1
1




 x=1
2
2 1
1 + 6 dx = x 5
x
5 x x= 1

2
 2
1 2
129
= 1 (1 32) =
.
2 5
10

Homework 3. Given 0 < a < b, find the arc length of y = ln

ex +1
ex 1

Solution. We compute
f (x) =

ex 1 (ex 1)ex (ex + 1)ex


2ex

=
ex + 1
(ex 1)2
e2x 1
1

for a x b.
(98 )

1 + (f (x))2 =
=

1+

2ex
e2x 1

e2x + 1
.
e2x 1

2

(e2x 1)2 + (2ex )2


=
(e2x 1)2

(e2x + 1)2
(e2x 1)2

So


Z b x
Z b  2x
Z bp
e + ex
e +1

2
dx =
dx
1 + (f (x)) dx =
Length =
e2x 1
ex ex
a
a
a
 b


Z b
h
b
i x=b
1
e

e
x
x
x
x
=
= ln a
d(e e ) = ln e e
.
x=a
ex ex
e ea
a

8.2

Area of a Surface of Revolution (page 545)

Homework 4.
Z
(a) Find sec3 x dx.
(b) The curve y = ln x, 0 < x < 1, is rotated about y-axis. Find the area of the
resulting surface.
(99 )
Solution.
(a) Since
Z
Z
Z
3
I = sec x dx = sec d tan = sec tan tan d sec
Z
Z
2
= sec tan tan sec d = sec tan (sec2 1) sec d
Z
Z
Z
2
3
= sec tan (sec 1) sec d. = sec tan sec d + sec d
= sec tan I + ln | sec + tan | + C,

we get
I=
We compute f (x) =

1
1
sec tan + ln | sec + tan | + C.
2
2
1
x

and

p
1 + (f (x))2 =

 2 r
1
1
= 1 + 2,
1+
x
x

so
Z 1 r
p
1
x 1 + 2 dx
Surface Area =
2x 1 + (f (x))2 dx = 2
x
0
0
Z 1
Z p
4
= 2
x2 + 1 dx = 2
tan2 + 1 sec2 d
0
0
 = 4

Z

4
1
1
3
sec tan + ln | sec + tan |
= 2
sec d = 2
2
2
=0
0


=
2 + ln( 2 + 1) .
Z

Remark that we use the Substitution Rule x = tan , dx = sec2 dx, upper
limit u = 4 , lower limit u = 0 in the calculation.
Homework 5. Find the exact area of the surface obtained by rotating the curve
3

x = 13 (y 2 + 2) 2 , 1 y 2 about the x-axis.


Solution. We compute

So

1
1
1
x (y) = (y 2 + 2) 2 2y = y(y 2 + 2) 2
2
p
p
p
p

2
1 + (x (y)) = 1 + y 2(y 2 + 2) = y 4 + 2y 2 + 1 = (y 2 + 1)2 = y 2 + 1.

Z 2
Z 2
p
2

2
y(y + 1) dy = 2
(y 3 + y) dy
Surface Area =
2y 1 + (x (y)) dy = 2
1
1
1


 y=2


1 4 1 2
1
1
21
= 2
= 2
y + y
(16 1) + (4 1) = .
4
2
4
2
2
y=1
Z

Homework 6.

(a) The ellipse


x2 y 2
+ 2 = 1,
a2
b

a>b

is rotated about the x-axis to form a surface called an ellipsoid, or prolate


spheroid. Find the surface area of this ellipsoid.
(b) If the ellipse in part (a) is rotated about its minor axis (the y-axis), the resulting ellipsoid is called an oblate spheroid. Find the surface area of this
ellipsoid.

Solution.
(a) First we write the equation of the ellipse above the x-axis:
r
b 2
x2
a x2
y =b 1 2 =
a
a
Then
b
2x
bx

=
2
2
a2 a x
a a2 x2
s
p
2 x2
p
a4 (a2 b2 )x2
b

1 + (y )2 = 1 + 2 2
.
=
a (a x2 )
a a2 x2
y =

Hence

Z
p
4b a p 4

2
a (a2 b2 )x2 dx.
Surface Area =
2y 1 + (y ) dx = 2
a
a
0

Let a2 b2 x = a2 sin , then a2 b2 dx = a2 cos d. Upper limit is =

2
2
sin1 a ab and lower limit is = 0, so
Z
4b a p 4
a (a2 b2 )x2 dx
Surface Area = 2
a
0


Z sin1 a2 b2
a
4b
=
a2 cos a2 cos d
2
2
2
a a b 0



Z sin1 a2 b2 
2
a
4ba
1 + cos 2
d
=
2
a2 b2 0


a2 b2
1


=sin
a
4ba2
sin 2
1
=
+

4
a2 b2 2
=0

 2



2 b2
a
a b2 b
4ba2
1
+

=
sin
a
a
a
a2 b2



 2
a
a b2
b
= 2ab
+
.
sin1
a
a
a2 bb
Z

(b) First we write the equation of the ellipse above the x-axis:
r
y2
ap 2
b y2
x=a 1 2 =
b
b
Then

a
ay
2y
p
= p
b 2 b2 y 2
b b2 y 2
s
p
2y2
p
b4 + (a2 b2 )y 2
a
p
=
.
1 + (x )2 = 1 + 2 2
b (b y 2)
b b2 y 2
x =

Hence
Z
p
4a b p 4

2
b + (a2 b2 )y 2 dy.
Surface Area =
2x 1 + (x ) dy = 2
b
0
b

Let a2 b2 y =b2 tan , then a2 b2 dy = b2 sec2 d. Upper limit is

a2 b2
and lower limit is = 0, so
= tan1
b
Z

4a
Surface Area = 2
b

Z bp

b4 + (a2 b2 )y 2 dy

tan1

a2 b2
b

4a

b2 sec b2 sec d
2
2
a b 0


a2 b2
2 Z tan1
b
4ab
sec3 d
=
2
2
a b 0


a2 b2
1
h
i
2
=tan
2ab
b

=
sec tan + ln | sec + tan |
2
2
=0
a b




2
2 b2
a
a2 b2
a
2ab
a

+ ln +
=

b
b
b
a2 b2 b



2ab2
a + a2 b2
2
= 2a +
.
ln
b
a2 b2
=

8.3

b2

Applications to physics and engineering


(page 554)

Homework 7 (page 560). A vertical plate is submerged in water and has the
indicated shaped. Explain how to approximate the hydrostatic force against one
side of the plate by a Riemann sum. Then express the force as an integral and
evaluate it.
a

Figure 1: Find the hydrostatic force.


Solution. We set up a coordinate axes such that the origin is the middle of the
square. In this case, each edge lies in one quadrant and we have a good equation
5

expression for each edge. For example, edge in quadrant one is y = x + 22 a, and

edge in quadrant four is y = x 22 a. h


i

In y direction, we divide the interval 22 a, 22 a into n-subinterval with y =

2a
.
n

For each subinterval on the top half, we can take sample point yi such that the

area of strip is equal to one piece of the subinterval of the square, which is shaped

as a trapezoid. That is, we can get the area Ai = 2( 22 a yi )y. The pressure on

the strip is approximately gdi = g( 22 a yi ). So the total force ton the top half
is
F1 = lim
n

n
X

= 2g

2
a
2

2
2
a yi 2
a yi y
2
2
i=1

!2
!3 y= 22 a


2
2
2a3 g
1
=
ay
a y
.
dy = 2g
2
3
2
6

Pi Ai = lim

i=1

n
X

y=0

Next, for each subinterval on the top half, we can take sample point yi such that the
area of strip is equal to one piece of the subinterval of the square, which is shaped

as a trapezoid. That is, we can get the area Ai = 2( 22 a + yi)y. The pressure on

the strip is approximately gdi = g( 22 a yi ). So the total force ton the top half
is
!
!

n
n
X
X
2
2
g
Pi Ai = lim
F2 = lim
a + yi 2
a yi y
n
n
2
2
i=1
i=1

2
 y=0

Z 0

2
1
1
2
3
2
a y dy = 2g
a y y
= 2g

2
2
3
22 a
y= 22 a

2 3
2 3
2 2a3 g
= 2g
a
a =
.
4
12
6

Hence the total force is F = F1 + F2 =

2a3 g
2

(N).

Homework 8. A vertical dam has semicircular gate as shown in the Figure 2. Find
the hydrostatic force against the gate.
Solution. We set up a coordinate system such that the x-axis is the bottom of the

dam. Then the semicircular gate has a good equation expression: y = 4 x2 .


In y direction, we divide the interval [0, 2] into n-subintervals with y = n2 . In
each strip, we choose sample point yi such that the area of the rectangle equals
the area of the one piece of semicircular gate. That is, we can get the area Ai =
p
2 4 (yi )2 y.
6

}2 m

water level

12 m

4m
Figure 2: Find the hydrostatic force.
Next, we compute the pressure of each strip: Pi = gdi = g(10 yi ). Hence
the hydrostatic force against the gate is
F = lim
n

n
X

Pi Ai = lim
n

i=1
2

= 2g

Since
Z

and
Z

2
0

we get

i=1

p
g(10 yi )2 4 (yi )2 y

Z
p
2
(10 y) 4 y dy = 20g

n
X

y 2 dy

y 2 dy

2g

p
4 y 2 dy.

Z
p
2
2
2
2
2 2 sin 2 d cos =
4 cos2 d
=
0
0


 =
Z 
2
1
sin 2 2
1 + cos 2
d = 4 +
= ,
4=

2
2
4
0
=0
Z

 y=2

Z 2p
p
3
8
1
1
2
2
2
= ,
4 y 2 d(4 y ) =
y 4 y 2 dy =
(4 y ) 2
2 0
2 3
3
y=0




16
16
16
= 9800 20
(N).
F = 20g g = g 20
3
3
3

Homework 9 (page 558). Find the centroid of the region bounded by the curves
y = cos x, y = 0, x = 0, and x = 2 .
Solution. First of all, we calculate the area of the region:
Z
h
i x= 2
2

cos x dx = sin x
= 1.
A=
x=0

Next, we calculate
Z
Z
h
i x= 2 Z 2
2
2

My =
x cos x dx =
x d sin x = x sin x
sin x dx

x=0
0
0
0
i x= 2
h


= + cos x
= 1.
2
2
x=0
7

1
Mx =
2

1
cos x dx =
2
0

Hence (
x, y) = 2 1, 8 .
Z

1 + cos 2x
2


 x=
1 1
sin 2x 2

dx =
x+
= .

2 2
4
8
x=0

Homework 10. Let R be the region bounded by y = sin x, y = 0, x = 0, and x = 2 .


(a) Find the centroid of the region R.
(b) Find the volume of the solid obtained by revolving R about 8x + 6y = 25.
(96 )
Solution.
(a) First of all, we compute the area of the region:

Area of R =

sin x dx =

Next, we compute
My =
=

Z
Z

x sin x dx =

x d cos x =

cos x dx =

i x= 2

= 1.
sin x

1
sin dx =
2
2

x=0

h
i x= 2 Z

x cos x
x=0

and
1
Mx =
2

i x= 2

cos x
= 1.

cos x dx

x=0

1 cos 2x
2

Hence (
x, y) = (1, 8 ).


 x=

1 1
sin 2x 2
= .
dx =
x

2 2
4
8
x=0

(b) The distance between (


x, y) and the line 8x + 6y = 25 is
|8 1 + 6 8 25|
17 34
17
3

=
.
d=
=
2
2
10
10 40
8 +6
By the Theorem of Pappus, the volume of solid is
Volume = (Area of R) 2d =

17
3
2.
5
20

Homework 11. Let R be the region bounded by y = cos x, y = 0, x = 0, and


x = 2 .
(a) Find the volume of the solid obtained by revolving R about x-axis.
8

(102 )

(b) Find the centroid of R.


Solution.
(a) The volume of the solid is
Volume =

cos x dx =

0
2

= .
4

1 + cos 2x
2

 x=
1
sin 2x 2
dx =
x+

2
4
x=0


(b) We can compute the volume of the solid obtained by revolving R about y-axis
by the method of cylindrical shell:
Volume =

2x cos x dx = 2

= 2

x d sin x
!

h
i x= 2 Z

x sin x
x=0

sin x dx

= 2 2.

h
i x= 2

= 2 + 2 cos x
x=0

Then by the theorem of pappus, we have


2
x = Volumey 2
x = 2 2 x =
2
y = Volumex 2
y=

1
2

2
y = .
4
8

The result is the same as Homework 9.


Homework 12 (page 561). Find the centroid of the region bounded by y =
sin x, y = cos x, x = 0, and x = 4 .
Solution. First of all, we compute the area of the region:

Z
h
i x= 4

4
2
2

A=
(cos x sin x) dx = sin x + cos x
=
0+
1 = 2 1.
2
2
x=0
0

Next, we compute
Z
Z
4
4
x(cos x sin x) dx =
x d(sin x + cos x)
My =
0
0

h
i x= 4
h
i x= 4 Z 4
2


(sin x + cos x) dx =

+ cos x sin x
= x(sin x + cos x)
4
x=0
x=0
0

2
=
1.
4
9

and

Z 
1 4 1 + cos 2x 1 cos 2x
(cos x sin x) dx =
dx

2 0
2
2
0

 x=
Z
1 sin 2x 4
1 4
1
cos 2x dx =
=
= .

2 0
2
2
4
x=0

1
Mx =
2

Hence

(
x, y) =

21

1
4

21

2
1
1 ( 2 + 1), ( 2 + 1) .
4
4

10

Calculus A (1) Homework 13 ANSWER


10.1

Curves defined by parametric equations


(page 636)

Homework 1 (page 644). Find parametric equations for the set of all points P as
shown in the Figure 1 such that |OP | = |AB|. This curve is called the cissoid of
Diocles after the Greek scholar Diocles, who introduced the cissoid as a graphical
method for constructing the edge of a cube whose volume is twice that of a given
cube.
y
B
x = 2a

Figure 1: The cissoid of Diocles.


Solution. Denote is the angle of OA and x-axis. We have
B = (2a, 2a tan ),

A = (2a cos2 , 2a cos sin ),

and |OA| = 2a cos .

Let P = k(2a, 2a tan ), where k > 0 satisfies k 2 (2a sec )2 = |AB|2 . Since
|AB|2 = (2a sin2 )2 + (2a(tan cos sin ))2
= 4a2 (sin4 + tan2 2 sin2 + (1 sin2 ) sin2 )
= 4a2 (tan2 sin2 ) = 4a2 sec2 sin4 ,
it implies k = sin2 =
Diocles is

tan2
.
1+tan2

(t) =

Denote t = tan , we know the trace of cissoid of




2at2 2at3
,
1 + t2 1 + t2

t R.

Homework 2 (page 653). A string is wound around a circle and then unwound
while being held taut. The curve traced by the point P at the end of the string
is called the involute of the circle. If the circle has radius r and center O and the
initial position of P is (r, 0), an if the parameter is chosen as the Figure 2, show
that parametric equations of the involute are
x = r(cos + sin ),

y = r(sin cos ).
1

y
T
r
P
x

Figure 2: The involute of the circle.


Solution. We know T = (r cos , r sin ) and |T P | = r. Since the unit tangent
~ = (sin , cos ), we get
vector of the circle at T is N
~ = (r cos , r sin ) + r(sin , cos )
P = T + rN
= (r(cos + sin ), r(sin cos )).

10.2

Calculus with Parametric Curves (page 645)

Homework 3 (page 651). Find


concave upward?

dy
dx

and

d2 y
.
dx2

For which values of t is the curve

(a) x = t2 + 1, y = t2 + t.
(b) x = cos 2t, y = cos t, 0 < t < .
Solution.
(a) We have
dy
dt
dx
dt

2t + 1
1
=1+ .
2t
2t


d
1+
d2 y
1
d
dt
1
+
=
=
dx
2
dx
dx
2t
dt
dy
=
dx

1
2t

2t12
1
= 3.
2t
4t

When t < 0, the curve is concave upward.


(b) We have
dy
=
dx

dy
dt
dx
dt

sin t
1
sin t
=
=
.
2 sin 2t
4 sin t cos t
4 cos t



1
d
1 sin t
d2 y
1
d
1
dt 4 cos t
4 cos2 t
=
=
=
=
.
dx
2
dx
dx 4 cos t
4 sin t cos t
16 cos3 t
dt
When

< t < , the curve is concave upward.


2

Homework 4 (page 651). Using the parametric equation to find the area of the
2

region enclosed by the astroid x 3 + y 3 = a 3 .


Solution. The parametric equation of the astroid is x(t) = a cos3 t, y(t) = a sin3 t.
By symmetry, we just compute the area in the first quadrant. So the area is
Z 0
Z 0
Z 0

y(t) dx(t) = 4
y(t)x (t) dt = 4
a sin3 t (3a cos2 t sin t) dt
A=4

= 12a2

(sin4 t cos2 t) dt = 12a2

(sin4 t sin6 t) dt.

We compute
Z

2

Z 
1 cos 2t
1 cos 2t cos2 2t
sin t dt =
dt

+
dt =
2
4
2
4



Z 
Z 
3 cos 2t cos 4t
1 cos 2t 1 1 cos 4t
dt =
dt

=
4
2
4
2
8
2
8
3
1
1
= t sin 2t
sin 4t + C,
8
4
32
Z 

so
Z

0
4

sin t dt =

Next, we compute
Z

so
Z

 t=0

1
1
3
3
t sin 2t
sin 4t
= .
8
4
32
16
t=
2

3

Z 
1 cos 2t
3
1 3
2
3
sin t dt =
cos 2t + cos 2t cos 2t dt
dt =
2
8 8
8



Z 
3 1 + cos 4t
1 3
3
cos 2t dt
cos 2t +
=
8 8
8
2

Z 
5
3
3
1
3
=
cos 2t +
cos 4t cos 2t dt
16 8
16
8


1 3
3
3
1
5
sin 2t sin 2t + C,
sin 2t +
sin 4t
= t
16
16
64
8
3
6

0
6

sin t dt =

Z 

5
3
3
1
t
sin 2t +
sin 4t
16
16
64
8

Hence
2

A = 12a

0
4

(sin t sin t) dt = 12a

 t=0

1 3
5
sin 2t sin 2t
= .
3
32
t=
2

3
5
+
16
32

3a2
.
8

Homework 5 (page 653). A cow is tied to a silo with radius r by a rope just long
enough to reach the opposite side of the silo. Find the area available for grazing by
the cow.
Hint: Part of the curve is the involute of the circle. (See Homework in section
10.1.)
Solution. The available region for grazing by the cow is shown in Figure 3, where
region I is bounded by a semicircle with radius r, and the boundary of regions II,
III is the involute of the circle. Recall the parametric equation of the involute is
P = (x(), y()) = (r(cos + sin ), r(sin cos )),

(, ).

II

III

Figure 3: The involute of the circle.


For region I, we know the area is 12 (r)2 = 21 3 r 2 . By symmetry, we only need
to compute the area of region II.
Z 0
1
Area of region II =
r(sin cos ) d(r(cos + sin )) r 2
2

Z 0
1
= r2
(sin cos )( cos ) d r 2
2

Z 0
1
( sin cos 2 cos2 ) d r 2 .
= r2
2

Since
Z 0

Z
Z
Z
1 0
1 0
1
sin cos d =
sin 2 d =
sin 2 d =
d cos 2
2
2
4 0


i = Z
i =
1
1h
1 h

cos 2 d =
cos 2
sin 2
=
4
4
8
=0
=0
0
1
= ,
4
4

and
Z 0



 =0
Z
1 0 2
1 3
1 + cos 2
+
d =

d sin 2
cos d =

2
6
4

=

 =0
Z

1 3
1 0
1 2

= +

sin 2 d
sin 2
6
4
2
=

 =0
Z
Z

1 3 1 0
1 0
1 3
1

= +

cos 2
d cos 2 +
cos 2 d
6
4
6
4
4
=

 =0

1
1 3
1

= 3 .
=
sin 2
6
4
8
6
4
2

we get the area of region II is





1
1 3
1
1
2
r 2 = 3 r 2 .

r
4
6
4
2
6
Hence the area available for grazing by the cow is
1
5
1 3 2
r + 2 3 r2 = 3r2.
2
6
6

Homework 6. Using the parametric equation to find the length of the astroid
2
2
2
x3 + y 3 = a3 .
(95 )
Solution. The parametric equation of the astroid is x(t) = a cos3 t, y(t) = a sin3 t.
By symmetry, we just compute the length in the first quadrant. In this case, we
know 0 t 2 . Since x (t) = 3a cos2 t sin t and y (t) = 3a sin2 t cos t, we have
p
p
(x (t))2 + (y (t))2 = 9a2 sin2 t cos4 t + 9a2 cos2 t sin4 t
p
= 9a2 sin2 t cos2 t = 3a sin t cos t.
So the total length is
Z p
Z
2

2
Length = 4
(x (t)) + (y (t)) dt. = 4
0

3a sin t cos t dt

 t=
sin2 t 2
= 12a
= 6a.
2 t=0


Homework 7. Consider the parametric curve with x (t) =


t 1. Find the arc length of this curve.
Solution. We compute
Z 1p
Z

2
Length =
(x (t)) + (y (t)) dt =

3t, y (t) = 2t t t2 , 0
(99 )

3t2 + 4t2 (t t2 ) dt
s
2

Z 1
Z 1
1
4
3
2
dt.
4t + 4t + 3t dt =
2t 1 t
=
2
0
0
0

Let t 21 = sin , then dt = cos d. Upper limit is = 6 and lower limit is = 6 .


So
s

2
Z
Z 1
6
1
(2 sin + 1) cos cos d
dt =
Length =
2t 1 t
2
6
0
Z
Z
6
6
2
=2
sin cos d +
cos2 d.
6

Since the first integrand is an odd function, the integration is 0. Hence



 =
Z 
Z
6
6

1 + cos 2
1
sin 2 6
3
2
= +
cos d =
Length =
d =
+
.

2
2
4
6
4
6
6
=
6

Homework 8. Find the arc length of the curve x = t sin 2t, y = t cos 2t, 0 t 1.
(102 )
Solution. Since x = sin 2t + 2t cos 2t and y = cos 2t 2t sin 2t, we have
p
p

(x )2 + (y )2 = (sin 2t + 2t cos 2t)2 + (cos 2t 2t sin 2t)2 = 1 + 4t2 .

Hence by letting 2t = tan , 2 dt = sec2 d, upper limit = tan1 (2) and lower
limit = 0, we get
Z tan1 (2)
Z 1
1
2
sec3 d
Length =
1 + 4t dt =
2
0
0

 =tan1 (2)


1
1
1
1


=
= 5 2 + ln 5 + 2
sec tan + ln | sec + tan |
4
4
4
4
=0

1
1

5 + ln 5 + 2 .
=
2
4

Homework 9 (page 652). Find the exact area of the surface obtained by rotating
the curve x = a cos3 , y = a sin3 , 0 2 about the x-axis.
Solution. The parametric equation of the astroid is x(t) = a cos3 t, y(t) = a sin3 t.
By symmetry, we just compute the length in the first quadrant. In this case, we
know 0 t 2 . Since x (t) = 3a cos2 t sin t and y (t) = 3a sin2 t cos t, we have
p
p
(x (t))2 + (y (t))2 = 9a2 sin2 t cos4 t + 9a2 cos2 t sin4 t
p
= 9a2 sin2 t cos2 t = 3a sin t cos t.

So

Surface Area =

2(a sin t)3a sin t cos t dt = 6a

 t=
1 5 2
6
2
sin t
= 6a
= a2 .
5
5
t=0


sin4 d sin t

Homework 10. Find the volume of the solid generated by revolving the region
bounded by one arch of the cycloid: x = r( sin ), y = r(1 cos ), and y = 0
about the x-axis.
(101 )
Solution.
Volume =

2
2

(r(1 cos )) d(r( sin )) =


Z 2
3
=r
(1 cos )3 d.
0

r 2 (1 cos )2 r(1 cos ) d


0

Since
Z 2


1 3 cos + 3 cos2 cos3 d
0



Z 2 
1 + cos 2
2
(1 sin ) cos d
=
1 3 cos + 3
2
0
 =2

3
1 3
3
= 5,
= 3 sin + + sin 2 sin + sin
2
4
3

(1 cos ) d =

=0

we get Volume = 5r 3 2 .

10.3

Polar Coordinates (page 654)

Homework 11. r = 1 + cos x

(96 )

Solution. We want to find the maximum of y() = r sin = (1 + cos ) sin =


sin + cos sin for 0 . So
dy
= cos + cos2 sin2 = cos + 2 cos2 1.
d
= 0 and get cos = 12 or cos = 1, which mean = 3 or = . Since
We solve dy
d

dy

1
3 3
3
changes
sign
from
positive
to
negative
at

=
,
y(
)
=
(1
+
)
=
is the
d
3
3
2 2
4
maximum value of the height on the cardioid.
Homework 12 (page 663). Find a formula for the distance between the points with
polar coordinates (r1 , 1 ) and (r2 , 2 ).
Solution. Let P1 = (r1 , 1 ) and P2 = (r2 , 2 ) in polar coordinates. We get their
Cartesian coordinates are
P1 = (x = r1 cos 1 , y = r1 sin 1 ),

P2 = (x = r2 cos 2 , y = r2 sin 2 ).

So the distance is
p
|P1 P2 | = (r1 cos 1 r2 cos 2 )2 + (r1 sin 1 r2 sin 2 )2
q
= r12 (cos2 + sin2 ) + r22 (cos2 + sin2 ) 2r1 r2 (cos 1 cos 2 + sin 1 sin 2 )
q
= r12 + r22 2r1 r2 cos(1 2 ).
Homework 13 (page 664). Find the slope of the tangent line to the given polar
curve at the point specified by the value of . (a) r = 1 , = . (b) r = cos 2, = 4 .

Solution.
(a) We compute
dy
=
dx

dy
d
dx
d

12 sin + 1 cos
f () sin + f () cos
sin cos
=
= 1
.
=
1
f () cos f () sin
cos + sin
2 cos sin

So

dy

= .
=

dx = 1

(b) We compute
dy
=
dx

dy
d
dx
d

f () sin + f () cos
2 sin 2 sin + cos 2 cos
=

f () cos f () sin
2 sin 2 cos cos 2 sin

5 sin2 cos + cos3


4 sin2 cos + (cos2 sin2 ) cos
=
.
=
4 sin cos2 (cos2 sin2 ) sin
5 sin cos2 + sin3
So


5 22 12 +
dy

=
2
1
dx =

+
5

2
2
4

2
4

2
4

= 1.

Homework 14 (page 664). Find the points on the given curve the tangent line is
horizontal or vertical. (a) r = 3 cos . (b) r = 1 sin .
Solution.
(a) We compute
dy
=
dx

dy
d
dx
d

f () sin + f () cos
3 sin2 + 3 cos2
cos 2
=
=
.

f () cos f () sin
3 sin cos 3 cos sin
sin 2

The condition for the tangent line being horizontal is cos 2 = 0 and sin 2 6= 0,

that is, = 4 , 34 and 6= 0, 2 . Hence ( 3 2 2 , 4 ) and ( 3 2 2 , 34 ) are two points


that their tangent lines are horizontal.
The condition for the tangent line being vertical is sin 2 = 0, that is, = 0, 2 .
Since cos 2 6= 0 at = 0 and = 2 , (3, 0) and (0, 2 ) are two points that their
tangent lines are vertical.
8

(b) We compute
dy
d
dx
d

f () sin + f () cos
cos sin + (1 sin ) cos
=

f () cos f () sin
cos cos (1 sin ) sin
cos (1 2 sin )
.
=
(1 + 2 sin )(1 + sin )

dy
=
dx

The condition for the tangent line being horizontal is


(
(
cos = 0, (1 2 sin ) = 0
= 2 , 23 , 16 , 65

6= 76 , 11
1 + 2 sin 6= 0, 1 + sin 6= 0
, 2 .
6
So ( 21 , 16 ), ( 12 , 56 ), (2, 23 ) are points that their tangent lines are horizontal.
. If = 67
The condition for the tangent line being vertical is = 2 , 76 , 11
6
dy
, numerators of dx
are nonzero, so they have vertical tangent line.
and = 11
6
If = 21 , we have to check
lim

lim
+

cos (1 2 sin )
cos sin 2
= lim
(1 + 2 sin )(1 + sin ) 2 2 sin2 sin 1
( 00 ,L )
sin 2 cos 2
= ,
= lim

2 sin 2 cos
2
cos (1 2 sin )
cos sin 2
= lim
(1 + 2 sin )(1 + sin ) 2 + 2 sin2 sin 1
( 00 ,L )
sin 2 cos 2
= ,
= lim
2 + 2 sin 2 cos

so the curve at =

also has vertical tangent line. Hence (0, 2 ), ( 32 , 67 ),

( 23 , 11
) are points that their tangent lines are vertical.
6

Calculus A (1) Homework 14 ANSWER


10.4

Areas and Length in Polar Coordinates


(page 665)

Homework 1 (page 659). Find the area of the region that lies inside both curves
r = sin 2 and r = cos 2.
Solution. Both of curves r = sin 2 and r = cos 2 are four leaved roses, where
black one is r = sin 2 and gray one is r = cos 2 in Figure 1. The symmetry shows
1

0.5

I
-1

-0.5

0.5

-0.5

-1

Figure 1: Four leaved roses r = sin 2 and r = cos 2.


there are eight small regions with equal area, so we consider one of them, region I
in Figure 1 for example. First, we will find the intersection of curves, region I in
particular. If we solve sin 2 = cos 2, which implies tan 2 = 1, and one solution is
= 8 . Hence one intersection is (
Area of region I =

Hence the total area is

2
, ).
2 8

The area of region I is

Z
Z
4 1
8
1
2
2
sin2 2 d
(sin 2) d +
(cos 2) d =

2
2
0
8


 = 8


sin 4
1
1 cos 4
d =
=

.

2
2
8
16 8
=0

Area = 8

16 8

1.
2

Homework 2.
(a) Find all points of intersection of the curves r = cos and r = 1 cos .
(b) Find the area of the region that lies inside the circle r = cos and outside the
(95 )

cardioid r = 1 cos .
1

1
0.5

-2

-1.5

-1

-0.5

0.5

-0.5
-1

Figure 2: The circle r = cos and cardioid r = 1 cos .


Solution.
(a) We plot two curves r = cos and r = 1 cos (see Figure 2), and know there
are three intersections. If r = 0, then cos = 0, which implies = 2 , 32
solvable, and 1 cos = 0 implies = 0 solvable, so (0, 0) is an intersection.
Next, we solve cos = 1 cos , then cos = 12 and = 3 , 3 . So the other
two intersections are ( 21 , 3 ) and ( 21 , 3 ).
(b) The area is
Area =

1
1
cos2 (1 cos )2
2
2

(cos2 1 + 2 cos cos2 ) d

(1 + 2 cos ) d =

i = 3


+ 2 sin
= + 3.
=0
3

Homework 3. Consider the cardioid by r = 1 cos , 0 2.


(a) Find the area enclosed by this curve.

(97 )

(b) Find the length of this curve.


Solution.

(a) The area enclose by the cardioid is


Z
Z 2
1 2
1
2
(1 cos ) d =
1 2 cos + cos2 d
Area =
2
2
0
0

 =2

Z 2 
1
1 3
sin 2
3
1
2 cos + cos 2 d =
2 sin +
=

2 0
2
2
2 2
4
=0
3
= .
2
2

(b) The length of the cardioid is


Z 2
Z 2 p

2
2
(1 cos ) + (sin ) d =
2 2 cos d
Length =
0
0

Z 2
Z 2
Z 2 r

1 cos



sin d
d = 2
sin d = 2
=2

2
2
2
0
0
0

 =2

= 4 cos
= 4(1 1) = 8.
2
=0

Homework 4.

(a) Plot the region A which is inside the circle r = 6 cos and outside the cardioid
r = 2(1 + cos ).
(b) Find the area of A.
(c) Find the length of the boundary of the region A.

(98 )

Solution.
(a) The region A is the gray part in Figure 3.
3
2
1

A5

-1
-2
-3

Figure 3: The circle r = 6 cos and outside the cardioid r = 2(1 + cos ).
(b) From Figure 3, we know two curves intersections at three points, but we only
need to find the intersections in region A. If we solve 6 cos = 2(1 + cos ),
then cos = 12 or = 35 . Hence two curves intersect at (3, 3 ) and (3, 35 ).
So the area of A is

Z 
3
1
1
2
2
(6 cos ) (2(1 + cos )) d
Area of A =
2
2
0

Z 2 
1
1
2
2
+
(6 cos ) (2(1 + cos )) d
5
2
2

3
Z
Z
3
3

2
=2
(3 + 4 cos 2 2 cos ) d
16 cos 2 4 cos d = 4
0
0
h
i = 3

 = 3

3
= 4 3 + 2 sin 2 2 sin
=4 +
= 4.
=0

=0

(c) The length of the boundary of the region A is

Z
=2 2
=4

(2(1 +

cos ))2

(2 sin )2

1 + cos d + 6

d +

1 d = 4

(6 cos )2 + (6 sin )2 d





cos d + 4

2

 =

3

+ 4 = 8 + 4.
cos d + 4 = 8 sin
2
2 =
3

Homework 5. Consider the limacons de Pascal () r = 1 2 cos .


(a) Find the slope of the tangents to the limacons de Pascal at the pole.
(b) Find the area of the region between the inner and outer loops of the limacons
de Pascal.
(100 )
Solution. The curve limacons de Pascal is in Figure 4.
1.5
1
0.5
-3

-2.5

-2

-1.5

-1

-0.5
-0.5
-1
-1.5

Figure 4: The limacons de Pascal r = 1 2 cos .


(a) First, we solve r = 1 2 cos = 0, then cos = 21 , so = 3 and 53 . So there
are two tangent lines at the pole. The slope of the tangents to the curve is




dy

dr
sin

+
r
cos

dr
dy


d
d
= dx
= dr
6= 0.
= tan if


dx r=0
d r=0
cos r sin
d
d
r=0

r=0

dr
= 2 sin 6= 0 at = 3 and = 53 . Thus the slope of the
We compute d

tangents to the limacons de Pascal at the pole are 3 or 3.

(b) The area is


Z

Area = 2
=
=

Z
Z

1
(1 2 cos )2 d
2

2
5

(1 4 cos + 4 cos ) d

(3 4 cos + 2 cos 2) d

1
(1 2 cos )2 d
2

(1 4 cos + 4 cos2 ) d
5

(3 4 cos + 2 cos 2) d
5

h
i =2
h
i =


= 3 4 sin + sin 2 3 4 sin + sin 2 5
= 3
= 3
!

3
3
3
3
2
1
=3 4

= + 3 3.
3 4
+
3
2
2
3
2
2
Homework 6. Find the area both inside r 2 = 2 cos 2 and inside r = 1.
(101 )
Solution. We plot two curves in Figure 5. To find the area both inside r 2 = 2 cos 2
1

0.5

II
I

-1

-0.5

0.5

-0.5

-1

Figure 5: The lemniscate r 2 = 2 cos 2 and circle r = 1


and inside r = 1, by symmetry, we only need to consider the areas of region I and
II in Figure 5. First, we will find the intersection of curves. If we solve 2 cos 2 = 1,
which implies = 6 , so one intersection is (1, 6 ). Thus, the area both inside
r 2 = 2 cos 2 and inside r = 1 is
Area = 4 (Area of region I + Area of region II)
Z
Z
h
i = 4
6 1
4 1


2
1 d + 4
(2 cos 2) d = + 4 sin 2
=4

= 6
2
2
3
0
6
!

= + 2 3.
= +2 1
3
2
3

Homework 7. Find the surface area of the solid generated by revolving the cardioid
(99 )

r = 1 + sin about = 2 .
Solution. We plot the cardioid r = 1 + sin , 2

in Figure 6. When we

1.5

0.5

-1

-0.5

0.5

Figure 6: The cardioid r = 1 + sin .


rotate the curve about the line = 2 , the surface area is
Surface area =

2x ds = 2

= 2

(1 + sin ) cos

(1 + sin )2 + (cos )2 d

(1 + sin ) cos 2 + 2 sin d

 = 2
5
2
(1 + sin ) 2
= 2 2
(1 + sin ) d(1 + sin ) = 2 2
5
2
= 2

5
2
32
= 2 2 (2) 2 = .
5
5

3
2

Homework 8. Find the area of the surface generated by revolving the curve r =
sin , 0

(101 )

about the polar axis = 0.

Solution. We plot the curve r = sin , 0 2 , which is a semi-circle in Figure 7.


1

0.8

0.6

0.4

0.2

-0.4

-0.2

0.2

0.4

Figure 7: The semi-circle r = sin , 0 2 .

When we rotate the curve about the polar axis = 0, the surface area is
Surface area =

2y ds = 2

= 2

10.6

sin2

1 cos 2
2

(sin )2 + (cos )2 d

 =
1
sin 2 2
= 2.
d = 2


2
4
2
=0


Conic Sections in Polar Coordinates


(page 678)

Homework 9. Show that each conic in Figure 8 has the corresponding polar equation.
y
y

P
r

L:y=d
x
r

L : x = d
(a)

L : y = d
(c)

(b)

Figure 8: Conic sections. (a) r =

ed
1e cos

(b) r =

ed
1+e sin

(c) r =

ed
.
1e sin

Solution.
(a) In Figure 8 (a), the condition

|P F |
|P L|

= e becomes

ed
r
= e r = e(d + r cos ) r(1 e cos ) = ed r =
.
d + r cos
1 e cos
(b) In Figure 8 (b), the condition

|P F |
|P L|

= e becomes

ed
r
= e r = e(d r sin ) r(1 + e sin ) = ed r =
.
d r sin
1 + e sin
(c) In Figure 8 (c), the condition

|P F |
|P L|

= e becomes

ed
r
= e r = e(d + r sin ) r(1 e sin ) = ed r =
.
d + r sin
1 e sin

Homework 10 (page 684). The orbit of Halleys comet, last seen in 1986 and due
to return in 2062, is an ellipse with eccentricity 0.97 and one focus at the sun.
The length of its major axis is 36.18 AU. (An astronomical unit (AU) is the mean
distance between the earth and the sun, about 93 million miles.) Find a polar
equation for the orbit of Halleys comet. What is the maximum distance from the
comet to the sun?
Solution.
(a) The length of the major axis is 2a = 36.18 AU, so a = 18.09 AU. We are given
that e = 0.97, so a polar equation for the orbit of Halleys comet is
r=

18.09(1 (0.97)2 )
1.06912
a(1 e2 )
=
=
.
1 + e cos
1 + 0.97 cos
1 + 0.97 cos

(b) The maximum distance is the aphelion distance:


a(1 + e) (18.09)(1 + 0.97) 35.6373 (AU).

9.3

Separable Equations (page 594)

Homework 11. Solve the differential equation:

1 x2
dy
=
0<x1
.
dx
x

y(1) = 0

(89 )

Solution. This is a separable equation, and we have


Z
Z
Z
Z
dy
1 x2
1 x2
dx =
dx dy =
dx.
dx
x
x

x
Let u = 1 x2 , then x2 = 1 u2 and du = 22x
dx = 1x
2 dx, so
1x2

Z
Z
Z
1 x2
1 x2
1 x2
u2
dx =

du =
du

x
x
x
1 u2


Z 
Z 
1
1
1
1
du = u +
du

=
1 2
u 1
2
u+1 u1



1 x2 + 1
1 u + 1
1
+ C.
+ C = 1 x2 + ln
= u + ln
2
u 1
2
1 x2 1
So we get

1 x2 + 1
1
+ C.
y(x) = 1 x2 + ln
2
1 x2 1
8

Initial condition y(1) = 0 implies 0 = 0 +

1
2

ln | 1| + C, so C = 0. Hence the

solution is





2

1 x2 + 1
1

x
1
+
1
1
= 1 x2 + ln

.
y(x) = 1 x2 + ln
2
2
1 x2 1
1 1 x2

Homework 12. Solve the differential equation:


(
dy
sec x dx
= ey+sin x x ( 2 , 6 )
.
y(0) = 0

(95 )
Solution. We change the equation into
y dy

y dy

= cos x e
e
dx = cos x esin x dx
dx
Z
Z
y
e dy = esin x d sin x ey = esin x + C.
e

sin x

dx

The initial condition y(0) = 0 gives 1 = 1 + C, so C = 2. Hence the solution is


ey = esin x 2 ey = esin x + 2 y = ln(esin x + 2)
y = ln(esin x + 2).
Homework 13. Solve

y 2 sin x
dy
=
with the initial condition y(0) = 1.
dx
1 + y3
(99 )

Solution. We change the equation into



Z
Z
Z 
Z
1 + y 3 dy
1 + y 3 dy
1
= sin x
dx = sin x dx
y + 2 dy = sin x dx
y 2 dx
y 2 dx
y
1 2 1
y = cos x + C.
2
y
The initial condition y(0) = 1 gives

1
2

1 = 1 + C, so C = 12 . Hence the solution

is
1
1 2 1
y = cos x + .
2
y
2

Homework 14. Solve the initial value problem y =

(y4)(2x+1)
,
x2 +1

y(0) = 1.
(97 )

Solution. We change the equation into


Z
Z
2x + 1
y
y
2x + 1
= 2

dx =
dx
y4
x +1
y4
x2 + 1

Z 
Z
2x
1
1
dx
dy =
+

y4
x2 + 1 x2 + 1
ln |y 4| = ln |x2 + 1| + tan1 x + C.

The initial condition y(0) = 1 gives ln 5 = C, so the solution is


ln |y 4| = ln |x2 + 1| + tan1 x + ln 5.
Homework 15.
(a) Show that the orthogonal trajectories of the family of curves x2 + y 2 = ax is
x2 + y 2 = by.
(b) Show that the orthogonal trajectories of the family of curves y = cx2 is x2 +
2y 2 = k.
(c) Show that the orthogonal trajectories of the family of curves y = ax3 is x2 +
3y 2 = b.
(98 )
Solution.
(a) We implicit differentiate x2 + y 2 = ax with respect to x and get 2x + 2yy = a,
so


1
1 x2 + y 2
y 2 x2

y = (a 2x) =
2x =
.
2y
2y
x
2xy
We want to find a family of curves (x, y(x)) satisfies
2 xy
dy
2xy
= 2
=
2 .
dx
x y2
1 xy

Let y = v(x)x, then y = v x + v and the differential equation becomes


2v
2v v + v 3
v(v 2 + 1)
2v

xv =
v =
=
.
xv + v =
1 v2
1 v2
1 v2
1 v2

So we have
1 v2 1
v =
v(v 2 + 1)
x

10

1 v2
dv =
v(v 2 + 1)

1
dx
x

We deal with the left hand side integration





Z 
Z
v
2v
1
1 v2
2


dv = ln |v| ln |v + 1| = ln 2
dv =
+
v(v 2 + 1)
v v2 + 1
v + 1




y


xy


x
.
= ln y 2
= ln 2


x + y2
+
1
x
So we get




y
xy
= ln |x| + C ln
= C |y| = C |x2 + y 2 |
ln 2


2
2
2
x +y
x +y
x2 + y 2 = by,

where R. Hence the orthogonal trajectories of x2 + y 2 = ax is x2 + y 2 = by.


(b) We differentiate y = cx2 with respect to x and get y = 2cx =
want to find a family of curves (x, y(x)) satisfies

2xy
x2

2y
,
x

so We

dy
x
1
= 2y dy = x dx y 2 = x2 + C x2 + 2y 2 = k.
dx
2y
2
So the orthogonal trajectories of the family of curves y = cx2 is x2 + 2y 2 = k.
(c) We differentiate y = ax3 with respect to x and get y = 3ax2 =
We want to find a family of curves (x, y(x)) satisfies

3x2 y
x3

3y
,
x

so

dy
x
3
1
= 3y dy = x dx y 2 = x2 + C x2 + 3y 2 = bk.
dx
3y
2
2
So the orthogonal trajectories of the family of curves y = ax3 is x2 + 3y 2 = b.
Homework 16 (page 601). Experiments show that the reaction H2 + Br2 2HBr
satisfies the rate law
1
d[HBr]
= k[H2 ][Br2 ] 2
dt

and so this reaction the differential equation becomes


1
dx
= k(a x)(b x) 2 ,
dt

where x = [HBr] and a and b are the initial concentrations of hydrogen and bromine.
(a) Find x as a function of t in the case where a = b. Use the fact that x(0) = 0.
(b) If a > b, find t as a function of x.

11

Solution.
(a) If a = b, the differential equation becomes
3
3
1
dx
= k(a x)(a x) 2 = k(a x) 2 (a x) 2 dx = k dt.
dt

After integration, the equation becomes


Z
Z
1
23
(a x) d(a x) = k
dt 2(a x) 2 = kt + C.
1

Since x(0) = 0, we have 2a 2 = C, so


2(a x)

12

12

= kt + 2a

=
ax

2
kt +
a

2

x(t) = a

4a
.
(k at + 2)2

(b) We want to find t as a function of x, so we will solve


Z
Z
1
1
dt
dt
=
dx =
1
1 dx.
dx
dx
k(a x)(b x) 2
k(a x)(b x) 2

To deal with the integration of the right hand side, we let u = b x, then
du = 21bu dx, which implies dx = 2u du. Furthermore, we have a x =
a b + u2 , so we get
Z
Z
Z
1
1
2
2u

du
du =
1 dx =
2
2
k(a b + u )u
k
u + ( a b)2
k(a x)(b x) 2


2
u
1

=
+C
tan
k ab
ab


bx
2
1

+ C.
tan
=
k ab
ab
Hence
2
tan1
t=
k ab

12



bx

+ C.
ab

Calculus A (1) Homework 15 ANSWER


9.5

Linear Equations (page 616)

Homework 1. Solve the differential equation xy 2y = x3 sec x tan x, x > 0, y( 4 ) =


0.
(96 )
Solution. We change this linear differential equation to the standard form:
2
y y = x2 sec x tan x,
x

(1)

and find the integrating factor:


R

e
We multiply

1
x2

x2 dx

= e2 ln x = x2 =

1
.
x2

on both sides of the equation (1) and get

1
2
d y
y
y

y
=
sec
x
tan
x

= sec x tan x 2 = sec x + C.


2
3
2
x
x
dx x
x

Since y( 4 ) = 0, we get 0 = sec 4 + C, so C = 2. Thus the solution of the initial

value problem is y(x) = x2 (sec x 2).


Homework 2. Solve the differential equation x(x + 1)y + y + (x + 1)2 cos x = 0,
x > 0, with y() = + 1.
(97 )
Solution. We change this linear differential equation to the standard form:
y +

1
(x + 1) cos x
y=
,
x(x + 1)
x

(2)

and find the integrating factor:


R

1
x(x+1)

dx

1
1
= e ( x x+1 ) dx = eln xln(x+1) =

x
.
x+1

on both sides of the equation (2) and get




xy
xy
1
d
x
= cos x
y +
y = cos x
= sin x + C.
2
x+1
(x + 1)
dx x + 1
x+1

We multiply

x
x+1

Since y() = + 1, we get


C=

( + 1)
= .
+1

Thus the solution of the initial value problem is


y(x) =

x+1
( sin x + ).
x
1

dy
+ 3x(x2 + 1)y = 2x with the initial condition
Homework 3. Solve (x2 + 1)2
dx
y(0) = 3.
(99 )
Solution. We change this linear differential equation to the standard form:
3x
2x
dy
+ 2
y= 2
,
dx x + 1
(x + 1)2

(3)

and find the integrating factor:


R

3x
x2 +1

dx

= e2

1
x2 +1

d(x2 +1)

= e 2 ln(x

2 +1)

= (x2 + 1) 2 .

We multiply (x2 + 1) 2 on both sides of the equation (3) and get



3
1
3
d  2
2x
2x
2y
(x
+
1)

=
(x2 + 1) 2 y + 3x(x2 + 1) 2 y =
1
1
dx
(x2 + 1) 2
(x2 + 1) 2
Z
Z
1
3
1
2x
2
2
2 + C.
(x2 + 1) 2 y =
1 dx =
1 d(x + 1) = 2(x + 1)
(x2 + 1) 2
(x2 + 1) 2
Since y(0) = 3, we get 3 = 2 + C, so C = 1. Hence the solution of the initial value
problem is
y(x) =

x2

2
1
.
+
2
+ 1 (x + 1) 32

Homework 4. Solve the initial value problem:


(
(sec x)y + y = (tan x)ecos xsin x 0 x < 2 ,
y(0) = 0

.
(101 )

Solution. We change this linear differential equation to the standard form:


y + (cos x)y = (sin x)ecos xsin x ,

(4)

and find the integrating factor:


R

cos x dx

= esin x .

We multiply esin x on both sides of the equation (4) and get


d sin x 
esin x y + (cos x) esin x y = (sin x)ecos x
e y = (sin x)ecos x
dx
Z
Z
sin x
cos x
e y = (sin x)e
dx = ecos x d(cos x) = ecos x + C.
Since y(0) = 0, we get 0 = e + C, so C = e. Hence the solution of the initial value
problem is
y(x) = ecos xsin x + e1sin x .
2

Homework 5. Solve xy 3y = 5x3


(a) with the initial condition y(1) = 2.
(102 )

(b) with the initial condition y(1) = 2.


Solution.

(a) We change this linear differential equation to the standard form:


3
y y = 5x2 ,
x

(5)

and find the integrating factor:


R

x3 dx

= e3 ln x = x3 =

1
.
x3

1
x3

on both sides of the equation (5) and get


Z
3
5
d y 5
y
5
1
= 3 =
y 4y =
dx = 5 ln |x| + C.
3
3
x
x
x
dx x
x
x
x

We multiply

Since y(1) = 2, we get 2 = 0 + C, so C = 2. Hence the solution of the initial


value problem is
y(x) = 5x3 ln |x| + 2x3 .
(b) Since y(1) = 2, we get 2 = 0 + C, so C = 2. Hence the solution of the
initial value problem is
y(x) = 5x3 ln |x| 2x3 .
Homework 6. Let y = h(x) be decreasing on [0, 2 ) and is continuously differentiable on (0, 2 ) with h(0) = 0. Let s(x) denote the arc length of y = h(x) from (0, 0)
to (x, h(x)).
(a) Write down the formula for s(x).
(b) Suppose that s(x) is also given by s(x) =

eh(t) dt. Find the function h(x)

explicitly.
(c) Find the function s(x) explicitly.

(102 )

Solution.
(a) The arc length of y = h(x) is
Z xp
1 + (h (t))2 dt,
s(x) =

0<x<

.
2

(b) We solve
s(x) =

1+

(h (t))2

dt =

eh(t) dt.

By the Fundamental Theorem of Calculus, we have


p
1 + (h (x))2 = eh(x) 1 + (h (x))2 = e2h(x) (h (x))2 = e2h(x) 1.

Since h(x) is a decreasing function, we get

p
h (x)
h (x) = e2h(x) 1
= 1.
e2h(x) 1
We integrate the equation on both sides and get
Z
Z
Z
1
h (x)

dx = 1 dx
dh(x) = x + C.
e2h(x) 1
e2h(x) 1

Let u = e2h(x) 1, then e2h(x) = u2 + 1 and


1
u
du =
(2)e2h(x) dh(x) dh(x) = 2
du.
2h(x)
u +1
2 e
1
So the left hand side of the equation becomes


Z
Z
Z
1
1
u
1

du =
dh(x) =
2
du
2
2h(x)
u
u +1
u +1
e
1
p
= tan1 u + C = tan1 e2h(x) 1 + C.
Thus we get

tan1

e2h(x) 1 = x + C.

Since h(0) = 0, which implies 0 = 0 + C, so C = 0, Hence


p
p
tan1 e2h(x) 1 = x e2h(x) 1 = tan x

e2h(x) = 1 + tan2 x = sec2 x 2h(x) = 2 ln sec x


h(x) = ln sec x = ln cos x.

(c) The arc length is


Z x
Z
h(t)
s(x) =
e
dt =
0

x
ln sec t

dt =

sec t dt = ln | sec x + tan x|.


0

Homework 7 (page 619). Suppose that the resistance is 12 and the inductance
is 4 H. If a battery gives a constant voltage of E(t) = 60 V and the switch is closed
when t = 0.
(a) Find I(t).
(b) Find the current after 1 s.
(c) Find the limit value of the current.
Solution.
(a) Recall the Electric Circuits equation is
L

dI
+ RI = E(t).
dt

When we put L = 4, R = 12, and E(t) = 60, then the equation becomes
4

dI
dI
+ 12I = 60
+ 3I = 15.
dt
dt

We also have the initial value condition I(0) = 0. Multiplying the integrating
R

factor e

3 dt

= e3t , we get

dI
d 3t 
e
e I = 15e3t e3t I =
+ 3 e3t I = 15e3t
dt
dt
3t

15e3t dt = 5e3t + C.

The initial condition I(0) = 0 implies 0 = 5 + C, so C = 5. Hence


I(t) =

1
(5e3t 5) = 5 5e3t .
e3t

(b) After 1 second the current is I(1) = 5(1 e3 ).


(c) The limiting value of the current is

lim I(t) = lim 5 5e3t = 5.

9.1

Modeling with Differential Equations


(page 580, 621)

Homework 8 (page 585). Psychologists interested in learning theory study learning


curves. A learning curve is the graph of a function P (t), the performance of someone
learning a skill as a function of the training time t. The derivative
rate at which performance improves.

dP
dt

represents the

If M is the maximum level of performance of which the learner is capable, then


dP
= k(M P (t))
dt
is a model for learning, where k is a positive constant. Solve it as a linear differential
equation and use your solution to graph the learning curve.
Solution. We solve the learning curve by the method of integrating factor:
dP
dP
= k(M P (t))
+ kP (t) = kM.
dt
dt
R

The integrating factor is e

k dt

= ekt , so we multiply ekt on both sides of the equation

and get
ekt


d kt
dP
e P (t) = kMekt ekt P (t) = Mekt + C.
+ kekt P (t) = kMekt .
dt
dt

Since P (0) = 0, we get 0 = M + C, so C = M. Hence the solution is


1
(Mekt M) = M(1 ekt ).
ekt

P (t) =

Figure 1 shows a graph of the learning curve. There is a horizontal asymptotes


P = M that the performance can not exceed the learners capacity.
P
M
P (t) = M(1 ekt )

t
Figure 1: Learning curves.

Calculus A (2) Homework 1 ANSWER


11.1

Sequences (page 690)

Homework 1 (page 700). Determine whether the sequence converges or diverges.


2

If it converges, find the limit. (a) an = nn3 +4n (b) bn =


n

(d) dn = 1 + n2
(e) en = n n + 1 n + 3.

(2n1)!
(2n+1)!

(c) cn =

cos2 n
2n

Solution.

(a) For n 2, n3 + 4n = n3 + 22 n n3 + n2 n = 2n3 , so


r
n2
n
n2
an =
=
=a
n .

3
3
2
n + 4n
2n

Since {an }
n=1 is divergent, {an }n=1 is divergent.

(b) Since
0
1
n n

and lim

1
1
(2n 1)!
=
,
(2n + 1)!
(2n + 1) 2n
n

(2n1)!
n (2n+1)!

= 0, we get lim



2
(c) Since |cn | = cos2n n

1
2n

1
n
n 2

and lim

(d) Consider the function f (x) = 1 +


lim f (x) = lim
x

= 0 by the Squeeze Theorem.

2
1+
x

x

lim

= 0, we have lim
n


2 x
,
x

= 0.

then

 x !2

2 2
1+
=
x

= lim

cos2 n
2n

lim

1+

1
x
2

 x2 !2

Thus

2
1+
n

n

x

2
= lim 1 +
= e2 .
x
x

(e) Since

n + 1 n + 3) (n + n + 1 n + 3)

en = n n + 1 n + 3 =
n+ n+1 n+3
4 n3
n2 (n2 + 4n + 3)
4n 3

=
=
=
,
n+ n+1 n+3
n + n2 + 4n + 3
1 + 1 + 4 + 32

(n

we have lim en =
n

40

1+ 1+0+0

4
2

= 2.
1

= e2 .

Homework 2. Show that lim nr n = 0 if |r| < 1.


n

Solution. Let an = nr n . We know lim r x = if r > 1; lim r x = 0 if 0 < r < 1.


x

(1) If r 1, then an = nr n n and lim n = , so lim nr n does not exist.


n

(2) If 0 < r < 1, consider f (x) = xr x and we get


lim nr n = lim xr x = lim
n

x

1 x

),L
(

lim
x

= lim
x

lim r
rx
= x = 0.
ln r
ln r
x


1 x
r

1
ln( 1r )

(3) If r = 0, then lim nr n = lim 0 = 0.


n

(4) If 1 < r < 0, we compute


|an | = |nr n | = |n((r))n | = |n(1)n (r)n | = |n(1)n (r)n | = n(r)n ,
where 0 < r < 1, so
lim |an | = lim n(r)n = 0
n

by case (2). Hence lim nr n = 0.


n

(5) If r 1, then r 1. We take subsequence


a2n = (2n)(r)2n = 2n((r))2n = 2n(1)2n (r)2n = 2n(r)2n 2n,
and lim 2n = , so lim nr n does not exist.
n

Homework 3 (page 701). Show that the sequence defined by a1 = 1, an+1 = 3 a1n
is increasing and an < 3 for all n. Deduce that {an } is convergent and find its limit.
Solution. Increasing: We claim: an+1 > an for all n N.
(1) When n = 1, a1 = 1 and a2 = 3

1
1

= 2, so a2 > a1 .

(2) Assume that it is true for n = k, that is, ak+1 > ak .




1
3 a1k =
(3) When n = k + 1, ak+2 ak+1 = 3 ak+1

1
ak

1
ak+1
=

ak+1 ak
ak ak+1

> 0.

Remark that the denominator is positive because ak+1 > ak > > a1 > 0.

(4) By mathematical induction, we know {an }


n=1 is increasing.
2

Bounded: We claim: an < 3 for all n N.


(1) When n = 1, a1 = 1 < 3 is true.
(2) Assume that it is true for n = k, that is, ak < 3.
(3) When n = k + 1, ak+1 = 3

1
ak

< 3 because ak > 0.

(4) By mathematical induction, we know {an }


n=1 is bounded above by 3.
Limit: By Monotonic Sequence Theorem, we know lim an exists. Let lim an = L.
n

Since an+1 = 3
L = lim an+1
n

1
,
an

we have


1
1
1
=3
= lim 3
= 3 L2 3L + 1 = 0.
n
an
lim an
L
n

The solution is L =

3 5
.
2

Since L > 1, we get L =

3+ 5
.
2

Homework 4 (page 702). Let a and b be positive numbers with a > b. Let a1 be
their arithmetic mean () and b1 their geometric mean ():

a+b
a1 =
,
b1 = ab.
2
Repeat this process so that, in general
an+1 =

an + bn
,
2

bn+1 =

p
an bn .

(a) Use mathematical induction () to show that an > an+1 > bn+1 > bn .
(b) Deduce that both {an } and {bn } are convergent.
(c) Show that lim an = lim bn . Gauss called the common value of these limits
n

the arithmetic-geometric mean of the numbers a and b.


Solution.
(a) Let a0 = a and b0 = b.
(1) When n = 0, we compute


a+b
ab
a0 a1 = a
=
>0
2
2
 1 

2
a+b
1
a 2 ab + b =
ab =
a1 b1 =
a b >0
2
2
2



a b > 0.
b1 b0 = ab b = b
So a0 > a1 > b1 > b0 .

(2) Assume that it is true for n = k, that is, ak > ak+1 > bk+1 > bk .
(3) When n = k + 1, we compute


ak+1 bk+1
ak+1 + bk+1
=
>0
ak+1 ak+2 = ak+1
2
2

p
1
ak+1 + bk+1 p
ak+1 2 ak+1 bk+1 + bk+1
ak+2 bk+2 =
ak+1 bk+1 =
2
2
2
p
1 
ak+1 bk+1 > 0
=
2


p
p
p
bk+2 bk+1 = ak+1 bk+1 bk+1 = bk+1
ak+1 bk+1 > 0.
(4) By mathematical induction, we know an > an+1 > bn+1 > bn .

(b) Since the sequence {an }


n=1 is decreasing and bounded below by b, and {bn }n=1

is increasing and bounded above by a, by Monotonic Sequence Theorem, we

know both {an }


n=1 and {bn }n=1 are convergent.

(c) Let lim an = A and lim bn = B. From the relation


n

an+1 =
we get A =

11.2

lim an + lim bn
an + bn
an + bn
n
lim an+1 = lim
= n
,
n
n
2
2
2

A+B
2

and it implies A = B.

Series (page 703)

Homework 5 (page 707). Discuss the series

def.

xn = 1 + x+ x2 + x3 + + xn +

n=0

for x R. If the series is convergent, find the sum of the series.


Solution. For fixed x, the series is a geometric series with common ratio x, so the
series is convergent if |x| < 1, and divergent if |x| 1.
For |x| < 1, we have
1 lim xn
1(1 xn )
1
n
x = lim
x = lim
=
=
.
n
n
1

x
1

x
1

x
n=0
k=0

n
X

Homework 6. Write the number 0.285714, 2.317, and 0.9 as a ratio of integers.
Solution.
n
X
1
= lim
= lim 285714
0.285714 =
6n
6k
n
n
10
10
106k
n=1
k=1
k=1
n 


n 
1
1 1016
1
2
285714
106
1
=
.
= lim 285714
= lim
1
n
n 999999
106
7
1 106

X
285714

n
X
285714

n
n
X
X
17
1
17
= 2.3 + lim
= 2.3 + lim 17
2.317 = 2.3 +
2n+1
2k+1
2k+1
n
n
10
10
10
n=1
k=1
k=1





n
n
1
1 1012
17
1
103
= 2.3 + lim 17
= 2.3 + lim
1
n
n 990
106
1 1012
17
2 990 + 3 99 + 17
2294
= 2.3 +
=
=
.
990
990
990

n
n

X
X
X
9
1
9
= lim
= lim 9
= lim 9
0.9 =
n
k
n
n
10
10
10k n
n=1
k=1
k=1

 n 
 n
1
1
= lim 1
= 1 lim
= 1.
n
n 10
10

1
10

1
1

 
1 n
10
1
10

Homework 7 (page 707).


(a) Show that the series

1
n(n+1)

is convergent, and find its sum.

n=1

(b) Show that the Euler series

1
n2

is convergent.

n=1

Solution.

(a) We compute the partial sum


sn =

n
X
k=1

X
1
=
k(k + 1) k=1
=

n
X
k=1

1
1

k k+1

n+1
X
k=2

n
X
1
k=1

1
1
=1
.
k
n+1

n
X
k=1

1
k+1

So

X
n=1



1
1
1
= lim 1 lim
= lim sn = lim 1
= 1.
n
n n + 1
n
n(n + 1) n
n+1

(b) Let an =

1
n2

> 0. We know the partial sum {sn }


n=1 of the Euler series is an

increasing sequence. Furthermore,


n
n
n
X
X
X
1
1
1
1
sn =
=1+
1+
= 1 + 1 < 2.
2
2
k
k
(k 1)k
n
k=1

k=2

k=2

The partial sum {sn }


n=1 is bounded above by 2. By the Monotonic Sequence

P
1
= lim sn is convergent.
Theorem, the Euler series
n2
n=1

Homework 8 (page 711). Determine whether the series is convergent or divergent.


If it is convergent, find its sum.


X
2
3
(b)
+
5n n
n=1

X
1 + 2n

(a)

3n

n=1

n
n+1

are geometric series with common ratio

1
3

(c)

ln

n=1

Solution.

(a) Both

1
3n

and

2n
3n

and 32 , re-

n=1

n=1

spectively, so

1+2n
3n

is convergent, and

n=1

 n
1
X
X
2
1
3
=
+
=
n
3
3
1

n=1
n=1

X
1 + 2n

3n

n=1

(b) Since

2
n

1
n

>

and

divergent. Since

n=1

1
n

1
3

2
3

2
3

5
1
+2= .
2
2

is harmonic series, which is divergent, We know

2
n

is

n=1
3
5n

1
5

is a geometric sereis with common ratio

< 1, which

n=1

is convergent, we can show that

3
5n

n=1

that

3
5n

n=1

2
n

is convergent. Then

2
n

is divergent as follows: Suppose


 X



X
3
2
3
2
3
2 X 3
n
=
+ n =
+
n
n
n n=1 5
n 5
5
n
5
n=1
n=1



X
X
3
2
3
=

+
5n n
5n
n=1
n=1
is convergent, and it contradicts to

2
n

is divergent.

n=1

(c) We compute the partial sum of the series

ln

n
n+1

n=1

sn =
=

n
X

k=1
n
X

ln

k
k+1

ln k

k=1

n+1
X

n
X

(ln k ln(k + 1)) =

k=1

n
X

ln k

k=1

n
X

ln k = ln 1 ln(n + 1) = ln(n + 1).

k=2

The limit lim sn = lim ln(n + 1) is divergent, so the series


n

ln(k + 1)

k=1

n=1

divergent.

ln

n
n+1

is

Homework 9 (page 714). Consider the series

n
.
(n+1)!

n=1

(a) Find the partial sums s1 , s2 , s3 , an s4 . Do you recognize the denominators?


Use the patten to guess a formula for sn .
(b) Use mathematical induction to prove your guess.
(c) Show that the given infinite series is convergent, and find its sum.
Solution.
(a) We compute
1
1
= ,
12
2
2
3+2
5
1
=
=
s2 = +
2 123
123
6
5
3
45+3
23
s3 =
+
=
=
123 1234
1234
24
4
5 23 + 4
119
23
+
=
=
.
s4 =
1234 12345
12345
120
s1 =

The denominators is (n + 1)!, so we guess sn =


(b) (1) When n = 1, s1 =

1
2

2!1
,
2!

(n+1)!1
.
(n+1)!

so the formula holds.

(2) Assume that it is true for n = k, that is, sk =

(k+1)!1
.
(k+1)!

(3) When n = k + 1, we compute


(k + 1)! 1
k+1
k+1
=
+
(k + 2)!
(k + 1)!
(k + 2)!
(k + 2)! k 2 + k + 1
(k + 2)((k + 1)! 1) + k + 1
=
=
(k + 2)!
(k + 2)!
(k + 2)! 1
.
=
(k + 2)!

sk+2 = sk+1 +

The formula is true.


(4) By mathematical induction, we know sn =

(n+1)!1
.
(n+1)!

(c) We compute

X
n=1



n
X
n
1
k
(n + 1)! 1
= lim
= lim
= lim 1
n
(n + 1)! n
(k + 1)! n (n + 1)!
(n + 1)!
k=1

1
= 1,
n (n + 1)!

= 1 lim
which is convergent.

11.3

The Integral Test and Estimates of Sums


(page 714)

Homework 10. Determine the value of p 0 such that the series

1
n(ln n)p

con-

n=2

verges.

(89, 97 )
1
,
x(ln x)p

Solution. Let f (x) =

which is a positive and continuous function on [2, ).

We find the region that the function is decreasing by calculating


(ln x)p px(ln x)p1 x1
(ln x + p)
= 2
< 0,
2
2p
x (ln x)
x (ln x)p+1

f (x) =

so f (x) is decreasing on [2, ). Next we compute the integral


Z t
Z t
Z
Z
1
1
1
dx = lim
dx = lim
d ln x.
f (x) dx =
p
p
t 2 x(ln x)
t 2 (ln x)p
x(ln x)
2
2
If p = 1, then
Z

f (x) dx = lim
t

t
2

is divergent.

h
i x=t
1

d ln x = lim ln(ln x)
= lim (ln(ln t) ln(ln 2))
t
t
ln x
x=2

If p 6= 1, then
Z

f (x) dx = lim

 x=t



1
1
1
1

=
.
lim

(p + 1)(ln x)p1 x=2 p + 1 t (ln x)p1 2p1

The improper integral is convergent if p > 1, and divergent if 0 p < 1.

P
1
By the Integral Test, the series
is convergent if p > 1.
n(ln n)p
n=2

Homework 11. Test the following two series

ln n
np

where p = 1 and p = 32 , for

n=1

convergence.
Solution. Let f (x) =

ln x
,
xp

(101 )

where p = 1 or p = 32 , which is a positive, continuous

function on [2, ). We find the region that the function is decreasing by calculating
f (x) =

xp

1
x

(ln x)pxp1
1 p(ln x)
=
,
2p
x
xp+1

and f (x) < 0 is true for p = 1 and p =

3
2

on [2, ). So f (x) =

function on [2, ).
Next, we compute the integral

R
2

ln x
xp

dx:

ln x
xp

is a decreasing

(a) If p = 1, then
 x=t

Z t
Z t
Z

1
ln x
ln x
2
dx = lim
dx = lim
(ln x)
ln x d ln x = lim
t 2
t 2
t 2
x
x
2
x=2


1
1
= lim
(ln t)2 (ln 2)2
t
2
2
is divergent. By the Integral Test, we get the series

ln n
n

is divergent.

n=1

(b) If p = 23 , then
Z

ln x

ln x
3
2

dx = lim 2

ln x dx 2

t

xh
Z2 t
i
x=t

21
21

x d ln x
= lim 2 x ln x
t
x=2
2


Z t
ln t ln 2
32
= lim 2 + +
x dx
t
t
2
2
h 1 i x=t


= 2 ln 2 + lim 4 x 2
t
x=2

4
4
= 2 ln 2 + lim + = 2 ln 2 + 2 2
t
t
2

P
ln n
is convergent. By the Integral Test, we get the series
3 is convergent.
2

3
2

dx = lim

n=1 n 2

Remark that

ln t
lim
t
t

),L
(

1
t
1
t
2 t

lim

1
= lim = 0.
t 2 t

Homework 12 (page 721). How many terms of the series


need to add to find its sum to within 0.01?
Solution. Let f (x) =

1
,
x(ln x)2

1
n(ln n)2

would you

n=2

whcih is a positive and continuous function on [2, ).

Furthermore, we compute
f (x) =

(ln x)2 2x(ln x) x1


ln x + 2
= 2
< 0,
2
4
x (ln x)
x (ln x)3

so f (x) is decreasing on [2, ). By the Integral Test, we can estimate the remainder
of the series
Z
Z
Rn
f (x) dx =
n

Z t
Z t
1
1
1
dx = lim
dx = lim
d ln x
2
2
t n x(ln x)
t n (ln x)2
x(ln x)
n

 x=t


1
1
1
1
1
=
lim

=
= lim
+
<
.
t
ln x x=n t
ln t ln n
ln n
100

So we require ln n > 100, which is equivalent to n > e100 .


9

11.4

The Comparison Tests (page 722)

Homework 13. Determine the convergence of the following series:


(a)

n+1n .
n+1+ n

n=0

(b)

q
P
sin n13 .

(99 )

n=1

Solution.

(a) Let an =

n+1n ,
n+1+ n

then

n+1 n
n+1 n
n+1+ n
n+1n
an =
=

=

n+1+ n
n+1+ n
n+1+ n
( n + 1 + n)2
1

.
=
2n + 1 + 2 n2 + n
Let bn = n1 , then
an
lim
= lim
n bn
n

1
2n+1+2 n2 +n
1
n

= lim
n

Since

bn =

n=1

1
n

2+

1
n

n 2n + 1 + 2 n2 + n
1
1
1
q

=
= > 0.
4
2+0+2 1+0
+2 1+ 1
= lim

is harmonic series, whcih is divergent, by the Limit Com-

n=1

parison Theorem, we know

n+1 n

n+1+ n

is also divergent.

n=0

q
q
q
sin n13 . Since sin x x for x 0, we have sin n13 n13 = 13 .
n2

P
P
3
1
bn is convergent.
bn is a p-series with p = 2 > 1, so
Let bn = 3 , then
n2
n=1
n=1

q
P
P
By the Comparison Test, we get
an =
sin n13 is convergent.

(b) Let an =

n=1

n=1

Homework 14.

(a) Prove that ln(n + 1) < 1 + 21 + +


(b) Test for convergence of

n=1

Solution.

1
1 .
1+ 21 ++ n

10

1
n

< 1 + ln n.
(102 )

f (x) =

1
x

f (x) =

n n+1

1 2 3

1 2 3

Figure 1: The graph of f (x) =


(a) We plot the graph of f (x) =

1
x

1
x

1
x

and rectangles.

on [1, n+1] and rectangles in Figure 1. The total

area of these gray rectangles is greater than the area of the region bounded by
f (x) = x1 , x-axis, x = 1, and x = n + 1. So we have
Z n+1
h
i x=n+1
1
1
1 1

= ln(n + 1).
dx = ln x
1+ + ++ >
x=1
2 3
n
x
1

The total area of these red rectangles is less than the area of the region bounded
by f (x) = x1 , x-axis, x = 1, and x = n. So we have
Z n
h
i x=n
1
1
1 1

= 1 + ln n.
dx = 1 + ln x
1+ + ++ < 1+
2 3
n
x
x=1
1

(b) Since an =

1
1+ 12 ++ n

>

1
1+ln n

>

1
1+n

= bn and

1
1+n

n=1

1
n

is harmonic

n=2

series, which is divergent. By the Comparison Test, we know

n=1

divergent.

1
1
1+ 12 ++ n

is

Remark that we can show ln n < n for all n N by proving f (x) = xln x > 0
for x 1. This is because f (x) = 1 x1 =

x1
x

> 0 and f (1) = 1ln 1 = 1 > 0,

we know f (x) > f (1) = 1 > 0.


Homework 15 (page 726). Determine whether the following series converges or
diverges.
2

X
1
(a)
1+
en
n
n=1

(b)

X
n=1

1
1

n1+ n

(c)

X
en

n=1

Solution.
(a) Let an = 1 +


1 2 n
e
n

and bn = en , then
2
2

1 + n1 en
1
an
= lim
= lim 1 +
= 1 > 0.
lim
n
n
n bn
en
n
11

Since

bn =

n=1


1 n
e

n=1

is a geometric series with common ratio

convergent. By the Limit Comparison Test,


1 2 n
e
n

1+

n=1

(b) Let an =

and bn = n1 , then

1
1

n1+ n

an
lim
= lim
n bn
n
Since

bn =

n=1

parison Test,

1
n

n=1

n1+ n
1
n

= lim
n

1
n

lim n

1
n

1
e

< 1, it is

is also convergent.

1
= 1 > 0.
1

is harmonic series, which is divergent, by the Limit Com-

1
1

1+
n=1 n n

is also divergent.
1

Remark that the limit lim n n = 1 is a corollary of the following limit:


n

ln n
ln x
= lim
n n
x x
lim

),L
(

lim
x

1
x

1
= 0,
n x

= lim

and f (x) = ln x is a continuous for x > 0, so


 1
1
1
1
ln n
= lim ln n = lim ln n n = ln lim n n = 0 lim n n = e0 = 1.
n n
n
n
n
n n
lim

(c) Let an =

en
n

and bn = n1 , then
1

an
= lim
lim
n
n bn
Since

n=1

parison Test,

1
n
n=1

1
P
en
n
n=1

bn =

en
n
1
n

= lim e n = 1 > 0.
n

is harmonic series, which is divergent, by the Limit Comis also divergent.

Homework 16 (page 726). Use

10
P

1
3n +4n

.
= 0.19788 to estimate the error of the

n=1

sum of the series

1
.
3n +4n

n=1

Solution. Since an =

1
3n +4n

<

1
3n +3n

R10 T10

1
23n

= bn , so the error is

1
1
1
2311
=

1 = 10 .
n
23
3
1 3
n=11

A better estimate is using the Integral Test:


 x=t

Z t x
Z
3
1
1 3x
R10 T10
dx = lim
dx = lim
x
t 10 2
t
2 ln 3 x=10
10 2 3


3t
1
310
lim
=
+
.
10
t
2 ln 3 2 ln 3
2 3 ln 3
12

Calculus A (2) Homework 2 ANSWER


11.5

Alternating Series (page 727)

Homework 1. Test the series

 1

(1)n e n 1 for convergence or divergence.

n=1

(97 )

Solution. We observe the function f (x) = ex for x [0, 1]. By the Mean Value
Theorem, we know there exists c (0, x) such that f (x) f (0) = f (c)(x 0),
that is, ex 1 = ec (x 0). When we put x =

1
,
n

it implies that the behavior of

1
n

e 1 looks like n1 . This observation inclines us to guess the series is conditionally


convergent.
 1

Let an = (1)n e n 1 , bn = n1 , and f (x) = ex . Then
1

en 1
ex 1
|an |
= lim
= f (0) = 1.
=
lim
lim
1
n
x0
n bn
x
n

Since

1
n

is a harmonic series, which is divergent, by the Limit Comparison The-

n=1

orem, we know

|an | is divergent.
 P
 1

(1)n cn , it is an alternating series, so we


(1)n e n 1 =

n=1

For the series

n=1

n=1

investigate the following conditions:

x
(a) {cn }
n=1 is decreasing: This is because e is an increasing function, so we know
1

cn = e n 1 > e n+1 1 = cn+1 .


(b) lim cn = 0: We compute the limit directly:
n
1

lim cn = lim (e n 1) = lim (ex 1) = 1 1 = 0.


n

x0


 1

P
By the Alternating Series Test, we know
(1)n e n 1 is convergent. Hence
n=1
 1


P
n
the series
(1) e n 1 is conditionally convergent.
n=1

Homework 2 (page 731). How many terms of the series do we need to add in order

P
to find the sum of the series
(1)n+1 n16 correct to four decimal place?
n=1

1
Solution. By the Alternating Series Estimation Theorem, we require bn+1 = (n+1)
6 <

6
1
0.00005 = 20000 , and it implies (n + 1) > 20000 5.21. So we need to add at least

5 terms.
1

11.6

Absolute Convergence and the Ratio and Root


Test (page 732)

Homework 3. Determine the series (a)

(1)n1

n+1 n1
n

and (b)

(1)n
n(ln n)

is

n=2

n=1

(94 )

absolutely convergent, conditionally convergent, or divergent.


Solution.

(a) We observe that

n+1 n1
n+1 n1
n+1+ n1

n
n
n+1+ n1
2
2
q

=
= 3 q
.
n( n + 1 + n 1)
n2
1+ 1 + 1 1
n

The behavor looks like

Let an = (1)n1

, so we guess the series is absolutely convergent.

n2

n+1 n1
n

and bn =

1
3

n2

, then

|an |
2n 2

= lim q
= lim
n bn
n n( n + 1 +
n 1) n 1 +

lim

Since

1
n

= 1 > 0.
1

1
n

bn is a p-series with p = 23 , which is convergent, by the Limit Compar-

n=1

ison Theorem,
convergent.

an is convergent. Hence

n=1

(1)n1

n+1 n1
n

is absolutely

n=1

(b) Consider the function f (x) =

1
x ln x

on [2, ), which is a positive and continuous

function. We show that the function is decreasing on [2, ) by calculating


f (x) =

ln x x
x2 (ln x)2

1
x

(ln x + 1)
< 0.
x2 (ln x)2

Next, we compute that the integral


Z
Z
Z t
Z t
1
1
1
f (x) dx =
dx = lim
dx = lim
d ln x
t 2 x ln x
t 2 ln x
x ln x
2
2
h
i x=t

= lim (ln(ln t) ln(ln 2))
= lim ln(ln x)
t

x=2

is divergent. By the Integral Test, the series

1
n(ln n)

is divergent.

n=2

We still need to check whether the original series

(1)n
n(ln n)

is convergent or

n=2

not. The series


series satisfies

n=2

(1)n
n(ln n)

(1)n an is an alternating series, we check the

n=2

(1) {an }
n=1 is decreasing: This is because both x and ln x are increasing
functions, so we know
an =

1
1
>
= an+1 .
n ln n
(n + 1) ln(n + 1)

(2) lim an = 0: We can prove it by the Squeeze Theorem:


n

1
1
lim = 0.
n n ln n
n n

0 lim an = lim
n

By the Alternating Series Test, we know the series

(1)n
n(ln n)

is convergent.

n=2

Hence the series

(1)n
n(ln n)

is conditionally convergent.

n=2

Homework 4 (page 738). Determine whether the series is absolutely convergent,

n2
P
P
(n!)2
2
(b)
. (101 )
conditionally convergent, or divergent. (a)
n!
(5n)!
n=1

n=1

Solution.

(a) Let an =

2n
n!

and f (x) =

22x+1
.
x+1

We compute





2(n+1)2

an+1
n!
22n+1

= lim

=
lim
lim

n
an n (n + 1)! 2n2 n n + 1
22x+1
x x + 1

= lim

By the Ratio Test, the series

),L
(

2n
n!

22x+1 (2 ln 2)
= .
x
1
lim

is divergent.

n=1

(b) Let an =

(n!)2
.
(5n)!

We compute





((n + 1)!)2 (5n)!
an+1
= lim

lim
n (5(n + 1))! (n!)2
n an
(n + 1)2
= lim
n (5n + 5)(5n + 4)(5n + 3)(5n + 2)(5n + 1)
1
+ n24 + n15
n3
= lim 5
= 0 < 1.
n 5 (1 + 1 )(1 + 4 )(1 + 3 )(1 + 2 )(1 + 1 )
n
4n
5n
5n
5n
By the Ratio Test, the series

(n!)2
(5n)!

n=1

is absolutely convergent.

Homework 5 (page 737). Determine whether the series is absolutely convergent,


n2

P
P
(b)
( n n 1)2n .
conditionally convergent, or divergent. (a)
1 + n1
n=1

n=1

(101 )

Solution.
(a) Let an = 1 +
lim
n

 2
1 n
.
n

p
n

We compute

|an | = lim
n

By the Root Test, the series



n2 ! n1
n
1
1
1+
= lim 1 +
= e > 1.
n
n
n

1+

n=1

 2
1 n
n

is divergent.

(b) Let an = ( n n 1)2n . We compute


lim
n

p
n

1

|an | = lim ( n n 1)2n n = lim ( n n 1)2


n
n
2

1

= 0 < 1.
= lim
n
n+ n1

By the Root Test, the series

P
( n n 1)2n is convergent.

n=1

11.7

Strategy for Testing Series (page 739)

Homework 6. Discuss the following series converges or diverges when a = 1 and


a = 2:

X
n=1

123n
.
(a + 1)(a + 2)(a + 3) (a + n)
(89 )

Solution.
(1) If a = 1, then the series is

X
n=1

X 1
X1
123n
=
=
,
2 3 4 (1 + n) n=1 n + 1 n=2 n

which is harmonic sereis, so the series is divergent.

(2) If a = 2, then the series is

X
n=1

Let an =

2
(n+1)(n+2)

an
= lim
lim
n
n bn
Since

1
n2

X
123n
2
=
.
3 4 5 (2 + n) n=1 (n + 1)(n + 2)
and bn =

2
(n+1)(n+2)
1
n2

1
,
n2

then

2n2
2
= lim
3
2
n n + 3n + 2
n 1 +
+
n

= lim

2
n2

= 2 > 0.

is a p-series with p = 2, whcih is convergent, by the Limit Com-

n=1

parison Theorem, we know the series

123n
345(2+n)

is convergent.

n=1

Homework 7. Find the values of p for the convergence of the series below
(a)

en(p

2 p2)

n=0

(b)

e n 1
.
np

(102 )

n=1

Solution.
(a) Fixed p, the series

en(p

2 p2)

is a geometric series with common ratio r =

n=0

ep

2 p2

. If |r| < 1, the series is convergent. So we solve


ep

2 p2

< 1 p2 p 2 < 0 1 < p < 2.

Hence if 1 < p < 2, the series


If p 2 or p 1, the series

en(p

2 p2)

n=0

P
n(p2 p2)

is convergent.

is divergent.

n=0

(b) Let an =

e n 1
, bn
np

1
,
np+1

and f (x) = ex . Then

an
lim
= lim
n bn
n
Since

1
np+1

e n 1
np
1
np+1

= lim

en 1
1
n

ex 1
= f (0) = 1 > 0.
x0
x

= lim

is a p-series, which is convergent if p > 0 and divergent if p 0,

n=1

by the Limit Comparison Theorem, we know

n=1

and divergent if p 0.

e n 1
np

is convergent if p > 0

Homework 8. Determine the value of x for which the series

1
n ln n

1+

n=2


1 n
x

converges absolutely, converges conditionally or diverges.


(100 )
n
Solution. Let an = n ln1 n 1 + x1 . First we compute

n+1





1
1 + x1

(n+1) ln(n+1)


an+1
1
ln
n
n
1 +
= lim
= lim


lim
n
n n + 1 ln(n + 1)
1
n
an n
x
1 + x1

n ln n




1
= 1 + .
x
Remark that we consider a continuous function f (y) =
(
),L
ln y
ln n

= lim
lim
= lim
y ln(y + 1)
n ln(n + 1)
y


1
= lim 1 +
= 1.
y
y

By the Ratio Test, we know that



P
(a) If 1 + x1 < 1 x < 21 , the series

1
n ln n

n=2

1+

ln y
ln(y+1)

1
y
1
y+1


1 n
x



P
(b) If 1 + x1 > 1 x > 21 and x 6= 0, the series

= lim
y

y+1
y

is absolutely convergent.

1
n ln n

n=2

(c) If x = 21 , the series

on [2, ), then

1+


1 n
x

is divergent.

(1)n n ln1 n is an alternating series, and it is condition-

n=2

ally convergent. (The argument is the same as Homework 3)

11.8

Power Series (page 741)

Homework 9. Find the radius of convergence of the power series

X
n=0

n!(3n + 1)
xn .
1 3 5 (2n + 1)
(95 )

n!(3 +1)
xn . We compute
Solution. Let an = 135(2n+1)




an+1
(n + 1)!(3(n+1) + 1)xn+1

1

(2n
+
1)
= lim

lim




n
n 1 3 5 (2(n + 1) + 1)
an
n!(3n + 1)xn
1 + n1 3 + 31n
n + 1 3 3n + 1
3
n
|x| = lim

= lim
|x| = |x| < 1.
2
1
n 2 +
n 2n + 3
3 +1
2
1 + 3n
n

When 32 |x| < 1, that is |x| < 23 , the series

n=0

so the radius of convergence is R = 32 .

n!(3n +1)
xn
135(2n+1)

is absolutly convergent,

Homework 10. Consider the power series

1
xn .
n ln n

n=2

(a) Find the radius of the convergence r.

(b) Discuss whether the power series is convergent or divergent at x = r and


(96 )

x = r.
Solution.
1
xn
n ln n

and f (y) = ln y. We compute






1

xn+1
an+1
ln n
n
(n+1)
ln(n+1)

= lim
lim

|x| = |x|.
= lim
1

n
n
n
n n + 1 ln(n + 1)
an
x
n ln n

(a) Let an =

If |x| < 1, then the power series is convergent, so the radius of convergence is
r = 1.
(b) If x = 1, then the series is
decreasing function f (x) =
Z

1
.
n ln n

n=2
1
x ln x

Consider the positive, continuous, and

on [2, ) and compute

Z t
Z t
1
1
1
dx = lim
dx = lim
d ln x
f (x) dx =
t 2 x ln x
t 2 ln x
x ln x
2
h
i x=t

= lim (ln(ln t) ln(ln 2))
= lim ln(ln x)
Z

x=2

is divergent. By the Integral Test, the series


If x = 1, then the series is
ing series. Since bn+1 =
Alternating Series Test,

(1)n n ln1 n =

n=2

1
n ln n

is divergent.

(1)n bn , whcih is an alternat-

n=2
n=2
1
1
<
= bn ,
(n+1) ln(n+1)
n ln n

P
the series
(1)n n ln1 n is
n=2

1
n n ln n

and lim

= 0, by the

convergent.

Homework 11. Find the interval of convergence of the series

ln

n+1
n

n=2

Solution. Let an = ln

xn . We compute




ln n+2  xn+1
an+1
ln


n+1
= lim

=
|x|
lim
lim

n ln
n
an n ln n+1
xn
n
n+1
n

n+2
n+1

n+1
n

xn .

(101 )

= |x|.

ln( y+2
y+1 )

Remark that we choose a smooth function f (y) =

lim
n

ln
ln

n+2
n+1

n+1
n

ln
= lim
y

ln

1
y 1 +

= lim

y+2
y+1
y+1
y

2
y

on [2, ), then

y+1 (y+1)1(y+2)1

y+2
(y+1)2
lim
y1(y+1)1
y
y

y+1
y2

(
),L

ln( y+1
y )

y
y y + 2

= lim

= 1.

If |x| < 1, the power series is convergent.


If x = 1, we get

X
n=2

ln

n+1
n

= lim
n

= lim
n

n
X

(ln(k + 1) ln k) = lim

k=2
n+1
X

(1)n ln

n=2

ln k

n
X

ln k

k=2

n+1
n

We investigate the following conditions:

1
y2
1+ y1

n
X

ln k

k=2

= lim ln(n + 1) = .
n

(1)n bn , which is an alternating series.

n=2

(1) {bn }
n=1 is decreasing: Let f (y) = ln
f (y) =

ln(k + 1)

k=2

k=3

If x = 1, we get

n
X

y+1
y



= ln 1 + y1 on [2, ), then

< 0. So f (y) is decreasing and hence {bn }


n=1 is decreasing.

(2) lim bn = 0: We compute


n



1
lim bn = lim f (y) = ln lim 1 +
= ln 1 = 0.
n
y
y
y
By the Alternating Series Test, we know

(1)n ln

n+1
n

n=2

Hence the interval of convergence of

ln

n=2

n+1
n

xn is [1, 1).

is convergent.

Calculus A (2) Homework 3 ANSWER


11.9

Representations of Function as Power Series


(page 747)

Homework 1 (page 751). Find a power series representation for the following
function and determine the interval of convergence.
(1) f (x) =

3
.
x2 x2

(2) g(x) =

x2 +x
.
(1x)3

(partial fraction first)

(3) h(x) = x2 tan1 (x3 ).


Solution.
(1) We compute
1
1
1
1

=
x2 x+1
2 1


X
1
n+1
=
(1)
n+1 xn .
2
n=0

f (x) =

x
2

1
1 X  x n X
=

(x)n
1 (x)
2 n=0 2
n=0

Since f is represented as the sum of two geometric series and one is convergent
on (2, 2) and the other is convergent on (1, 1), the interval of convergence
of f (x) =

3
x2 x2

is (1, 1).

(2) We compute


x2 + x
2
x2 + x d2
1
x2 + x
=

=
2
g(x) =
(1 x)3
2
(1 x)3
2
dx
1x
!
!

x2 + x d2 X n
x2 + x X d2 n
=
=
x
x
2
dx2 n=0
2
dx2
n=0
!

X
n(n 1) n X n(n 1) n1
x2 + x X
n2
=
n(n 1)x
x +
x
=
2
2
2
n=2
n=2
n=2
=

X
n(n 1)
n=2

xn +

X
(n + 1)n
n=1

xn = x +

n2 xn =

n=2

The series is convergent if x (1, 1). If x = 1, the series is

n2 xn .

n=1

n2 , which is

n=1

divergent. If x = 1, the series is


interval of convergence of g(x) =

n2 (1)n , which is divergent. Hence the

n=1
x2 +x
(1x)3

is (1, 1).

(3) We compute

X
(1)n 3 2n+1 X (1)n 6n+5
h(x) = x tan (x ) = x
(x )
=
x
.
2n + 1
2n + 1
n=0
n=0
2

The series is convergent if |x3 | < 1 |x| < 1. If x = 1, the series is

(1)n
.
2n+1

n=0

If x = 1, the series is

2n+1

n=0
1
,
2n+1

nating series with bn =


1
n 2n+1

and lim bn = lim


n

(1)n

(1)6n+5 =

(1)n+1
2n+1

. Both series are alter-

n=0

which is decreasing bn =

1
2n+1

1
2(n+1)+1

= bn+1

= 0. By the Alternating Series Test, both series are

convergent. Hence the interval of convergence of h(x) = x2 tan1 (x3 ) is [1, 1].
Homework 2.
(a) Find the radius of convergence and the interval of convergence of the power

P
(x1)n
.
series
(2)n n
n=1

(b) Let f (x) =

(x1)n

(2)n n

when the power series is convergent. Evaluate f (3) (1).

n=1

(102 )

Solution.
(a) Let an =

(x1)n
.
(2)n n

We compute




r

n
an+1
(x 1)n+1

|x 1|
(2)
n
n
= lim
= lim

lim

n
an n (2)n+1 n + 1 (x 1)n n n + 1
2
|x 1|
1
|x 1|
=
lim q
.
=
2 n 1 + 1
2
n

The series is convergent if

|x1|
2

< 1 |x 1| < 2 1 < x < 3, and the

radius of convergence is R = 2.

P
(1)n
. Let bn =
If x = 3, the series is
n

1 ,
n

then bn =

1
n

1
n+1

= bn+1

n=1

is decreasing and lim bn = lim 1n = 0. By the Alternating Series Test, the


n
n

P
n
(1)

series
is convergent.
n
n=1

If x = 1, the series is

1 ,
n

which is a p-series with p =

1
2

< 1, so it is

n=1

divergent. Hence the interval of convergence of

n=1

(x1)n

(2)n n

is (1, 3].

(b) When the power series is convergent, we have


f (x) =

X
n=1

X
f (m) (1)
1
n

(x

1)
=
(x 1)m .
(2)n n
m!
m=0
1
(2)3 3

We compare the coefficients of (x 1)3 to get

f (3) (1)
,
3!

so f (3) (1) =

3
.
4

Homework 3 (page 753). Find the sum of each of the following series.
(1)

n(n 1)x ,

|x| < 1

(2)

n=1

X
n2 n

2n

n=1

(3)

X
n2
n=1

2n

Solution.
(1) We compute for |x| < 1,

n(n 1)xn =

n=1

n(n 1)xn = x2

n(n 1)xn2 = x2

n=2

n=2

 2 
d2
x
=x 2
x
dx
1x
n=2


2x2
1
d2
=
.
= x2 2 x 1 +
dx
1x
(1 x)3
d
=x 2
dx
2

1
2

(1, 1), from (1), we get


#
"


X
X
1
n2 n X
n
n(n

1)
=
=
n(n

1)x

n
n

2
2
n=1
n=1
n=1

(2) Since


2x2
=
= 4.
(1 x)3 x= 1

x= 21

(3) We compute for |x| < 1,

X
d
d n
(x ) = x
nxn1 = x
nxn = x
dx
dx
n=1
n=1
n=1


d
1
x
=x
1 +
=
.
dx
1x
(1 x)2

Since

X
d2 n
(x )
2
dx
n=2

1
2

(1, 1), we have


"
#


X
X
n
n
=
nx

n

2
n=1
n=1

x= 21

and hence

X
n2
n=1

2n

X
n2 n + n
n=1

2n

n=1

d
= x
dx



x

=
= 2,
2
(1 x) x= 1
2

X
n2 n
n=1

2n

X
n
+
= 4 + 2 = 6.
2n
n=1

x
1x

11.10

Taylor and Maclaurin Series (page 753)

Homework 4. Let f (x) = sin2 x.


(a) Find the Maclaurin series for f (x). (Hint: sin2 x = 12 (1 cos 2x). page 765)
(b) Find f (94) (0).

(94 )

Solution.

(a) Since the Maclaurin series of cos x is

(1)n 2n
x ,
(2n)!

which is defined on R and

n=0

cos x =

(1)n 2n
x ,
(2n)!

we have

n=0

1
1
sin2 x = (1 cos 2x) =
2
2
=

X
(1)n+1 22n1

(2n)!

n=1

X
(1)n
n=0

(2n)!

(2x)2n

1
=
2

X
(1)n 22n
n=1

(2n)!

x2n

x2n .

(b) Since the Maclaurin series of sin2 x can be represented as

f (m) (0) m
x .
m!

When

m=0

m = 94 and n = 47, the coefficients of x94 must be the same, so


f (94) (0)
(1)48 293
=
f (94) (0) = 293 .
94!
94!
Homework 5.

(a) Find the Taylor series for f (x) = (x2 + x + 1) x + 1 at x = 0 up to the third
power of x.
(b) Let f (x) = ln
Solution.

1+x2
.
1x2

Find f (10) (0).

(101 )

(a) From binomial series, we know

1
2

x + 1 = (1 + x) =

Cn2 xn ,

n=0

where
1
2

Cn =

)( 21 (n1))

1
21 1
2

n!

if n N
if n = 0

The radius of convergence of

x + 1 is R = 1, so

X
1

2
f (x) = (x + x + 1) x + 1 = (x + x + 1)
Cn2 xn
2

n=0

Cn2 (xn+2 + xn+1 + xn ) =

n=0

Cn2 xn+2 +

n=0

1
2

Cn2xn +

n=2

1
2

Cn1xn +

n=1

Cn2 xn+1 +

n=0

1
2

Cn2 xn

n=0

Cn xn

n=0


 1
X
1
1
1
1
1
2
2
2
2
2
= C0 + C0 + C1 x +
Cn2 + Cn1 + Cn2 xn
n=2

1
1
1 3
( 12 )
1
1
2 ( 2) ( 2)
2
=
=

,
and
C
= 16
, the
3
2!
8
3!

Taylor series for f (x) = (x2 + x + 1) x + 1 at x = 0 up to the third power of


1

Since C02 = 1, C12 = 21 , C22 =

x is
 1
 1
 1
1
1
1
1
1
1
C02 + C02 + C12 x + C02 + C12 + C22 x2 + C12 + C22 + C32 x3 +

11
7
3
= 1 + x + x2 + x3 + .
2
8
16
(b) Since f (x) = ln

1+x2
1x2

= 21 ln(1 + x2 ) 21 ln(1 x2 ), and

1
1 X (1)n1 2 n X (1)n1 2n
ln(1 + x2 ) =
(x ) =
x ,
2
2 n=1
n
2n
n=1

R=1

X 1
1
1 X (1)n1
ln(1 x2 ) =
(x2 )n =
x2n ,
2
2 n=1
n
2n
n=1

R=1

we get
f (x) =

X
((1)n1 + 1)

2n

n=1

x2n .

Since the Maclaurin series of f (x) can be represented as

f (m) (0) m
x ,
m!

m=0

m = 10 and n = 5, the coefficients of x10 must be the same, so


1
10!
f (10) (0)
= f (10) (0) =
.
10!
5
5

when

Homework 6. Find the first three terms of the Maclaurin series of


1

((1 x2 )(1 + x)) 4 .


(99 )
Solution. From binomial series, we have
2 14

(1 x )

= (1 + (x ))

41

1
Cn 4 (x2 )n

n=0

(1 + x) 4 =

Cn 4 (1)n x2n ,

R=1

n=0

R = 1,

Cm 4 xm ,

m=0

so

((1 x2 )(1 + x))

14

1
Cn 4 (1)n x2n

n=0

1
Cm 4 xm

m=0

where
1
4

Cn =
We compute

14 ( 41 1)( 41 (n1))
n!

if n N

if n = 0

41

41

41

coefficients of x0 : C0 4 (1)0 C0
coefficients of x1 : C0 4 (1)0 C1
coefficients of x2 : C1 4 (1)1 C0

= 1.
= 41 .
1

41

+ C0 4 (1)0 C2

1
4

4
32

13
.
32
1

So the first three terms of the Maclaurin series of ((1 x2 )(1 + x)) 4 is
13
1
1 x + x2 .
4
32
(x tan1 x)(e3x 1)
.
x0
2x2 1 + cos 2x

(100 )

Homework 7. Evaluate lim


Solution. Since
tan

X
(1)n 2n+1
x=
x
2n + 1
n=0

R=1

X
X
X
1 n
1
3n n
n
3x
e =
x e =
(3x) =
x
n!
n!
n!
n=0
n=0
n=0

R=

cos x =

X
(1)n
n=0

(2n)!

2n

cos 2x =

X
(1)n
n=0

(2n)!

(2x)

2n

X
(1)n 22n
n=0

(2n)!

x2n , R = ,

we have for |x| < 1,


(x tan1 x)(e3x 1)
x0
2x2 1 + cos 2x


x x + 31 x3 15 x5 + 1 + 3x + 29 x2 + 1
= lim
x0
2x2 1 + 1 2x2 + 32 x4


1 3
x 15 x5 + 3x + 29 x2 +
x4 + 23 x5 35 x6 +
3
=
lim
= lim
2 4
2 4
x0
x0
x
x
3
3


3 9
9
3
= lim
+ x x2 + = .
x0 2
4
10
2
lim

sin x tan1 x x2 +
Homework 8. Find lim
x0
x6

x4
2

(95 )

Solution. Since

X
(1)n 2n+1
x
,
sin x =
(2n + 1)!
n=0

tan

X
(1)n 2n+1
x=
x
,
2n + 1
n=0

R=
R = 1,

we have for |x| < 1


4

sin x tan1 x x2 + x2
lim
x0
x6



5
7
3
5
7
4
3
x x3! + x5! x7! + x x3 + x5 x7 + x2 + x2
= lim
x0
x6 
 4


1
1
1
1
1
1
2
+ 35!
+ 7!1 x8 + x2 +
x 3 + 3! x + 5 + 33! + 5!1 x6 71 + 53!
= lim
x0
x6 



19
1
1
1
1
19
x2 + = .

+
+
+
= lim
x0
72
7 5 3! 3 5! 7!
72

x4
2

Homework 9.
(a) Find the Maclaurin series for f (y) = sin y.
Z
2
(b) Evaluate
sin(cos x) dx correct to within an error of 0.01.

(98 )

Solution.
(a) We compute for k = 0, 1, 2, . . .,
f (4k) (y) = sin y f (4k+1) (y) = cos y f (4k+2) (y) = sin y f (4k+3) (y) = cos y
f (4k) (0) = 0

f (4k+1) (0) = 1

f (4k+2) (0) = 0
7

f (4k+3) (0) = 1,

so the Maclaurin series for f (y) = sin y is

(1)n 2n+1
y
.
(2n+1)!

Remark that the

n=0

Maclaurin series of f (y) = sin y is convergent for all y R and f (y) =

P
(1)n 2n+1
y
for all y R.
(2n+1)!

n=0

(b) Since 0 cos x 1 for 0 x 2 , we know the following equality holds:

X
(1)n
cos2n+1 x,
sin(cos x) =
(2n
+
1)!
n=0

and
Z

X
(1)n
cos2n+1 x dx
sin(cos x) dx =
0 n=0 (2n + 1)!
R

2n+1
2
X
x dx def. X
n 0 cos
=
(1)
=
(1)n bn .
(2n + 1)!
n=0
n=0

This is an alternating series. If we can show that {bn }


n=1 is decreasing and
lim bn = 0, then the series is convergent, and we can use the Alternating
n

Series Estimation Theorem to estimate the error.


2n+3
x
(1) {bn }
n=1 is decreasing: Since 0 cos x 1, we have 0 cos

cos2n+1 x 1 and
R
2

bn+1 =

cos2n+3 x dx

(2n + 3)!

R
2

cos2n+1 x dx
= bn .
(2n + 1)!

(2) lim bn = 0: We compute


n

R

2
R
2n+1

x dx
2
0 cos
|cos2n+1 x| dx

2
0
=

.
|bn |
(2n + 1)!
(2n + 1)!
(2n + 1)!
2(2n + 1)

= 0, by the Squeeze Theorem, we know lim bn = 0.


Since lim 2(2n+1)
n

From the Alternating Series Estimation Theorem, we know that the error of
n
P
(1)k bk can be estimated by bn+1 , and we hope bn+1 < 0.01. We compute

k=0

1
bn =
(2n + 1)!
1
=
(2n + 1)!
=

1
(2n + 1)!

1
=
(2n + 1)!

cos

2n+1

1
x dx =
(2n + 1)!

cos2n x d sin x

1
(1 sin x) d sin x =
(2n + 1)!
0
Z
n
X
2
k n
sin2k x d sin x
(1) Ck
Z

k=0
n
X
k=0

n
X

Ckn ( sin2 x)k d sin x

k=0

(1)

Ckn

 x=
n
X
sin2k+1 x 2
(1)k Ckn
1
=
.
2k + 1 x=0
(2n + 1)! k=0 2k + 1
8

Since
(1)0
1
C00
=1
1! 
1

0
1
1
1
1 (1)
1 (1)
C0
=
+ C1
b1 =
3!
1
3
9


0
1
2
1
1
1
2 (1)
2 (1)
2 (1)
C0
=
+ C1
+ C2
<
= 0.01,
b2 =
5!
1
3
5
225
100

b0 =

the integral
0.01.

R
2

sin(cos x) dx (1)0 b0 + (1)1 b1 = 1

1
9

8
9

with error less than

Calculus A (2) Homework 4 ANSWER


11.11

Applications of Taylor Polynomials


(page 768)

Homework 1. Approximate

240 with error less than 0.0001.

Solution. From the binomial series f (x) = (1 + x)m =

Cnm xn with m =

1
5

and

n=0

x = 314 , we have

240 =

243 3 =

s
5



1
n



X
1
1 5
3
1
5
Cn 4
=3 1 4
=3
,
243 1
243
3
3
n=0

where C05 = 1, C15 = 15 and for n N, n 2,





1
4
9
1
1

(n

1)

1 4 9
5n 6
5
5
5
= (1)n1

Cn5 = 5
n!
5 10 15
5n

n 
n1 Y
6
(1)
1
=
5
5k
k=2
so

240 = 3

1
5

Cn

n=0

1
4
3

n

!


n 
X
(1)n1 Y
1 1
6 (1)n
=3 1 4 +
1
5 3
5
5k
34n
n=2
k=2
!

n 
X
6
1 1
1 Y
.
1
=3 1 4
5 3
5 34n
5k
n=2
k=2


If we approximate 5 240 3 1 15 314 2.9926, then the error is

!


n 
n 

X
X
1 Y
6
6
1 Y


|error| = 3
1
1
=3
4n
4n


5

3
5k
5

3
5k
n=2
n=2
k=2

X
n=2

1
5 34n

k=2

1
4
6 X 1
6
1
1
38

=
=

<
.
1 =
4n
10
25 n=2 3
25 1 34
25 27 40
10000

Homework 2 (page 776). If a surveyor measures differences in elevation when making plans for a highway across a desert, corrections must be made for the curvature
of the earth.
(a) If R is the radius of the earth and L is the length of the highway, show that
the correction is
 
L
R.
C = R sec
R
(b) Use a Taylor polynomial to show that
C

L2
5L4
+
.
2R 24R3

(c) Compare the corrections given by the formulas in parts (a) and (b) for a
highway that is 100 km long. (Take the radius of the earth to be 6370 km.)
L
R

Figure 1: Surveyors measures differences in elevation of highway.


Solution.
L
, and
(a) From Figure 1, we know = R


 
C +RR
C +R
L
C =R
R.
=R
1 = R sec R = R sec
R
R
R

(b) Since cos x =

(1)n 2n
x ,
(2n)!

we have

n=0

so

 2n
  X

L2
(1)n L
L4
L6
L
=
=1
+

+
cos
2
4
6
R
(2n)!
R
2R
24R
720R
n=0

 2
L4
L6
L

+
,
=1
2R2 24R4 720R6
C=R

1
cos

L
R

 1

=R

L2
2R2

 1
L6
+ 720R
6
!
n
1
L4
24R4


X
L2
L4
L6
=R 1+

+
2R2 24R4 720R6
n=1
n

X
L4
L6
L2

+
.
=R
2R2 24R4 720R6
n=1

When expanding the above eries and collect terms in power of L, we have
C

5L4
L2
+
.
2R 24R3

(c) We put L = 100 km and R = 6370 km into the formula in (a) and get
 


L
100
C = R sec
R = 6370 sec
6370 0.78500996544 km.
R
6370
The formula in (b) gives
5L4
1002
5 1004
L2
+
=
+
0.78500995736 km.
2R 24R3
2 6370 24 63703

The difference is only 0.00000000808 km.


Homework 3. Find the sum of the series (a)

(1)n n
32n (2n)!

n=0

(b)

(ln 2)2n
.
(2n)!

n=0

(100 )

Solution.
(a)

(1)n n
32n (2n)!

(1)n
(2n)!

n=0

n=0

(b) Since

 2n

n=0

(1)n 2n
x
(2n)!

x=

= cos

 

.
3

X 1
x2 x3 x4 x5 x6
e =1+x+
+
+
+
+
+ =
xn
2!
3!
4!
5!
6!
n!
n=0

X (1)n
x2 x3 x4 x5 x6

+
+ =
xn ,
2!
3!
4!
5!
6!
n!
n=0

ex = 1 x +

X 1

1 x
x2 x4 x6
e + ex = 1 +
+
+
+ =
x2n ,
2
2!
4!
6!
(2n)!
n=0

we have
#



1 ln 2
1 x
1 2n
x
=
=
e +e
e + e ln 2
=
x

(2n)!
(2n)!
2
2
x=ln 2
n=0
x=ln 2


1
1
5
=
2+
= .
2
2
4

X
(ln 2)2n
n=0

"
X

12.6

Cylinders and Quadric Surfaces (page 827)

Homework 4 (page 833, 836). Classify the following quadric surfaces.


(a) 4x2 + y 2 + 4z 2 4y 24z + 36 = 0.
(b) x2 y 2 + z 2 4x 2y 2z + 4 = 0.
(c) 4y 2 + z 2 x 16y 4z + 20 = 0.
(d) z = x2 y 2.

(e) y 2 + z 2 = 1 + x2 .

(f) 4x2 + y 2 4z 2 = 4.

Solution.
(a) Since 4x2 + y 2 + 4z 2 4y 24z + 36 = 4x2 + (y 2)2 + 4(z 3)2 4 = 0, we
get
x2 +

(y 2)2
+ (z 3)2 = 1,
4

which is an ellipsoid with center (0, 2, 3).


(b) Since x2 y 2 + z 2 4x 2y 2z + 4 = (x 2)2 (y + 1)2 + (z 1)2 = 0, we
get
(y + 1)2 = (x 2)2 + (z 1)2 ,
which is a circular cone with center (2, 1, 1) and axis the horizontal line
x = 2, z = 1.
(c) Since 4y 2 + z 2 x 16y 4z + 20 = x + 4(y 2)2 + (z 2)2 = 0, we get
(z 2)2
x
= (y 2)2 +
,
4
4
which is an elliptic paraboloid with vertex (0, 2, 2) and axis horizontal line
y = 2, z = 2.
(d) The equation z = x2 y 2 represents a hyperbolic paraboloid with center
(0, 0, 0).
(e) The equation y 2 + z 2 = 1 + x2 is equivalent to x2 + y 2 + z 2 = 1, which is a
hyperboloid of one sheet with axis the x-axis.
(f) The equation 4x2 + y 2 4z 2 = 4 is equivalent to x2 +
is a hyperboloid of two sheet with axis the y-axis.
4

y2
4

z 2 = 1, which

13.1

Vector Functions and Space Curves


(page 840)

Homework 5 (page 847). Find a vector function that represents the curve of intersection of the two surfaces.
p
(a) The cone z = x2 + y 2 and the plane z = 1 + y.
(b) The semiellipsoid x2 + y 2 + 4z 2 = 4, y 0, and the cylinder x2 + z 2 = 1.

Solution.
(a) First we observe that z 2 = x2 + y 2 = (1 + y)2, and it gives x2 = 1 + 2y. If we
2
let x = t, then y = t 21 , and
s
 2 s 2
2
 2
p
t
+
1
t2 + 1
t

1
=
=
.
z = x2 + y 2 = t2 +
2
2
2
2 
2 
Hence a vector function is r(t) = t i + t 21 j + t 2+1 k, t R.

(b) We can first assume that x = cos t and z = sin t, and then solve y from the
equation x2 + y 2 + 4z 2 = 4, y 0:

y 2 = 4 x2 4z 2 = 4 cos2 t 4 sin2 t = 3 cos2 t y = 3 cos t, t .


2
2

Hence a vector function is r(t) = cos t i + 3 cos t j + sin t k, 2 t 2 .

13.2

Derivatives and Integrals of Vector Functions


(page 847)

Homework 6 (page 850). Show that


(3)

d
(f (t) u(t))
dt

(5)

d
(u(t)
dt

= f (t) u(t) + f (t) u (t).

v(t)) = u (t) v(t) + u(t) v (t).

Solution.
(3) Let u(t) = u1 (t) i + u2 (t) j + u3 (t) k, then
d
d
(f (t) u(t)) = (f (t)u1(t) i + f (t)u2 (t) j + f (t)u3 (t) k)
dt
dt

= (f (t)u1 (t) + f (t)u1(t)) i + (f (t)u2 (t) + f (t)u2 (t)) j + f (t)u3 (t) + f (t)u3(t)) k
= f (t)(u1 (t) i + u2 (t) j + u3 (t) k) + f (t)(u1 (t) i + u2 (t) j + u3 (t) k)
= f (t) u(t) + f (t) u(t).
5

(5) Let u(t) = u1 i + u2 j + u3 k and Let v(t) = v1 i + v2 j + v3 k, where ui and vi ,


i = 1, 2, 3 are functions of t, then
d
d
(u(t) v(t)) = ((u2 v3 u3 v2 ) i + (u3 v1 u1 v3 ) j + (u1 v2 u2 v1 ) k)
dt
dt
= (u2 v3 u3 v2 ) i + (u3 v1 u1 v3 ) j + (u1 v2 u2 v1 ) k
+ (u2 v3 u3 v2 ) i + (u3v1 u1 v3 ) j + (u1 v2 u2v1 ) k
= u (t) v(t) + u(t) v (t).
Homework 7 (page 853). If r(t) 6= 0, show that

d
r(t) r (t)
|r(t)| =
.
dt
|r(t)|

Solution. We compute

13.3

d
r (t) r(t) + r(t) r (t)
r(t) r (t)
dp
p
r(t) r(t) =
=
|r(t)| =
.
dt
dt
|r(t)|
2 r(t) r(t)

Arc Length and Curvature (page 853)

Homework 8. Consider a curve r(t) = (3t t3 ) i + 3t2 j + (3t + t3 ) k, t R.


(a) Find the unit tangent T(t).
(b) Find the arc length function s(t).
(c) Find the unit normal N(t).
(100 )

(d) Find the curvature (t).


Solution.
(a) We compute
r (t) = (3 3t2 ) i + 6t j + (3 + 3t2 ) k,

t R,

and
p

(3 3t2 )2 + (6t)2 + (3 + 3t2 )2 = 18t4 + 36t2 + 18


p

= 18(t2 + 1)2 = 3 2(t2 + 1).

|r (t)| =

So the unit tangent is


T(t) =

1 t2
2t
1
r (t)

=
i+
j + k,

2
2
|r (t)|
2(t + 1)
2(t + 1)
2
6

t R.

(b) The arc length function is


Z t
Z t
h
i u=t

2u3 + 3 2u
s(t) =
|r (u)| du =
3 2(u2 + 1) du =
u=0
0
0
3

= 2t + 3 2t.
(c) We compute
(t2 + 1)2 2t(2t)
(t2 + 1)(2t) (1 t2 )(2t)

i+
j+0k
2(t2 + 1)2
2(t2 + 1)2

2(1 t2 )
2 2t
i
+
j + 0 k,
= 2
(t + 1)2
(t2 + 1)2

T (t) =

and
1
|T (t)| = 2
(t + 1)2

(2

2t)2

+ ( 2(1

t2 ))2

02

2(t2 + 1)2
2
= 2
,
2
2
(t + 1)
t +1

so the unit normal is


N(t) =

T (t)
2t
1 t2
=
i
+
j + 0 k.
|T (t)|
t2 + 1
t2 + 1

(d) The curvature is

1
|T (t)|
2
= t +1
.
=
(t) =
|r (t)|
3(t2 + 1)2
3 2(t2 + 1)
Homework 9. Find the curvature , the unit tangent vector T, the unit normal
vector N, and binormal vector B of the curve r(t) = (cos t, sin t, ln(cos t)) at r(0) =
(1, 0, 0).
(97 )
Solution. We compute


sin t

r (t) = sin t, cos t,


= ( sin t, cos t, tan t)
cos t
p
|r (0)| = 1,
|r (t)| = ( sin t)2 + (cos t)2 + ( tan t)2 = sec t
so

T(t) =

r (t)
= ( sin t cos t, cos2 t, sin t)
|r (t)|

T(0) = (0, 1, 0).

Since
T (t) = ( cos2 t + sin2 t, 2 cos t sin t, cos t),
7

we have
(1, 0, 1)
T (0)
=
=p
N(0) =
|T (0)|
(1)2 + 02 + (1)2


1
1
B(0) = T(0) N(0) = , 0, ,
2
2



1
1
, 0, ,
2
2

and
(0) =

|T (0)|
= 2.
|r (0)|

Homework 10. Find the curvature of the graph of the function y = ln(cos x) at
(95 )

x = 4 .

Solution. The graph of the function y = ln(cos x) can be thought as a vector


function r(x) = (x, ln(cos x), 0) in R3 . We compute


sin x

, 0 = (1, tan x, 0),


r (x) = 1,
cos x
 
p

2
2
|r (x)| = 1 + ( tan x) + 0 = sec x r
= 2,
4
r (x) = (0, sec2 x, 0),
and

 

(r r )(x) = (0, 0, sec x) (r r )


= 2.
4

So the curvature at x =

is

14.1

 
4



(r r )
1
4
=
= .
 3
r

2
4

Functions of Several Variables, page 878

Homework 11. Find and sketch the domain of the function f (x, y) = sin1 (x2 +
y 2 2).
Solution. Domain of the function is
{(x, y)| 1 x2 + y 2 2 1} = {(x, y)|1 x2 + y 2 3},
which is an annulus. See Figure 2.

x
3

Figure 2: Domain of f (x, y) = sin1 (x2 + y 2 2).


Homework 12 (page 891). Match the function (a),(b),(c) with its graph (A),(B),(C)
and its contour map (I), (II), (III). Give reasons for your choices.
(a) f (x, y) = sin x sin y

60
40
20
0
-20
-4

(b) g(x, y) =

xy
1 + x2 + y 2

2
1
0
-1
-2

-2

-2

y0

0
2

-4

-4
-2

44

0
2

-4

(B)

-2

-2

-2

(I)

-4
-4

-2

(C)
4

0
2
44

-2

-2

y0

-4

-4

-2

44

(A)

-4

0.5
0
-0.5

-2

y0

(c) h(x, y) = ex cos y.

(II)

-4
-4

-2

(III)

Figure 3: Match functions, graphs, and contour maps.


Solution.
(a) Fix x0 , f (x0 , y) is a trigonometric function with period 2. Fix y0 , f (x, y0) is
a trigonometric function with period 2. So its graph would be (B) and the
contour map is (III).
9

(b) We know the symmetry g(y, x) = g(x, y) and the function is not periodic
function, so the graph of g(x, y) is (C) and the contour map is (I).
(c) For cos y > 0 (cos y < 0), f (x, 0) = ex is exponentially increasing (decreasing),
so the graph of h(x, y) is (A) and the contour map is (II).

14.2

Limits and Continuity, page 892

Homework 13 (page 899900). Find the limit, if it exists, or show that the limit
does not exist.
(a)
(b)
(c)
(d)

lim

xy

x2 +y 2

(x,y)(0,0)

(94 )

x2 y ey
4 +4y 2 .
x
(x,y)(0,0)

lim

x2 +y 2

lim

lim

(x2 + y 2 ) ln(x2 + y 2 ).

(x,y)(0,0)

(x,y)(0,0)

x2 +y 2 +11

Solution.
(a) Let x = r cos and y = r sin , then for r 6= 0, we have


r 2 cos sin

xy


= r 0 as r 0+ .
=
p

2
2
x +y
r
By the Squeeze Theorem, we get

xy

lim

(x,y)(0,0)

x2 +y 2

= 0.

(b) We take a path C1 (x) = (x, 0) for x 6= 0, then f (x, y)|C1 (x) 0. We take
another path C2 (x) = (x, x2 ) for x 6= 0, then
2

f (x, y)|C2(x)
so the limit

x2 y ey
4 +4y 2
x
(x,y)(0,0)

lim

x2 x2 ex
ex
1
= 4
=
as x 0,
2
2
x + 4(x )
5
5
does not exist.

(c) Let x = r cos and y = r sin , then we have


x2 + y 2
r2
p
= lim+
(x,y)(0,0)
r2 + 1 1
x2 + y 2 + 1 1 r0

r2 + 1 + 1
r2

= lim+ ( r 2 + 1 + 1) = 2.
= lim+
r0
r2 + 1 1
r 2 + 1 + 1 r0
lim

10

(d) Let x = r cos and y = r sin , then we have


lim

(x,y)(0,0)

= lim+

(x2 + y 2 ) ln(x2 + y 2 ) = lim+ r 2 ln(r 2 ) = lim+ 2r 2 ln r


r0

2 ln r

r0

1
r2

,L )
(

lim+

r0

2 1r
r23

r0

= lim+ r 2 = 0.
r0

Homework 14 (page 900). Determine the set of points at which the function is
continuous.
( 2 3
x y
if (x, y) 6= (0, 0)
2x2 +y 2
f (x, y) =
.
1
if (x, y) = (0, 0)
1
2

Solution. Let x =

r cos and y = r sin , then for r 6= 0, we have





x2 y 3 12 r 2 cos2 r 3 sin3 1 3


= r cos2 sin3 1 r 3 0 as r 0+ ,
=
2x2 + y 2 2 1 r 2 cos2 + r 2 sin2 2
2
2

so

x2 y 3
= 0 6= f (0, 0),
(x,y)(0,0) 2x2 + y 2
lim

and the function is not continuous at (0, 0). For (x, y) 6= (0, 0), the function is a
rational function, so it is continuous. Hence the function is continuous on R2
{(0, 0)}.
Homework 15. Let
f (x, y, z) =

xy+yz 3
x2 +z 6

if (x, y, z) 6= (0, 0, 0)

if (x, y, z) = (0, 0, 0).

Determine the set of points at which f (x, y, z) is continuous.

(92 )

Solution. We take a path C1 (x) = (x, 0, x) for x 6= 0, then f (x, y, z)|C1 (x) 0. We
take another path C2 (z) = (z 3 , z 3 , z) for z 6= 0, then
f (x, y, z)|C2 (z) =
so the limit

lim

(x,y,z)(0,0,0)

z3 z3 + z3 z3
1,
(z 3 )2 + z 6

f (x, y, z) does not exist and hence the function is not con-

tinuous at (x, y, z) = (0, 0, 0).


For (x, y, z) 6= (0, 0, 0) or x2 + z 6 6= 0, the function is a rational function, so it is
continuous. Hence the function is continuous on R3 {(0, 0, 0)}.
Remark that if (x, y, z) 6= (0, 0, 0), the function is not defined on the region
x2 + z 6 = 0, that is, x = 0, z = 0, y 6= 0.
11

Calculus A (2) Homework 5 ANSWER


14.3

Partial Derivative, page 900

Homework 1 (page 915). If f (x, y) =

x esin(x

2 y)
3

(x2 + y 2 ) 2

, find fx (1, 0).

Solution. Putting y = 0 into the function first, we get



x
1
2
= 2.
f (x, 0) = 3 = 2 fx (1, 0) = fx (x, 0)|x=1 = 3
x
x
x x=1

Solution 2. By definition, we have

(1+h)esin((1+h)

2 0)

2 0)

esin(1

3
f (1 + h, 0) f (1, 0)
(12 +02 ) 2
= lim
h0
h0
h
h
1
2
1
1 (1 + h)
2h h2
2 h
(1+h)2
= lim
= lim
=
lim
=
lim
= 2.
h0
h0 h(1 + h)2
h0 h(1 + h)2
h0 (1 + h)2
h
3
((1+h)2 +02 ) 2

fx (1, 0) = lim

Solution 3. Direct computation gives





1
2
sin(x2 y)
sin(x2 y)
2
2
2 32
+ xe
cos(x y)2xy x esin(x y) 32 (x2 + y 2 ) 2 2x
(x + y ) e

fx (1, 0) =

(x2 + y 2)3

(1,0)

3
2

1 2
= 2.
1

Homework 2. Let r(x, y) =


and ryy .

x2 + y 2 . For (x, y) 6= (0, 0), compute rx , ry , rxx , rxy , ryx ,

Solution. Direction computation gives


y
2x
x
2y
=p
rx = p
=p
,
ry = p
,
2 x2 + y 2
x2 + y 2
2 x2 + y 2
x2 + y 2
p
x2 + y 2 x 2x2 2
y2
x2 + y 2 x2
2 x +y
=
=
rxx =
3
3 ,
x2 + y 2
(x2 + y 2 ) 2
(x2 + y 2 ) 2
x 2y
xy
y 2x
xy
rxy =
ryx =
3 =
3 ,
3 =
3 ,
2(x2 + y 2) 2
(x2 + y 2 ) 2
2(x2 + y 2 ) 2
(x2 + y 2 ) 2
p
x2 + y 2 y 2y2 2
x2 + y 2 y 2
x2
2 x +y
ryy =
=
=
3
3 .
x2 + y 2
(x2 + y 2) 2
(x2 + y 2) 2

Homework 3. Let f (x, y) =

x3 xy 2
.
x2 + y 2

(a) Determine the value f (0, 0) such that f (x, y) is continuous at (0, 0).
(89 )

(b) Find fx (x, y), fy (x, y), fx (0, 0) and fy (0, 0).
(c) Compute fxy (0, 0) and fyx (0, 0).
Solution.

(a) Let x = r cos , y = r sin , then




3
x xy 2 r 3 (cos3 cos sin2 )

2r 0 as r 0.


x2 + y 2 =
r2
So we define f (0, 0) =

lim

(x,y)(0,0)

f (x, y) = 0.

(b) If (x, y) 6= (0, 0), direct computation gives


(x2 + y 2 )(3x2 y 2 ) (x3 xy 2 )(2x)
x4 + 4x2 y 2 y 4
=
(x2 + y 2)2
(x2 + y 2 )2
4x3 y
(x2 + y 2 )(2xy) (x3 xy 2 )(2y)
=
.
fy (x, y) =
(x2 + y 2)2
(x2 + y 2)2

fx (x, y) =

By definition, we know
f (h, 0) f (0, 0)
fx (0, 0) = lim
= lim
h0
h0
h
f (h, 0) f (0, 0)
fy (0, 0) = lim
= lim
h0
h0
h

h3
h2

0
= lim 1 = 1
h0
h
0

0
h2
= lim 0 = 0.
h0
h

(c) By definition, we know


4

h
2
fx (0, h) fx (0, 0)
4 1
= lim h
= lim
does not exist,
fxy (0, 0) = lim
h0
h0 h
h0
h
h
fy (h, 0) fy (0, 0)
00
0
fyx (0, 0) = lim
= lim
= lim = 0.
h0
h0
h0 h
h
h

14.4

Tangent Planes and Linear Approximations,


page 915

Homework 4. Find the tangent plane to the surface x2 + y 2 + z 2 = 6xyz 3 at


(99 )

the point (1, 1, 1).

Solution. We differentiate the equation F (x, y, z) = F (x, y, z(x, y)) = x2 + y 2 +


z 2 6xyz + 3 = 0 as follows:
z
z
6yz 6xy
=0
Fx (x, y, z(x, y)) = 0 2x + 2z
x
x


z
x + 3yz
(1) + 3 1 (1)

=
=
= 1.


x (1,1,1)
z 3xy (1,1,1)
(1) 3 (1) 1
z
z
Fy (x, y, z(x, y)) = 0 2y + 2z
6xz 6xy
=0
y
y


z
y + 3xz
1 + 3 (1) (1)

= 1.
=
=


y (1,1,1)
z 3xy (1,1,1)
(1) 3 (1) 1

So the tangent plane is z + 1 = (x + 1) + 1(y 1), or x y + z = 3.

Homework 5. Let z = y ex + cos xy . Find the equation of the tangent plane and
the normal line equation to the surface at (x, y, z) = (1, 2 , e
).
2

(96 )

Solution. First we compute



e
y  y 
z
x
2
+ ,
=
= y e sin

x (x,y)=(1, )
x
x (x,y)=(1, 2 )
2
2
2

 
z
y 1
x
=
e

sin
= e 1.
y (x,y)=(1, )
x x (x,y)=(1, )
2

So the tangent plane is



e

(x 1) + (e 1) y
(z ) = 0.
+
2
2
2
2

 e

Normal line equation is


y 2
z e
x1
2
=
.
=
e
2
e1
1
2

14.5

The Chain Rule, page 924

Homework 6 (page 928). If g(s, t) = f (s2 t2 , t2 s2 ) and f is differentiable, show


that g satisfies the equation t g
+ s g
= 0.
s
t
Solution. Let g(s, t) = f (u(s, t), v(s, t)), then by the Chain Rule, we have




g
f
f
f u f v
f
f
t
= t 2s
= 2st
=t
+
2s
2st
s
u s
v s
u
v
u
v




f
f
f
f
f u f v
g
= s 2t
= 2st
=s
+
+ 2t
+ 2st .
s
t
u t
v t
u
v
u
v
+ s g
= 0.
Hence g(s, t) satisfies t g
s
t
Homework 7. Suppose that w = f (x, y) satisfies the Laplace equation: wxx +wyy =
0. Let x = 21 (u2 v 2 ) and y = uv. Show that w also satisfies the Laplace equation:
(90 )

wuu + wvv = 0.

Solution. We compute

xu xv
u v
xuu xuv
1 0
yuu yuv
0 1

=
,
=
,
=
.
yu yv
v u
xvu xvv
0 1
yvu yvv
1 0
By the Chain Rule, we have
wu = wx xu + wy yu

wv = wx xv + wy yv

wuu = (wx xu + wy yu )u = (wxx xu + wxy yu )xu + wx xuu + (wyx xu + wyy yu )yu + wy yuu
= x2u wxx + 2xu yu wxy + yu2 wyy + xuu wx + yuu wy
= u2 wxx + 2uvwxy + v 2 wyy + wx ,
wvv = (wx xv + wy yv )v = (wxx xv + wxy yv )xv + wx xvv + (wyx xv + wyy yv )yv + wy yvv
= x2v wxx + 2xv yv wxy + yv2 wyy + xvv wx + yvv wy
= v 2 wxx 2uvwxy + u2 wyy wx .
Hence
wuu + wvv = (u2 + v 2 )wxx + (u2 + v 2 )wyy = (u2 + v 2 )(wxx + wyy ) = 0.

Homework 8 (page 932). Let z = f (x, y) be a differentiable function. From


relations x = r cos and y = r sin , we get the identity:
 2
 2
 2  2
z
z
z
z
+
= A(r, )
+ B(r, )
.
x
y
r

(96 )

Find A(r, ) and B(r, ).

Solution. First we compute

x x
r r
1 0
cos r sin
r r
1 0
r x y =

x y =
,
yt y
x y
0 1
sin r cos
x y
0 1
we have


r r
cos r sin
r cos r sin
cos sin
.
x y =
= 1
=
r sin cos
x y
sin r cos
sinr cosr

By the Chain Rule, we have


 2 
2  2  2
 2  2
z
z
z

z r z
r
z z r
+
+
=
=
+2
x
r x x
x
r
r x x
x

2
 2
2 
z
2 sin cos r sin z
+ 2

= cos2
r
r
x x
r

2  2  2
 2  2
 2 
z
z
z r z
r
z z r

z
=
=
+2
+
+
y
r y y
y
r
r y y
y


 2

2
z
2 sin cos r cos2 z
= sin2
+
+
.
r
r
y y
r2

So we get


z
x

2

that is, A(r, ) = 1, B(r, ) =

z
y

2

1
.
r2

z
r

2

1
+ 2
r

2

Homework 9 (page 932). If f (x, y) = 0 define y as a function of x, show that


fxx fy2 2fxy fx fy + fyy fx2
d2 y
.
=
dx2
fy3
(92 )
Solution. We differentiate f (x, y(x)) = 0 with respect to x and get
fx + fy y = 0.
So we know y = ffxy . We differentiate (1) with respect to x and get
fxx + fxy y + (fyx + fyy y )y + fy y = 0


fx
fx
fxx
fxy + fyx fyy
+ fy y = 0
fy
fy
fxx fy2 2fx fy fxy + fyy fx2
+ fy y = 0

fy2
y =

fxx fy2 2fx fy fxy + fyy fx2


.
fy3

(1)

Calculus A (2) Homework 6 ANSWER


14.6

Directional Derivatives and the Gradient Vector, page 933

Homework 1. Let f (x, y) = x ey + cos(xy).


(a) Find the direction (a unit vector u) in which f (x, y) increases most rapidly at
(2, 0) (that is, Du f (2, 0) is maximal).
(b) Find the direction in which f (x, y) decreases most rapidly at (2, 0).
(c) What are the direction of zero change in f (x, y) at (2, 0)?

(101 )

Solution.
(a) The direction in which f (x, y) increases most rapidly is parallel to the gradient
vector:
f (2, 0) = (fx (2, 0), fy (2, 0)) = (ey sin(xy)y, x ey sin(xy)x)|(2,0) = (1, 2).


1 2

, 5 .
Since we require the vector to be unit length, we get u =
5

(b) The direction in which f (x, y) decreases most rapidly at (2, 0) is parallel to


f and opposite direction, so the direction is u = 15 , 25 .
(c) The direction of zero change in f (x, y) at (2, 0) is orthogonal to u, so we get




15 , 25 or 15 , 25 .

Homework 2. In what direction does f (x, y, z) =

z2
xy

attain the maximum rate of

change at the point (1, 2, 4)? What is this maximum rate of change? (99 )
Solution. We compute
f (1, 2, 4) =


z2
z 2 2z
2 , 2,
= (8, 4, 4).
x y xy xy (1,2,4)

The direction in which f (x, y) attains the maximum rate of change is parallel to the
gradient vector, and we require the vector to be unit length, so it will be
1
(8, 4, 4)
= (2, 1, 1).
u= p
2
2
2
6
(8) + (4) + 4
p

The maximum rate of change is |f (1, 2, 4)| = (8)2 + (4)2 + 42 = 4 6.


1

Homework 3 (page 945). Find the equations of the tangent plane and normal line
at P (0, 0, 1) to the surface x + y + z = exyz .
Solution. Let F (x, y, z) = x + y + z exyz = 0. We compute
F (0, 0, 1) = (1 yz exyz , 1 xz exyz , 1 xy exyz )|(0,0,1) = (1, 1, 1).
So the tangent plane equation is x + y + (z 1) = 0. The normal line equation is
x = y = z 1.
Homework 4. Two surfaces x3 + 3x2 y 2 + y 3 + 4xy z 2 = 0 and x2 + y 2 + z 2 11 = 0
intersect a curve. Find the parametric equation for the tangent line to the curve at
(95 )

the point (1, 1, 3).

Solution. Let F (x, y, z) = x3 + 3x2 y 2 + y 3 + 4xy z 2 = 0 and G(x, y, z) = x2 +


y 2 + z 2 11 = 0. We compute
F (0, 0, 1) = (Fx , Fy , Fz )(1,1,3) = (3x2 + 6xy 2 + 4y, 6x2y + 3y 2 + 4x, 2z)|(1,1,3)
= (13, 13, 6)
def.

G(0, 0, 1) = (Gx , Gy , Gz )(1,1,3) = (2x, 2y, 2z)|(1,1,3) = (2, 2, 6)//(1, 1, 3) = n.


The directional vector of the tangent line is parallel to
F (1, 1, 3) n = (45, 45, 0)//(1, 1, 0),
so the parametric equation for the tangent line is

x=1+t
y = 1 t , t R.

z=3

14.7

Maximum and Minimum Values, page 946

Homework 5. Find and classify the critical points of f (x, y) = xyex

2 y 2

(97 )
Solution. We first solve

f = yex2 y2 2x2 yex2 y2 = yex2 y2 (1 2x2 ) = 0


x
fy = xex2 y2 2xy 2 ex2 y2 = xex2 y2 (1 2y 2) = 0,
and fx = 0 implies y = 0 or x =

2
.
2

(a) If y = 0, we solve fy = 0 and get x = 0.


(b) If x =

2
,
2

we solve fy = 0 and get y =

2
.
2

we solve fy = 0 and get y = 22 .


  

  

So all critical points are (0, 0), 22 , 22 , 22 , 22 , 22 , 22 , and 22 , 22 .
(c) If x =

2
,
2

Next, we compute the Hessian matrix

2
x2 y 2
2
2 x2 y 2
fxx fxy
2xy(2x 3)e
(1 2x )(1 2y )e
=

Hess(f ) =
2
2
2
2
fyx fyy
(1 2x2 )(1 2y 2)ex y
2xy(2y 2 3)ex y
If p1 = (x, y) = (0, 0), D = 1 < 0 so (0, 0) is a saddle point.
 
 
If p2 = (x, y) = 22 , 22 , D = 4e2 > 0 fxx (p2 ) = 2e1 < 0, so 22 , 22 is

a local maximum.




If p3 = (x, y) = 22 , 22 , D = 4e2 and fxx (p3 ) = 2e1 > 0, so 22 , 22

is a local minimum.
 
 
If p4 = (x, y) = 22 , 22 , D = 4e2 and fxx (p4 ) = 2e1 > 0, so 22 , 22
is a local minimum.


If p5 = (x, y) = 22 , 22 , D = 4e2 and fxx (p5 ) = 2e1 < 0, so


22 , 22 is a local maximum.

Homework 6. Find the local maximum, and local minimum values and saddle
point(s) of f (x, y) = y 3 + 3x2 y 3x2 3y 2 + 3.

(102 )

Solution. We first solve

f = 6xy 6x = 6x(y 1) = 0
x
fy = 3y 2 + 3x2 6y = 0

(a) If x = 0, we solve 3y 2 6y = 3y(y 2) = 0 and get y = 0 or y = 2.

(b) If y = 1, we solve 3x2 3 = 3(x + 1)(x 1) = 0 and get x = 1 or x = 1.


So all critical points are (0, 0), (0, 2), (1, 1), (1, 1).
Next, we compute the Hessian matrix

fxx fxy
6y 6
6x
=

Hess(f ) =
fyx fyy
6x
6y 6
3

If (x, y) = (0, 0), D = 36 > 0 and fxx (0, 0) = 6 < 0, so (0, 0) is a local
maximum, and f (0, 0) = 3.
If (x, y) = (0, 2), D = 36 > 0 and fxx (0, 2) = 6 < 0, so (0, 2) is a local
maximum, and f (0, 2) = 1.
If (x, y) = (1, 1), D = 36 < 0, so (1, 1) is a saddle point.
If (x, y) = (1, 1), D = 36 < 0, so (1, 1) is a saddle point.
Homework 7 (page 955). Find the maximum volume of a rectangular box that is
inscribed in a sphere of radius r.
Solution. First we set Cartesian coordinates that the center of the sphere is at the
origin, so we can write the equation of the sphere is x2 + y 2 + z 2 = r 2 . Furthermore,
we can orient the inscribed rectangular box so that its edges are parallel to the
coordinate axes. So we can assume that the box has length 2x, width 2y, height
p
2z = r 2 x2 y 2 , and volume

 p
p
V (x, y) = (2x)(2y) 2 r 2 x2 y 2 = 8xy r 2 x2 y 2 , 0 x2 + y 2 r 2 .
We compute

p
2x
8y(r 2 2x2 y 2)
+ r 2 x2 y 2 8y = p
Vx = 8xy p
2 r 2 x2 y 2
r 2 x2 y 2
p
8x(r 2 2x2 y 2 )
2y
Vy = 8xy p
+ r 2 x2 y 2 8x = p
.
2 r 2 x2 y 2
r 2 x2 y 2

Solving Vx = 0 gives y = 0 or 2x2 + y 2 = r 2 . Similarly, we solve Vy = 0 and get


x = 0 or x2 + 2y 2 = r 2 . So the only critical points inside the region 0 x2 + y 2 r 2
is to solve 2x2 + y 2 = x2 + 2y 2 x2 = y 2 y = x, and we get x =

r
3

= y.

(a) If x = 0 or y = 0, the volume is 0.


(b) If x2 + y 2 = r 2 , then z = 0, so the volume is 0.
(c) If x = y =
V

r
3

and the volume is

r
r
,
3 3

=8

so the maximum volume is



s
2 
2

8
r
r

= r3,
r2
3
3
3 3

r3.
3 3

14.8

Lagrange Multipliers, page 957

Homework 8. Find the absolute minimum and maximum values of f (x, y) = xy 2


on the curve x2 + 7xy + y 2 = 45, x 0, y 0.

(94 )

Solution. Consider Lagrange function F (x, y, ) = xy 2 (x2 + 7xy + y 2 45).


The critical points of F (x, y, ) satisfy
Fx = y 2 (2x + 7y) = 0
Fy = 2xy (7x + 2y) = 0
F = x2 + 7xy + y 2 45 = 0.
Since x2 + 7xy + y 2 = 45, x 0, y 0, we know 2x + 7y > 0 and 7x + 2y > 0, so
=

2xy
y2
=
y 2(7x + 2y) = 2xy(2x + 7y) y(x + 2y)(4x y) = 0.
2x + 7y
7x + 2y

We get y = 0, x = 2y (not in the case), or y = 4x.

(a) If y = 0, we get x2 = 45 y = 3 5, and f (3 5, 0) = 0.


(b) If y = 4x, we solve x2 + 28x2 + 16x2 = 45 x = 1, y = 4, and f (1, 4) = 16.

Hence the absolute maximum is f (1, 4) = 16; absolute minimum is f (3 5, 0) = 0.


Homework 9. Let be the curve which is the intersection of 2y + 4z = 5 and
z 2 = 4x2 + 4y 2 . Find the point on that is closest from the origin (0, 0, 0).
(91 )
Solution. Instead of finding the maximum or minimum values of the function
p
d(x, y, z) = x2 + y 2 + z 2 on the curve , we consider its square function d2 (x, y, z) =

x2 + y 2 + z 2 because they both attain maximum or minimum at the same places.

Consider Lagrange function F (x, y, z, , ) = x2 + y 2 + z 2 (2y + 4z 5)


(z 2 4x2 4y 2 ). The critical points of F (x, y, z, , ) satisfy
Fx = 2x + 8x = 2x(1 + 4) = 0

(1)

Fy = 2y 2 + 8y = 2(y(1 + 4) ) = 0

(2)

Fz = 2z 4 2z = 2(z(1 ) 2) = 0

(3)

F = (2y + 4z 5) = 0

(4)

F = (z 2 4x2 4y 2) = 0

(5)

From (1), we know x = 0 or = 14 .


5

(a) If x = 0, from (5) we get z = 2y. Putting into (4) and get 2y 8y 5 = 0, so
y=

1
2

or y = 65 . Hence we get (x, y, z) = (0, 12 , 1) or (0, 65 , 53 ). We compute




1
5
d 0, , 1 =
,
2
2



5 5
5 5
=
d 0, ,
.
6 3
6

(b) If = 41 , from (2) we know = 0. From (3) we get z = 0. From (5) it


implies x = 0 and y = 0, but it is impossible because of (4).
So the point on that is closest from the origin is (0, 21 , 1).
Homework 10. The plane x + y +

2z = 0 and the ellipsoid

x2
4

y2
4

z2
2

= 1

intersect an ellipse.
(a) Find the farthest distance and the closest distance from the point on the ellipse
to the origin.
(96 )

(b) Find the area of the ellipse.


Solution.

(a) Instead of finding the maximum or minimum values of the function d(x, y, z) =
p
x2 + y 2 + z 2 on the ellipse, we consider its square function d2 (x, y, z) =
x2 + y 2 + z 2 because they both attain maximum or minimum at the same

places.
Consider Lagrange function
2

F (x, y, z, , ) = x + y + z (x + y +

2z)


x2 y 2 z 2
+
+
1 .
4
4
2

The critical points of F (x, y, z, , ) satisfy


1
Fx = 2x x = 0
2
1
Fy = 2y y = 0
2

Fz = 2z 2 z = 0

F = (x + y + 2z) = 0

 2
y2 z2
x
+
+
1 = 0.
F =
4
4
2

(6)
(7)
(8)
(9)
(10)

From (6), (7) and (8), we get


1
1
z
= x(4 ) = y(4 ) = (2 ),
2
2
2
and 21 x(4 ) = 12 y(4 ) implies 21 (x y)(4 ) = 0, so we get x = y or
= 4.

If x = y, from (9) we get z = 2x. From (10) we get x2 =

36 . So

d = 236.

we get (

2 3
6
6
,
,

)
3
3
3

or (

6
6 2 3
,

,
),
3
3
3

2
3

x=

and the distance is

If = 4, it implies = 0 and we get z = 0 and x+y = 0 x = y. From

2
(10), we get x2 = 1 x = 2 y = 2. So we get ( 2, 2, 0) or

( 2, 2, 0), and the distance is d = 2.
The farthest distance is 2 and the closest distance is

2 6
.
3

(b) Since the semi-major axis is 2 and the semi-minor axis


ellipse is

2 6
,
3

the area of the

4 6
.
3

Homework 11. The plane 2x + y + z = 10 intersects the paraboloid z = x2 + y 2


in an ellipse. Find the highest point on the ellipse.

(92 )

Solution. Consider the Lagrange function F (x, y, z, , ) = z (2x + y + z 10)


(z x2 y 2 ). The critical points of F (x, y, z, , ) satisfy
Fx = 2 + 2x = 2( x) = 0

(11)

Fy = + 2y = ( 2y) = 0

(12)

Fz = 1 = 0

(13)

F = (2x + y + z 10) = 0

(14)

F = (z x2 y 2) = 0.

(15)

From (11) and (12), we get x = 2y (x 2y) = 0 = 0 or x = 2y.


(a) If = 0, from (11) we get = 0 and it is impossible because of (13).
(b) If x = 2y, from (14) we get z = 105y. From (15) we solve 105y 4y 2 y 2 =
0, that is, y 2 + y 2 = (y 1)(y + 2) = 0 y = 1 or y = 2. So we get
(x, y, z) = (2, 1, 5) or (4, 2, 20).
Hence the highest point on the ellipse is (4, 2, 20).
7

Calculus A (2) Homework 7 ANSWER


Definition (page 1046). Let T be a transformation that maps a region S in uv-space
onto a region R in xy-space by means of the equations
x = g(u, v),

y = h(u, v).

The Jacobian of T is the following 2 2 determinant:

x
x
(x, y)
v
= det u
.
y
y
(u, v)
u

Homework 1 (Polar Coordinates, page 1044). The transformation T from the


r-plane to the xy-plane is given by
x = g(r, ) = r cos ,

y = h(r, ) = r sin .





Compute (x,y)
(in terms of r, ).
(r,)

Solution.




(x, y) cos r sin


(r, ) =
sin r cos




= r.

Homework 2 (page 1044). The transformation T from the uv-plane to the xy-plane
is given by
x = g(u, v) = u2 v 2 ,

y = h(u, v) = 2uv.





Compute (x,y)
(in terms of u, v).
(u,v)
Solution.






(x, y) 2u 2v
= 4(u2 + v 2 ).



(u, v) =
2v 2u

Homework 3 (page 1044). The transformation T from the xy-plane to the uv-plane
is given by
u = g(x, y) = x + y,

v = h(x, y) = x y.




(x,y)
(u,v)
Compute (u,v) (in terms of u, v) and (x,y) (in terms of x, y).
1

and y = uv
.
Solution. We solve x = u+v
2
2





(x, y) 21 21 1
= ,



(u, v) = 1
2 12 2

So



(u, v) 1 1


(x, y) =
1 1




= 2.

Definition (page 1046). Let T be a transformation that maps a region S in uvw-

space onto a region R in xyz-space by means of the equations


x = g(u, v, w),

y = h(u, v, w),

z = k(u, v, w).

The Jacobian of T is the following 3 3 determinant:

(x, y, z)

= det

(u, v, w)

x
u
y
u
z
u

x
v
y
v
z
v

x
w
y
w
z
w

Homework 4 (Cylindrical Coordinates, page 1028). The transformation T from


the rz-plane to the xyz-plane is given by
x = r cos , y = r sin ,




Compute (x,y,z)
(in terms of r, , z).
(r,,z)

z=z

Solution.



cos r sin 0




(x, y, z)

= sin r cos 0 = r.

(u, v, w)


0
0
1

Homework 5 (Spherical Coordinates, page 1033). The transformation T from the


r-plane to the xyz-plane is given by
x = sin cos , y = sin sin ,




Compute (x,y,z)
(in terms of , , ).
(,,)

z = cos .

Solution.




sin cos sin sin cos cos
(x, y, z)


(u, v, w) = sin sin sin cos cos sin

cos
0
sin

= |2 cos sin cos ( sin2 cos2 ) 2 sin sin sin (cos2 + sin2 )|
= | 2 sin (cos2 + sin2 )| = 2 sin .

Calculus A (2) Homework 8 ANSWER


15.2

Iterated Integrals, page 982

Homework 1 (page 987). Calculate the integral.



Z 1 Z 1
ZZ

1
dA, R = [1, 3] [1, 2].
(1)
x + y dx dy.
(2)
R 1+x+y
0
0

Solution.

(1) We compute
Z 1 Z



Z 1 Z 1
1
(x + y) 2 d(x + y) dy
x + y dx dy =
0
0
0
0
 x=1
Z
Z 1

3
3
3
2 1
2

dy =
(x + y) 2
(y + 1) 2 y 2 dy
=
3
3 0
0
x=0

 y=1

5
4  5
2 5
8 3
2 2
2 2 1 1 + 0 = (2 2 1).
=
(y + 1) 2 y 2
=
3 5
5
15
15
1

y=0

(2) We compute
ZZ
Z 3Z 2
Z 3h
i y=2
1
1

dx
dA =
dy dx =
ln(1 + x + y)
1
+
x
+
y
1
+
x
+
y
y=1
R
1 1
1
Z 3
=
ln(x + 3) ln(x + 2) dx.
1

Since
Z

ln x dx = (ln x)x

we have
ZZ

1
dA =
1+x+y

x d ln x = (ln x)x

1 dx = (ln x)x x + C,

ln(x + 3) d(x + 3)
ln(x + 2) d(x + 2)
1
1
R
h
i x=3 h
i x=3


= ln(x + 3)x (x + 3)
ln(x + 2)x (x + 2)
x=1

x=1

= 6 ln 6 4 ln 4 6 + 4 (5 ln 5 3 ln 3 5 + 3)
= 6 ln 6 5 ln 5 4 ln 4 + 3 ln 3.

Homework 2 (page 988). Find the volume of the solid in the first octant bounded
by the cylinder z = 16 x2 and the plane y = 5.
Solution. The volume is
 x=4

Z 4Z 5
Z 4
640
1 3
2
2
=
.
V =
(16 x ) dy dx = 5
(16 x ) dx = 5 16x x
3
3
0
0
0
x=0
1

15.3

Double Integrals over General Regions, page


988
Z 8Z

Homework 3. Find the integral

(95 )

1 + x4 dx dy.

3y

Solution. We change the iterated integral and get


Z 2 Z y=x3
Z
Z 8Z 2
4
4
1 + x dx dy =
1 + x dy dx =

x3 1 + x4 dx

i
3 x=2
1
3
1
1h
1
= (17 2 1).
=
(1 + x4 ) 2
(1 + x4 ) 2 d(x4 + 1) =
x=0
4 0
6
6
ZZ
Homework 4. Find the integral
|y x2 | dA, R = [1, 1] [0, 2].
Z

Solution. The integral is


ZZ
Z
2
|y x | dA =
R

Since
Z 1Z
1

y=2

(96 )

(y x ) dy dx +

y=x2

and
Z 1Z
1

y=x2

(y x ) dy dx =

y=0

1
1

we have
ZZ

1 2
y x2 y
(y x ) dy dx =
y=x2
1 2
Z 1
=
2 2x2 +
Z

y=2

y=x2

(y x2 ) dy dx.
y=0

 y=2
Z 1

1

dx
=
(4 x4 ) x2 (2 x2 ) dx
2
1 2
y=x


 1
43
1 4
2 3
1 5
x dx = 2x x + x =
2
3
10
15
1

 y=x2
Z 1

1
1 2
2

dx =
x4 + x2 (x2 ) dx
y +x y
2
2
1
y=0
 1

1
1 4
1 5
x dx =
x = ,
2
10
5
1


|y x2 | dA =

Homework 5. Evaluate the integral

43 1
46
+ = .
15 5
15

sin(y 2 ) dy dx.

Solution. We change the iterated integral and get


Z
Z Z
Z Z x=2y
2
2
2
2
sin(y ) dx dy =
sin(y ) dy dx =
0

x
2

2y sin(y 2 ) dy

x=0

(97 )

x
2

sin(y 2 ) dy 2 =

i 2
2

+ 1.
cos(y 2)
=
2
y=0

Homework 6 (page 996). Find the volume of the solid bounded by the cylinder
y 2 + z 2 = 4 and the planes x = 2y, x = 0, z = 0 in the first octant.
Solution. The volume is
Z 2
Z 2 Z x=2y p
Z 2 p
1
4 y 2 dx dy =
(4 y 2) 2 d(4 y 2)
V =
2y 4 y 2 dy =
0
0
x=0
0

 2

3
2
2 3
16
= (4 y 2) 2 = 4 2 = .
3
3
3
0

Calculus A (2) Homework 9 ANSWER


15.4

Double Integrals in Polar Coordinates, page


997

Homework 1. Find the integral


r = 4 + 3 cos .

RR

|y| dA, where R is the region enclosed by


(94 )

Solution. We use the polar coordinates x = r cos , y = r sin , then y 0 if


[0, ], and y 0 if [, 2]. Hence
Z Z r=4+3 cos
ZZ
Z
|y| dA =
r sin r dr d
0

We compute
Z Z

2Z r=4+3 cos

r sin r dr d.

 r=4+3 cos
1 3
d
r sin dr d =
sin
r
3 r=0
0 0
0
Z
Z
1
1
3
=
sin (4 + 3 cos ) d =
(4 + 3 cos )3 d(4 + 3 cos )
3 0
9 0
i =
1
1
1 h

(4 + 3 cos )4
= (1 74 ) = (74 1).
=
36
36
36
=0
r=4+3 cos

Similarly, we have
Z 2Z r=4+3 cos
i =2
1
1 h
4
2
= (74 1).
(4 + 3 cos )

r sin dr d =
36
36
=

0
RR
1
Hence R |y| dA = 18
(74 1).
ZZ
1
dA, where R is the region
Homework 2. Find the integral
2
2 2
R (1 + x + y )
bounded by the Lemniscate r 2 = cos 2.
(95 )
Solution. Using the polar coordinates x = r cos , y = r sin and we get
1

cos 2

1
r dr d
+
(1 + r 2 )2
R (1 +
4 0
 r=cos 2
Z Z cos 2
Z 

4
4
1
1

=
d(1 + r 2 ) d =
d

2
2
2

(1
+
r
)
1
+
r
4 0
4
r=0

Z
Z
Z 
4
4
4 2 cos2 1
cos 2
1
1 d =
d =
d
=
1 + cos 2
2 cos2
4
4 1 + cos 2
4


 4
Z 

4
1
1

1 sec2 d = tan = 1.
=
2
2
2
4

ZZ

x2

y 2 )2

dA = 2

Homework 3. Find the volume bounded by the sphere x2 + y 2 + z 2 = 4 and the


cylinder (x 1)2 + y 2 = 1.

(95 )

Solution. When the solid project to xy-plane, the region is (x 1)2 + y 2 = 1. We


use the polar coordinates x = r cos , y = r sin , then the region is inside the circle
r = 2 cos . So by symmetry, we get
Z Z 2 cos
Z Z 2 cos
2
2
1
2
(4 r 2 ) 2 d(4 r 2 ) d
4 r r dr d = 2
V =4
0
0
0
0
Z h
Z 
i

r=2
cos

2
3
3
4
4 2
2 23
2
2
2
(4 r )
(4 4 cos ) 4 d
=
d =
3 0
3 0
r=0
Z

32 2
sin3 1 d.
=
3 0
Since

sin d =

0
2
3

we get V = 32
3

Z


(cos 1) d cos =

16

64
.
9

Homework 4. Evaluate the integral

Z 2Z

4x2

 = 2

2
1
3
= ,
cos cos
3
3
=0
e(x

2 +y 2 )

dy dx.

(99 )

2 0

Solution. The region is an half disk, so we use the polar coordinates x = r cos , r =
sin and get
Z 2Z
2

15.6

1
2

4x2

Z 0
0

h 2 i r=2

er
r=0

Z Z
1 2 r2 2
e dr d
e r dr d =
2 0 0
0


e4 1 d = (1 e4 ).
2

Z Z

(x2 +y 2 )

dy dx =
Z 0
1
d =
2 0

r 2

Surface Area, page 1013

Homework 5 (page 1016). Find the area of the surface created by the cylinder
y 2 + z 2 = r 2 that lies within another cylinder x2 + z 2 = r 2 .

(91 )

Solution. By symmetry, we first compute the area of the surface which is a graph
p
p
, fz = 0, and 1 + fy2 + fz2 = 2r 2 , we
of f (x, y) = r 2 y 2 . Since fy = y
2
2
r y

r y

have

# x=y
Z r

2ry

p
p
p
A=
dy =
dx dy =
dy

r2 y 2
r 2 y 2 x=y
r2 y 2
0
y
0
0
Z r
h
i r
2
2
2
2 12
2
2 21
= r
(r y ) d(r y ) = 2r(r y ) = 2r 2 .
Z rZ

"

rx

Hence the total surface area is 8 2r = 16r .


2

Homework 6 (page 1016). Find the area of the surface z = 32 (x 2 +y 2 ), 0 x, y 1.


(98 )
p

1
1
Solution. First, we compute zx = x 2 , zy = y 2 , and 1 + zx2 + zy2 = 1 + x + y.

So the surface area is


Z 1Z 1 p
Z 1Z 1
1
S=
1 + x + y dx dy =
(1 + x + y) 2 d(1 + x + y) dy
0 0
0 0
 x=1
Z
Z 1
3
3
3
2 1
2

dy =
(1 + x + y) 2
(2 + y) 2 (1 + y) 2 dy
=
3
3 0
0
x=0
i
h
y=1
5
5
5
7
5
4 5
4 5
3
= (3 2 2 2 2 2 + 1) = (3 2 2 2 + 1).
=
(2 + y) 2 (1 + y) 2
5
15
15
y=0
2
2
2
Homework 7. Find the area of the surface {x +y +z = 4, 1 x2 +y 2 3, z 0}.
(102 )

Solution. The surface can be written as the graph of z =

p
4 x2 y 2 in the

region 1 x2 + y 2 3. We compute
q
y
x
2
, zy = p
, and
.
1 + zx2 + zy2 = p
zx = p
4 x2 y 2
4 x2 y 2
4 x2 y 2

We use the polar coordinate x = r cos , y = r sin to compute the surface area
Z 2Z 3
Z 2Z 3
1
2

A=
(4 r 2 ) 2 d(4 r 2 ) d
r dr d =
2
4r
0
1
1
0

Z 2 h
i
r=
3

1
d = 2( 3 1) 2 = 4( 3 1).
=
2(4 r 2 ) 2
r=1

15.7

Triple Integrals, page 1017

Homework 8 (page 1026). Write five other iterated integrals that are equal to the
Z 1Z 1Z y
iterated integral
f (x, y, z) dz dx dy.
0

Solution. The solid projecting to xy-plane is a region bounded by y = 0, y = x,


and x = 1, so we get
Z 1Z xZ
0

f (x, y, z) dz dy dx.

If we iterated the integral along x first, we get


Z 1Z yZ 1
Z 1Z 1Z
f (x, y, z) dx dz dy and
0

f (x, y, z) dx dy dz.

If we iterated the integral along y first, we get


Z 1Z xZ x
Z 1Z 1Z
f (x, y, z) dy dz dx and
0

f (x, y, z) dy dx dz.

Homework 9. Evaluate the integral

Z 2Z

4x2Z x
0

sin 2z
dy dz dx.
4z

Solution. We compute
Z 2Z 4x2Z x
Z 2Z 4x2
Z 4Z 4z
x sin 2z
sin 2z
dy dz dx =
dz dx =
4z
0 0
0 0
0 0
0 4z
 x=4z
Z 4
Z 4
h
i z=4
1
sin 2z 2
1

cos 2z
sin 2z dz =
x
=
=
dz =
2(4 z)
2
4
z=0
0

x=0

(96 )

x sin 2z
dx dz
4z
1
(1 cos 8).
4

Homework 10 (page 1025). Use a triple integral to find the volume of the solid
enclosed by the cylinder y = x2 and the planes z = 0 and y + z = 1.
Solution. The volume of the solid is
Z 1 Z 1 Z 1y
Z 1Z 1
Z
V =
dz dy dx =
(1 y) dy dx =
1

x2 0
1 

x2

 y=1
Z

1 1
1
2
dx =
(1 y)
(1 x2 )2 dx =
=
2
2
2
1
1
y=x

 x=1

2 1
2
1
8
=1 + = .
= x x3 + x5
3
5
3 5
15
x=0
Z

1
Z 1

(1 y) d(1 y) dx
x2

(1 2x2 + x4 ) dx

Homework 11. The region E is bounded by z = (x2 + y 2) 2 , x2 + y 2 = 9, and z = 0.


Suppose the density function (x, y, z) = z. Find the mass of E and the center of
(92 )

mass (
x, y, z).

Solution. The solid projecting to the xy-plane is a disk with center (0, 0) and radius
3, so we use the polar coordinate x = r cos , y = r sin . We first compute the mass:
ZZZ
Z 2Z 3Z x2 +y2
z r dz dr d
m=
(x, y, z) dV =
E
0
0 0
 z=x2 +y2
Z 2Z 3
Z 2Z 3 
1
1 2
dr d =
=
rz
r(x2 + y 2) dr d
2
2
0
0
0
0
z=0
 r=3
Z 2Z 3
Z 2 
1 3
81
1 4
=
r dr d =
r d =
.
2
8
4
0

r=0

The symmetry of E and about xz-plane and yz-plane implies the moments Mxz = 0
and Myz = 0. So we still need to compute
ZZZ
Z 2Z 3Z x2 +y2
z 2 r dz dr d
Mxy =
z(x, y, z) dV =
E
0
0 0
 z=x2 +y2
Z 2Z 3
Z 2Z 3 
3
1
1 2
dr d =
rz
r(x2 + y 2) 2 dr d
=
3
0
0 3
0
0
z=0
 r=3
Z 2 
Z 2Z 3
162
1 5
1 4
r dr d =
r d =
.
=
3
15
5
0

r=0

Hence the center of mass is (


x, y, z) =

Myz Mxz Mxy


, m , m
m

= (0, 0, 85 ).

Calculus A (2) Homework 10 ANSWER


15.8

Triple Integrals in Cylindrical Coordinates,


page 1027

Homework 1 (page 1031). Evaluate the integral by changing to cylindrical coordinates.


Z

Z 4y2 Z

4y 2

xz dz dx dy.
x2 +y 2

Solution. We use the cylindrical coordinates x = r cos , y = r sin , z = z and get


Z 2Z 2Z 2
Z 2 Z 4y2 Z 2
xz dz dx dy =
r cos z r dz dr d

=
=

2
0

2Z

4y 2

x2 +y 2

 z=2
Z 2Z 2
2
1 2
1
2
r cos
dr d =
z
cos (4r 2 r 4 ) dr d
2
2
0
0
0
z=r
 r=2

Z 2
i =2

32
4 3 1 5
32 h
1

d =
= 0.
sin
cos
r r
cos d =
=0
2
3
5
15 0
15
r=0


Homework 2 (page 1032). Find the volume of the solid enclosed by the three
cylinders x2 + y 2 = 1, x2 + z 2 = 1, and y 2 + z 2 = 1.
Solution. We use the cylindrical coordinates x = r cos , y = r sin , z = z and get
Z Z 1
Z Z 1Z 1x2
4
4
r 1 r 2 cos2 dr d
r dz dr d = 16
V = 16
0 0
0
0
0
Z
Z 1
4
1
1
= 8
(1 r 2 cos2 ) 2 d(1 r 2 cos2 ) d
2
0 cos 0
Z
Z
i
3 r=1
16 4 1 h
16 4 1
2
2
=
(1 r cos ) 2
(sin3 1) d
d =
3 0 cos2
3 0 cos2
r=0
Z

16 4
=
(sec2 1) sin sec2 d
3 0
Z

16 4
sec2 (sin 1) sin d
=
3 0
!
Z
i 4 Z 4
4
16 h

=
(sin 1) tan
tan cos d
sin d
3
0
0
0
!
!

Z
h
i 4
4
16
2
2
16

sin d =
12
1 + 2 cos
=
0
3
2
3
2
0
!

2
= 16
1 = 16 8 2.
2
1

15.9

Triple Integrals in Spherical Coordinates,


page 1033

Homework 3. Evaluate the integral


Z Z
Z
4x2

4x2

16x2 y 2

3(x2 +y 2 )

x2 + y 2 + z 2 dz dy dx.
(94 )

Solution. We use the spherical coordinates x = sin cos , y = sin sin , z =


p
cos , then the solid is bounded below by the cone z = 3(x2 + y 2) and above by
p
the sphere z = 42 x2 y 2 . So we have
Z 2 Z 4x2 Z 16x2 y2 p
x2 + y 2 + z 2 dz dy dx
2 2

2
2 4x
3(x +y )
 =4
Z Z 2Z 4
Z Z 2 
6
6
1 4
3
=
sin d d
sin d d d =
4
0
0
0
0
0
=0
!
Z
h
i = 6
6
1 4
3

.
= 4 2
sin d = 128 cos
= 128 1
4
2
=0
0
Homework 4. Evaluate the integral

ZZZ

z3

x2 + y 2 + z 2 dV , where E is the

solid that lies above the xy-plane and below the sphere x2 + y 2 + z 2 = 1.
(92 )
Solution. We use the spherical coordinates x = sin cos , y = sin sin , z =
cos and get
ZZZ

Z Z
2

2Z 1

6 cos3 sin d d d
+ + dV =
0
0
0
E
 =1
Z
Z Z 2 

2
2 2
1 7
3
cos sin d d d =
=
cos4 d cos
7
7
0
0
0
=0

h 4 i 2
cos = .
=
0
14
14
z

x2

y2

z2

Homework 5. Find the volume of the solid bounded below by the cone z 2 =
4(x2 + y 2) and above by the ellipsoid 4(x2 + y 2 ) + z 2 = 8.

(101 )

Solution. We assume that x = 2 sin cos , y = 2 sin sin , z = 2 2 cos .


Then the volume element is 2 2 2 2 2 sin d d d = 4 22 sin d d d.
2

So the volume is
Z Z 2Z
4
V =
0

15.10

Z 4Z
4 2 sin d d d = 4 2

4 2
2
3

i = 4
8 2 h

cos
sin d =
3
=0

 =1
1 3
sin d d
3
=0

8
=
( 2 1).
3


Change Variables in Multiple Integrals,


page 1040

Homework 6. Evaluate the integral

RR

sin(x + y) cos(2x y) dA, where R is the

region bounded by y = 2x 1, y = 2x + 3, y = x, and y = x + 1. (90 )


Solution. Let u = 2x y and v = x + y, then

u u
2 1
x y =
.
vx vy
1 1





(u,v)
(x,y) 1
Since (x,y) = 3, we have (u,v) = 3 . After the transformation, the region becomes

S = {(u, v)| 3 u 1, 0 v 1}. So


Z Z
ZZ
1 1 1
sin(x + y) cos(2x y) dA =
sin v cos u du dv
3 0 3
R
Z
Z 1
h
i u=1
1 1
1

=
sin v sin u
sin v dv
dv = (sin 1 + sin 3)
3 0
3
u=3
0
h
i v=1 1
1

= (sin 1 + sin 3) cos v
= (sin 1 + sin 3)(1 cos 1).
3
3
v=0
RRR
Homework 7. Evaluate the integral
(x2 y + 3xyz) dV , where E is the region
E
(91 )

1 x 2, 0 xy 2, and 0 z 1.

Solution. Let u = x, v = xy, w = z, then


1 0 0
ux uy uz

vx vy vz = y x 0

0 0 1
wx wy wz





(x,y,z)

Since (u,v,w)
=
x,
we
have
=


(x,y,z)
(u,v,w)

1
x

= u1 . After the transformation, the region

becomes S = {(u, v, w)|1 u 2, 0 v 2, 0 w 1}. So we get


Z 1Z 2Z

1
(x y + 3xyz) dV =
(uv + 3vw) du dv dw
u
E
0 0 1

Z 1Z 2
Z 1Z 2Z 2 
3vw
du dv dw =
=
(v + 3 ln 2 vw) dv dw
v+
u
0 0
0 0 1
 v=2
Z 1
Z 1
h
i w=1
1 2 3 ln 2 2

dw =
(2 + 6 ln 2w)dw = 2w + 3 ln 2w 2
v +
v w
=
w=0
2
2
0
0
v=0
ZZZ

= 2 + 3 ln 2.

Homework 8. Evaluate the integral


y 2 a2 }.

RR

ex

2 xy+y 2

dA, where R = {(x, y)|x2 xy +


(92 )

Solution. From the relation x2 xy + y 2 = x


and v =

3
y.
2

Then



(u,v)
Since (x,y)
=

3
,
2

ux uy
vx vy





=
we have (x,y)
(u,v)

2 .
3

becomes u2 + v 2 a2 . So we get
ZZ
ZZ
Z
x2 xy+y 2
u2 +v2
e
dA =
e
dA =

1 21
0

3
2


y 2
2

 2
3y
2

, we let u = x

y
2

After the transformation, the region

Z Z
1 2 a r2 2
e dr d
e r dr d =
2 0 0
R
u2 +v2 a2
0
0
Z
 2

1 2 h r2 i a
=
e d = ea 1 .
2 0
0
RRR
Homework 9. Evaluate the integral
(x+y+z)2 dV , where E = {(x, y, z)|2x2 +
E
2Z a

r2

3y 2 + 5z 2 + 6yz + 2xz 1}.

(96 )

Solution. From the relation


2x2 + 3y 2 + 5z 2 + 6yz + 2xz = (x + y + z)2 + x2 + 2y 2 + 4z 2 2xy + 4yz
= (x + y + z)2 + (x y)2 + (y + 2z)2 .
We let u = x + y + z, v = x y, w = y + 2z, then

1 1 1
ux uy uz

vx vy vz = 1 1 0 .

0 1 2
wx wy wz
4





(x,y,z) 1


=
3,
we
get
Since (u,v,w)
(u,v,w) = 3 . So
(x,y,z)
ZZZ

(x + y + z) dV =

ZZZ

u2 dV
u2 +v2 +w 2 1

We use the spherical coordinates w = sin cos , v = sin sin , u = cos


ZZZ
Z Z 2Z 1
2
(x + y + z) dV =
2 cos2 2 sin d d d
E
0 0
0
 =1
Z
Z Z 2 

1
1 5
2
cos sin d d = 2
cos2 d cos
=
5
5
0
0 0
=0

 =

4
2 1
=
cos3
.
=
5 3
15
=0

Calculus A (2) Homework 11 ANSWER


16.1

Vector Fields, page 1056

Homework 1 (page 1062). Match the functions f1 , f2 , f3 , and f4 with the plots of
their gradient vector fields labeled I - IV. Give reasons for your choices.
(a) f1 (x, y) = x2 + y 2 .
(b) f2 (x, y) = x(x + y).
(c) f3 (x, y) = (x + y)2 .
(d) f4 (x, y) = sin

x2 + y 2 .

1
1

0.5

0.5

-0.5

-0.5

-1

-1
-1

-0.5

0.5

II

-1.5

0.5

0.5

-0.5

-0.5

-1

-1

III

-1

-0.5

0.5

IV

-1

-1

-0.5

-0.5

0.5

0.5

1.5

Figure 1: Gradient vector fields.


Solution.
(a) Since f1 (x, y) = 2x i + 2y j, which is two times of the position vector, the
gradient vector field of f1 is IV.
1

(b) Since f2 (x, y) = (2x + y) i + x j, the vectors point upward in quadrants I and
IV and downward in quadrants II and III. The gradient vector fields of f2 is
II.
(c) Since f3 (x, y) = 2(x + y) i + 2(x + y) j, two components are the same, so each
vector is parallel to i + j, and hence the gradient vector fields of f3 is I.

y cos x2 +y 2
x cos x2 +y 2
(d) Since f4 (x, y) = 2 2 i + 2 2 j, which is parallel to the position
x +y

x +y

vector. When x and y large, two components tend to zero, so the gradient
vector fields of f3 is III.

16.2

Line Integrals, page 1063

Homework 2. Evaluate

xy dx + yz dy + zx dz, where C consists of the line

segment C1 from (0, 0, 0) to (1, 1, 0) followed by another line segment C2 from (1, 1, 0)
(90 )

to (1, 1, 1).
Solution. Let

C1 : r1 (t) = t i + t j + 0 k, 0 t 1 r1 (t) = 1 i + 1 j + 0 k
C2 : r2 (t) = 1 i + 1 j + t k, 0 t 1 r2 (t) = 0 i + 0 j + 1 k.
Then
Z

C
Z 1
0

xy dx + yz dy + zx dz
xy dx + yz dy + zx dz =
C1 C2
Z 1
5
1 1
2
t dt +
t dt = + = .
3 2
6
0

Homework 3. Find the work done by the force field F(x, y) = x2 y i xy 2 j on a


particle that starts at (2, 0) and moves once around the circle x2 + y 2 = 4 oriented
(92 )

in the counterclockwise direction.

Solution. Let C : r(t) = 2 cos t i + 2 sin t j, 0 t 2 r (t) = 2 sin t i + 2 cos t j.


The force field on the curve is F(x, y) = 8 cos2 t sin t i 8 cos t sin2 t j. So the work
Z
Z 2
Z 2
2
2
2
2
W =
F dr =
(16 cos t sin t 16 cos t sin t) dt = 32
cos2 t sin2 t dt
C
0
0


Z 2 
Z 2
1 cos t
1 + cos 2t
= 32
dt = 8
(1 cos2 2t) dt
2
2
0
0


 2

Z 2 
t sin 4t
1 + cos 4t
= 8.
dt = 8
= 8

1
2
2
8 0
0
2

Homework 4. Evaluate
(0, 1).

x ds, where C is part of unit circle from ( 12 , 12 ) to


(92 )

Solution. Let C : r(t) = cos t i + sin j, 4 t 2 , then ds =

cos2 t + sin2 t dt =

dt. So
Z

x ds =

cos t dt =

i 2
2

sin t = 1 +
.
2
4

Homework 5. Find the work done by the force field F(x, y, z) = z i + y j x k in


moving a particle along the circular helix given by the equations x(t) = cos t, y(t) =
(92 )

sin t, z(t) = t, 0 t 2.
Solution. The circular helix is

r(t) = cos t i + sin t j + t k, 0 t 2 r (t) = sin t i + cos t j + 1 k.


The force field on the helix is F(x, y, z) = t i + sin t j cos t k. So the work is
Z
Z 2
W =
F dr =
(t sin t + sin t cos t cos t) dt.
0

We compute
Z 2

h
i t=2 Z 2

t sin t dt =
cos t dt = 2,
t d cos t = t cos t
t=0
0
0
0

 t=2
Z 2
Z 2
1 2
= 0,
sin t cos t dt =
sin t d sin t =
sin t
2
0
0
t=0
Z 2
i t=2
h

= 0.
cos t dt = sint
2

t=0

So W = 2.

Homework 6. Find the line integral

6y 2 dx + 4x3 dy, where C is the arc of the

parabola y = 1 x2 from (1, 0) to (0, 1) and then to (1, 0).

(99 )

Solution. Let C : r(t) = t i + (1 t2 ) j, 1 t 1 r (t) = 1 i 2t j. So the line


integral is
Z
Z
2
3
6y dx + 4x dy =
C

1
2 2

(6(1 t ) 8t ) dt =

(6 12t2 2t4 ) dt



 t=1
4
16
2 5
3
= 12 8
= .
= 6t 4t t
5
5
5
t=1


Calculus A (2) Homework 12 ANSWER


16.3

The Fundamental Theorem for Line Integrals, page 1075

Homework 1. Let
F(x, y, z) =

y
i+
1 + x2 y 2

z
x
+p
2
2
1+x y
1 y 2z 2

j+

(a) Find a potential function of F.


(b) Find

y
p
+z
1 y 2z 2

k.

F dr, where C is the upper cardioid r = 1 + cos , 0 ,


(93 )

counterclockwise.
Solution.

(a) The potential function f (x, y, z) of F satisfies the following equations:


f
y
=
,
x
1 + x2 y 2

f
x
z
=
+p
,
2
2
y
1+x y
1 y2z2

y
f
=p
+ z.
z
1 y2z2

From the first equation, we have


Z
Z
y
y
f (x, y, z) =
dx =
d(xy) = tan1 (xy) + g(y, z).
2
2
1+x y
1 + (xy)2
Next, we compute


f
x

g
x
z
tan1 (xy) + g(y, z) =
=
+
=
+p
,
2
2
2
2
y
y
1+x y
y
1+x y
1 y 2z 2

so it implies

g(y, z) =

g
y

z
1y 2 z 2

and

p
dy =
1 y 2z 2

Finally, we calculate

1
p

1 (yz)2

d(yz) = sin1 (yz) + h(z).

 g
f

y
y
tan1 (xy) + g(y, z) =
=
=p
+ h (z) = p
+z
2
2
z
z
z
1y z
1 y 2z 2

and it gives h (z) = z h(z) = 21 z 2 + C. Hence the potential function is


1
f (x, y, z) = tan1 (xy) + sin1 (yz) + z 2 + C.
2
1

(b) For the curve r = 1 + cos , 0 , the initial point is r = 2, so r( = 0) =


2 i + 0 j + 0 k, and the terminal point is r = 0, so r( = 0) = 0 i + 0 j + 0 k By
the Fundamental Theorem of Line Integral, we get
Z
F dr = f (0 i + 0 j + 0 k) f (2 i + 0 j + 0 k) = 0 0 = 0.
C

Homework 2. Let

F = yz i + (xz + ze ) j + 2xyz + p(y, z) +
2

yz

1
1+z

k,

where p(y, z) is a smooth functions with p(0, z) = 0. Suppose that F is a conservative


field.
(a) Find p(y, z).
(b) Find the potential function of F.
(c) Let C be a curve r(t) = t i + t2 j + t3 k, 0 t 1. Find

F dr.
(96 )

Solution.
(a) Since F is a conservative field, there exists a smooth function f (x, y, z) satisfying the following equations:
f
= yz 2 ,
x

f
= xz 2 + zeyz ,
y

f
1
= 2xyz + p(y, z) +
.
x
1+z

From the first equation, we know


Z
f (x, y, z) = yz 2 dx = xyz 2 + g(y, z).

Next, we compute


g

g
f
xyz 2 + g(y, z) = xz 2 +
=
= xz 2 + zeyz
= zeyz ,
y
y
y
y

so we get

g(y, z) =
Finally, we calculate

z eyz dy = eyz + h(z).


g

f
xyz 2 + g(y, z) = 2xyz +
=
= 2xyz + yeyz + h (z)
z
z
z
1
= p(y, z) +
,
1+z
Since h (z) is independent of y, we get p(y, z) = y eyz .
2

(b) We solve h (z) =

1
1+z

and get h(z) = ln |1 + z| + C. Hence the potential

function of F is f (x, y, z) = xyz 2 + eyz + ln |1 + z| + C.


(c) For the curve C, the initial point is r(0) = 0 i + 0 j + 0 k, and the terminal
point is r(1) = 1 i + 1 j + 1 k By the Fundamental Theorem of Line Integral,
we get
Z

F dr = f (1 i + 1 j + 1 k) f (0 i + 0 j + 0 k)

= (1 + e + ln 2) (0 + e0 + ln 1) = e + ln 2.

Homework 3. Let F(x, y, z) = y 2 i + (2xy + e3z ) j + 3ye3z k.


(a) Find the potential function of F.
(b) Compute the line integral
t 2 .

Fdr, where C : r(t) = 4t i+3 cos t j+3 sin t k, 0


(98 )

Solution.
(a) The potential function f (x, y, z) of F satisfies the following equations:
f
= y2,
x

f
= 2xy + e3z ,
y

f
= 3y e3z .
z

From the first equation, we have


f (x, y, z) =

y 2 dx = xy 2 + g(y, z)

Next, we compute

and it implies


f
g

xy 2 + g(y, z) = 2xy +
=
= 2xy + e3z ,
y
y
y
g
= e3z g(y, z) =
y

e3z dy = y e3z + h(z).

Finally, we calculate
 g
f
xy 2 + g(y, z) =
= 3y e3 z + h (z) = 3y e3z
z z
z

So we solve h (z) = 0 h(z) = C. Hence the potential function is f (x, y, z) =


xy 2 + ye3z + C.
3

(b) For the curve C, the initial point is r(0) = 0 i + 3 j + 0 k, and the terminal
point is r( 2 ) = 2 i + 0 j + 3 k By the Fundamental Theorem of Line Integral,
we get
Z
F dr = f (2 i + 0 j + 3 k) f (0 i + 3 j + 0 k) = (0 + 0) (0 + 3e0 ) = 3.
C

Homework 4. Find the line integral


Z
(2x sin(y) ez ) dx + (x2 cos(y) 3ez ) dy xez dz
C

along the curve C = {(x, y, z)|z = ln

1 + x2 , y = x, 0 x 1}.

(101 )

Solution. Let
F(x, y, z) = P i + Q j + R k = (2x sin(y) ez ) i + (x2 cos(y) 3ez ) j xez k.
We compute
Q
R
Q
R
=0
= 3ez
6=
y
z
y
z
P
R
R
P
= ez
= ez
=
z
x
z
x
Q
P
Q
P
= 2x cos(y)
= 2x cos(y)
=
,
x
y
x
y
so F is not a conservative field. However, we consider another vector field
y, z) = P i + Q
j+R
k
F(x,
= (2x sin(y) ez ) i + (x2 cos(y) 3ez ) j + (xez 3yez ) k.
is a conservative field, and F(x, y, z) = F(x,
y, z) + 3yez k.
Then F
then it satisfies the
Next, we want to find the potential function f(x, y, z) of F,
following equations:
f
= 2x sin(y) ez
x

f
= x2 cos(y) 3ez
y

f
= xez 3yez .
z

For the first equation, we have


Z

f (x, y, z) = (2x sin(y) ez ) dx = x2 sin(y) x ez + g(y, z).


4

We compute


f
x2 sin(y) xez + g(y, z) = x2 cos(y) +
=
= x2 cos(y) 3ez .
y
y
y

So we have

g
= 3ez g(y, z) =
y

3ez dy = 3yez + h(z).

Finally, we calculate

f

x2 sin(y) xez + g(y, z) = xez +


=
z
z
z
z
z

z
z
(z) = xe 3ye .
= xe 3ye + h
(z) = 0 h(z) = C. The potential function of F
is
So we solve h
f(x, y, z) = x2 sin(y) xez 3yez + C.
The line integral is
Z
Z 
Z
Z

z

F dr =
F + 3ye k dr =
F dr +
3yez k dr
C
C
C
C
Z 1

t
2
dt
= f(1, 1, ln 2) f(0, 0, 0) +
3teln 1+t
1 + t2
0
Z 1

1
1
3t2
ln 2
ln 2
2
2

= (e
31e
)+
dt = 4 2 + I.
1 + t2
0
For the integral I, we let t = tan , then dt = sec2 d and the upper limit is =

and lower limit is = 0. So we have


Z
Z
Z
4 3 tan2
4
4
2
2
I=
3 tan sec d = 3
(sec3 sec ) d.
sec d =
sec

0
0
0
Since
Z

i = 4 Z

sec tan

sec x dx =
sec d tan =
tan d sec
=0
0
0
Z
Z

4
4
2
= 2
tan sec d = 2
(sec2 1) sec d
0
0
Z
Z
h
i = 4

4
4

3
= 2
sec d +
sec d = 2 II + ln | sec + tan |

II =

2 II + ln | 2 + 1|.

=0

R
we get II = 21 2 + 12 ln | 2 + 1|. Hence I = 32 2 32 ln | 2 + 1| and C F dr =

52 2 32 ln | 2 + 1|.
5

16.4

Greens Theorem, page 1084

Homework 5. Find the positive-oriented simple closed curve C such that the inR
tegral C (y 3 y) dx x3 dy is maximum. Find the maximum value.
(93 )

Solution. Let P = y 3 y and Q = x3 , then Qx = 3x2 and Py = 3y 2 1. By


Greens Theorem, we know
Z
ZZ
3
3
(y y) dx x dy =
C


3x2 3y 2 + 1 dA.

We want to maximize the line integral, and it is equivalent to maximize the right
hand side double integral. Notice that the function f (x, y) = 3x2 3y 2 + 1 is
nonnegative in the region 3x2 + 3y 2 1, and this region will maximize the double
integral. Hence the boundary of the region will maximize the line integral, and the
curve is 3x2 + 3y 2 = 1.
We use the polar coordinates x = r cos and y = r sin to compute the double
integral:
Z

=2

=2

=0

r= 1

(3r + 1)r dr d =

r=0

=2
=0

r= 1

(3r 3 + r) dr d

r=0

 r= 1

Z 2
1

3 4 1 2 3
d =
d = .
=
r + r
4
2
12 0
6
=0
r=0
H
Homework 6. Evaluate the line integral C F dr counterclockwise around the
region bounded by 0 x 1 and x2 y 1, where F(x, y) = sin(x3 ) i +

x2
1+y 2

j.

(100 )
Solution. Let P = sin(x3 ) and Q =

x2
,
1+y 2

then Qx =

2x
1+y 2

and Py = 0. By Greens

Theorem, we have

Z y=1 Z x=y
I
ZZ 
2x
2x
dA =
dx dy
F dr =
2
1+y
1 + y2
y=0
C
R
x=0
Z y=1  2  x=y
Z
Z 1

x
1 1 1
y

=
dy =
d(1 + y 2 )
dy =
2
1 + y 2 x=0
2 0 1 + y2
y=0
0 1+y


y=1 1
1
2
= ln 2.
ln 1 + y
=
2
2
y=0

Homework 7. Evaluate
counterclockwise.

r=1cos

(x2 y + y) dx(xy 2 x) dy with the curve oriented


(101 )

Solution. Let P = x2 y + y and Q = xy 2 + x, then Qx = y 2 + 1 and Py = x2 + 1.


By Greens Theorem, we have
I
ZZ
2
2
(x y + y) dx (xy x) dy =
=

r=1cos
=2 Z r=1cos

=2


y 2 x2 dA

 r=1cos
1 4
d
r
4
r=0

r r dr d =
=0
Z 2
Z 2
1
1
=
(1 cos )4 d =
(1 + 6 cos2 + cos4 ) d
4 0
4 0
 
2 !

Z
1 2
1 + cos2
1 + cos 2
=
+
d
1+6
4 0
2
2


Z 
1 + cos 4
1 2
1
1 + 2 cos 2 +
d
=
1 + 3(1 + cos 2) +
4 0
4
2

Z 
1 2 35 7
1
35
=
+ cos 2 + cos 4 d = .
4 0
8
2
8
16
=0

r=0

Homework 8. Let D be the bounded region in the first quadrant enclosed by y =

0, x = 1, and y = x with positively oriented boundary C (i.e. counter clockwise.)





H 
1
3
3
3
Evaluate C 9x2 y(x3 + 1) 2 xy 2 (x3 + 1) 2 dx + 2(x3 + 1) 2 + 2(y 3 + 1) 2 dy.
(102 )

Solution. Let P = 9x2 y(x3 + 1) 2 xy 2 (x3 + 1) 2 and Q = 2(x3 + 1) 2 + 2(y 3 + 1) 2 ,


then
1
1
Q
= 3(x3 + 1) 2 3x2 = 9x2 (x3 + 1) 2
x

3
1
P
= 9x2 (x3 + 1) 2 2xy(x3 + 1) 2 .
y

By Greens Theorem, we have


I 



1
3
3
3
2
3
2 3
3
3
2
2
2
2
9x y(x + 1) xy (x + 1) dx + 2(x + 1) + 2(y + 1) dy
C

Z x=1 Z y=x
ZZ 
3
Q P
2xy(x3 + 1) 2 dy dx
dA =

=
x
y
y=0
x=0
R

Z 1
Z x=1 h
i
y=
x
3
3
2
3
2
dx =
x2 (x3 + 1) 2 dx
=
x(x + 1) y
y=0
0
x=0
 x=1

Z 1

5
3
2  5
1
1
2
3
3
3
=
=
22 1 .
(x + 1) 2
(x + 1) 2 d(x + 1) =
3 0
3 5
15
x=0
7

Calculus A (2) Homework 13 ANSWER


16.5

Curl and Divergence, page 1091

Homework 1 (page 1097). Let f be a scalar field and F a vector field. State
whether each expression is meaningful. If not, explain why. If so, state whether it is
a scalar field or a vector field. (a) curl f . (b) grad f . (c) div F. (d) curl(grad f ). (e)
grad F (f) grad(div F) (g) div(grad f ) (h) grad(div f ) (i) curl(curl F) (j) div(div F)
(k) (grad f ) (div F) (l) div(curl(grad f )).
Solution.
(a) curl f is meaningless because f is a scalar field.
(b) grad f is a vector field.
(c) div F is a scalar field.
(d) curl(grad f ) is a vector field.
(e) grad F is meaningless because F is not a scalar field.
(f) grad(div F) is a vector field.
(g) div(grad f ) is a scalar field.
(h) grad(div f ) is meaningless because f is a scalar field.
(i) curl(curl F) is a vector field.
(j) div(div F) is meaningless because div F is a scalar field.
(k) (grad f ) (div F) is meaningless because div F is a scalar field.
(l) div(curl(grad f )) is a scalar field.

Homework 2. Let F =

p

x2 + y 2

x
1+y 2

i + (ex + tan1 y) j. Find

F n ds,

where C is the cardioid r = 1 + cos .


(95 )

p
x
x2 + y 2 1+y
and Q = (ex + tan1 y), then
Solution. Let P =
2
P
x
1
=p
p
x
x2 + y 2
x2 + y 2

1
Q
=
y
1 + y2

P
Q
x
+
=p
.
x
y
x2 + y 2

We use the polar coordinates x = r cos and r = sin and get



Z =2 Z r=1+cos
I
ZZ 
r cos
Q
P
dA =
+
r dr d
F n ds =
x
y
r
=0
r=0
C
R

 r=1+cos
Z =2
Z =2
1 2
1
=
cos
d =
r
cos (1 + cos )2 d
2
2
=0
=0
r=0
Z 2
Z =2

1
2
3
cos + 2 cos + cos d =
cos2 d
=
2
0
=0


 =2
Z 2 
1 + cos 2
1
sin 2
=
= .
d =
+

2
2
4
0
=0




H
yln(1+y 2 )
x+2y tan1 x
Homework 3. Let F =
i
+
j.
Find
F n ds, where C
2
2
1+y
1+x
C 1
is the boundary of the rectangular region R = {(x, y)|1 x 3, 3 y 1}
(96 )
Solution. Let P =

x+2y tan1 x
1+y 2

and Q =

yln(1+y 2 )
,
1+x2

1
2y
P
=
+
2
2
x
1+y
(1 + y )(1 + x2 )

then

1
2y
Q
=

,
2
2
y
1+x
(1 + x )(1 + y 2)

and we get
P
Q
1
1
+
=
+
.
2
x
y
1+x
1 + y2
So
I

F n ds =

x= 3

ZZ 
R

Q
P
+
x
y

dA =

x= 3

x=1

y=1

y= 1

1
1
+
2
1+x
1 + y2

dy dx

h
i y=1
1 h i y=1

1
y 1 + tan y 1 dx
=
2
1+x
y=
y=
x=1
3
3




h
Z 3 
i x=3
1
h i x= 3
1

1

1
1
+
dx = 1
x
+
tan x
=
x=1
x=1
12
3 1 + x2 12
3
1



1

3
= 1
+ ( 3 1) =
.
18
3 12 12
Z

16.6

Parametric Surfaces and Their Areas,


page 1099

Homework 4 (page 1110).


(a) Find a parametric representation for the torus obtained by rotating about the
z-axis the circle in the xz-plane with center (b, 0, 0) and radius a < b.
(95 )

(b) Find the surface area of the torus.


Solution.

(a) r(u, v) = (b cos u + a cos v cos u)i + (b sin u + a cos v sin u)j + a sin v k, where
0 2, 0 v 2.
(b) We compute
ru (u, v) = (b sin u a cos v sin u)i + (b cos u + a cos v cos u)j + 0 k
rv (u, v) = (a sin v cos u)i + (a sin v sin u)j + a cos v k
ru rv (u, v) = a cos v(b cos u + a cos v cos u) i
+ a cos v(b sin u a cos v sin u) j
+ (b sin u a cos v sin u)(a sin v sin u)
(a sin v cos u)(b cos u + a cos v cos u) k
= a(b + a cos v) ((cos u cos v) i + (sin u cos v) j + (sin v) k)
Then
p
|ru rv | = a(b + a cos v) cos2 u cos2 v + sin2 u cos2 u + sin2 v = a(b + a cos v).
Hence

A(S) =

u=2

u=0

v=2
v=0

h
i v=2

= 4 2 ab.
a(b + a cos v) dv du = 2 abv + a2 sin v
v=0

16.7

Surface Integrals, page 1110


RR

y 2z dS, where S is the part of the


p
sphere x2 + y 2 + z 2 = 4 between the cone z = x2 + y 2 .
(90 )
Homework 5. Find the surface integral

Solution. The surface is r(u, v) = 2 sin u cos v i + 2 sin u sin v j + 2 cos u k, where

0 u 4 , 0 v < 2, then we have


ru (u, v) = 2 cos u cos v i + 2 cos u sin v j 2 sin u k
rv (u, v) = 2 sin u sin v i + 2 sin u cos v j + 0 k
ru rv (u, v) = 4 sin2 u cos v i + 4 sin2 u sin v j + 4 sin u cos u k
p
|ru rv | = 4 sin4 u cos2 v + sin4 u sin2 v + sin2 u cos2 u = 4 sin u.

The surface integral becomes


ZZ

y z dS =

u= 4

u=0

v=2

4 sin2 u sin2 v 2 cos u 4 sin u dv du

v=0
u= 4 Z v=2

sin3 u cos u sin2 v dv du


u=0
v=0


Z u= Z v=2
4
1 cos 2v
3
dv du
= 32
sin u cos u
2
u=0
v=0

 v=2
Z u=
4
sin 2v
3
= 32
sin u cos u
du


2
4
u=0
v=0

 u=
Z
4
1 4 4
3
= 2.
sin u d sin u = 32
= 32
sin u
4

= 32

Homework 6. Find the surface integral

u=0

RR

x2 +y 2 +z 2 =R2

z 4 dS.

(91 )

Solution. The surface is r(u, v) = R sin u cos v i + R sin u sin v j + R cos u k, where
0 u , 0 v 2, then we have
ru (u, v) = R cos u cos v i + R cos u sin v j R sin u k
rv (u, v) = R sin u sin v i + R sin u cos v j + 0 k
ru rv (u, v) = R2 sin2 u cos v i + R2 sin2 u sin v j + R2 sin u cos u k
p
|ru rv | = R2 sin4 u cos2 v + sin4 u sin2 v + sin2 u cos2 u = R2 sin u.

The surface integral becomes


ZZ
Z u= Z
4
z dS =
x2 +y 2 +z 2 =R2

u=0

v=2

R4 cos4 u R2 sin u dv du
v=0

= 2 R

u=
4

cos u d cos u = 2 R

u=0

 u=

1
5
cos u
5
u=0

4 6
R .
5
RR
Homework 7. Evaluate S (x2 + y 2 )z dS, where S is the upper hemisphere x2 +
=

y 2 + z 2 = 1, z 0.

(93 )

Solution. The surface is r(u, v) = sin u cos v i + sin u sin v j + cos u k, where 0 u

,0
2

v 2, then we have
ru (u, v) = cos u cos v i + cos u sin v j sin u k
rv (u, v) = sin u sin v i + sin u cos v j + 0 k
ru rv (u, v) = sin2 u cos v i + sin2 u sin v j + sin u cos u k
p
|ru rv | = sin4 u cos2 v + sin4 u sin2 v + sin2 u cos2 u = sin u.

The surface integral becomes


ZZ

(x + y )z dS =
S

u= 2
u=0

= 2

v=2

sin2 u cos u sin u dv du

v=0
u= 2

u=0

 u=
1 4 2

sin u d sin u = 2
= .
sin u
4
2
u=0
3

Calculus A (2) Homework 14 ANSWER


16.7

Surface Integrals, page 1110

RR
Homework 1. Evaluate S F dS, where F = xz i + yz j + z 2 k, and S is the part
of sphere x2 + y 2 + z 2 = 1, z 12 with upward orientation.
(91 )
Solution. We parameterize the surface S by
r(u, v) = sin u cos v i + sin u sin v j + cos u k, 0 u

, 0 v 2,
4

then we have
ru (u, v) = cos u cos v i + cos u sin v j sin u k
rv (u, v) = sin u sin v i + sin u cos v j + 0 k
ru rv = sin2 u cos v i + sin2 u sin v j + sin u cos u k
F(u, v) = sin u cos u cos v i + sin u cos u sin v j + cos2 u k.
So we get
ZZ

F dS =

v=2

u= 4

(sin3 u cos u + sin u cos3 u) du dv


v=0
u=0
Z v=2 Z u=
Z v=2 Z u=
4
4
sin u cos u du dv =
sin u d sin u dv
=

v=0
v=2

v=0

u=0

v=0

u=0

 u=
1
1 2 4

dv = 2 = .
sin u
2
4
2
u=0

Solution 2. We can use the Divergence Theorem:


ZZZ
ZZ
ZZ
F dS,
div F dV =
F dS +
E

S1

where S1 is the part of the plane {(x, y, z)|x + y 1, z =

1 }.
2

First we compute

div F = z + z + 2z = 4z,
then we use the spherical coordinates x = sin cos , y = sin sin , z = cos
to get
ZZZ
Z = Z =2 Z =1
4
div F dV =
4 cos 2 sin d d d
=0

=0

= 4

=0

=2

=0

1
2 cos

h i =1

4

1
2 cos

cos sin d d


1
cos +
d cos
= 2
4 cos3
0
 = 4




1 1

1
1
2

= 2
= .

= 2 cos

2
2
8 cos =0
4 8
4
Z

Next, we parameterize the surface S1 by


1
1
r(u, v) = u cos v i + u sin v j + k, 0 u , 0 v < 2.
2
2
Then
ru (u, v) = cos v i + sin v j + 0 k
rv (u, v) = u sin v i + u cos v j + 0 k
ru rv = 0 i + 0 j + u k // 0 i + 0 j u k
1
1
1
F(u, v) = u cos v i + u sin v j + k
2
2
2
So we get
u= 1

1
F dS =
u du dv =
2
S1
v=0

 u=0

1
= .
= 2
8
4
Z

ZZ

v=2

v=2

v=0

 u= 1

2
1 2
u
dv
4
u=0

So
ZZ

F dS =

ZZZ

div F dV
E

ZZ

 
F dS =
= .
4
4
2
S1

Solution 3. We can use the Divergence Theorem:


ZZZ
ZZ
ZZ
F dS,
div F dV =
F dS +
E

S1

where S1 is the part of the sphere {(x, y, z)|x2 + y 2 + z 2 = 1, z 12 }. First we


compute div F = z + z + 2z = 4z, then we use the spherical coordinates x =
sin cos , y = sin sin , z = cos to get
ZZZ
Z = Z =2 Z =1
div F dV =
4 cos 2 sin d d d = 0.
E

=0

=0

=0

Next, we parameterize the surface S1 by


r(u, v) = sin u cos v i + sin u sin v j + cos u k,

u , 0 v < 2.
4

Then
ru (u, v) = cos u cos v i + cos u sin v j sin u k
rv (u, v) = sin u sin v i + sin u cos v j + 0 k
ru rv = sin2 u cos v i + sin2 u sin v j + sin u cos u k
F(u, v) = sin u cos u cos v i + sin u cos u sin v j + cos2 u k
2

So we get
ZZ

F dS =

v=2

v=2

v=2

v=0

u=

So
ZZ

F dS =

(sin3 u cos u + sin u cos3 u) du dv


sin u cos u du dv =

u= 4

v=0

u=

u= 4

v=0

v=2
v=0

u=

sin u d sin u dv
u= 4

 u=

1
1 2
sin u
dv = 2 = .
2
4
2
u=
4

ZZZ

div F dV
E

RR

ZZ

 
= .
F dS = 0
2
2
S1

Homework 2. Evaluate S F dS, where F(x, y, z) = 2y 2 j z k and S is the


surface of the solid enclosed by y = x2 + z 2 and y = 1 with outward normal vector.
(98 )
Solution. Let S = S1 S2 , where S1 is the part of the paraboloid and S2 is the
part of the plane. First, we parameterize the surface S1 by
r(u, v) = u cos v i + u2 j + u sin v k, 0 u 1, 0 v < 2
ru (u, v) = cos v i + 2u j + sin v k
rv (u, v) = u sin v i + 0 j + u cos v k
ru rv = 2u2 cos v i u j + 2u2 sin v k
F(u, v) = 0 i + 2u4 j u sin v k
So we compute
Z
ZZ
F dS =
S1

v=2

v=0
v=2

u=1

(2u5 2u3 sin2 v) du dv

u=0


 u=1
1 6 1 4 2
=
u u sin v
dv
3
2
v=0
u=0


Z 2 
Z 2 
1
7
1 1 2
=
cos 2v dv
sin v dv =
3 2
12 4
0
0

 v=2

7
7
1
7
= v sin 2v
= 2 = .
12
8
12
6
Z

v=0

Next, we parameterize the surface S2 by

r(u, v) = u cos v i + 1 j + u sin v k, 0 u 1, 0 v < 2


ru (u, v) = cos v i + 0 j + sin v k
rv (u, v) = u sin v i + 0 j + u cos v k
ru rv = 0 i u j + 0 k // 0 i + u j + 0 k
F(u, v) = 0 i + 2 j u sin v k.
3

So we compute
Z
ZZ
F dS =
S2

v=2

v=0

Hence we get
ZZ

u=1

2u du dv =

u=0

F dS =

v=2

v=0

ZZ

F dS +

S1

Z
h i u=1
2
u
dv =
u=0

v=2

1 dv = 2.

v=0

5
7
F dS = + 2 = .
6
6
S2

ZZ

Solution 2. We can use the Divergence Theorem to get:


ZZ
ZZZ
ZZ
Z y=1
F dS =
div F dV =
(4y 1) dy dA
S

=
=

E
=2Z r=1

=0
r=0
=2Z r=1

=0

16.8

r=0

x2 +z 2 1

y=x2 +z 2

Z
i y=1

r dr d =
2y y
2
2

=2Z r=1


1 2r 4 + r 2 r dr d
y=r
=0
r=0
 r=1


5
1 6 1 4
1
2
5
3
= .
r r + r
r 2r + r dr d = 2
2
3
4
6
r=0

Stokes Theorem, page 1110

Homework 3. Suppose that S is the part of the paraboloid z = x2 + y 2 that lies


below the plane z = 1 and oriented upward. Let F = x2 y i xy 2 j + zex k. Evaluate
RR
F n dS.
(91 )
S

Solution. Let C is the boundary curve of S. We first parameterize the curve C by


r(t) = cos t i + sin t j + 1 k, 0 t < 2
r (t) = sin t i + cos t j + 0 k
F(t) = cos2 t sin t i cos t sin2 t j + ecos t k
By Stokes Theorem, we have
ZZ
I
Z 2
F n dS =
F dr =
2 cos2 t sin2 t dt
S
 0


ZC2
Z
1 + cos 2t
1 2
1 cos 2t
=
2
dt =
(1 cos2 2t) dt
2
2
2 0
0



Z 2 
Z 2 
1
1
1 + cos 4t
1 cos 4t
dt =
dt
=
1

2 0
2
2 0
2
2

 t=2

sin 4t
1 1
= .
t
=

2 2
8
2
t=0
4

Solution 2. Let C is the boundary curve of S and let S1 is the part of plane
x2 + y 2 1, z = 1. By Stokes Theorem twice, we get
ZZ
I
ZZ
F n dS.
F n dS =
F dr =
S

S1

We parameterize the surface S1 by


r(u, v) = u cos v i + u sin v j + 1 k, 0 u 1, 0 v < 2.
Then we have
ru (u, v) = cos v i + sin v j + 0 k
rv (u, v) = u sin v i + u cos v j + 0 k
ru rv = 0 i + 0 j + u k.
Next, we compute

i
j
k

curl F = x
y
z

x2 y xy 2 zex
So
ZZ

ZZ





= 0 i zex j + (y 2 x2 ) k = eu cos v j u2 k.

F n dS =
 
1

= 2
= .
4
2

F n dS =

S1

v=2Z u=1

v=0

u=0

 u=1
1 4
u du dv = 2 u
4
u=0
3

Solution 3. We parameterize the surface S by


r(u, v) = u cos v i + u sin v j + u2 k, 0 u 1, 0 v < 2.
Then we have
ru (u, v) = cos v i + sin v j + 2u k
rv (u, v) = u sin v i + u cos v j + 0 k
ru rv = 2u2 cos v i 2u2 sin v j + u k.
We compute

i
j
k

curl F = x
y
z

x2 y xy 2 zex





= 0 i zex j + (y 2 x2 ) k = u2 eu cos v j u2 k.


5

So
ZZ

u=1Z v=2

(2u4 sin veu cos v u3 ) dv du


S
u=0 v=0
Z u=1Z v=2
Z u=1Z v=2
4
u cos v
=
2u sin ve
dv du
u3 dv du
u=0 v=0
u=0 v=0
Z 1
Z u=1 h
i v=2

du 2
u3 du
=
2u3 eu cos v
v=0
0
u=0
 u=1


1

1 4
= 2 = .
u
= 2
4
4
2
u=0
RR
Homework 4. Evaluate S curl F n dS, where F = yz 5 i + sin(xyz) j + x k and
S is the part of paraboloid y = 1 x2 z 2 that lies to the right of the xz-plane,
F n dS =

(93 )

oriented in the direction of the positive y-axis.

Solution. Let C is the boundary curve of S. First we parameterize the curve C by


r(t) = cos t i + 0 j + sin t k
r (t) = sin t i + 0 j + cos t k
F(t) = 0 i + 0 j + cos t k.
By Stokes Theorem, we have

ZZ
I
Z 0
Z 0
1 + cos 2t
2
dt
curl F n dS =
F dr =
cos t dt =
2
S
C
2
2
 t=0

cos 2t
1
= .
t+
=

2
4
t=2

Remark that the orientation of the curve should be take from t = 2 to t = 0.


Solution 2. We will use Stokes Theorem twice to get
ZZ
I
ZZ
curl F n dS,
curl F n dS =
F dr =
S

S1

where S1 is the part of the plane x2 + z 2 1, y = 0. We parameterize the surface


S1 by
r(u, v) = u cos v i + 0 j + u sin v k, 0 u 1, 0 v < 2
ru (u, v) = cos v i + 0 j + sin v k
rv (u, v) = u sin v i + 0 j + u cos v k
ru rv = 0 i u j + 0 k // 0 i + u j + 0 k,

and then compute



i
j

curl F = x
y

yz 5 sin(xyz)


k

= xy cos(xyz) i (1 5yz 4 ) j + (yz cos(xyz) z 5 ) k
z
x
= j u5 sin5 v k.

So
ZZ

curl F n dS =

=
Homework 5. Evaluate

ZZ
Z

curl F n dS =

S1
v=2

v=0

v=2

v=0

u=1

u du dv

u=0

 u=1
1
1 2
dv = 2 = .
u
2
2
u=0

F dr around the curve

r(t) = cos t i + sin t j + (sin t + cos t) k,

0 t 2,

where F = (ex y 3) i + (ey + x3 ) j + (ez + x + y) k.

(100 )

Solution. First, we know the boundary of the surface S = {(x, y, z)|x2 +y 2 1, z =


x + y} is C. By Stokes Theorem, we know
I
ZZ
F dr =
curl F dS.
C

We parameterize the surface S by


r(u, v) = u cos v i + u sin v j + u(cos v + sin v) k, 0 u 1, 0 v < 2
ru (u, v) = cos v i + sin v j + (cos v + sin v) k
rv (u, v) = u sin v i + u cos v j + u( sin v + cos v) k
ru rv = u i u j + u k.
Next, we compute


i
j
k

curl F =
x
y
z

ex y 3 ey + x3 ez + x + y





= i j + (3x2 + 3y 2) k = i j + 3u2 k.

So we get
 u=1
I
ZZ
Z v=2 Z u=1
Z v=2 
3 4
2
F dr =
curl F dS =
u 3u du dv =
dv
u
4
C
S
v=0
u=0
v=0
u=0
Z 2
3
3
3
dv = 2 = .
=
4
4
2
0
7

Solution 2. We compute
r (t) = sin t i + cos t j + (cos t sin t) k,
F(t) = (ecos t sin3 t) i + esin t + cos3 t j + esin t+cos t (cos t + sin t) k,
and get
I

F dr = I + II + III + IV + V + VI,

where
Z 2
Z 2
i t=2
h

cos t
=0
e ( sin t) dt =
ecos t d cos t = ecos t
t=0
0
0

Z 2
Z 2 
1 3
3
cos 4t
4
sin t dt =
dt =
2 cos 2t +
4 2
2
4
0
0
Z 2
Z 2
i t=2
h

=0
cos tesin t dt =
esin t d sin t = esin t
t=0
0
0

Z 2
Z 2 
3
cos 4t
1 3
4
dt =
+ 2 cos 2t +
cos t dt =
4 2
2
4
0
0
Z 2
Z 2
i =2
h
sin t+cos t
sin t+cos t
sin t+cos t
=0
e
(cos t sin t) dt =
e
d(sin t + cos t) = e

=0
0
0


Z 2
Z 2 
1 cos 2t
1 + cos 2t
2
2
dt = 0.

(cos t sin t) dt =
2
2
0
0
Hence
3
F dr = .
2
C

16.9

The Divergence Theorem, page 1127

RR
Homework 6. Evaluate the surface integral S F n dS, where F = 2y 2 i + xez j +
3 k, S is the upper hemisphere x2 + y 2 + z 2 = 1, z 0, and n is the unit normal
(91 )

pointing upward.
Solution. Since div F = 0, by the Divergence Theorem, we have
ZZZ
ZZ
ZZ
F n dS,
0=
div F dV =
F n dS +
E

S1

where S1 is the part of the plane {x2 + y 2 1, z = 0}. We parameterize the surface
S1 by
r(u, v) = u cos v i + u sin v j + 0 k, 0 u 1, 0 v < 2.
8

Then we have
ru (u, v) = cos v i + sin v j + 0 k
rv (u, v) = u sin v i + u cos v j + 0 k
ru rv = 0 i + 0 j + u k // 0 i + 0 j u k
F(u, v) = 2u2 sin2 v i + u cos v j + 3 k.
So
ZZ

ZZ

v=2

F ndS =
F ndS =
S1
v=0
Z v=2
3
3
=
dv = 2 = 3.
2
2
v=0

u=1

3u du dv =

u=0

v=2

v=0

 u=1
3 2
dv
u
2
u=0

Solution 2. We parameterize the surface S by


r(u, v) = sin u cos v i + sin u sin v j + cos u k, 0 u

, 0 v < 2.
2

Then we have
ru (u, v) = cos u cos v i + cos u sin v j sin u k
rv (u, v) = sin u sin v i + sin u cos v j sin u k
ru rv = sin2 u cos v i + sin2 u sin v j + sin u cos u k
F(u, v) = 2 sin2 u sin2 v i + sin u cos vecos u j + 3 k.
So the surface integral is
ZZ
F n dS
S
Z v=2 Z u=
2
(2 sin4 u sin2 v cos v + sin3 u sin v cos vecos u + 3 sin u cos u) du dv
=
v=0
v=2

v=0

u=0
u= 2
u=0

Homework 7.

 u=
3 2 2
3
3 sin u cos u du dv = 2
= 2 = 3.
sin u
2
2
u=0


(a) Find the flux integral of the vector field F = (x2 4xy) i 6yz j + 12z k on
the surface of the box 0 x a, 0 y b, 0 z 1.
(b) Find a, b such that the flux integral attains maximum.
Solution.

(91 )

(a) By the Divergence Theorem, the flux integral is


ZZ
ZZZ
Z x=a Z y=b Z z=1
F dS =
div F dV =
(2x 4y 6z + 12) dz dy dx
S

=
=

B
x=a Z y=b

x=0
x=a

Zx=0
x=a

y=0
y=b

x=0

y=0

z=0

i z=1

dy dx
2xz 4yz 3z + 12z
2

z=0

(2x 4y + 9) dy dx

y=0

Z x=a
i y=b

dx =
(2xb 2b2 + 9b) dx
=
2xy 2y + 9y
y=0
x=0
hx=0
i x=a

= a2 b 2ab2 + 9ab.
= bx2 2b2 x + 9bx
h

x=0

(b) Let f (a, b) = a2 b 2ab2 + 9ab. We will first find all critical points of f (a, b)
for a 0 and b 0:
fa = 2ab 2b2 + 9b = b(2a 2b + 9)
fb = a2 4ab + 9a = a(a 4b + 9).
Solutions of fa = 0 and fb = 0 are (a, b) = (0, 0), (0, 29 ), (3, 32 ), and (9, 0).
, and f (9, 0) = 0, we know that
Since f (0, 0) = 0, f (0, 92 ) = 0, f (3, 23 ) = 27
2
(a, b) = (3, 32 ) attains the maximum of the flux integral.
Solution 2. For the part (a), we can integrate the surface integral directly:
Back: Since
r(u, v) = 0 i + u j + v k, 0 u b, 0 v 1
n(u, v) = 1 i + 0 j + 0 k
F(u, v) = 0 i 6uv j + 12v k,
we have

RR

S1

F dS = 0.

Front: Since
r(u, v) = a i + u j + v k, 0 u b, 0 v 1
n(u, v) = 1 i + 0 j + 0 k
F(u, v) = (a2 4au) i 6uv j + 12v k,
we have
Z
ZZ
F dS =
S2

v=1

v=0

u=b
2

a 4au du dv =

u=0

v=1

v=0

= a2 b 2ab2 .
10

i u=b

dv
a2 u 2au2
u=0

Left: Since
r(u, v) = u i + 0 j + v k, 0 u a, 0 v 1
n(u, v) = 0 i 1 j + 0 k
F(u, v) = u2 i + 0 j + 12v k,
we have

RR

S3

F dS = 0.

Right: Since
r(u, v) = u i + b j + v k, 0 u a, 0 v 1
n(u, v) = 0 i + 1 j + 0 k
F(u, v) = u2 i 6bv j + 12v k,
we have
ZZ

F dS =

S4

v=1
v=0
v=1

u=a

6bv du dv =

u=0

v=1

v=0

(6abv) dv =
v=0

Bottom: Since

i u=a

6bvu
dv
u=0

i v=1

= 3ab.
3abv 2
v=0

r(u, v) = u i + v j + 0 k, 0 u a, 0 v b
n(u, v) = 0 i + 1 j 1 k
F(u, v) = (u2 4uv) i + 0 j + 0 k,
we have

RR

S5

F dS = 0.

Top: Since

r(u, v) = u i + v j + 1 k, 0 u a, 0 v b
n(u, v) = 0 i + 1 j + 1 k
F(u, v) = (u2 4uv) i 6v j + 12 k,
we have
ZZ

F dS =
S6

v=b

v=0

u=a

12 du dv = 12ab.
u=0

Hence the surface integral is


ZZ
F dS = a2 2ab2 3ab + 12ab = a2 2ab2 + 9ab.
S

11

RR

Homework 8. Evaluate

F dS, where F(x, y, z) = xy i

y2
2

j + z k, and S =

S1 S2 S3 , S1 = {(x, y, z)|z = 4 3x 3y , 1 z 4}, S2 = {(x, y, z)|x2 + y 2 =


1, 0 z 1}, S3 = {(x, y, z)|x2 + y 2 1, z = 0}.
(93 )
Solution. By the Divergence Theorem, we get
ZZ
ZZZ
ZZZ
F dS =
div F dV =
(y y + 1)dV = Volume(E).
S

The solid consists of the part of rigid paraboloid E1 and part of rigid cylinder E2 ,
so we compute
Volume(E1 ) =

ZZ

x2 +y 2 1

=2

=0

z=43x2 3y 2

dz dx dy =

z=1
r=1
2

(3 3r )r dr d =

r=0

3
3
= 2 = ,
4
2

=2

=2

=0
=2 

r=1

(3r 3r 3 ) dr d
r=0

 r=1
3 2 3 4
d
r r
2
4
r=0

=0

and
Volume(E2 ) =
=
Hence
ZZ

ZZ
Z

x2 +y 2 1
=2 

=0

z=1

dz dx dy =

z=0
 r=1

2

1
d =
r
2 r=0

=0

r=1

r dr d
r=0

1
1
d = 2 = .
2
2

3
5
F dS = Volume(E) = Volume(E1 ) + Volume(E2 ) = + = .
2
2
S

Solution 2. We can compute the surface integral directly. First, we parameterize


the surface S1 by
r(u, v) = u cos v i + u sin v j + (4 3u2 ) k, 0 u 1, 0 v
Then
ru (u, v) = cos v i + sin v j 6u k
rv (u, v) = u sin v i + u cos v j + 0 k
ru rv = 6u2 cos v i + 6u2 sin v j + u k
1
F(u, v) = u2 sin v cos v i + u2 sin2 v j + (4 3u2 ) k.
2

12

.
2

So we have
Z
ZZ
F dS =

v=2

v=0
v=2

S1

v=0

u=1

(6u4 sin v cos2 v + 3u2 sin3 v + 4u 3u3 ) du dv

u=0
u=1

(4u 3u ) du dv =

u=0

5
5
2 = .
4
2

v=2
v=0

 u=1
3 4
dv
2u u
4
u=0
2

Next, we parameterize S2 by
r(u, v) = cos u i + sin u j + v k, 0 u < 2, 0 v 1.
Then
ru (u, v) = sin u i + cos u j + 0 k
rv (u, v) = 0 i + 0 j + 1 k
ru rv = sin u i + cos u j + 0 k
1
F(u, v) = sin v cos u i sin2 u j + v k.
2
So we have
ZZ

F dS =

S2

v=1
v=0

u=2

u=0



1 3
2
sin u cos u sin u du dv = 0.
2

For S3 , we use the following parametrization:


r(u, v) = u cos v i + u sin v j + 0 k, 0 u 1, 0 v < 2.
Then we have
ru (u, v) = cos v i + sin v j + 0 k
rv (u, v) = u sin v i + u cos v j + 0 k
ru rv = 0 i + 0 j + u k
1
F(u, v) = u2 sin v cos v i + u2 sin2 v j + 0 k.
2
So we have

RR

S3

ZZ

F dS = 0. Hence
F dS =

ZZ

F dS +
S1

ZZ

F dS +
S2

13

5
F dS = .
2
S3

ZZ

Calculus A (1) Quiz 1, 2014/10/1

ANSWER

Department:

Name:

I.D.:

True-False: Determine whether the statement is true (T) or false (F).


(T) 1. If f is an even function, then f (2x) = f (2x).
(T) 2. If x1 < x2 and f is a decreasing function, then f (x1 ) > f (x2 ).
(F) 3. If f is one-to-one, then f 1 (x) =

1
.
f (x)

(T) 4. If 0 < a < b, then ln a < ln b.


(F) 5. If x > 0, then (ln x)6 = 6 ln x.
(T) 6. cos1 ( 12 ) =
(F) 7. tan1 x =

2
.
3

sin1 x
.
cos1 x

(T) 8. ln x is the inverse function of ex .


(F) 9. xa is the inverse function of xa .
f (x)
xa g(x)

does not exist.

f (x)
xa g(x)

does not exist.

(T) 10. If lim f (x) = 2 and lim g(x) = 0, then lim


xa

xa

(F) 11. If lim f (x) = 0 and lim g(x) = 0, then lim


xa

xa

(F) 12. If neither lim f (x) nor lim g(x) exists, then lim (f (x) + g(x)) does not exist.
xa

xa

xa

(T) 13. If lim f (x) exists but lim g(x) does not exist, then lim (f (x) + g(x)) does not
xa

xa

xa

exist.
(F) 14. If lim f (x)g(x) exists, then the limit must be f (a)g(a).
xa

(F) 15. Consider lim


n

1
n2

2
n2

++

n
n2

. Student A gives the following argument:

Since for every k = 1, . . . , n, each term has lim nk2 = 0, by the Limit Sum
n

1
2
n
Law, we have lim n2 + n2 + + n2 = lim n12 + lim n22 + + lim nn2 =
n

0 + 0 + + 0 = 0. Is this proof correct?

(F) 16. If f (x) > 1 for all x and lim f (x) exists, then lim f (x) > 1.
x0

x0

(F) 17. If f (1) > 0 and f (3) < 0, then there exists a number c between 1 and 3 such
that f (c) = 0.

3
1+cx1
,
x
x0
2
2

Problem 18. Evaluate lim

where c is a constant.

(Hint: a3 b3 = (a b)(a + ab + b ))
Solution.

3
lim
x0

1 + cx 1
(1 + cx) 3 1 (1 + cx) 3 + (1 + cx) 3 + 1
= lim

1
2
x0
x
x
(1 + cx) 3 + (1 + cx) 3 + 1
1 + cx 1
= lim
1
2
x0 x((1 + cx) 3 + (1 + cx) 3 + 1)
c
= lim
2
1
x0 (1 + cx) 3 + (1 + cx) 3 + 1
lim c
c
x0
=
= .
2
1
3
lim (1 + cx) 3 + lim (1 + cx) 3 + lim 1
x0

x0

Problem 19. The function f (x) =

1
2

x2
2

x0

is called the Gaussian bell function.

0.4
0.3

g(x)

f (x)
0.2
0.1

h(x)
-4

-2

Figure 1: f (x) =

1
2

x2

e 2 .

The followings are possible expressions of g(x) and h(x):


2

(x+1)
1 e 2
2
1 x 2
1 e 2 ( 2 )
2
1 x 2
1 e 2 ( 2 )
2 2

 2 
x 2+1
1

e
2
1 x 2
2 e 2 ( 2 )
2
1 x2
1 e 2 2
2 2

(x1)
1 e 2
2
1
2
1 e 2 (2x)
2
1
2
1 e 2 (2x)
2 2

 2 
x 21
1

e
2
1
2
2 e 2 (2x)
2
1
2
1 e 2 2x
2 2

Solution.
1 x 2
1
h(x) = e 2 ( 2 ) .
2 2

(x1)2
1
g(x) = e 2 ,
2

Calculus A (1) Quiz 2, 2014/10/22


Department:

I.D.:

ANSWER
Name:

True-False: Determine whether the statement is true (T) or false (F).


(T) 1. If f (x) is continuous at a, so is |f (x)|.
(F) 2. If |f (x)| is continuous at a, so is f (x).
(F) 3. lim
x

sin x
x

sin x
x x

= 1.

(T) 4. lim

= 1.

(T) 5. lim

f (x)f (xh)
h

h0

(F) 6.

d
dx

(F) 7.

d
(ln 10)
dx

= f (x).

ex = x ex1 .
=

1
.
10

f (x)
g(x)

(F) 8. If f and g are differentiable, then

d
dx

(T) 9. If f and g are differentiable, then

d
f (g(x))
dx

(T) 10.

d
d

tan2 =

d
d

f (x)
.
g (x)

= f (g(x))g (x).

sec2 .

d
d

cos
sin1 = (1) sin
2 .

x2 if x < 1
(F) 12. Let f (x) =
. Find f (x).
2x if x 1

(F) 11.

Student A gives the following argument:

If x < 1, then f (x) = 2x. If x > 1, then f (x) = 2.


If x = 1, both limits lim f (x) = lim+ f (x) = 2, so f (1) = 2.
x1
x1

2x if x < 1
Hence f (x) =
. Is this argument correct?
2 if x 1

Problem 13 (10pts). Differentiate the function f (x) =


Solution.
(x 1) (x1)2
e 2
f (x) =
2
1

1
2

(x1)2
2

Problem 14 (10pts). Find the tangent line and normal line of the rotated ellipse
x2 xy + y 2 = 3 at the point (1, 1).
Solution. We differentiate x2 xy + y 2 = 3 with respect to x and get

dy
dy
y 2x
dy
dy
+ 2y
=0
=

= 1.
2x y x
dx
dx
dx
2y x
dx (x,y)=(1,1)

Hence the tangent line is y 1 = x + 1 and the normal line is y 1 = (x + 1).


Problem 15 (5pts+5pts).
(a) Find the limit lim
x0

cos x1
.
x2

Solution.
2 sin2
cos x 1
lim
= lim
x0
x0
x2
x2

x
2



sin x2 2
1
1
= lim
= .
x
2 x0
2
2

1
(b) Find the limit lim+ (cos x) x .
x0

Solution. Let y =

x, then y 0+ as x 0+ . Thus

1
1
1
cos y1
lim+ (cos x) x = lim+ (cos y) y2 = lim+ (1 + (cos y 1)) cos y1 y2

y0

y0

x0

lim+ (1 + (cos y 1))

y0

1
cos y1

lim
y0+

cos y1
y2

= e 2 .

Problem 16 (10pts). Suppose that f (x) is continuous on [0, 1] with f (0) = f (1).
Show that there exists c (0, 12 ] such that f (c + 21 ) = f (c).
(If you want to quote some Theorem, you should explain that the following F (x)
satisfies all assumptions of the theorem and write down the name of the theorem.)
Proof. Without loss of generality, we assume f (0) = f (1) = 0. Consider the function
F (x) = f (x + 21 ) f (x) for x [0, 21 ]. Since f (x + 21 ) and f (x) are continuous
functions, so is F (x). Furthermore, we have F (0) = f ( 21 ) f (0) = f ( 12 ) and
F ( 12 ) = f (1) f ( 21 ) = f ( 21 ). If f ( 12 ) = 0, then we choose c =

1
2

and get f ( 21 + 21 ) =

f ( 21 ) = 0. If f ( 21 ) 6= 0, by the Intermediate Value Theorem, there is a c (0, 12 ) such


that F (c) = f (c + 12 ) f (c) = 0, which implies f (c + 12 ) = f (c).

Calculus A (1) Quiz 3, 2014/12/3 ANSWER


Department:

I.D.:

Name:

True-False: Determine whether the statement is true (T) or false (F).


Z b
Z b
Z b
(F) 1. If f, g are continuous on [a, b], then
(f (x)g(x)) dx =
f (x) dx
g(x) dx.
a

(T) 2.

(ax2 + bx + c) dx = 2

(ax2 + c) dx.

(F) 3. Let f (t) = [[t]] be the greatest integer function


Z x(). By the
d
Fundamental Theorem of Calculus, we have
[[t]] dt = [[x]].
dx 0
(F) 4. By the Fundamental Theorem of Calculus and Chain Rule, we have
Z x2
d
(x2 t) dt = (x2 x2 )2x = 0.
dx 0
(F) 5. By the Fundamental Theorem, we have
Z x
d
sin t d sin t = sin x d sin x = sin x cos x dx.
dx 0
Z b

(T) 6. If f is continuous on [a, b], then


f (x) dx = f (b) f (a).
a

(T) 7. If f (x) g (x) on [a, b] and f (a) = g(a), then f (x) g(x) for all x [a, b].
(T) 8. If f, g are continuous on [0, 1], then
Z 1
Z
f (x)g(1 x) dx =
0

(F) 9.

f (1 x)g(x) dx.

x=

= tan tan 0 = 0 0 = 0.
sec2 x dx = tan x
x=0

sin2 d, but he found two facts: One


0 Z
2

fact is sin2 + cos2 1, which implies


(sin2 + cos2 ) d = . The other
2
0


fact is cos = sin
, so by the Substitution Rule and concept of dummy
2
Z
Z
Z
Z


2
2
2
2
2
2
2
cos d =
sin
sin d =
sin2 d.
d =
variable,
2
0
0
0
Z 0
2

Thus
sin2 d = . Is this argument correct?
4
0

(T) 10. Student A forgot how to integrate

1
=
ba

(T) 11. If f (x) is continuous on [a, b], then L = fave


f (x) dx achieves the
a
Z b
absolute minimum of the function F (L) =
(f (x) L)2 dx.
a

Problem 12 (17pts). Write down the integral formula for the area of enclosed
region. No absolute value in your formula.
(a) The region bounded by the curves y = sin x, y = cos x, x = 0, and x = 2 .
 
(2pts) Two curves y = sin x and y = cos x intersect at (x, y) = 4 , 22 .
Z
Z
4
2
(cos x sin x) dx +
(sin x cos x) dx.
(7pts) Area =

(b) The region enclosed by x = y 2 4y and x = 2y y 2 .


(3pts) Two curves intersect at (x, y) = (0, 0) and (3, 3).
Z 3
Z 3
2
2
(5pts) Area =
(2y y y + 4y) dy =
(2y 2 + 6y) dy.
0

Problem 13 (14pts). Write down the formula for the volume of solids of revolution:
(a) (7pts) The region bounded by f (x) and g(x) (f (x) > g(x) 0), x = a, x = b
about the x-axis.
Z b
Volume =
((f (x))2 (g(x))2 ) dx.
a

(b) (7pts) The region bounded by y = f (x) 0, y = 0, x = a, x = b, b > a 0


about x = 1 by the method of cylindrical shells.
Z b
Volume =
2(x + 1)f (x) dx.
a

Problem 14 (14pts). Evaluate

1
2

sin1 x dx. (Hint: Integration by parts first.)

Solution.
Z 1
Z
x= 1
2
2
1
1
sin x dx = x sin x

=
4 2

=
4 2

Z 1
2
1
x d sin x =
x d sin1 x
4
x=0
2
0
0
Z 1
Z 1
2
2
1
x
1

dx = +
d(1 x2 )
2
2
2
1

x
1

x
4 2
0
0
1

1 x= 2
1
= + 1.
+ (1 x2 ) 2
x=0
4 2
2
1
2

Quiz 3
1.

,
, f (x) = 1, g(x) = 1,
b a, (b a)2
:
(Cauchy inequality) ,
: ~u, ~v R3 , |h~u, ~v i| k~ukk~v k

11 29 , C[a, b],
R3 , C[a, b]
, , ,
h~u, ~ui 0, ~u = 0, ha~u + b~v , wi
~ = ah~u, wi
~ + bh~v , wi
~

, C[a, b] ,

, C[a, b] ? :
Z b
Z b
 12 Z b
 21


2
2

f (x)g(x) dx
(f (x)) dx
(g(x)) dx .

a

2. , ,

, () ,
(, ,
); () ,
, ,
, ,
, x = 0 (
)
, ,
,
3. 3 , (..:P) 3
5 (Fundamental Theorem of Calculus),
3

, ,

f (t) = [[t]] (jump


discontinuity), f (t) , , :
F (x) =

f (t) dt =
0

[[x]]1 Z k+1
X
k=0

[[t]] dt +

[[t]] dt =

[[x]]

[[x]]1 Z k+1
X
k=0

k dt +

[[x]] dt
[[x]]

[[x]]1
X
X   t=k+1   t=x
k + [[x]](x [[x]])
=
+ t
kt
=
[[x]]1

t=k

t=[[x]]

k=0

k=0

([[x]] 1)[[x]]
[[x]]
1
+ +[[x]](x [[x]]) =
+ x[[x]] ([[x]])2 .
2
2
2

F (x) ()
F (x) , ,

4. , ,

x, x,
x , x
, sense
R x2
, F (x) = 0 (x2 t) dt 0, F (x) C,

C , F (0) = 0 ( ), F (x) 0,
F (x) , 0,
0,
5.

:


 t=x

d 1 2
d 1 2
=
sin t d sin t =
sin t
sin x = sin x cos x.
dx 2
dx 2
0
t=0
Z x
Z x
d
d
sin t d sin t =
sin t cos t dt = sin x cos x.
(b)
dx 0
dx 0
d
(a)
dx

(c) sense , , ,

differential,
6. Net Change Theorem,

7. , f (x), g(x) ,

; : x [a, b],
Z x
Z x
Z x

g(x) f (x) =
g (t) dt
f (t) dt =
(g (t) f (t)) dt 0,
a

g(x) f (x) ( g (t) f (t) [a, b]


)
, ,
()
, () ,
, () (
) , , ,
( g(x)) ( f (x))

8. (substitution rule) (dummy variable) y = 1 x, x = 1 y,

dy = dx, y = 0 y = 1,
Z 1
Z 0
Z 1
f (x)g(1 x) dx =
f (1 y)g(y) dy =
f (1 y)g(y) dy
0
1
0
Z 1
=
f (1 x)g(x) dx.
0

:
(a) : f (x) F (x), g(x) G(x),
f (x)g(x) F (x)G(x),
( (F (x)G(x)) = F (x)G(x) + F (x)G (x) = f (x)G(x) + F (x)g(x))
(b) (convolution) : f (x), g(x)
Z
(f g)(t) =
f (x)g(t x) dx ( 7-8 )

Z
Z
(f g)(t) =
f (x)g(t x) dx =

f (t x)g(x) dx = (g f )(t),

9. sec2 x x =

(infinity discontinuity),
R
x1 dx ?
5

R
ln |x| + C, : x > 0 , x1 dx = ln |x| + C1 , x < 0
R
, x1 dx = ln |x| + C2 , C1 C2

() 7-8

tan x sec2 x dx,

1
tan2 x + C.
2
Z
Z
1
(b)
tan x sec2 x dx = sec x d sec x = sec2 x + C.
2
(a)

tan x sec x dx =

tan x d tan x =

? , , (a) (b)
C C sec2 x = tan2 x + 1, C 12

sense ( sense
, , ,
, )
, , sec2 x > 0, () ,
0
10. ...XD Student A

( sense) (),
R
R
: 02 sin2 , d 02 cos2 , d
, (), ,

: , :
r
r 2 x2 dx, ,

, 12 r2

11. (),
12, 13. , :
(a) , 12 , sin x cos x

x ,
(b) , x y , (x = c y = c

) 8
6

(c) , x y ; ,

,
(d) !

,
; , ,
1 2 ()
!
8 10 ,
, , ,

, ,
14. integration by parts ;

, ( 1 x2 x = sin )

: x = sin , dx = cos d, =
Z

1
2

dx =
1 x2

sin

= 0,
Z

cos d =
1 sin2

 = 4
1
= + 1.
= cos
=0
2
0

, !

sin d

Calculus A (1) Quiz 4, 2014/12/24 ANSWER


Department:

I.D.:

Name:

17. True-False (T)-(F).

810 Multiple Choice (a), (b), (c), or (d).


R
R
(T) 1. If f is continuous on [0, ) and 1 f (x) dx is convergent, then 0 f (x) dx is
convergent.

(F) 2. Suppose f, g are continuous on [0, ) and f (x) g(x). If


R
convergent, then 0 g(x) dx is convergent.

f (x) dx is

(F) 3. If f is a continuous and decreasing function on [0, ) and lim f (x) = 0, then
x
R
f
(x)
dx
is
convergent.
0
(T) 4. If f (x) is continuous on [0, ) and lim f (x) = 0, then
x

(F) 5.
(F) 6.
(T) 7.

R
0

sin x
x

R
1

dx is not integrable.

x dx = lim

t t

1
x

Rt

x dx = lim
t

dx is convergent.

(c) 8. Student A computed


Student B computed

1
2

 x=t
x2 x=t =

tan sec2 d =
tan sec2 d =

1
lim (t2
2 t

R
0

f (x) dx = f (0).

(t)2 ) = 0.

tan d tan = 21 tan2 + C.


sec d sec =

1
2

sec2 + C.

(a) Only Student A is correct. (b) Only Student B is correct.


(c) Both Students are correct. (d) Both Students are incorrect.
(c) 9. Consider a curve of the form (x, f (x)), where an increasing function y = f (x)
on [a, b], and its inverse function is called x = g(y) with range [c, d]. Student A
Rbp
said the length of the curve is a 1 + (f (x))2 dx. Student B said the length
Rdp
of the curve is c 1 + (g (y))2 dy.

(a) Only Student A is correct. (b) Only Student B is correct.

(c) Both Students are correct. (d) Both Students are incorrect.
(a) 10. Consider a curve of the form (x(t), y(t)), Student A said
Student B generalized Student As idea and get

d2 y
dx2

dy
dx

d2 y
dt2
d2 x
dt2

dy
dt
dx
dt

(a) Only Student A is correct. (b) Only Student B is correct.


(c) Both Students are correct. (d) Both Students are incorrect.
1

Problem 11 (5pts). What is the partial fraction assumption of the following


integration, where P (x) is a polynomial of degree 9?
Z
P (x)
dx
4
2
2
(x x )(x + 1)(x2 x + 2)2

Z 
C1 C2
C3
C4
C5 x + C6
C7 x + C8
C9 x + C10
+ 2 +
+
+
+ 2
+
dx.
x
x
x+1 x1
x2 + 1
x x + 2 (x2 x + 1)2
Problem 12 (15pts). Write down the integral formula for the length of given
curves.
(a) The length of the arc of the hyperbola xy = 1 from (1, 1) to (2, 21 ).
Z 2r
Z 1r
1
1
1 + 4 dx =
1 + 4 dy.
Length =
1
x
y
1
2
(b) The circumference () of an ellipse x = 2 cos , y = sin .
Z 2 p
Circumference =
4 sin2 + cos2 d.
0

(c) The total length of four-leaved rose r = cos 2.


Z 2 p
Total length =
4 sin2 2 + cos2 2 d.
0

Problem 13 (4pts+6pts). Write down the integral formula for the enclosed area.
The region lies inside r = 3 sin (circle) and outside r = 1 + sin (cardioid).


Two curves intersect at (0, 0), 23 , 6 , and 23 , 65 .
Enclosed area =

1
1
(3 sin )2 (1 + sin )2
2
2

d.

Problem 14 (10pts). The area of the parabola y = x2 from (1, 1) to (2, 4) is


rotated about the y-axis. Write down the integral formula of the surface area of
the resulting surface.
Surface Area =

2x 1 + 4x2 dx =

2 y

1+

1
dy.
4y

Problem 15 (10pts). Write down the integral formula for the centroid of the

region bounded by y = x and y = x.

x x) dx
x(
x = R0 1
,
( x x) dx
0

R1

R1

1
x
2


12 x2 dx

y = 0R 1
.
( x x) dx
0
2

Calculus A (2) Quiz 1, 2014/3/18


Department:

I.D.:

19 True-False (T)-(F).

Name:

1011 Multiple Choice (a), (b), (c), or (d).

(T) 1. lim |an | = 0 if and only if lim an = 0.


n

(F) 2. 0.9 = 0.999999 . . . < 1.


def.

(F) 3. If a sequence {an }


n=1 is increasing and bounded below, then the limit exists.

P
P
(T) 4. A series
an is convergent if and only if
an is convergent.
n=1

n=100

(T) 5. If lim an 6= 0 or lim an does not exist, then


n

an is divergent.

n=1

(F) 6. If {an }
n=1 and {bn }n=1 are divergent, then {an + bn }n=1 is divergent.

P
n

P
P
P
P
P
(T) 7. If
an and
bn are absolutely convergent,
ak bnk =
an
bn .
n=0

n=0

n=0 k=0

n=0

n=0

1
n

{an }
n=1

(F) 8. Consider a sequence


satisfying an+1 = (1)an + and a1 = 1.
Student A gives the following argument: Let lim an = L, then
n
1
L = L + 0 L = 0.
lim an+1 = (1) lim an + lim
n
n
n n
Is this argument correct?
(F) 9. Consider a series

1
n2

ln

n=2

. Student B gives the following argument: Since

ln x x for all x > 0, we know ln

1
n2

1
.
n2

Because

1
n2

is convergent, by

n=1


P
ln n12 is also convergent. Is this argument correct?
the Comparison Test,
n=2

P
(d) 10. If a series
an is convergent, then
n=1

(a)

(c)

n=1

|an | is convergent

(b)

(1)n an is convergent.

n=1

an an+1 is convergent

(d)

n=1

an +an+1
2

is convergent.

n=1

(b) 11. Consider a series

an with an > 0.

n=1

(a) If
(b) If

an+1
an
an+1
an

< 1, then

> 1, then

n=1

an is convergent.
an is divergent.

n=1

(c) If

an+1
an

= 1, then the convergence of

n=1

(d) All above are correct.


1

an is inconclusive.

Blank filling. ()
Problem 12 (5pts+5pts).

P
1
is convergent if p > 1 and divergent if p 1.
(1) p-series
np
n=1

P
(2) Geometric series
r n1 is convergent if |r| < 1 and divergent if |r| 1.
n=1

Problem 13 (5pts). We say two sequences {an }


n=1 and {bn }n=1 have order an bn
if lim abnn = 0. List the correct order of the following sequences:
n

{n2 }
n=1

{n!}
n=1

ln n

{1}
n=1
n2

{ln n}
n=1

2n

{2n }
n=1

n!

{nn }
n=1

nn

The Integral Test (5pts). Suppose that f (x) is a continuous, positive, decreasing

P
function on [1, ) and let an = f (n). Then the series
an is convergent if and
n=1
R
only if the improper integral 1 f (x) dx is convergent.

P
P
The Comparison Test (5pts). Suppose that
an and
bn are series with
n=1

n=1

positive terms and an bn for all n.

P
P
an is also convergent.
bn is convergent, then
(a) If
(b) If

n=1

n=1

an is divergent, then

bn is also divergent.

n=1

n=1

The Limit Comparison Test (5pts). Suppose that

an and

n=1

with positive terms. If lim

an
n bn

bn are series

n=1

= c, where c is a finite number and c > 0, then either

both series converge or both diverge.


Alternating Series Test (5pts). If the alternating series

(1)n bn , where bn > 0,

n=1

satisfies bn+1 bn and lim bn = 0, then the series is convergent.


n

The Ratio Test (5pts).



P
an+1
an is (absolutely) convergent.
(a) If lim an = L < 1, then the series
n


n=1

P
an+1


or
lim
then
the
series
an is divergent.
=
L
>
1
=
,
(b) If lim an+1



n an
n an
n=1




(c) If lim an+1
= 1, the Ratio Test is inconclusive.
an
n

The Root Test (5pts).

p
P
(a) If lim n |an | = L < 1, then the series
an is (absolutely) convergent.
n
n=1

p
p
P
an is divergent.
(b) If lim n |an | = L > 1 or lim n |an | = , then the series
n
n
n=1
p
(c) If lim n |an | = 1, the Root Test is inconclusive.
n

Quiz 1
1. : f (x) = |x| , lim an =
n




0, lim |an | = lim an = |0| = 0 , |an | an |an |,
n

lim |an | = 0, lim an = 0

lim an = 0 , an ,
n

, ,
lim |an | = 0 lim an = 0, f (x)
n
n
(n)
P
f (a)
n

(xa) , : lim Rn (x) =


n!
n

n=1

0, |Rn (x)| ,

11.10 ex sin x
,

bn = lim Sn = lim
n

n=1

|bn | = lim Sn = lim


n

n=1

n
X

k=1
n
X

bk
|bk |,

k=1



n

X


bk ,
|Sn | =

Sn =

k=1

n
X

|bk |

k=1

2. ? ? ?

, ,
 
n
1 n
9
X
1

9
10
0.9 = 0.9(9 ) = lim
= lim 10
1
k
n
n
10
1

10
k=1

 n 
 n
1
1
= lim 1
= lim 1 lim
= 1 0 = 1
n
n
n 10
10

3. , (Monotonic Sequence Theorem) :

, :
;
3

{an }
n=1 , a1 , a1
{an }
n=1 ,
4. {an }
n=1

an , : an ,

n=1

P
P
, sn =
ak ,
an lim sn ;
n

n=1

k=1

an lim (sn s99 ) , :


n

n=100

lim sn = lim (sn s99 + s99 ) = lim (sn s99 ) + s99 ,


n

()
5.

an : lim an = 0, L =
n

n=1

an = lim sn ,

n=1

lim an = lim sn sn1 = lim sn lim sn1 = L L = 0


n

, ,
(Test of Divergence)
6. , ; ,

P
P
1

n1 ,
n
n=1

n=1

0 = 0

n=1

7. , ,

,
, ,
,

? 11.10
2

1 + x + x2! + x3! + 1 x
ex 1 x
= lim
lim
n
n
x2
x2
2
3
1 1 + x x + x2! + x3! +
= lim
n
x2

1
x
1
= lim
+ + = ,
n 2
3!
2

(1)

(1) , 1 x ,
, ex , (1)
4

8. ,

,
, , ,
( a2n+1 1 a2n 1), A
, L,
9. , :


P
B ,
ln n12
n=1
, , lim ln n12 = ,
n


P
(Test of Divergence)
ln n12
n=1

10. , : p-

() , :
(A)
(B)

n=1

(1)n
n
(1)n
n2

:
:

n=1

n=1

(1)n
n
(1)n
n2

,
,

n=1

n=1

1
n

1
n2

n=1

(a) (b) (,
(Ratio Test) (Root Test) 1
)
(d) ,

an ,

n=1

an+1 , :

n=1

X
an + an+1

n=1

1X
1X
an +
an+1
2 n=1
2 n=1

(c) , , ,
, , n1p , p 1

P
P
(1)n
, , 1
an =

n
n
n=1

n=1

0,

an

n=1

an an+1

n=1

n=2

1
n

X
X
X
(1)n (1)n+1 X
1
1
1

p
=
=
,
=

n
+
1
n
n
n
+
1
n(n
+
1)
n=1
n=2
n=1
n=1

an an+1

n=1

11. (Ratio Test), (d);

, , lim
n

,
lim
n

an+1
an

, (), ,

P
(common ratio) (c) , 1 ,
an = lim n a1
n=1

( an > 0); (b),

an = lim
n

n=1

n
X

ak > lim
n

n
X

an+1
an

> 1,

a1 = lim n a1
n

k=1

k=1

(b)
(a) , :

1
n

n=1
an+1
an

n
n+1

< 1

, ,

, ,

,
, (Ratio Test)


p
an+1
(Root Test) (b), lim an lim n |an |
n

? : ,

, , ,

P
(2+(1)n )n

, :
4n
n=1

(2 + 1)n
(2 + (1)n )n

=
an =
4n
4n

bn

3
4

 n
3

= bn
4

, (

n=1

(Comparison Test)

an

n=1

3
4

1 34

= 3)

, (Ratio Test)

an ,

n=1



an+1
(2 + (1)n+1 )n+1
1 (2 + (1)n+1 )n+1
4n


=
lim
=
lim
lim
n
n an
4n+1
(2 + (1)n )n n 4 (2 + (1)n )n

lim 1 3n+1 n
n 4
=
,
lim 1 1n
n
4 3
n





lim an+1
( )
an
n

(Limit Comparison Theorem),

lim an = lim bn = c , ,
n

lim an = lim bn = 0, ,
n

Calculus A (2) Quiz 2, 2015/4/8 ANSWER


Department:

I.D.:

Name:

17 True-False (T)-(F).
(T) 1.
(F) 2.

n=0

(1)n
n!

= 1e .

(1)n1 2n
n

= ln(1 + 2) = ln 3.

n=1

(F) 3. If f (x) has continuous derivatives of all orders, then f (x) =

f (n) (0) n
x
n!

in the

n=0

interval of convergence.
(F) 4. If |r(t)| = 1 for all t, then |r (t)| is a constant.

(T) 5. If |r(t)| = 1 for all t, then r (t) is orthogonal to r(t) for all t.
(F) 6. If T(t) is the unit tangent vector of a smooth curve, then the curvature is

.
(t) = dT
dt
(F) 7. Consider the limit

lim

(x,y)(0,0)

f (x, y) =

xy 2
2 +y 4 .
x
(x,y)(0,0)

lim

Student A gives the

following argument: We take the path Cm (x) = (x, y = mx), x 6= 0, then


f (x, y)|Cm(x) =

x(mx)2
m2 x3
m2 x
=
=
0 as x 0.
x2 + (mx)4
x2 + m4 x4
1 + m4 x2

We take vertical path C (y) = (x = 0, y), y 6= 0, then f (x, y)|C (y) 0. Since
2
the limit tends to 0 along each direction, we get
lim x2xy+y4 = 0. Is this
(x,y)(0,0)

argument correct?

Problem 8 (15pts). Compute a space curve r(t) = t i + 22 t2 j + 13 t3 k as follows:

r (t) = 1 i + 2t j + t2 k
|r (t)| = t2 + 1

r (t) = 0 i + 2 j + 2t k

|r (t) r (t)| = 2(t2 + 1)


r (t) r (t) = 2t2 i 2t j + 2 k

2t
1
t2
r (t)
= 2
i+ 2
j+ 2
k
T(t) =
|r (t)|
t +1
t +1
t +1

2t
2(1 t2 )
2t

i
+
j+ 2
k
T (t) = 2
2
2
2
(t + 1)
(t + 1)
(t + 1)2
So the curvature (t) =

(t2 +1)2

T(1) =

1
2

N(1) =
B(1) =
1

i+

1
2

2
2

2
2

j+

j+

i+
i+

2
2

j+

1
2

k,

2
2

k,

1
2

k.

Problem 9 ((4 + 1) 6 = 30 pts). Write down the Maclaurin series and the radius
of convergence of the following functions.

X
1
=
xn
(1)
1 x n=0
(2) ln(1 + x) =

R=1

X
(1)n1 xn

n=1

(3) tan

x=

(4) e =

R=1

n+1

n=0

X
(1)n x2n+1

R=1

2n + 1

n=0

X
(1)n xn+1

X
xn
n=0

(5) sin x =

X
(1)n x2n+1
n=0

(6) cos x =

R=

n!

R=

(2n + 1)!

X
(1)n x2n
n=0

R=

(2n)!

Problem 10 (3 + 4 + 5 + 1 + 5 + 1 + 1 pts). To find the Maclaurin series of sin1 x,


first we know that
d
1
sin1 x =
,
dx
1 x2
21

Define C0
we get

= 1 and Cn 2 =

( 21 )( 32 )( 21 (n1))
n!

for n N. By the binomial series,

X 1
X
1
d
1
sin1 x = (1 + (x2 )) 2 =
(1)n Cn 2 x2n ,
Cn 2 (x2 )n =
dx
n=0
n=0
where the radius of convergence is R = 1. We can integrate the power series terms
by terms to get
1

sin1 x = C +

X
(1)n Cn 2
n=0

2n + 1

x2n+1 ,

where the radius of convergence is R = 1. Finally, since sin1 0 = 0, we get the


integral constant is 0.

Quiz 2
1. f (x) = ex ( Maclaurin series) , 14 (3):
e =
x

X
xn
n=0

n!

R, R = , lim Rn (x) = 0,
n

; x = 1
:
, ,
? ?
(derangement), :
= 1 + 2 3 + 4 5
= C05 5! C15 4! + C25 3! C35 2! + C45 1! C55 0!
= 120 120 + 60 20 + 5 1 = 44,

P5 =

44
11
=
120
30

(inclusion-exclusion principle) m
m , :
m =

m
X

(1)n Cnm (m n)!

n=0

m
X

X (1)n m!
m!
(m n)! =
,
=
(1)
n!(m

n)!
n!
n=0
n=0
n

m
Pm =

m
P

(1)n m!
n!

n=0

m!

m
X
(1)n
n=0

n!

m , ; lim Pm = 1e
m

1e , , ,
? , ,
:
3

http://pansci.tw/archives/73913
http://en.wikipedia.org/wiki/Secretary_problem

, ,

2. f (x) = ln(1 + x) , 14

(2):
ln(1 + x) =

X
(1)n1 xn

n=1

X
(1)n xn+1

n+1

n=0

x (1, 1], R = 1, ,
lim Rn (x) = 0, (1, 1]
n

, x = 2 ( (Ratio Test) ),
ln 3

f (x) = ln(1 + x) x = 1 ,
(1)n1
1 1 1 1 1 1
+ + ++
+ = ln 2,
1 2 3 4 5 6
n

3. C (R) C (R) ; 11.11


Appendix , :

e x12
f (x) =
0

x 6= 0
x = 0,

f (x) , f (n) (0) 0, f (x)


() : T (x) 0, T (x) = 0 R (
R), f (x) x 6= 0 , f (x) 6= T (x) x 6= 0
(R > 0) ,

P
Cnm xn , 11.10
9 (1 + x)m =
n=0

4. |r(t)| = 1 : r(t) S2

, , (),
()
, ,
:
r(t) = (cos(t3 ), sin(t3 ), 0)

r(t) = (cos t, sin t, 0),

, r (t) = (3t2 sin(t3 ), 3t2 cos(t3 ), 0) |r (t)| = 3t2


,
, (gauge invariance),
(diffeomorphism), , ,
, ; , ,
, ,
,
s , ,
, ,
, , , T, N, B

r(t) T(t), t s
, ,

5. , :
|r(t)| = 1 |r(t)|2 = r(t) r(t) = 1

d
(r(t) r(t)) = 2 r(t) r(t) = 0
dt

T, N, B, ,

6. ;

; ,

, , = dT ,
ds

r(t) ,


dT dT dt dT
=
= dt = |T (t)| ,
(t) =
ds
ds dt ds
|r (t)|
dt

|T (t)| |r (t)|
7. ,

, , C(y) =
(y 2 , y), y 6= 0,
f (x, y)|C(y) =

1
y2 y2

,
(y 2)2 + y 4
2

(0, 0) 12
,
;
(Squeeze Theorem), x = r cos , y = r sin ,
r (order), ,
(bounded),
8. (t) T, N, B , ,

( 12.4)
|r (t) r (t)|
|T (t)|
=
,
|r (t)|
|r (t)|3
T (t)
N(t) =
,
B(t) = T(t) N(t)
|T (t)|

(t) =
T(t) =

r (t)
,
|r(t)|

9-10. , (1 + x)m =

Cnm xn , R = 1

n=0

, , 10 sin1 x
cos1 x
, q: sec x ( 4 2 ), tan x (11.10
2
( 3 5 ), tan1 x ( 3 8
), cos 2x ( 3 4,7 ), ln 1+x
1x2
)

:
http://mathworld.wolfram.com/MaclaurinSeries.html

Calculus A (2) Quiz 4, 2015/6/3 ANSWER


Problem 1 (4%+16%). Consider the double integral

ZZ

(x y)ex

2 y 2

dA, where

R = {(x, y)||x 1| + |y 1| 1}.


(a) Let u = x y and v = x + y.
(b) The double integral can be changed as

u=1

u=1

v=3

v=1

1 uv
ue dv du.
2

(, , )
Z
z2
p
Problem 2 (6%+6%+6%+2%). Compute the line integral
ds, where
2x2 + y 2
C
C is the intersection of the circular cylinder x2 + y 2 = 1 and the plane z = y.
(a) Write down the parametric equations of C: r(t) = cos t i + sin t j + sin t k.

(b) From (a), we know the arc-length ds = sin2 t + 2 cos2 t dt.


Z t=2
p
sin2 t

sin2 +2 cos2 t dt.


(c) The line integral can be changed as
2
2
2 cos t + sin t
t=0
(d) The value of the line integral is .
Problem 3 (5%+15%). Find the work done by the force field
F(x, y) = (x y) i + xy j
in moving a particle along the quarter-circle x2 + y 2 = 4 from (2, 0) to (0, 2).
(a) The parametric equations of the curve: r(t) = 2 cos t i + 2 sin t j, 2 t 0.
(b) The work is

(2(cos t sin t)(2 sin t) + 8 cos t sin t cos t) dt.

(, , )
Problem 4 (15%+5%).
(a) Find the potential function of the vector field
F(x, y, z) = (y + z sin(xz)) i + (x + z eyz ) j + (x sin(xz) + y eyz ) k.
()
Z

(b) Compute
F dr, where C is any path from (0, 0, 0) to 1, 2, 2 .
C

Solution.
R
(a) fx = y + z sin(xz) f (x, y, z) = (y + z sin(xz)) dx = xy cos(xz) + g(y, z).
R
Next, fy = x + gy = x + z eyz gy = z eyz g(y, z) = z eyz dy = eyz + h(z).
Finally, fz = x sin(xz) + y eyz + h (z) = x sin(xz) + y eyz h (z) = 0 h(z) =
C. So the potential function of F is f (x, y, z) = xy cos(xz) + eyz + C.

(b) By the Fundamental Theorem of Calculus, we know


Z

h
i (1,2, 2 )

= (2 0 + e ) (0 1 + 1) = 2 + e .
F dr = f (x, y, z)
(0,0,0)

Problem 5 (8%+12%). Compute the line integral



I
I 


1 2
3
y 2
P (x, y) dx + Q(x, y) dy =
cos(x ) + y dx + xy + xe
dy,
2
C
C

where C is the triangle region with vertices (0, 0), (1, 1), and (0, 1). It is impossible
to calculate the line integral directly, so we will use Greens Theorem. Greens
Theorem tells us

so we get

ZZ 
P

(A)

Z ZD  x

(B)

Z ZD  x
I

P
(C)
P (x, y) dx + Q(x, y) dy =

Z ZD  y

(D)

Z ZD  x

(E)
y
D

y=1

y=0

x=y


Q
+
y 
Q

y 
Q

x 
P

y 
P

dA
dA
dA ,
dA
dA

ey dx dy.

x=0

(, , , )

Calculus A (2) Quiz 4, 2015/6/3 ANSWER


Problem 1. Evaluate

ZZ

(x y)ex

2 y 2

dA, where R = {(x, y)||x 1| + |y 1| 1}.

Solution. Let u = x y and v = x + y, then we compute


"
#





(u, v)
(x, y) 1
1
1





(x, y) = det 1 1 = 2 (u, v) = 2 .

The region R will be mapped to S = {(u, v)| 1 u 1, 1 v 3}, so we have


Z

u=1

u=1

v=3

v=1

Z
h i v=3
1 u=1 3u
uv
du =
e
(e eu ) du
v=1
2
u=1
u=1

 u=1



1 1 3
1 1 3u
u
3
1
=
=
e e
(e e ) (e e ) .
2 3
2 3
u=1

1
1 uv
ue dv du =
2
2

u=1

Problem 2. Compute the line integral

z2

ds, where C is the intersection


2x2 + y 2
of the circular cylinder x2 + y 2 = 1 and the plane z = y.
C

Solution. We first parameterize the curve C: r(t) = cos t i + sin t j + sin t k, where
0 t < 2. Then we get the arc-length
ds =
=

(x (t))2 + (y (t))2 + (z (t))2 dt =


sin2 t + 2 cos2 t dt.

p
( sin t)2 + (cos t)2 + (cos t)2 dt

So the line integral is


Z
Z 2
p
z2
sin2 t
p

ds =
sin2 +2 cos2 t dt
2
2
2
2
2x + y
2 cos t + sin t
C
0


 2
Z 2
Z 2 
1
sin 2t
1 + cos 2t
2
= .
dt =
t+
=
sin t dt =
2
2
4 0
0
0

Problem 3. Find the work done by the force field


F(x, y) = (x y) i + xy j
in moving a particle along the quarter-circle x2 + y 2 = 4 from (2, 0) to (0, 2).
Solution. First we parameterize the curve C by r(t) = 2 cos t i+2 sin t j, 2 t 0.
So the work is
Z
2
W =
(2(cos t sin t)(2 sin t) + 8 cos t sin t cos t) dt
0
Z
2
=
(4 cos t sin t + 4 sin2 t + 8 cos2 t sin t) dt
0

Z
Z
Z 
2
2
2
1 cos 2t
dt
=
4 cos t d cos t +
4
8 cos2 t d sin t
2
0
0
0
 t= 2


8
= 2 cos2 t + 2t sin 2t cos3 t
3
t=0


 8

2
= 2 (0 1) + 2 0 (0 1) = + .
2
3
3
Problem 4.
(a) Find the potential function of the vector field
F(x, y, z) = (y + z sin(xz)) i + (x + z eyz ) j + (x sin(xz) + y eyz ) k.
(b) Compute

Solution.


F dr, where C is any path from (0, 0, 0) to 1, 2, 2 .

(a) We will find the smooth function f (x, y, z) such that f = (fx , fy , fz ) = F.
From the first equation fx = y + z sin(xz), we get
Z
f (x, y, z) = (y + z sin(xz)) dx = xy cos(xz) + g(y, z).
Next, we compute
fy = x + gy = x + z e

yz

gy = z e

yz

g(y, z) =

z eyz dy = eyz + h(z).

Finally, we calculate
fz = x sin(xz) + y eyz + h (z) = x sin(xz) + y eyz h (z) = 0 h(z) = C.
So the potential function of F is f (x, y, z) = xy cos(xz) + eyz + C.
2

(b) By the Fundamental Theorem of Calculus, we know


Z
h
i (1,2, 2 )

= (2 0 + e ) (0 1 + 1) = 2 + e .
F dr = f (x, y, z)
(0,0,0)

Problem 5. Compute the line integral



I
I 


1 2
3
y 2
P (x, y) dx + Q(x, y) dy =
cos(x ) + y dx + xy + xe
dy,
2
C
C
where C is the triangle region with vertices (0, 0), (1, 1), and (0, 1).

Solution. By Greens Theorem,



Z y=1 Z x=y
I
ZZ 
Q P
2
dA =

ey dx dy
P (x, y) dx + Q(x, y) dy =
x
y
y=0
x=0
C
D
Z 1
Z y=1 h
i
x=y
2
2
dy =
=
yey dy
xey
x=0
0
y=0
Z 1

1 h y2 i y=1 1
1
y 2
2
=
1 e1 .
e d(y ) = e
=

2 0
2
2
y=0

Problem 6. Find the area of the region R bounded by xy = 4, xy = 8, xy 3 = 5,


and xy 3 = 15 in the first quadrant.

Solution. Let u = xy and v = xy 3 . We first compute


!





(u, v)
(x, y)
y
x
1
1

3




(x, y) = det y 3 3xy 2 = 2xy (u, v) = 2xy 3 = 2v .
So the area is
Z u=8 Z
Area =
u=4

v=15

v=5

1
dv du =
2v

u=8

u=4

 v=15
h i u=8

1
1

du = ln 3 u
= 2 ln 3.
ln |v|
2
2
u=4
v=5

Solution 2. Let u = xy and v = y 2 , then xy 3 = uv. We first compute


!





(x, y)
(u, v)
y
x

2

= det
= 1 = 1.

= 2y
(x, y)
(u, v) 2y 2
2v
0 2y
So the area is

Area =

u=8
u=4
u=8
u=4

 v= 15u
Z u=8 

1
1
du
dv du =
ln |v|
5
2v
2
5
v= u
u=4
v= u
h i u=8
1
1

= 2 ln 3.
ln 3 du = ln 3 u
u=4
2
2

v= 15
u

Problem 7. Compute

Z p

x2 + y 2 ds, where C is a curve satisfying x2 + y 2 = 2y.

Solution. Let x = cos t, y = 1 + sin t, 0 t < 2. Then


p
p
ds = (x (t))2 + (y (t))2 dt = ( sin t)2 + (cos t)2 dt = dt,

and

p
p

(x(t))2 + (y(t))2 = cos2 t + (1 + sin t)2 = 2 2 sin t.

So we have
Z p
Z
2
2
x + y ds =

r
Z 2
t
t
t
t
2 2 sin t dt = 2
sin2 + cos2 2 sin cos
2
2
2
2
0
0


Z
2



sin t cos t dt
= 2
2
2
0




Z 2
Z 2
t
t
t
t
sin cos
= 2
cos sin
dt + 2
dt

2
2
2
2
0
2

 2



t 2
t
t
t
= 2 2 sin + cos + 2 2 cos sin
2
2 0
2
2
 2

 

 

1
1
1
1
0 + 1 1
0
=2 2
2
2
2
2

= 8.
Solution 2. We first use the polar coordinates x = r cos , y = r sin to find the
parametric equation of the circle C:
r 2 cos2 + r 2 sin2 = 2r sin r(r + 2 sin ) = 0 r = 0 or r = 2 sin .
So the parametric equation of C is
y = (2 sin ) sin = 2 sin2 ,

x = (2 sin ) cos = sin 2,

0 .

We compute
p
p
(x ())2 + (y ())2 d = (2 cos 2)2 + (4 sin cos )2 d
p
= 2 cos2 2 + sin2 2 d = 2 d.

ds =

So

Z p
C

x2

y 2 ds

=2

q
( sin 2)2 + (2 sin2 )2 2 d

Z0 p
sin2 cos2 + sin4 d
=4
Z0
h
i =

= 8.
=4
| sin | d = cos
=0

Problem 8. Compute

xy dx, where C is the closed curve consisting of the upper

half circle (x 1)2 + y 2 = 1, y 0, and the x-axis.


Solution. Let C = C1 C2 , where C1 is 0 x 1, y = 0 and C2 is the upper half
circle (x 1)2 + y 2 = 1, y 0. We parameterize C1 by r(t) = t i + 0 j, 0 t 2,
R
then r (t) = 1 i + 0 j and get C1 xy dx = 0.
For C2 , we use the parameterize equation r(t) = (1 + cos t) i + sin t j, then r (t) =
( sin t) i + cos t j and
Z
Z
Z
xy dx =
(1 + cos t) sin t( sin t) dt =
sin2 t sin2 t cos t dt
C2
0
0

Z 
Z
1 cos 2t
=
dt
sin2 t d sin t
2
0
0
 


1 3

sin 2t
1

sin t = .
= t

2
4
3
2
0
0

Hence we get

xy dx =

xy dx +

xy dx = 0

C2

C1

= .
2
2

Solution 2. By Greens Theorem and let x = 1 + r cos , y = r sin , we have


I
ZZ
Z = Z r=1
xy dx =
(x) dA =
(1 + r cos )r dr d
C
D
=0
r=0
 r=1
Z = Z r=1
Z = 

1
1 3
2
=
(r r cos ) dr d =
r r cos
d
2
3
=0
r=0
=0
r=0



Z 

1
1 1
=
cos d = sin = .
2 3
2 3
2
0
0
Solution 3. By Greens Theorem, we have
I

= 2

r=2 cos

r cos r dr d
(x) dA =
=0
r=0
 r=2 cos

Z
Z
8 2
1 3
cos d =
=
r
cos4 d
3
3
=0
0
r=0
2

Z 
Z 
1 + cos 4
2 2
8 2 1 + cos 2
1 + 2 cos 2 +
d
d =
=
3 0
2
3 0
2



2 3
sin 4
= .
=
+ sin 2 +

3 2
8
2
0

xy dx =
C

ZZ

D
= 2

Problem 9. Find the potential function of the following vector field


F(x, y) = (y exy sin(xy) + y exy cos(xy)) i + (x exy sin(xy) + x exy cos(xy)) j.
Solution. We want to find a smooth function f such that
f
= y exy sin(xy) + y exy cos(xy) and
x

f
= x exy sin(xy) + x exy cos(xy).
y

First, we compute
Z
Z
Z
Z
xy
xy
xy
f (x, y) = y e sin(xy) dx + y e cos(xy) dx = sin(xy) de + cos(xy) dexy
i Z
i Z
h
h
xy
xy
xy
+ y exy sin(xy) dx + g(y)
y e cos(xy) dx + cos(xy)e
= sin(xy)e
i Z
i Z
h
h
xy
xy
xy
+ sin(xy) dexy + g(y)
cos(xy) de + cos(xy)e
= sin(xy)e
h
i h
i Z
xy
xy
= sin(xy)e
cos(xy)e
y exy sin(xy) dx + g(y)
i Z
i h
h
xy
xy
y exy cos(xy) dx + g(y)
+ sin(xy)e
+ cos(xy)e
h
i
= 2 sin(xy)exy f (x, y) + g(y).

So f (x, y) = sin(xy)exy + g(y). Next, we compute

f
(x, y) = x cos(xy)exy + x sin(xy)exy + g (y) = x exy sin(xy) + x exy cos(xy).
y
So we get g(y) = 0 g(y) = C. Hence the potential function is f (x, y) =
sin(xy)exy + C.
Z


Problem 10. Compute
F dr, where F = y + x2 + 1 i + (tan1 x) j and C is
C

the triangle from (0, 0), to (0, 1) to (1, 1) to (0, 0).

1
Solution. Let P = y + x2 + 1 and Q = tan1 x, so Qx = 1+x
2 and Py = 1. Notice
that the curve C is oriented clockwise, by Greens Theorem, we have

Z
Z
ZZ 
1
1 dA
F dr =
P dx + Q dy =
1 + x2
C
C
D

Z x=1 Z y=1 
1
=
1 dy dx
1 + x2
x=0
y=x

Z x=1 
x
1

1 + x dx
=
1 + x2 1 + x2
x=0
 x=1

1
1
1 2
1
1
2
= + ln 2 + .
= tan x ln |1 + x | x + x
2
2
4 2
2
x=0
6

Solution 2. Let P = y +

x2 + 1 and Q = tan1 x, so Qx =

1
1+x2

and Py = 1.

Notice that the curve C is oriented clockwise, by Greens Theorem, we have



Z
Z
ZZ 
1
F dr =
P dx + Q dy =
1 dA
1 + x2
C
C
D

Z y=1 Z x=y 
Z y=1 h
i x=y
1

1
dy
=

1
dx
dy
=

tan
x

x

2
x=0
1+x
y=0
y=0
x=0
 y=1

Z 1
h
i y=1 Z 1 y
1 2

1
1
dy +
y
+
=
( tan y + y) dy = y tan y
2
2
y=0
0 1+y
0
y=0
Z 1
h
i
y=1
1
1
1
1
1

= +
+
ln(1 + y 2 )
d(1 + y 2) = +
2
y=0
4 2 0 1+y
2
4 2
2
1
1
= + ln 2 + .
4 2
2
Solution 3. Notice that the closed curve is C = C1 C2 C3 , where
r1 (t) = 0 i + t j, 0 t 1,
r2 (t) = t i + 1 j, 0 t 1,
r3 (t) = 0 i + t j, 0 t 1,

0 + 1) i + tan1 0 j

r1 (t) = 1 i + 0 j, F(t) = (1 + t2 + 1) i + tan1 t j

r1 (t) = 1 i + 1 j, F(t) = (t + t2 + 1) i + tan1 t j.


r1 (t) = 0 i + 1 j, F(t) = (t +

So the line integral is


Z
Z
Z
Z
F dr3
F dr2 +
F dr1 +
F dr =
C3
C2
C
C1
Z 1
Z 1

1
2
2
(1 t tan1 t) dt
=
(1 + t + 1 t t + 1 tan t) dt =
0
0

 t=1 h
i t=1 Z 1  t 
1

1
= t t t tan t +
dt
2 t=0
1 + t2
t=0
0

 t=1

1 1
1
1
2
ln(1 + t ) = + ln 2.
= +
2 4
2
2 4 2
t=0

1.1

(x, y = f (x))

y = f (x), :
(1) y = f (x) + d : d
d ; d
(2) y = af (x) : y a
|a| > 1 y ; 0 < |a| < 1 y ; a x-
(3) y = f (x + c) : c
c ; c
(4) y = f (bx) : x b
|b| > 1 x ; 0 < |b| < 1 x ; b y-

a, b, c, d , ,
, y = x, y = x + 1, y = 2x;
y = x2 , y = x2 1, y = 2x2 , y = (x 1)2 , y = ( 12 x)2 , ,

,
, , : y = f (x),
(1) a , d , y = af (x) + d
(2) d , a , y = a(f (x) + d)
(3) b , c , y = f (b(x + c))
(4) c , b , y = f (bx + c)
(5) , ,
(6) c b a d a d c b, y = af (bx + c) + d
(7) b c a d a d b c, y = af (b(x + c)) + d
1

, , ,
1.3 , ,
1. sin ?
(a) cos( 2 ) (b) sin( + ) (c) cos( + 2 ) (d) sin( )

. :



cos
= cos cos + sin sin = sin
2
2
2
sin( + ) = (sin cos + cos sin ) = sin





= cos cos sin sin
= sin
cos +
2
2
2
sin( ) = sin() cos cos() sin = sin ,
sin () :

1
-6

-4

-2

-1

1: cos ()

-6

-4

-2

-1

()

2: sin () (), x- (a = 1, )

-6

-4

-2

3: cos ()

-6

-4

-2

-1

(), x- (a = 1, )

-1

4: sin () (), y- (b = 1, )

1.2

F (x, y) = 0

F (x, y) = 0 , :
(1) F (Ax, y) = 0 , 0 < |A| < 1 ; |A| > 1 ; y-
(2) F (x, Cy) = 0 , 0 < |C| < 1 ; |C| > 1 ; x-
(3) F (x + B, y) = 0 , B , B
(4) F (x, y + D) = 0 , D , D
(5) A B , F (A(x + B), y) = 0
(6) B , A , F (Ax + B, y) = 0
(7) , ,

, , ,
, : A, B, C, D
y = af (bx + c) + d b, c ; ,
a, d

2. F (x, y) = x2 + y 2 1 = 0 x- 3 , y-
2 , 1 , 2 ?
. 3 2, (1, 2) , :


1
(x 1)2 (y + 2)2
1
(x 1), (y + 2) =
+
1 = 0
F
3
2
32
22
2
1
-3

-2

-1

-1
-2
-3
-4

5: () () () () ()

1.3

() :
,
, () ,
, , ()
e1 = (1, 0) e2 = (0, 1), e1 =
(cos , sin ) e2 = ( sin , cos )
e2 = (0, 1)

e2 = ( sin , cos )

e1 = (cos , sin )

e1 = (1, 0)

6: e1 e1 ; e2 e2
e1 , e2 [
e1 |
e2 ] ,

cos sin
,
R =
sin cos

v = (v1 , v2 ) v :

v1
cos sin
v1
v1 cos v2 sin
= R (v) =

v =
v2
sin cos
v2
v1 sin + v2 cos

v = R (v) = R (v1 e1 + v2 e2 ) = v1 R (e1 ) + v2 R (e2 ) = v1 e1 + v2 e2


= v1 (cos e1 + sin e2 ) + v2 ( sin e1 + cos e2 )
= (v1 cos v2 sin )e1 + (v1 sin + v2 cos )e2

, ,
e1 , e2 , ,
, v = v1 e1 + v2 e2 , v1 , v2
4

3.

x2
32

y2
22

= 1

. : x(t) = 3 cos t, y(t) = 2 sin t, 0 t 2,


v(t) = (x(t), y(t)) , 4 v(t) = (
x(t), y(t)):

2
2

x(t) 2 y(t)
x(t)
x(t)
cos 4 sin 4

= 2

2
2
sin 4 cos 4
x(t) + 2 y(t)
y(t)
y(t)
2

3 2
cos t 2 sin t
2

3 2
cos
t
+
2
sin
t
2

(
x(t), y(t)) , :

x(t)
x(t)
cos 4 sin 4

sin 4 cos 4
y(t)
y(t)

2
2

x(t) + 2 y(t)
x(t)
cos 4 sin 4
x(t)
=

= 2
,

sin 4 cos 4
2 x(t) + 22 y(t)
y(t)
y(t)

x2
32

y2
22

= 1,


2
x
2

2

2
y
2

32


22 x +

2

2
y
2

22

= 1,

13
x2 10
xy + 13
y 2 72 = 0 ( 13x2 10xy + 13y 2 72 = 0)

2
1

-3

-2

-1

-1
-2

7:

x2
32

+ y22 = 1

13x2 10xy + 13y 2 72 = 0

1.4

(r = f (), )

r = f () , :
(1) r = f () + d : d ; d > 0 , d < 0
(2) r = af () : a
|a| > 1 ; 0 < |a| < 1 ; a < 0
(3) r = f ( + c) : c ; c > 0 , c < 0
(4) r = f (b) : b ()
|b| > 1 ; 0 < |b| < 1 ; b < 0

4. r = sin r = 1 + sin
. 8 (0, 12 )

1
2

r = sin

( y-) ,
: x = r cos , y = r sin ,
2  2

1
1
=

r = sin r = y x + y = y x + y
2
2
2

1.5

0.5

-1

-0.5

0.5

8: r = sin () r = 1 + sin ()
r = 1 + sin r = sin 1 ,
, , 1 () r = 1 + sin
0 , , 2 ,
( r < 0, ),
1 () 2
6

5. r = 1 + sin , r = 1 sin , r = 1 + cos , r = 1 cos

. sin = sin( ), cos = sin +

, cos = sin

, 1)

, (

(1) r = 1 sin r = 1 + sin () ()


(2) r = 1 + cos r = 1 + sin

()

(3) r = 1 cos r = 1 + sin

()

-2

-1

-1

-2

9: r = 1 + sin (); r = 1 sin (); r = 1 + cos (); r = 1 cos ()


6. r = sin , 0 r = sin 2, 0

. 10 , 0 , = 0 = ,
r = sin (); = 0 =
r = sin 2, 0

()

()

10: r = sin , 0 () r = sin 2, 0


r = f (b) , b
7

. x2 y 2 = 1

xy = 1

. 10.6 r =
ed
,
1+e sin

r=

ed
1e sin

ed
,
1+e cos

r=

ed
,
1e cos

r=

? ?

. , :
(1)

(x1)2
32

(y+2)2
22

= 1 6

(2) 13x2 10xy + 13y 2 72 = 0 6


(3) r = sin 2, 0

. , ?
. r =

1 + sin 2 ,

1 + sin 2 = 1 + 2 sin cos = sin2 + 2 sin cos + cos2




p


1
1
= (sin + cos )2 = | sin + cos | = 2 sin + cos
2
2


= 2 sin +

4

r=

. r = 1 + sin r = 2(1 + sin )

. r = 1 + 2 cos 2 ,
r = cos 2, r = 1 + 2 cos 2

2.1

, , :
.

(1) 1, 2, 3, 4, 5, 6, 7, 8, 9,

(2) 2, 4, 8, 16, 32, 64, 128, 256, 512,


(3) 1, 21 , 13 , 41 , 51 , 61 , 17 , 81 , 91 ,

(4) 1, 1, 2, 3, 5, 8, 13, 21, 34,


(5) 1, 4, 1, 4, 2, 1, 3, 5, 6,

(6) 0, 9, 5, 2, 3, 2, 7, 9, 5,


1
10, 1024, 10
, 55, ,

21
? , , , ,

, ,
, , 1 9,
10 ? 10 ,
A, , :
, A ! , A ? , 9
A ?
, ,
,
,

, ()
, ()
1

, , ,
, : 0, 1, 1, 0, 0, 1, 0, 1, 1, 1, . . .
, 0, 1, 0 1,
, () , , ,
,
, ,
n = 1, 2, 3, . . . , , ,2 :

(1) an = n, {an }
n=1 = {n}n=1
n
(2) bn = 2n , {bn }
n=1 = {2 }n=1

(3) c1 = c2 = 1, cn = cn1 + cn2 , n = 3, 4, . . .,


{cn }
n=1 = {c1 = c2 = 1, cn = cn1 + cn2 , n 3}
1
(4) dn = n1 , {dn }
n=1 = { n }n=1
n
(5) en = (1)n , {en }
n=1 = {(1) }n=1
n

(6) fn = (1)n n, {fn }


n=1 = {(1) n}n=1

, , ;
, , : {gn }5n=1 = {n}5n=1 ,

() , ?
, , {an }, {bn }, {cn }
, {dn } 0, {en } 1
1 ( 0 ), {fn } ,
0 , ,

( ), ,

2.2

, ,
, ,
, , :
2

0, 1, 2, 3, . . . , , 2, 3, 4, . . . ,

, , , :
1. {an }
n=1 ,
, : ,
(), ,
,
, , ,
, ,
,
,

,
, :
1
(1) {an }
n=1 = { n }n=1 , lim an = 0
n

(2) {bn }
n=1 = {
(3)

{cn }
n=1

(1)n
n

}
n=1 , lim bn = 0
n

{(1)n }
n=1 ,

lim cn ()
n

(4) {dn }
n=1 = {n}n=1 , lim dn = ()
n

(5) {en }
n=1 = {n}n=1 , lim en = ()
n

2. {an }
n=1 = {n}n=1 {bn }n=1 = {2n}n=1

. lim an = lim n = , lim bn = lim 2n = ,


n

lim
n

2n
bn
= lim
= lim 2 = 2
n
n
an
n

: {n}
n=1 {2n}n=1 ,

, , {2n}
n=1 {n}n=1

2
:

2
3. {an }
n=1 = {n}n=1 {bn }n=1 = {n }n=1

. lim an = lim n = , lim bn = lim n2 = ,


n

n2
bn
= lim
= lim n =
lim
n n
n
n an

(1)

2
: {n}
n=1 {n }n=1 ,

, {n2 }
n=1 {n}n=1

, (1)
, , ,
, ,
, order, ,
, 3
:
1 log n n n log n n2 n3 2n 3n n!

(2)

(2), :
1 ; 1 ( {1}
n=1 )
{1 + n1 }
n=1 {2 +

(1)n
}n=1
n

{1}
n=1

(2) , 1 , n
, ,
(2) log , :
logn b
loga n
= lim
= lim log b = loga b
n
n logb n
logn a n a
lim

(2) ? ,
: :
(1) lim
n

(2) lim
n

log n
1

n log n
n

n3
2
n
n

(3) lim
3

= lim log n = , 1 log n


n

= lim log n = , n n log n


n

= lim n = , n2 n3
n

order ,

3n
n
n 2

(4) lim

= lim
n


3 n
2

= , 2n 3n

? :
4. n 2n
. n > 2,
2 = (1 + 1) =
n

n
X

Cin 1ni 1i > C2n =

i=0

n(n 1)
,
2

2n
n1
>

n
2

lim

n1
n 2

2n
n n

= , lim

= , n 2n

4 (2) , ,
, , 4
5. nk an , a > 1
. a = 1 + a , a > 0 n > 2 , n 1 > n2 , ,
a = (1 + a ) =
n

n
X

n(n 1) 2 n2 2 n2
a > a = (a 1)2
2
4
4

Cin 1ni ai > C2n a2 =

i=0

:
an
k
n n

(a) k 0, lim

= lim an nk =
n

(b) k = 1,
an
n
an
=
> (a 1)2 ,
k
n
n
4
an
n n

lim n4 (a 1)2 = , lim


n

= , n an

(c) k > 0,
1

ak
n

an
=
nk
1
1
k

a > 1, (b) lim


n

ak
n

n

n !k

an
k
n n

= , lim
5

= , nk an

6. an n!, a > 0
. k 2a , n > k ,
n n1
k+1 k
2 1
1
2n
n!
nk
=

>
2

=
,
an
a
a
a
a
a a
ak
(2a)k
an
k
(2a)
n

5 lim

= , lim

n!
n
n a

= , an n!

(2), log n n n log n n2 ,


lim n
n log n

= lim
n

log n
n

= 0 ,

, ,
, , (
),

. {an }
n=1 {bn }n=1 (), c ,

(1) lim (an + bn ) = lim an + lim bn


n

(2) lim (an bn ) = lim an lim bn


n

(3) lim c an = c lim an


n

(4) lim (an bn ) = lim an lim bn


n

(5) lim bn 6= 0, lim


n

an
n bn

lim an
lim bn

(6) p > 0 an > 0, lim apn =


n

lim an

p

. {an }
n=1 , {bn }n=1 {cn }n=1 , n0 ,

n n0 , an bn cn , lim an = lim cn = L, lim bn = L


n

7. f (x) lim f (x) = L f (n) = an , lim an = L


x

8. lim an = L f (x) x = L ,
n

lim f (an ) = f ( lim an ) = f (L)


n

9. : {an }
n=1
a1 a2 a3 . . . an . . . a1 a2 a3 . . . an . . . ,

M n |an | M, lim an
n

, log n n n log n n2 , :

{ n n}
n=1 :
10. lim

n = 1

. an =

n an > 1, an = 1 + an, n > 2 , n 1 > n2 ,

,
n=

ann

= (1 + an, ) =
n

n
X

Cin 1ni akn,

i=0

> C2n a2n, =

2
n(n 1) 2
n2
n2
n2
n
an, > a2n, = (an 1)2 =
n1 ,
2
4
4
4

lim 2n
n

lim n n = 1

2
n1< ,
n

= 0, : lim ( n n 1) = 0,
0<

11. lim

log n
n

= 0 log n n n log n n2

1
. f (x) = log x x = 1 , an = n n = n n, 8 :




1
1
log n
= lim log n n = log lim n n = log 1 = 0
lim
n
n n
n

. 2048 , App ,
, 2048 , :
(1) 32 64 128 256 512 1024 2048


(2) ?
(3) ( 2048) ? ?

. : lim |an | = 0, lim an = 0


n

. lim

n
n!

= 0

. : |r| < 1, lim nrn = 0


n

. {an }
n=1 , a1 =

2, an+1 =

2 + an

.
(1)
(2) lim
n

1
2

1
2

34

3
4

2n1
2n

2n1
2n

<

2n+1

= 0

. lim f (x) , lim an ,


x

. 8 (2)?

3.1

, :
, 1 + 2 + 3 + + 98 + 99 + 100,
, 5050
, () ,
:
S=

1 + 2 + 3 + + 98 + 99 + 100

S = 100 + 99 + 98 + + 3 + 2 +

1,

2S = (1 + 100) + (2 + 99) + (3 + 98) + + (98 + 3) + (99 + 2) + (100 + 1)


= 101 + 101 + 101 + + 101 + 101 + 101 = 101 100 = 10100,
1
S = 10100 = 5050
2

, , ,
S = 1 + 2 + 3 + + 98 + 99 + 100 1 1
n
P
, ,
ak , ak a1
k=1

d , ak = a1 + (k 1)d,

?
: 12 ( + ) , ( + )
, ,
, , k ak 1,
1 , 1
,

1:

3.2

, , :
, , , ,
, , 1 ,
2 , 4 ,
, ,
,
S = 1 + 2 + 22 + + 261 (
) , ,
:
S = 1 + 2 + 22 + 23 + + 261
2S =

2 + 22 + 23 + + 261 + 262 ,

S = 1 262 ,

S = 262 1

262 1 = 4611686018427387903 62 ,
,
,

3.3

, :
1+

1
1 1 1 1 1 1 1
+ + + + + + ++ +
2 3 4 5 6 7 8
n

, ,
, , ,
, (,
, )
:
1 1 1 1 1 1 1
1
+ + + + + + ++ +
2 3 4 5 6 7 8
n
1
1 1 1 1 1 1 1
> 1+ + + + + + + ++ +
2 |4 {z 4} |8 8 {z 8 8}
n
1
1
1
=1+ +
+
+
2
2
2
1
1
1
+
+
>1+ +
2
3
4
1+

(1)

(2)
(3)

(1) (3) , (1) > (3) ,


?
, ,
, :
(1) , ,
n x1 , x2 , . . . , xn
1
x1

1
x2

n
++

1
xn


, + , ,
, ,
, ,
, (1)
:
1+

n
X
1 1 1 1 1 1 1
1
1
+ + + + + + + + + = lim
,
n
2 3 4 5 6 7 8
n
k
k=1

an ,

n=1

X
n=1

sn =

n
P

def.

an = lim

n
X

ak = lim sn ,

k=1

(4)

ak (4) ,

k=1


,
, (1) (3)
( ), (2) ,

1
2

, , (2)

1
{sn = 1 + 12 (n 1)}
n=1 , (1)

P
,
an ,
n=1

, f (x) g(x) > 0,

R
1

g(x) dx ,

R
1

f (x) dx

, ,
() :
x , x2 , x2 x, x(x1) 0
0 x 1 x , x = 1
, 2, 3, 4 1, ,
,
( ),
,
, ()
() () ,
,
,

3.4

p-

X
1
1
1
1
1
1
1
= 1+ 2 + 2 + 2 + 2 + 2 ++ 2 + ,
2
n
2
3
4
5
6
n
n=1
, :
n
o
n
P
1
,
:, sn =
2
k
k=1

n=1


n
n
n 
X
X
X
1
1
1
1
1
sn =
= 1 + 1 < 2,
<1+
=1+

2
k
(k 1)k
k1 k
n
k=1

2 ,

k=2

n=1

1
n2

k=2

, (Euler) , ,

, p-:

X
1
1
1
1
1
1
1
1
= p + p + p + p + p + p ++ p + ,
p
n
1
2
3
4
5
6
n
n=1

p p-, 7.8
R
, 1 x1p dx: p > 1 , 0 < p < 1 , p-

3.5

:
m
X

an bn

n=1

an bn ,

n=1

{an }m
n=1 , {an }n=1 , d, {bn }n=1 , {bn }n=1 ,

r

sn =

n
X

an bk = a1 b1 + a2 b2 + a3 b3 + a4 b4 + + an bn

k=1

rsn =

n
X

a1 b2 + a2 b3 + a3 b4 + + an1 bn + an bn+1

an (bk r) =

k=1

r 6= 1,
n
n
X
X
(1 r)sn = a1 b1 +
(ak ak1 )bk an bn+1 = a1 b1 + d
bk an bn+1
k=2

k=2

n1

= a1 b1 + d

b2 (1 r
1r

(a1 + (n 1)d)b1 r n

r(1 r n1 ) (n 1)r n (1 r)
1r
n
r nr + (n 1)r n+1
= a1 b1 (1 r n ) + db1
,
1r

= a1 b1 (1 r n ) + db1

sn = a1 b1

1 rn
r nr n + (n 1)r n+1
+ db1

1r
(1 r)2

r
1 rn
n 1 r
lim

r nr n + (n 1)r n+1
n
(1 r)2
lim

: |r| < 1 , |r| > 1 ()


5 , |r| < 1 , |r| > 1
r = 1, , ,

n
a1 b1 + db1 r
X
X
|r| < 1
1 r (1 r)2
ak bk =
an bn = lim
n

n=1
k=1

|r| 1
5

3.6

, , ,
,
, , ,
,
, , , ,
?
, , , ?
, , 10 , , :
, 20 :, 20 , ,
! , 40 , : 40 :? :
, 40 80 :!
, , ,
, 80 , 80
, , , 10
,
? , ,
, 10 , : , , 10 ;
, , ,
, 10 + 20 + 40 = 70 , 80 , 10
?
,
,

1
4

1
2

10 = 5;

10 ( 14 ,

10 ); , n 1 , n ,

1
2n

10,

(, 0 )
, ? ?

: , ,
, (? ?),

3.7

, : , 17
, :, ,
,
, , 8 12 , 5 23 , 1 98 ,
?
, , , 18
, 12 , 9 , 13 , 6 , 19 , 2
17 , 1
, , ,
? ,
,
, 12 , 31 , 91
1 , , ,
1
: ()
18
181 12 , 31 , 19 ,

,
,
:

a1 = 17
a2
a3
..
.

1
2

1
3

1
9

1
18

1
a
2 1
1
a
2 2
1
a
2 3

1
a
3 1
1
a
3 2
1
a
3 3

1
a
9 1
1
a
9 2
1
a
9 3

1
a
18 1
1
a
18 2
1
a
18 3

a2 =

1
a
2 n

1
a
3 n

1
a
9 n

1
a
18 n

an+1 =
..
.

..
.

an
..
.

..
.

..
.

..
.

..
.

..
.

..
.
..
.

a3 =
a4 =
..
.

1
a
18 1
1
a
18 2
1
a
18 3

1
a
18 n

lim

n
X
1
k=1

n
X
1

1
17 X 1
a
=
lim
1
n
n 2
2 18k1
18k1
k=1
k=1



 n 
1 n
1
17 1 1 18
= lim 9 1
= lim
= 9

1
n
n 2
18
1 18

ak = lim

6 , 2

3.8

0.9 1?

,
:
 
n
9
1 n
X
1 10
9
10
0.9 = 0.9(9 ) = lim
= lim
1
k
n
n
10
1 10
k=1

 n 
 n
1
1
= lim 1
= lim 1 lim
= 1 0 = 1
n
n
n
10
10

PTT : Math
, 0.9 = 1 ,
0.9 6= 1, ,
,
!

0.142857 = 0.142857(142857 ) = lim

n
X
142857
k=1
1
106

n
X
1
= lim 142857
= lim 142857
n
106k n
k=1


n 
1
1
142857
1
= lim
=

n 999999
106
7

3.9

106k

1 1016
1 1016

n 

, , ,
11.2 ,
? 11
, :
(1) (11.3 11.6)
(2) ? (11.6)
(3) () (11.8)
(4) ? (11.9 11.11)

n
P

. :

ak =

k=1

X
(1)n
n=1

=1

(2a1 +(n1)d)n

1 1 1 1 1
+ + + ,
2 3 4 5 6

:
1 1 1 1 1
+ + +
2 3 4 5 6
1
1 1 1 1
+
S1 = 1 + + + + +
3 5 7 9 11
1
1
1 1 1 1
+
+
S2 = + + + +
2 4 6 8 10 12
1 1 1 1 1
2S2 = 1 + + + + + +
2 3 4 5 6

S =1

S1 + S2 = 2S2 , S1 = S2 , S = S1 S2 = 0
?
. r
r nr n + (n 1)r n+1
n
(1 r)2
lim

. :

X
2n 1
n=1

2n

1
3
5
7
2n 1
+ 2 + 3 + 4 ++
+
2 2
2
2
2n

. ?
. , 6 2
. 0.285714, 0.428571, 0.571428, 0.714285, 0.857142
. 0.123

4
4.1

, , 10.5

(a) x2 = 4py, p > 0 , p < 0 ; (0, p), y = p


(b) y 2 = 4px, p > 0 , p < 0 ; (p, 0), x = p
y

y 2 = 4px

x = 4py
F (0, p)

F (p, 0)

x
y = p
x = p

1: x2 = 4py, p > 0 y 2 = 4px, p > 0

x2
a2

y2
b2

= 1,

(a) a > b > 0, () , (c, 0), a2 = b2 + c2


(b) b > a > 0, () , (0, c), a2 = b2 + c2
y
y
F1
F2

F1

x
F2

2:

x2
a2

y2
b2

= 1, a > b > 0

x2
a2

y2
b2

= 1, b > a > 0


(a)

x2
a2

yb2 = 1 , (c, 0), c2 = a2 + b2 , (a, 0),

y = ab x
(b)

y2
a2

xb2 = 1 , (0, c), c2 = a2 + b2 , (0, a),

y = ab x
y

F1

F1

F2

x
F2

3:

4.2

x2
a2

y2
b2

=1

y2
a2

x2
b2

= 1

ax2 + 2bxy + cy 2 + dx + ey + f = 0,

(1)

, a, b, c, d, e, f ,
,
b = 0, (1) x y
, (x h)2 = 4p(y k), (y k)2 =
2

+
4p(x h), (xh)
a2

(yk)2
b2

= 1, (xh)

a2

(yk)2
b2

= 1,

(yk)2
a2

(xh)2
b2

= 1

b 6= 0, 3, ,
xy ( ); ,
, ,
,




h i
a b
x
d
x
= ,
, X = , B = , C = f , X
A=
b c
y
e
y

(1)

X T AX + B T X + C = 0,
2

(2)

,
3 , P , X = P X,


x
cos sin
x
=
,
y
sin cos
y

(x, y) (
x, y) , (2)

T A(P X)
+ BT P X
+C =X
T P T AP X
+ BT P X
+C
X T AX + B T X + C = (P X)
T (P T AP )X
+ (P T B)T X
+ C = 0,
=X

P T AP ,

D = P T AP =

T DX
+ (P T B)T X
+C
X T AX + B T X + C = X
x + ey + f = 0,
= 1 (
x)2 + 2 (
y )2 + d

PTB =

d
e

x y
P T AP ?
P T AP

cos sin
a b
cos sin

=
sin cos
b c
sin cos

a cos2 + 2b sin cos + sin2


b(cos2 sin2 ) (a + c) sin cos

=
b(cos2 sin2 ) (a + c) sin cos
a sin2 2b sin cos + c cos2

2
a+c
2
a cos + b sin 2 + sin
b cos 2 2 sin 2

=
2
2
b cos 2 a+c
sin
2
a
sin

b
sin
2
+
c
cos

P T AP :
b cos 2

= 21 tan1

2b
a+c
sin 2 = 0 tan 2 =
,
2
(a + c)

2b
a+c

4.3

:
Ax2 + By 2 + Cz 2 + 2Dxy + 2Exz + 2F yz + Gx + Hy + Iz + J = 0,

A, B, C, . . . , J , :

x
A
D
E
i x h i h i
h
i
h


x y z
D B F y + G H I y + J = 0
z
z
E F C

D = E = F = 0, x, y, z

D, E, F , , ,
,
:
Ax2 + By 2 + Cz 2 + J = 0

Ax2 + By 2 + Iz = 0

, ? ,
, ,
, , ,
, , ,
? , ,
()
, ,
, 12.6
,
(a) (ellipsoid):

x2
a2

y2
b2

z2
c2

= 1

(1) z = k ,

x2
a2

y2
b2

=1

k2
,
c2

k 2 < c2 ,

k 2 = c2 , k 2 > c2
(2) y = k x = k ,
(b) (cone):

z2
c2

x2
a2

y2

b2

(1) z = k ,

x2
a2

y2
b2

k2
,
c2

k 6= 0 , k = 0

(2) y = k ,

z2
c2

x2
a2

k2
,
b2

k 6= 0 ,

k = 0 ; x = k
4

y
x

4:

x2
a2

y2
b2

(c) (elliptic paraboloid):

z2
c2
z
c

(1) z = k ,

k
c

= 1
x2
a2

z2
c2

x2
a2

y2

b2

y2

b2

x2
a2

y2
b2

= kc ,

z
c

x2
a2

k
c

> 0 , k = 0

< 0

(2) y = k ,

k2
,
b2

k ;

x = k
z
c

(d) (hyperbolic paraboloid):


(1) z = k ,

x2
a2

y2
b2

x2
a2

y2

b2

= kc , k 6= 0 , k = 0

(2) y = k ,

z
c

x2
a2

k2
,
b2

k ;

x = k
z

y
y
x

5:

x2
a2

y2
b2

z2
c2

= 1

(e) (hyperboloid of one sheet):

x2
a2

y2
b2

z2
c2

=
z2
c2

(1) z = k ,

x2
a2

y2
b2

=1+

(2) y = k ,

x2
a2

z2
c2

=1

x2
a2

= 1

k2
,
c2
k2
,
b2

k
k 2 6= b2 ,

k 2 = b2 ; x = k
5

y2

b2

(f) (hyperboloid of two sheets): xa2


x2
a2

(1) z = k ,

y2
b2

y2
b2
k2
c2

z2
c2

= 1

1, k 2 > c2 ,

k 2 = c2 , k 2 < c2
z2
c2

(2) y = k ,

x2
a2

= 1+

k2
,
b2

; x = k
z

6: :

x2
a2

y2
b2

z2
c2

= 1 : xa2

y2
b2

z2
c2

= 1

. :
(a) x2 2xy + y 2 5y = 0
(b) 5x2 6xy + 5y 2 4x 4y 4 = 0
(c) x2 + 4xy + y 2 8x + 2y 8 = 0

. (1) , ?
.

x2
a2

+ yb2 zc2 = 1 xa2 yb2 + zc2 = 1

. :
(a) (elliptic cylinder):

x2
a2

y2
b2

=1

y2
a2

=1

(c) (hyperbolic cylinder):

x2
a2

(b) (circular cylinder):

x2
a2

y2
b2

=1

(d) (parabolic cylinder): x2 + 2ay = 0

(linear algebra) ,
, ,
, ; , ,
, , ,
,

, (matrix) n
, 1 4
, ,

, ,
,
, ,
;
, ,
, ,
(tensor)
? , ,
,
, , ,

,

, ,
,

, , ?
? dx (differential)
, ; ,
, ,
,
1

5.1

(vector space)

, ,
, , ?
, ;
,
1. R V
, x, y V , x + y V ; a R x V , ax V ,
:
(1) x, y V , x + y = y + x

()

(2) x, y, z V , (x + y) + z = x + (y + z)

()

(3) V , 0, x V , x + 0 = x ()
(4) x V , y V x + y = 0
(5) x V , 1x = x

()
()

(6) a, b R x V , (ab)x = a(bx)

()

(7) a R x, y, V , a(x + y) = ax + ay ()
(8) a, b R x V , (a + b)x = ax + bx ()

, R
field, C p- Zp ,

2. Rn n = 3 , R3
~v = (v1 , v2 , v3 ) , ~0 =
(0, 0, 0), 1, ~v

(v1 , v2 , v3 ), ~v
3. m n Mmn (R), (
) 0 ,
1
, Mmn (R) , Rmn (
)
, ,
,
2

, () ,

4. I = [0, 1] R F (I, R), f, g


F (I, R), f = g x I, f (x) = g(x); , (f + g)(x) =
f (x) + g(x) (cf )(x) = c(f (x)), c R, F (I, R)

, I S, F (S, R)
5. , f (x) = an xn + an1 xn1 + + a1 x + a0 ,
n , ai , i = 0, 1, 2, . . . , n , xi (coefficient)

f (x) = an xn + an1 xn1 + + a1 x + a0


g(x) = bm xm + bm1 xm1 + + b1 x + b0 ,
f (x) = g(x) m = n, ai = bi , i = 1, 2 . . . , n

: m n, bm+1 = bm+2 = = bn = 0, g(x)


g(x) = bn xn + bn1 xn1 + + b1 x + b0 ,

f (x) + g(x) = (an + bn )xn + (an1 + bn1 )xn1 + + (a1 + b1 )x + (a0 + b0 )


cf (x) = can xn + can1 xn1 + + ca1 x + ca0 ,

, (P (f ), R),

6. 2 (sequence), ,
,
R , (n) = an , n = 1, 2, 3, . . ., {an } V

an {an } , {an } {bn } , c R,


{an } + {bn } = {an + bn } c{an } = {can }, V

() ,
, , ; ,
,
, ,

7. S = {(a1 , a2 ), a1 , a2 R}, (a1 , a2 ), (b1 , b2 ) S c R,


(a1 , a2 ) + (b1 , b2 ) = (a1 + b1 , a2 b2 ) c(a1 , a2 ) = (ca1 , ca2 ),

(1), (2), (8), S


8. S = {(a1 , a2 ), a1 , a2 R}, (a1 , a2 ), (b1 , b2 ) S c R,
(a1 , a2 ) + (b1 , b2 ) = (a1 + b1 , 0) c(a1 , a2 ) = (ca1 , 0),

(3), (4), (5), S


, R3 u, v u v,
u v, ?
, , (1) (8) ,
, ; ,
(inner produce space)
; R3 , ,

5.2

R3 ,
i = (1, 0, 0),

j = (0, 1, 0),

k = (0, 0, 1)

? R3 , i, j, k
R3 ; , i, j, k
, {i, j, k} R3
, ,
(a) ,

(b) ,

(c) ,
(d) , ,

, R3 , {2i, j, k} {i+j, j+k, k+i},


, ,
4

5.3

R V W , T : V W x, y V
c R,
(1) T (x + y) = T (x) + T (y),
(2) T (cx) = cT (x),

T V W
(1) , : x, y V ,
x + y, T W , T (x + y); :
x, y T W , T (x) T (y), W ,
T (x) + T (y) (1)
, (2) , : x c ,
T W , T (cx); : x T W

, T (x), T (x) c cT (x) (2)

, : T ,

? R3 ; Mmn (R)
Mnm (R) ; , (
)

5.4

, V R
, L(V, R), L(V, R) V
(linear functional)
, :
9. V = Mnn (R), f : V R f (A) = tr(A) A (trace),
f

10. V [0, 2] , f V ,
Z 2
Z 2
1
1
an (f ) =
f (x) sin nx dx, bn (f ) =
f (x) cos nx dx,
2 0
2 0
an bn , n ( ) (n-th Fourier coefficient)
5

11. R3 , L(R3 , R) R3 {i, j, k}


, , i , j , k , i, j, k
, :
i (i) = 1

i (j) = 0

i (k) = 0

j (i) = 0

j (j) = 1

j (k) = 0

k (i) = 0

k (j) = 0

k (k) = 1,

x = xi i + xj j + xk k, i (x) = xi , j (x) = xj , k (x) = xk


{i , j , k } L(R3 , R) () :
(1) f (x) L(R3 , R), x = xi i + xj j + xk k,
f (x) = f (xi i + xj j + xk k) = f (i)xi + f (j)xj + f (k)xk
= f (i) i (x) + f (j) j (x) + f (k) k (x) = (f (i) i + f (j) j + f (k) k )(x),

f = f (i) i + f (j) j + f (k) k , , L(R3 , R)


i , j , k
(2) i , j , k :

xi i + xj j + xk k = 0,

, v 0,
(xi i + xj j + xk k )(i) = xi i (i) + xj j (i) + xk k (i) = xi = 0(i) = 0,
(xi i + xj j + xk k )(j) = xi i (j) + xj j (j) + xk k (j) = xj = 0(j) = 0,
(xi i + xj j + xk k )(k) = xi i (k) + xj j (k) + xk k (k) = xk = 0(k) = 0

, L(R3 , R) ,2 {i , j , k },
{i, j, k}
V , L(V, R) , :
12. V , V (dual space) V
V R L(V, R)

, {e1 , e2 , . . . , en } V ,
V = L(V, R) {e1 , e2 , . . . , en },

1 i = j

ei (ej ) = ij =
0 i 6= j

i , j , k , xi i + xj j + xk k = 0, xi = xj = xk = 0,
i , j , k
2

5.5

, V ,
,
13. V R V h, i V V R
, x, y, z V , c R,
(1) hx + z, yi = hx, yi + hz, yi
(2) hcx, yi = chx, yi
(3) hx, yi = hy, xi
(4) hx, xi > 0 x 6= 0

, C ,
14. (V, h, i) (inner product space)
, ,
15. V = Mnn (R), A, B, hA, Bi = tr(B T A), (V, h, i)

,
16. (V, h, i) , x V , x (norm)
p
kxk = hx, xi
:

17. V R , x, y V c R, :
(1) kcxk = |c| kxk
(2) kxk 0, kxk = 0 x = 0
(3) |hx, yi| kxk kyk

()

(4) kx + yk kxk + kyk

()

,
7

. R ,
(f + g)(x) = f (x) + g(x) (cf )(x) = c(f (x)),

.
. Mnn (R), n 2 ?
. L(V, R)
. V = C([0, 1]) [0, 1] , f, g V ,

hf, gi =

f (x)g(x) dx,

h, i V = C([0, 1])

6.1

3.10 (differential)
1. x > 0 , x + x , ?
. , x + x x
(x + x)2 x2 = x2 + 2xx + (x)2 x = 2xx + (x)2

(1)

, ,
? ,
x

(x)2 x

xx

x2

xx

1: x x + x , 2xx

(a) 1 (1), 2xx

, (x)2 , ,
2xx (x)2 ,
2 , ,
, x , 2xx (x)2
, (x)2 2xx ( x 0),
(x)2
x
= lim
= 0
x0 2xx
x0 2x
lim

:
x + x x 2xx
1

(b) : (x)2 x , (x)2


x ( x 0); , 2xx x ,

5 , ,
1
(c) , , :

,
,
, x
, ,
11
(d) , dx = x, y = f (x),

x , dy = df = f (x) dx
1, y = f (x) = x2 , x
dy = df = f (x) dx = 2x dx

,
(V, R) f : V R (V , R), 1
V = R, dx : R R R ,1 dx
R = L(R, R) , f : dy = df
R = L(R, R) , , f (x) dx, f (x)
y
df = f (x) dx
y = f (x)

2: df : R R (x, f (x)) ()
2 : df R R ( R
), (x, f (x)), 2
, df (x, f (x))
1

dx x Tx R Tx R R ,

(e) f (x) df = f (x) dx z =


f (x, y) ? x, y , R2 (

) R2 (R2 ) = L(R2 , R) (R2 ) dx


dy, df ,
, , ,

f
x

f
,
y

,
df =

f
f
dx +
dy
x
x

3 z = f (x, y) (x, y, f (x, y)) ,


z = f (x, y) (x, y) , dz = df , (
), (x, y, f (x, y)) x ,
y ,

f
x

dy, , dz = df =

f
x

dx +

f
x

f
x

dx,

dy

y
x

(x, y, 0)

3: df : R2 R (x, y, f (x, y))


, z = f (x, y) dz = df (total
differential)
(f)

f
x

f
y

14 , :

f
f (x0 + h, y0 ) f (x0 , y0 )
(x0 , y0) = lim
h0
x
h
f (x0 , y0 + h) f (x0 , y0 )
f
(x0 , y0) = lim
,
h0
y
h

R3 y = y0 (x, y, f (x, y)), ,


(x0 , y0 , f (x0 , y0 ))

f
(x0 , y0)
x

, R3

x = x0 (x, y, f (x, y)), ,


(x0 , y0 , f (x0 , y0 ))

f
(x0 , y0 )
y

(x0 , y0 , f
(x0 , y0 ))
x

(x0 , y0)) (x, y, f (x, y)) (x0 , y0 , f (x0 , y0))


(x0 , y0 , f
y

, 3
3

6.2

, 12
12.1 12.5
2. R3 u = u1 i + u2 j + u3 k v = v1 i + v2 j + v3 k, u v

u v = u1 v1 + u2 v2 + u3 v3

:
(1) u u = kuk2 = u21 + u22 + u23
(2) u v = v u
(3) u (v + w) = u v + u w
(4) (c u) u + c (u v) = u (c v)
(5) 0 u = 0

: u v , u v = kuk kvk cos


3. R3 u = u1 i + u2 j + u3 k v = v1 i + v2 j + v3 k, u v

u v = (u2 v3 v2 u3 , u3v1 v3 u1 , u1 v2 v1 u2 )








u2 u3 u3 u1 u1 u2






=

,
,
v2 v3 v3 v1 v1 v2

(1) u v = v u
(2) (c u) v = c (u v) = u (c v)
(3) u (v + w) = u v + u w
(4) (u + v) w = u w + v w
(5) u (v w) = (u v) w
4

(6) u (v w) = (v w)v (u v)w


(7) u v u , v
(8) u, v : a b = 0

: u v , ku vk = kuk kvk sin


4. R3
u = u1 i + u2 j + u3 k,

v = v1 i + v2 j + v3 k,

w = w1 i + w2 j + w3 k

?
. (u v) w,
(u v) w = ku vk kwk cos = kuk kvk sin kwk cos ,

u v w , u v ku vk = kuk kvk sin


u v , kwk cos u, v, w ,
(u v) w
uv

v
u

4: (u v) w
,



u u u




1



2
3


u2 u3 u3 u1 u1 u2
(w1 , w2, w3 ) = v1 v2 v3 ,
,
,
(u v) w =







v2 v3 v3 v1 v1 v2
w1 w2 w3

, (u v) w ,
, ,
, u, v, w u v w,
u v w = det(u, v, w) i j k
5

6.3

,
Rb
a f (x) dx f (x) x- ,

x-, x-

, f (x, y), R2 D ,

f (x, y) dA, xy-

, f (x, y) xy-, dA
, , dA
dx dy
, ,
, ; ,
(polar equation) ,
, ,
, R2 ; R3
,

R2 (x, y) (r, )
y = r sin

x = r cos ,

6.1 , x, y r, :

dx = dr cos r sin d
dx
cos r sin
dr
=

dy = dr sin + r cos d
dy
sin r cos
d

6.2 , 2




cos r sin
= r cos2 + r sin2 = r,



sin r cos

dA = dx dy = r dr d
2

R3 , u = (cos , r sin , 0), v = (sin , r cos , 0)


w = (0, 0, 1) , w ,


R3 (x, y, z) (r, , z )
x = r cos ,

y = r sin ,

z = z

, 6.1 , x, y, z r, , z

cos r sin
dx
dx
=
dr
cos

r
sin
d


dy = dr sin + r cos d
dy = sin r cos

0
0
dz
dz = dz

6.2 ,


cos r sin 0




sin r cos 0 = r,




0
0
1

dr
0

0
d
dz
1

dV = dx dy dz = r dr d dz

R3 (x, y, z) (r, , )
x = r sin cos ,

y = r sin sin ,

z = r cos

6.1 , x, y, z r, , :

dx = dr sin cos + r cos d cos r sin sin d


dy = dr sin sin + r cos d sin + r sin cos d

dz = dr cos r sin d

dr
sin cos r cos cos r sin sin
dx

dy = sin sin r cos sin r sin cos d


d
cos
r sin
0
dz

6.2 ,

sin cos r cos cos r sin sin


sin sin r cos sin r sin cos


cos
r sin
0





= r 2 sin ,


dV = dx dy dz = r 2 sin dr d d
7


R2 (x, y) u, v
x = x(u, v),

y = y(u, v),

6.1 , x, y u, v :


dx = x du + x dv
x
x
du
dx
u
v

= u v

y
y
y
y
dy = du + dv,
dv
dy
u
v
u
v
x, y u, v (Jacobian) :

x
(x, y) u
=
(u, v) y
u

dA = dx dy =

6.4

x
v
y
v

(x, y)
du dv
(u, v)

,
, , ,
, (
, ),
,
. R2 x = u2 v 2 , y = uv, dx dy du dv
. R3 x = x(u, v, w), y = y(u, v, w), z = z(u, v, w), dxdy dz
du dv dw ?

También podría gustarte